You are on page 1of 530

Downloaded From : www.EasyEngineering.

net

ww
w.E
a syE
ngi
nee
rin
g.n
et

**Note: Other Websites/Blogs Owners Please do not Copy (or) Republish


this Materials, Students & Graduates if You Find the Same Materials with
EasyEngineering.net Watermarks or Logo, Kindly report us to
easyengineeringnet@gmail.com

Downloaded From : www.EasyEngineering.net


Downloaded From : www.EasyEngineering.net
GATE EC BY RK Kanodia

MULTIPLE CHOICE QUESTION

ELECTRONICS & COMMUNICATION


Electronics &ENGINEERING
CIVIL Communication Engineering
ww ENGINEERING
w.E
asFifth
yE
Edition

ngi
nee
R. K. Kanodia
rin B.Tech.

Downloaded From : www.EasyEngineering.net

g.n
et

NODIA & COMAPNY


JAIPUR

www.gatehelp.com

Downloaded From : www.EasyEngineering.net


Downloaded From : www.EasyEngineering.net

TOPICS COVERED
( SOLUTIONS ARE GIVEN AT EACH TOPICS END )

Analog Electronics
Design of Steel Structure
Communication Systems
Engineering Mathematics

Control Systems
Environmental Engineering

wDigital Circuits
Fluid Mechanics and Hydraulic Machines
ww
.Ea
Electromagnetics
Geotechnical Engineering
s y
Electronic Devices E
Highway Engineering and Surveying
ngi
n ee
Irrigation Engg. and Engg. Hydrology
Engineering Mathematics
rin
RCC Structures and Prestressed Concrete
General Apitude
g.n
Strength of Materials
et
Network Analysis
Structural Analysis
Signals and Systems

Downloaded From : www.EasyEngineering.net


Downloaded From : www.EasyEngineering.net

GATE SOLVED PAPER - EC


ANALOG ELECTRONIC

2013 ONE MARK

Q. 1 In the circuit shown below what is the output voltage ^Vouth if a silicon transistor
Q and an ideal op-amp are used?

ww
w.E (A) - 15 V
(C) + 0.7 V asy (B) - 0.7 V
(D) + 15 V

Q. 2
En
In a voltage-voltage feedback as shown below, which one of the following statements
is TRUE if the gain k is increased?

gin
eer
ing
.ne
t
(A) The input impedance increases and output impedance decreases
(B) The input impedance increases and output impedance also increases
(C) The input impedance decreases and output impedance also decreases
(D) The input impedance decreases and output impedance increases

2013 TWO MARKS

Q. 3 The ac schematic of an NMOS common-source state is shown in the figure below,


where part of the biasing circuits has been omitted for simplicity. For the n
-channel MOSFET M, the transconductance gm = 1 mA/V , and body effect and
channel length modulation effect are to be neglected. The lower cutoff frequency
in HZ of the circuit is approximately at

Downloaded From : www.EasyEngineering.net


Downloaded From : www.EasyEngineering.net
GATE SOLVED PAPER - EC ANALOG ELECTRONIC

(A) 8 (B) 32
(C) 50 (D) 200

Q. 4 In the circuit shown below, the knee current of the ideal Zener dioide is 10 mA
. To maintain 5 V across RL , the minimum value of RL in W and the minimum

ww power rating of the Zener diode in mW, respectively, are

w.E
asy
En
(A) 125 and 125
gin (B) 125 and 250

Q. 5
(C) 250 and 125
eer
(D) 250 and 250

In the circuit shown below the op-amps are ideal. Then, Vout in Volts is

ing
.ne
t

(A) 4 (B) 6
(C) 8 (D) 10

Q. 6 In the circuit shown below, Q1 has negligible collector-to-emitter saturation


voltage and the diode drops negligible voltage across it under forward bias. If Vcc
is + 5 V , X and Y are digital signals with 0 V as logic 0 and Vcc as logic 1, then
the Boolean expression for Z is

Downloaded From : www.EasyEngineering.net


Downloaded From : www.EasyEngineering.net
GATE SOLVED PAPER - EC ANALOG ELECTRONIC

(A) XY (B) XY
(C) XY (D) XY

Q. 7 A voltage 1000 sin wt Volts is applied across YZ . Assuming ideal diodes, the
voltage measured across WX in Volts, is

ww
w.E
asy
En
(A) sin wt
(C) ^sin wt - sin wt h /2 gin (B) _sin wt + sin wt i /2
(D) 0 for all t

Q. 8
eer
In the circuit shown below, the silicon npn transistor Q has a very high value of

ing
b . The required value of R2 in kW to produce IC = 1 mA is

.ne
t
(A) 20 (B) 30
(C) 40 (D) 50

2012 ONE MARK

Q. 9 The i -v characteristics of the diode in the circuit given below are


v - 0.7 A, v $ 0.7 V
i = * 500
0A v < 0.7 V

Downloaded From : www.EasyEngineering.net


Downloaded From : www.EasyEngineering.net
GATE SOLVED PAPER - EC ANALOG ELECTRONIC

The current in the circuit is


(A) 10 mA
(B) 9.3 mA
(C) 6.67 mA
(D) 6.2 mA

Q. 10 The current ib through the base of a silicon npn transistor is 1 + 0.1 cos (10000pt) mA
At 300 K, the rp in the small signal model of the transistor is

ww
w.E
(A) 250 W asy (B) 27.5 W
(C) 25 W
En (D) 22.5 W

Q. 11
the diode D1 is
gin
The diodes and capacitors in the circuit shown are ideal. The voltage v (t) across

eer
ing
.ne
Q. 12
(A) cos (wt) - 1
(C) 1 - cos (wt)
(B) sin (wt)
(D) 1 - sin (wt)

The impedance looking into nodes 1 and 2 in the given circuit is


t

(A) 50 W (B) 100 W


(C) 5 kW (D) 10.1 kW

Downloaded From : www.EasyEngineering.net


Downloaded From : www.EasyEngineering.net
GATE SOLVED PAPER - EC ANALOG ELECTRONIC

2012 TWO MARKS

Q. 13 The circuit shown is a

(A) low pass filter with f3dB = 1 rad/s


(R1 + R2) C
(B) high pass filter with f3dB = 1 rad/s
R1 C
(C) low pass filter with f3dB = 1 rad/s
ww (D) high pass filter with f3dB =
R1 C
1 rad/s

w.E
Q. 14
(R1 + R2) C
The voltage gain Av of the circuit shown below is

asy
En
gin
(A) Av . 200 eer
(B) Av . 100
(C) Av . 20
ing
(D) Av . 10

2011
.ne ONE MARK

Q. 15

t
In the circuit shown below, capacitors C1 and C2 are very large and are shorts at
the input frequency. vi is a small signal input. The gain magnitude vo at 10 M
rad/s is
vi

(A) maximum (B) minimum


(C) unity (D) zero

Downloaded From : www.EasyEngineering.net


Downloaded From : www.EasyEngineering.net
GATE SOLVED PAPER - EC ANALOG ELECTRONIC

Q. 16 The circuit below implements a filter between the input current ii and the output
voltage vo . Assume that the op-amp is ideal. The filter implemented is a

(A) low pass filter (B) band pass filter


(C) band stop filter (D) high pass filter

ww 2011 TWO MARKS

w.E
Q. 17 In the circuit shown below, for the MOS transistors, mn Cox = 100 mA/V 2 and the
threshold voltage VT = 1 V . The voltage Vx at the source of the upper transistor is

asy
En
gin
(A) 1 V
(C) 3 V
(B) 2 V
eer
(D) 3.67 V

Q. 18
ing
For a BJT, the common base current gain a = 0.98 and the collector base
junction reverse bias saturation current ICO = 0.6 mA . This BJT is connected in

.ne
the common emitter mode and operated in the active region with a base drive
current IB = 20 mA . The collector current IC for this mode of operation is

Q. 19
(A) 0.98 mA
(C) 1.0 mA
(B) 0.99 mA
(D) 1.01 mA t
For the BJT, Q1 in the circuit shown below, b = 3, VBEon = 0.7 V, VCEsat = 0.7 V
. The switch is initially closed. At time t = 0 , the switch is opened. The time t at
which Q1 leaves the active region is

Downloaded From : www.EasyEngineering.net


Downloaded From : www.EasyEngineering.net
GATE SOLVED PAPER - EC ANALOG ELECTRONIC

(A) 10 ms (B) 25 ms
(C) 50 ms (D) 100 ms

Statement for Linked Answer Questions: 20 and 21


In the circuit shown below, assume that the voltage drop across a forward biased
diode is 0.7 V. The thermal voltage Vt = kT/q = 25 mV . The small signal input
vi = Vp cos ^wt h where Vp = 100 mV.

ww
w.E
Q. 20 The bias current IDC through the diodes is
(A) 1 mA
(C) 1.5 mA
asy (B) 1.28 mA
(D) 2 mA

Q. 21

En
The ac output voltage vac is
(A) 0.25 cos ^wt h mV (B) 1 cos (wt) mV
(C) 2 cos (wt) mV
gin (D) 22 cos (wt) mV

2010
eer ONE MARK

Q. 22

ing
The amplifier circuit shown below uses a silicon transistor. The capacitors CC
and CE can be assumed to be short at signal frequency and effect of output

.ne
resistance r0 can be ignored. If CE is disconnected from the circuit, which one of
the following statements is true

(A) The input resistance Ri increases and magnitude of voltage gainAV


decreases
(B) The input resistance Ri decreases and magnitude of voltage gain AV
increases
(C) Both input resistance Ri and magnitude of voltage gain AV decreases

Downloaded From : www.EasyEngineering.net


Downloaded From : www.EasyEngineering.net
GATE SOLVED PAPER - EC ANALOG ELECTRONIC

(D) Both input resistance Ri and the magnitude of voltage gain AV increases

Q. 23 In the silicon BJT circuit shown below, assume that the emitter area of transistor
Q1 is half that of transistor Q2

The value of current Io is approximately

ww (A) 0.5 mA
(C) 9.3 mA
(B) 2 mA
(D) 15 mA

w.E
Q. 24 Assuming the OP-AMP to be ideal, the voltage gain of the amplifier shown below
is

asy
En
gin
(A) - R2
R1 R1eer
(B) - R 3
R || R 3
(C) - 2
R1 R1 ing
(D) -b R2 + R 3 l

2010 .ne TWO MARKS

Common Data For Q. 25 and 26


t
Consider the common emitter amplifier shown below with the following circuit
parameters:
b = 100, gm = 0.3861 A/V, r0 = 259 W, RS = 1 kW, RB = 93 kW,
RC = 250 kW, RL = 1 kW, C1 = 3 and C2 = 4.7 mF

Downloaded From : www.EasyEngineering.net


Downloaded From : www.EasyEngineering.net
GATE SOLVED PAPER - EC ANALOG ELECTRONIC

Q. 25 The resistance seen by the source vS is


(A) 258 W (B) 1258 W
(C) 93 kW (D) 3

Q. 26 The lower cut-off frequency due to C2 is


(A) 33.9 Hz
(B) 27.1 Hz
(C) 13.6 Hz
(D) 16.9 Hz

Q. 27 The transfer characteristic for the precision rectifier circuit shown below is
(assume ideal OP-AMP and practical diodes)

ww
w.E
asy
En
gin
eer
ing
.ne
t
2009 TWO MARKS

Q. 28 In the circuit below, the diode is ideal. The voltage V is given by

(A) min (Vi, 1) (B) max (Vi, 1)


(C) min (- Vi, 1) (D) max (- Vi, 1)

Downloaded From : www.EasyEngineering.net


Downloaded From : www.EasyEngineering.net
GATE SOLVED PAPER - EC ANALOG ELECTRONIC

Q. 29 In the following a stable multivibrator circuit, which properties of v0 (t) depend


on R2 ?

(A) Only the frequency


(B) Only the amplitude

ww (C) Both the amplitude and the frequency


(D) Neither the amplitude nor the frequency

w.E Statement for Linked Answer Question 30 and 31

asy
Consider for CMOS circuit shown, where the gate voltage v0 of the n-MOSFET is
increased from zero, while the gate voltage of the p -MOSFET is kept constant

En
at 3 V. Assume, that, for both transistors, the magnitude of the threshold voltage
is 1 V and the product of the trans-conductance parameter is 1mA. V - 2

gin
eer
ing
Q. 30 .ne
For small increase in VG beyond 1V, which of the following gives the correct
description of the region of operation of each MOSFET
(A) Both the MOSFETs are in saturation region
(B) Both the MOSFETs are in triode region
t
(C) n-MOSFETs is in triode and p -MOSFET is in saturation region
(D) n- MOSFET is in saturation and p -MOSFET is in triode region

Q. 31 Estimate the output voltage V0 for VG = 1.5 V. [Hints : Use the appropriate
current-voltage equation for each MOSFET, based on the answer to Q.4.16]
(A) 4 - 1 (B) 4 + 1
2 2
(C) 4 - 3 (D) 4 + 3
2 2

Q. 32 In the circuit shown below, the op-amp is ideal, the transistor has VBE = 0.6 V
and b = 150 . Decide whether the feedback in the circuit is positive or negative
and determine the voltage V at the output of the op-amp.

Downloaded From : www.EasyEngineering.net


Downloaded From : www.EasyEngineering.net
GATE SOLVED PAPER - EC ANALOG ELECTRONIC

(A) Positive feedback, V = 10 V


(B) Positive feedback, V = 0 V
(C) Negative feedback, V = 5 V
(D) Negative feedback, V = 2 V
Q. 33 A small signal source Vi (t) = A cos 20t + B sin 106 t is applied to a transistor

ww amplifier as shown below. The transistor has b = 150 and hie = 3W . Which
expression best approximate V0 (t)

w.E
asy
En
gin
(A) V0 (t) =- 1500 (A cos 20t + B sin 106 t) eer
(B) V0 (t) = - 1500( A cos 20t + B sin 106 t)
(C) V0 (t) =- 1500B sin 106 t ing
(D) V0 (t) =- 150B sin 106 t
.ne
Q. 34
2008
t ONE MARK

In the following limiter circuit, an input voltage Vi = 10 sin 100pt is applied.


Assume that the diode drop is 0.7 V when it is forward biased. When it is forward
biased. The zener breakdown voltage is 6.8 V
The maximum and minimum values of the output voltage respectively are

(A) 6.1 V, - 0.7 V (B) 0.7 V, - 7.5 V


(C) 7.5 V, - 0.7 V (D) 7.5 V, - 7.5 V

Downloaded From : www.EasyEngineering.net


Downloaded From : www.EasyEngineering.net
GATE SOLVED PAPER - EC ANALOG ELECTRONIC

2008 TWO MARSK

Q. 35 For the circuit shown in the following figure, transistor M1 and M2 are identical
NMOS transistors. Assume the M2 is in saturation and the output is unloaded.

The current Ix is related to Ibias as

ww (A) Ix = Ibias + Is
V
(C) Ix = Ibias - cVDD - out m
RE
(B) Ix = Ibias
(D) Ix = Ibias - Is

w.E
Q. 36 Consider the following circuit using an ideal OPAMP. The I-V characteristic of
the diode is described by the relation I = I 0 _eV - 1i where VT = 25 mV, I0 = 1m A
t
V

and V is the voltage across the diode (taken as positive for forward bias). For an

asy
input voltage Vi =- 1 V , the output voltage V0 is

En
gin
(A) 0 V (B) 0.1 V eer
(C) 0.7 V (D) 1.1 V
ing
Q. 37 The OPAMP circuit shown above represents a

.ne
t
(A) high pass filter
(B) low pass filter
(C) band pass filter
(D) band reject filter

Q. 38 Two identical NMOS transistors M1 and M2 are connected as shown below. Vbias
is chosen so that both transistors are in saturation. The equivalent gm of the pair
is defied to be 2Iout at constant Vout
2Vi
The equivalent gm of the pair is

Downloaded From : www.EasyEngineering.net


Downloaded From : www.EasyEngineering.net
GATE SOLVED PAPER - EC ANALOG ELECTRONIC

(A) the sum of individual gm ' s of the transistors


(B) the product of individual gm ’s of the transistors
(C) nearly equal to the gm of M1
g
(D) nearly equal to m of M2
g0
Q. 39 Consider the Schmidt trigger circuit shown below
A triangular wave which goes from -12 to 12 V is applied to the inverting input

ww of OPMAP. Assume that the output of the OPAMP swings from +15 V to -15 V.
The voltage at the non-inverting input switches between

w.E
asy
En
(A) - 12V to +12 V
gin (B) -7.5 V to 7.5 V
(C) -5 V to +5 V

eer
(D) 0 V and 5 V

Statement for Linked Answer Question 40 and 41


ing
In the following transistor circuit, VBE = 0.7 V, r3 = 25 mV/IE , and b and all the
capacitances are very large
.ne
t
Q. 40 The value of DC current IE is
(A) 1 mA (B) 2 mA
(C) 5 mA (D) 10 mA

Q. 41 The mid-band voltage gain of the amplifier is approximately


(A) -180 (B) -120
(C) -90 (D) -60

Downloaded From : www.EasyEngineering.net


Downloaded From : www.EasyEngineering.net
GATE SOLVED PAPER - EC ANALOG ELECTRONIC

2007 ONE MARK

Q. 42 The correct full wave rectifier circuit is

ww
Q. 43 In a transconductance amplifier, it is desirable to have
(A) a large input resistance and a large output resistance

w.E (B) a large input resistance and a small output resistance


(C) a small input resistance and a large output resistance

asy
(D) a small input resistance and a small output resistance

Q. 44
2007

En
For the Op-Amp circuit shown in the figure, V0 is
TWO MARKS

gin
eer
ing
(A) -2 V (B) -1 V .ne
Q. 45
(C) -0.5 V (D) 0.5 V
t
For the BJT circuit shown, assume that the b of the transistor is very large and
VBE = 0.7 V. The mode of operation of the BJT is

(A) cut-off (B) saturation


(C) normal active (D) reverse active

Q. 46 In the Op-Amp circuit shown, assume that the diode current follows the equation
I = Is exp (V/VT ). For Vi = 2V, V0 = V01, and for Vi = 4V, V0 = V02 .
The relationship between V01 and V02 is

Downloaded From : www.EasyEngineering.net


Downloaded From : www.EasyEngineering.net
GATE SOLVED PAPER - EC ANALOG ELECTRONIC

(A) V02 = 2 Vo1 (B) Vo2 = e2 Vo1


(C) Vo2 = Vo1 1n2 (D) Vo1 - Vo2 = VT 1n2

Q. 47 In the CMOS inverter circuit shown, if the trans conductance parameters of the
NMOS and PMOS transistors are
W
kn = kp = mn Cox Wn = mCox p = 40mA/V2
Ln Lp
and their threshold voltages ae VTHn = VTHp = 1 V the current I is

ww
w.E
(A) 0 A asy (B) 25 mA
(C) 45 mA
En (D) 90 mA

Q. 48

gin
For the Zener diode shown in the figure, the Zener voltage at knee is 7 V, the knee
current is negligible and the Zener dynamic resistance is 10 W. If the input voltage
(Vi) range is from 10 to 16 V, the output voltage (V0) ranges from

eer
ing
.ne
(A) 7.00 to 7.29 V
(C) 7.14 to 7.43 V t
(B) 7.14 to 7.29 V
(D) 7.29 to 7.43 V

Statement for Linked Answer Questions 49 and 50:


Consider the Op-Amp circuit shown in the figure.

Q. 49 The transfer function V0 (s)/ Vi (s) is


(A) 1 - sRC (B) 1 + sRC
1 + sRC 1 - sRC

Downloaded From : www.EasyEngineering.net


Downloaded From : www.EasyEngineering.net
GATE SOLVED PAPER - EC ANALOG ELECTRONIC

(C) 1 (D) 1
1 - sRC 1 + sRC

Q. 50 If Vi = V1 sin (wt) and V0 = V2 sin (wt + f), then the minimum and maximum values
of f (in radians) are respectively
(A) - p and p (B) 0 and p
2 2 2
(C) - p and 0 p
(D) - and 0
2

2006 ONE MARK

Q. 51 The input impedance (Zi) and the output impedance (Z0) of an ideal trans-
conductance (voltage controlled current source) amplifier are
(A) Zi = 0, Z0 = 0

ww (B) Zi = 0, Z0 = 3
(C) Zi = 3, Z0 = 0

w.E
Q. 52
(D) Zi = 3, Z0 = 3

An n-channel depletion MOSFET has following two points on its ID - VGs curve:
(i) VGS = 0 at ID = 12 mA and

asy
(ii) VGS =- 6 Volts at ID = 0 mA
Which of the following Q point will given the highest trans conductance gain for
small signals?
(A) VGS =- 6 Volts En (B) VGS =- 3 Volts
(C) VGS = 0 Volts
gin
(D) VGS = 3 Volts

2006
eer TWO MARKS

Q. 53

ing
For the circuit shown in the following figure, the capacitor C is initially uncharged.
At t = 0 the switch S is closed. The Vc across the capacitor at t = 1 millisecond is

.ne
In the figure shown above, the OP-AMP is supplied with !15V .

t
(A) 0 Volt (B) 6.3 Volt
(C) 9.45 Volts (D) 10 Volts

Q. 54 For the circuit shown below, assume that the zener diode is ideal with a breakdown
voltage of 6 volts. The waveform observed across R is

Downloaded From : www.EasyEngineering.net


Downloaded From : www.EasyEngineering.net
GATE SOLVED PAPER - EC ANALOG ELECTRONIC

ww Common Data For Q. 55 to 57


In the transistor amplifier circuit shown in the figure below, the transistor has

w.E the following parameters:


bDC = 60 , VBE = 0.7V, hie " 3
The capacitance CC can be assumed to be infinite.

asy
In the figure above, the ground has been shown by the symbol 4

En
gin
eer
Q. 55 ing
Under the DC conditions, the collector-or-emitter voltage drop is
(A) 4.8 Volts
(C) 6.0 Volts
(B) 5.3 Volts
(D) 6.6 Volts .ne
Q. 56
(A) increases by less than or equal to 10%
t
If bDC is increased by 10%, the collector-to-emitter voltage drop

(B) decreases by less than or equal to 10%


(C) increase by more than 10%
(D) decreases by more than 10%

Q. 57 The small-signal gain of the amplifier vc is


vs
(A) -10 (B) -5.3
(C) 5.3 (D) 10

Common Data For Q. 58 and 59


A regulated power supply, shown in figure below, has an unregulated input (UR)
of 15 Volts and generates a regulated output Vout . Use the component values
shown in the figure.

Downloaded From : www.EasyEngineering.net


Downloaded From : www.EasyEngineering.net
GATE SOLVED PAPER - EC ANALOG ELECTRONIC

Q. 58 The power dissipation across the transistor Q1 shown in the figure is


(A) 4.8 Watts (B) 5.0 Watts
(C) 5.4 Watts (D) 6.0 Watts

If the unregulated voltage increases by 20%, the power dissipation across the

ww
Q. 59
transistor Q1
(A) increases by 20% (B) increases by 50%

w.E (C) remains unchanged (D) decreases by 20%

Q. 60
2005

asy
The input resistance Ri of the amplifier shown in the figure is
ONE MARK

En
gin
eer
(A) 30 kW
4
(B) 10 kW
ing
Q. 61
(C) 40 kW (D) infinite

The effect of current shunt feedback in an amplifier is to .ne


(B) increases both input and output resistance
(C) decrease both input and output resistance
t
(A) increase the input resistance and decrease the output resistance

(D) decrease the input resistance and increase the output resistance

Q. 62 The cascade amplifier is a multistage configuration of


(A) CC - CB (B) CE - CB
(C) CB - CC (D) CE - CC

2005 TWO MARKS

Q. 63 In an ideal differential amplifier shown in the figure, a large value of (RE ).


(A) increase both the differential and common - mode gains.
(B) increases the common mode gain only.
(C) decreases the differential mode gain only.
(D) decreases the common mode gain only.

Downloaded From : www.EasyEngineering.net


Downloaded From : www.EasyEngineering.net
GATE SOLVED PAPER - EC ANALOG ELECTRONIC

Q. 64 For an npn transistor connected as shown in figure VBE = 0.7 volts. Given that
reverse saturation current of the junction at room temperature 300 K is 10 - 13 A,
the emitter current is

(A) 30 mA (B) 39 mA
(C) 49 mA (D) 20 mA

Q. 65 The voltage e0 is indicated in the figure has been measured by an ideal voltmeter.
Which of the following can be calculated ?

ww
w.E
asy
En
(A) Bias current of the inverting input only
(B) Bias current of the inverting and non-inverting inputs only
(C) Input offset current only
gin
Q. 66 eer
(D) Both the bias currents and the input offset current

The Op-amp circuit shown in the figure is filter. The type of filter and its cut. Off
frequency are respectively
ing
.ne
t
(A) high pass, 1000 rad/sec. (B) Low pass, 1000 rad/sec
(C) high pass, 1000 rad/sec (D) low pass, 10000 rad/sec

Q. 67 The circuit using a BJT with b = 50 and VBE = 0.7V is shown in the figure. The
base current IB and collector voltage by VC and respectively

Downloaded From : www.EasyEngineering.net


Downloaded From : www.EasyEngineering.net
GATE SOLVED PAPER - EC ANALOG ELECTRONIC

(A) 43 mA and 11.4 Volts (B) 40 mA and 16 Volts


(C) 45 mA and 11 Volts (D) 50 mA and 10 Volts

Q. 68 The Zener diode in the regulator circuit shown in the figure has a Zener voltage
of 5.8 volts and a zener knee current of 0.5 mA. The maximum load current
drawn from this current ensuring proper functioning over the input voltage range
between 20 and 30 volts, is

(A) 23.7 mA (B) 14.2 mA

ww
Q. 69
(C) 13.7 mA (D) 24.2 mA

Both transistors T1 and T2 show in the figure, have a b = 100 , threshold voltage of

w.E 1 Volts. The device parameters K1 and K2 of T1 and T2 are, respectively, 36 mA/V2
and 9 mA/V 2 . The output voltage Vo i s

asy
En
(A) 1 V gin(B) 2 V
(C) 3 V (D) 4 V
eer
Common Data For Q. 70 to 72 ing
Given, rd = 20kW , IDSS = 10 mA, Vp =- 8 V
.ne
t

Q. 70 Zi and Z0 of the circuit are respectively


(A) 2 MW and 2 kW (B) 2 MW and 20 kW
11
(C) infinity and 2 MW (D) infinity and 20 kW
11
Q. 71 ID and VDS under DC conditions are respectively
(A) 5.625 mA and 8.75 V (B) 1.875 mA and 5.00 V
(C) 4.500 mA and 11.00 V (D) 6.250 mA and 7.50 V

Downloaded From : www.EasyEngineering.net


Downloaded From : www.EasyEngineering.net
GATE SOLVED PAPER - EC ANALOG ELECTRONIC

Q. 72 Transconductance in milli-Siemens (mS) and voltage gain of the amplifier are


respectively
(A) 1.875 mS and 3.41
(B) 1.875 ms and -3.41
(C) 3.3 mS and -6
(D) 3.3 mS and 6

Q. 73 Given the ideal operational amplifier circuit shown in the figure indicate the
correct transfer characteristics assuming ideal diodes with zero cut-in voltage.

ww
w.E
asy
En
gin
eer
ing
.ne
t
2004 ONE MARK

Q. 74 An ideal op-amp is an ideal


(A) voltage controlled current source
(B) voltage controlled voltage source
(C) current controlled current source
(D) current controlled voltage source

Q. 75 Voltage series feedback (also called series-shunt feedback) results in


(A) increase in both input and output impedances
(B) decrease in both input and output impedances
(C) increase in input impedance and decrease in output impedance
(D) decrease in input impedance and increase in output impedance

Downloaded From : www.EasyEngineering.net


Downloaded From : www.EasyEngineering.net
GATE SOLVED PAPER - EC ANALOG ELECTRONIC

Q. 76 The circuit in the figure is a

(A) low-pass filter (B) high-pass filter


(C) band-pass filter (D) band-reject filter

2004 TWO MARKS

ww
Q. 77 A bipolar transistor is operating in the active region with a collector current of 1
mA. Assuming that the b of the transistor is 100 and the thermal voltage (VT ) is
25 mV, the transconductance (gm) and the input resistance (rp) of the transistor

w.E in the common emitter configuration, are


(A) gm = 25 mA/V and rp = 15.625 kW (B) gm = 40 mA/V and rp = 4.0 kW

Q. 78 asy
(C) gm = 25 mA/V and rp = 2.5 k W (D) gm = 40 mA/V and rp = 2.5 kW

The value of C required for sinusoidal oscillations of frequency 1 kHz in the

En
circuit of the figure is

gin
eer
ing
.ne
(A) 1 mF

(C)
2p
1
2p 6
mF
(B) 2p mF

(D) 2p 6 mF
t
Q. 79 In the op-amp circuit given in the figure, the load current iL is

(A) - Vs (B) Vs
R2 R2

Downloaded From : www.EasyEngineering.net


Downloaded From : www.EasyEngineering.net
GATE SOLVED PAPER - EC ANALOG ELECTRONIC

(C) - Vs (D) Vs
RL R1

Q. 80 In the voltage regulator shown in the figure, the load current can vary from 100
mA to 500 mA. Assuming that the Zener diode is ideal (i.e., the Zener knee
current is negligibly small and Zener resistance is zero in the breakdown region),
the value of R is

(A) 7 W (B) 70 W

ww
Q. 81
(C) 70 W
3
(D) 14 W

In a full-wave rectifier using two ideal diodes, Vdc and Vm are the dc and peak

w.E values of the voltage respectively across a resistive load. If PIV is the peak inverse
voltage of the diode, then the appropriate relationships for this rectifier are
(A) Vdc = Vm , PIV = 2Vm (B) Idc = 2 Vm , PIV = 2Vm
p
asy
(C) Vdc = 2 Vm , PIV = Vm
p
(D) Vdc Vm , PIV = Vm

Q. 82
p

En p
Assume that the b of transistor is extremely large and VBE = 0.7V, IC and VCE in
the circuit shown in the figure
gin
eer
ing
.ne
(A) IC = 1 mA, VCE = 4.7 V
(C) IC = 1 mA, VCE = 2.5 V
t
(B) IC = 0.5 mA, VCE = 3.75 V
(D) IC = 0.5 mA, VCE = 3.9 V

2003 ONE MARK

Q. 83 Choose the correct match for input resistance of various amplifier configurations
shown below :
Configuration Input resistance
CB : Common Base LO : Low
CC : Common Collector MO : Moderate
CE : Common Emitter HI : High
(A) CB - LO, CC - MO, CE - HI
(B) CB - LO, CC - HI, CE - MO
(C) CB - MO, CC - HI, CE - LO
(D) CB - HI, CC - LO, CE - MO

Downloaded From : www.EasyEngineering.net


Downloaded From : www.EasyEngineering.net
GATE SOLVED PAPER - EC ANALOG ELECTRONIC

Q. 84 The circuit shown in the figure is best described as a

(A) bridge rectifier (B) ring modulator


(C) frequency discriminator (D) voltage double

Q. 85 If the input to the ideal comparators shown in the figure is a sinusoidal signal of 8
V (peak to peak) without any DC component, then the output of the comparators
has a duty cycle of

ww
w.E (A) 1/2
(C) 1/6
(B) 1/3
(D) 1/2

Q. 86

asy
If the differential voltage gain and the common mode voltage gain of a differential
amplifier are 48 dB and 2 dB respectively, then common mode rejection ratio is
(A) 23 dB (B) 25 dB
(C) 46 dB
En (D) 50 dB

Q. 87
(A) internal capacitances of the devicegin
Generally, the gain of a transistor amplifier falls at high frequencies due to the

(B) coupling capacitor at the input


(C) skin effect eer
(D) coupling capacitor at the output
ing
2003
.ne TWO MARKS

Q. 88

t
An amplifier without feedback has a voltage gain of 50, input resistance of 1 k
W and output resistance of 2.5 kW. The input resistance of the current-shunt
negative feedback amplifier using the above amplifier with a feedback factor of
0.2, is
(A) 1 kW (B) 1 kW
11 5
(C) 5 kW (D) 11 kW

Q. 89 In the amplifier circuit shown in the figure, the values of R1 and R2 are such that
the transistor is operating at VCE = 3 V and IC = 1.5 mA when its b is 150. For
a transistor with b of 200, the operating point (VCE , IC ) is

Downloaded From : www.EasyEngineering.net


Downloaded From : www.EasyEngineering.net
GATE SOLVED PAPER - EC ANALOG ELECTRONIC

(A) (2 V, 2 mA) (B) (3 V, 2 mA)


(C) (4 V, 2 mA) (D) (4 V, 1 mA)

Q. 90 The oscillator circuit shown in the figure has an ideal inverting amplifier. Its
frequency of oscillation (in Hz) is

(A) 1 (B) 1

ww (C)
(2p 6 RC)
1
( 6 RC)
(D)
(2pRC)
6
(2pRC)

w.E
Q. 91 The output voltage of the regulated power supply shown in the figure is

asy
En
gin
(A) 3 V
eer
(B) 6 V
(C) 9 V (D) 12 V
ing
Q. 92

.ne
If the op-amp in the figure is ideal, the output voltage Vout will be equal to

t
(A) 1 V (B) 6 V
(C) 14 V (D) 17 V

Q. 93 Three identical amplifiers with each one having a voltage gain of 50, input
resistance of 1 kW and output resistance of 250 W are cascaded. The opened
circuit voltages gain of the combined amplifier is
(A) 49 dB (B) 51 dB
(C) 98 dB (D) 102 dB

Downloaded From : www.EasyEngineering.net


Downloaded From : www.EasyEngineering.net
GATE SOLVED PAPER - EC ANALOG ELECTRONIC

Q. 94 An ideal sawtooth voltages waveform of frequency of 500 Hz and amplitude 3 V


is generated by charging a capacitor of 2 mF in every cycle. The charging requires
(A) Constant voltage source of 3 V for 1 ms
(B) Constant voltage source of 3 V for 2 ms
(C) Constant voltage source of 1 mA for 1 ms
(D) Constant voltage source of 3 mA for 2 ms

2002 ONE MARK

Q. 95 In a negative feedback amplifier using voltage-series (i.e. voltage-sampling, series


mixing) feedback.
(A) Ri decreases and R0 decreases (B) Ri decreases and R0 increases
(C) Ri increases and R0 decreases (D) Ri increases and R0 increases

ww
Q. 96
(Ri and R0 denote the input and output resistance respectively)

A 741-type opamp has a gain-bandwidth product of 1 MHz. A non-inverting

w.E amplifier suing this opamp and having a voltage gain of 20 dB will exhibit a -3
dB bandwidth of
(A) 50 kHz (B) 100 kHz
(C) 1000
17
kHz
asy (D) 1000 kHz
7.07
Q. 97

En
Three identical RC-coupled transistor amplifiers are cascaded. If each of the
amplifiers has a frequency response as shown in the figure, the overall frequency
response is as given in
gin
eer
ing
.ne
t

Downloaded From : www.EasyEngineering.net


Downloaded From : www.EasyEngineering.net
GATE SOLVED PAPER - EC ANALOG ELECTRONIC

2002 TWO MARKS

Q. 98 The circuit in the figure employs positive feedback and is intended to generate
V (f) 1
sinusoidal oscillation. If at a frequency f0, B (f) = 3 f = +0c, then to sustain
V0 (f) 6
oscillation at this frequency

(A) R2 = 5R1 (B) R2 = 6R1

ww (C) R2 = R1
6
(D) R2 = R1
5

w.E
Q. 99 An amplifier using an opamp with a slew-rate SR = 1 V/m sec has a gain of 40
dB. If this amplifier has to faithfully amplify sinusoidal signals from dc to 20 kHz
without introducing any slew-rate induced distortion, then the input signal level

(A) 795 mV asy


must not exceed.
(B) 395 mV
(C) 79.5 mV
En (D) 39.5 mV

Q. 100

gin
A zener diode regulator in the figure is to be designed to meet the specifications:
IL = 10 mA V0 = 10 V and Vin varies from 30 V to 50 V. The zener diode has
Vz = 10 V and Izk (knee current) =1 mA. For satisfactory operation

eer
ing
(A) R # 1800W .ne
(B) 2000W # R # 2200W

Q. 101
(C) 3700W # R # 4000W (D) R $ 4000W
t
The voltage gain Av = v0 of the JFET amplifier shown in the figure is IDSS = 10
vt
mA Vp =- 5 V(Assume C1, C2 and Cs to be very large

(A) +16 (B) -16


(C) +8 (D) -6

Downloaded From : www.EasyEngineering.net


Downloaded From : www.EasyEngineering.net
GATE SOLVED PAPER - EC ANALOG ELECTRONIC

2001 ONE MARK

Q. 102 The current gain of a BJT is


gm
(A) gm r0 (B)
r
g
(C) gm rp (D) m
rp

Q. 103 Thee ideal OP-AMP has the following characteristics.


(A) Ri = 3, A = 3, R0 = 0
(B) Ri = 0, A = 3, R0 = 0
(C) Ri = 3, A = 3, R0 = 3
(D) Ri = 0, A = 3, R0 = 3

Q. 104 Consider the following two statements :

ww Statement 1 :
A stable multi vibrator can be used for generating square wave.
Statement 2:

w.E Bistable multi vibrator can be used for storing binary information.
(A) Only statement 1 is correct

asy
(B) Only statement 2 is correct
(C) Both the statements 1 and 2 are correct

En
(D) Both the statements 1 and 2 are incorrect

2001
gin TWO MARKS

An npn BJT has gm = 38 mA/V, C m = 10-14 F, C p = 4 # 10-13 F, and DC current


Q. 105
gain b0 = 90 . For this transistor fT and fb are
(A) fT = 1.64 # 108 Hz and fb = 1.47 # 1010 Hz eer
(B) fT = 1.47 # 1010 Hz and fb = 1.64 # 108 Hz
(C) fT = 1.33 # 1012 Hz and fb = 1.47 # 1010 Hz
ing
(D) fT = 1.47 # 1010 Hz and fb = 1.33 # 1012 Hz
.ne
Q. 106

t
The transistor shunt regulator shown in the figure has a regulated output voltage
of 10 V, when the input varies from 20 V to 30 V. The relevant parameters for
the zener diode and the transistor are : Vz = 9.5 , VBE = 0.3 V, b = 99 , Neglect the
current through RB . Then the maximum power dissipated in the zener diode (Pz )
and the transistor (PT ) are

(A) Pz = 75 mW, PT = 7.9 W


(B) Pz = 85 mW, PT = 8.9 W
(C) Pz = 95 mW, PT = 9.9 W
(D) Pz = 115 mW, PT = 11.9 W

Downloaded From : www.EasyEngineering.net


Downloaded From : www.EasyEngineering.net
GATE SOLVED PAPER - EC ANALOG ELECTRONIC

Q. 107 The oscillator circuit shown in the figure is

(A) Hartely oscillator with foscillation = 79.6 MHz


(B) Colpitts oscillator with foscillation = 50.3 MHz

ww (C) Hartley oscillator with foscillation = 159.2 MHz


(D) Colpitts oscillator with foscillation = 159.3 MHz

w.E
Q. 108 The inverting OP-AMP shown in the figure has an open-loop gain of 100.

asy
En
The closed-loop gain V0 is gin
(A) - 8
(C) - 10
Vs

eer
(B) - 9
(D) - 11

Q. 109
ing
In the figure assume the OP-AMPs to be ideal. The output v0 of the circuit is

.ne
t
t
(A) 10 cos (100t) (B) 10 #0 cos (100t) dt
t
(C) 10 - 4 #0 cos (100t) dt (D) 10 - 4 d cos (100t)
dt

2000 ONE MARK

Q. 110 Introducing a resistor in the emitter of a common amplifier stabilizes the dc


operating point against variations in
(A) only the temperature (B) only the b of the transistor
(C) both temperature and b (D) none of the above

Q. 111 In the differential amplifier of the figure, if the source resistance of the current

Downloaded From : www.EasyEngineering.net


Downloaded From : www.EasyEngineering.net
GATE SOLVED PAPER - EC ANALOG ELECTRONIC

source IEE is infinite, then the common-mode gain is

(A) zero (B) infinite


(C) indeterminate (D) Vin1 + Vin2

ww
Q. 112 In the circuit of the figure, V0 is
2VT

w.E
asy
(A) -1 V
(C) +1 V En (B) 2 V
(D) +15 V

Q. 113
(A) transistor capacitances gin
The current gain of a bipolar transistor drops at high frequencies because of
(B) high current effects in the base
(C) parasitic inductive elements
eer
(D) the Early effect

Q. 114 If the op-amp in the figure, is ideal, then v0 is


ing
.ne
(A) zero (B) (V1 - V2) sin wt
t
(C) - (V1 + V2) sin wt (D) (V1 + V2) sin wt

Q. 115 The configuration of the figure is a

Downloaded From : www.EasyEngineering.net


Downloaded From : www.EasyEngineering.net
GATE SOLVED PAPER - EC ANALOG ELECTRONIC

(A) precision integrator (B) Hartely oscillator


(C) Butterworth high pass filter (D) Wien-bridge oscillator

Q. 116 Assume that the op-amp of the figure is ideal. If vi is a triangular wave, then v0
will be

(A) square wave (B) triangular wave


(C) parabolic wave (D) sine wave

ww
Q. 117 The most commonly used amplifier is sample and hold circuits is
(A) a unity gain inverting amplifier

w.E (B) a unity gain non-inverting amplifier


(C) an inverting amplifier with a gain of 10
(D) an inverting amplifier with a gain of 100

2000
asy TWO MARKS

Q. 118
En
In the circuit of figure, assume that the transistor is in the active region. It has a

gin
large b and its base-emitter voltage is 0.7 V. The value of Ic is

eer
ing
.ne
(A) Indeterminate since Rc is not given (B) 1 mA
t
(C) 5 mA (D) 10 mA

Q. 119 If the op-amp in the figure has an input offset voltage of 5 mV and an open-loop
voltage gain of 10000, then v0 will be

(A) 0 V (B) 5 mV
(C) + 15 V or -15 V (D) +50 V or -50 V

Downloaded From : www.EasyEngineering.net


Downloaded From : www.EasyEngineering.net
GATE SOLVED PAPER - EC ANALOG ELECTRONIC

1999 ONE MARK

Q. 120 The first dominant pole encountered in the frequency response of a compensated
op-amp is approximately at
(A) 5 Hz (B) 10 kHz
(C) 1 MHz (D) 100 MHz

Q. 121 Negative feedback in an amplifier


(A) reduces gain
(B) increases frequency and phase distortions
(C) reduces bandwidth
(D) increases noise

Q. 122 In the cascade amplifier shown in the given figure, if the common-emitter
stage (Q1) has a transconductance gm1 , and the common base stage (Q2) has

ww a transconductance gm2 , then the overall transconductance g (= i 0 /vi) of the


cascade amplifier is

w.E
asy
(A) gm1 En (B) gm2
g
(C) m1
2
gin g
(D) m2
2
Q. 123 Crossover distortion behavior is characteristic of
(A) Class A output stage
eer
(B) Class B output stage
(C) Class AB output stage
ing
(D) Common-base output stage

Q. 124
1999
.ne TWO MARK

An amplifier has an open-loop gain of 100, an input impedance of 1 kW,and an

t
output impedance of 100 W. A feedback network with a feedback factor of 0.99 is
connected to the amplifier in a voltage series feedback mode. The new input and
output impedances, respectively, are
(A) 10 W and 1W (B) 10 W and 10 kW
(C) 100 kW and 1 W (D) 100 kW and 1 kW

Q. 125 A dc power supply has a no-load voltage of 30 V, and a full-load voltage of


25 V at a full-load current of 1 A. Its output resistance and load regulation,
respectively, are
(A) 5 W and 20% (B) 25 W and 20%
(C) 5 W and 16.7% (D) 25 W and 16.7%

1998 ONE MARK

Q. 126 The circuit of the figure is an example of feedback of the following type

Downloaded From : www.EasyEngineering.net


Downloaded From : www.EasyEngineering.net
GATE SOLVED PAPER - EC ANALOG ELECTRONIC

(A) current series (B) current shunt


(C) voltage series (D) voltage shunt

Q. 127 In a differential amplifier, CMRR can be improved by using an increased


(A) emitter resistance (B) collector resistance
(C) power supply voltages (D) source resistance

Q. 128 From a measurement of the rise time of the output pulse of an amplifier whose is

ww a small amplitude square wave, one can estimate the following parameter of the
amplifier
(A) gain-bandwidth product (B) slow rate

w.E
Q. 129
(C) upper 3–dB frequency (D) lower 3–dB frequency

The emitter coupled pair of BJT’s given a linear transfer relation between the

asy
differential output voltage and the differential output voltage and the differential
input voltage Vid is less a times the thermal voltage, where a is
(A) 4
(C) 2 En (B) 3
(D) 1

Q. 130
gin
In a shunt-shunt negative feedback amplifier, as compared to the basic amplifier
(A) both, input and output impedances,decrease

eer
(B) input impedance decreases but output impedance increases
(C) input impedance increase but output
(D) both input and output impedances increases. ing
1998 .ne TWO MARKS

Q. 131
t
A multistage amplifier has a low-pass response with three real poles at
s =- w1 - w2 and w3 . The approximate overall bandwidth B of the amplifier will
be given by
(A) B = w1 + w2 + w3 (B) 1 = 1 + 1 + 1
B w1 w2 w3
(C) B = (w1 + w2 + w3) 1/3 (D) B = w12 + w22 + w23

Q. 132 One input terminal of high gain comparator circuit is connected to ground and a
sinusoidal voltage is applied to the other input. The output of comparator will be
(A) a sinusoid (B) a full rectified sinusoid
(C) a half rectified sinusoid (D) a square wave

Q. 133 In a series regulated power supply circuit, the voltage gain Av of the ‘pass’
transistor satisfies the condition
(A) Av " 3 (B) 1 << Av < 3
(C) Av . 1 (D) Av << 1

Downloaded From : www.EasyEngineering.net


Downloaded From : www.EasyEngineering.net
GATE SOLVED PAPER - EC ANALOG ELECTRONIC

Q. 134 For full wave rectification, a four diode bridge rectifier is claimed to have the
following advantages over a two diode circuit :
(A) less expensive transformer,
(B) smaller size transformer, and
(C) suitability for higher voltage application.
Of these,
(A) only (1) and (2) are true (B) only (1) and (3) are true
(C) only (2) and (3) are true (D) (1), (2) as well as (3) are true

Q. 135 In the MOSFET amplifier of the figure is the signal output V1 and V2 obey the
relationship

ww
w.E
(A) V1 = V2 asy (B) V1 =-V2
2
(C) V1 = 2V2 En 2
(D) V1 =- 2V2

Q. 136
gin
For small signal ac operation, a practical forward biased diode can be modelled as
(A) a resistance and a capacitance in series
(B) an ideal diode and resistance in parallel
(C) a resistance and an ideal diode in series
eer
(D) a resistance
ing
1997 .ne ONE MARK

Q. 137
active region. Putting a capacitor across RE will t
In the BJT amplifier shown in the figure is the transistor is based in the forward

(A) decrease the voltage gain and decrease the input impedance
(B) increase the voltage gain and decrease the input impedance
(C) decrease the voltage gain and increase the input impedance

Downloaded From : www.EasyEngineering.net


Downloaded From : www.EasyEngineering.net
GATE SOLVED PAPER - EC ANALOG ELECTRONIC

(D) increase the voltage gain and increase the input impedance

Q. 138 A cascade amplifier stags is equivalent to


(A) a common emitter stage followed by a common base stage
(B) a common base stage followed by an emitter follower
(C) an emitter follower stage followed by a common base stage
(D) a common base stage followed by a common emitter stage

Q. 139 In a common emitter BJT amplifier, the maximum usable supply voltage is
limited by
(A) Avalanche breakdown of Base-Emitter junction
(B) Collector-Base breakdown voltage with emitter open (BVCBO)
(C) Collector-Emitter breakdown voltage with base open (BVCBO)

ww (D) Zener breakdown voltage of the Emitter-Base junction

w.E
Q. 140
1997 TWO MARKS

In the circuit of in the figure is the current iD through the ideal diode (zero cut
in voltage and forward resistance) equals

asy
En
gin
(A) 0 A
(C) 1 A eer
(B) 4 A
(D) None of the above

Q. 141
ing
The output voltage V0 of the circuit shown in the figure is

.ne
t
(A) - 4 V (B) 6 V
(C) 5 V (D) - 5.5 V

Q. 142 A half wave rectifier uses a diode with a forward resistance Rf . The voltage is
Vm sin wt and the load resistance is RL . The DC current is given by
(A) Vm (B) Vm
2 RL p (R f + RL)
(C) 2Vm (D) Vm
p RL

Downloaded From : www.EasyEngineering.net


Downloaded From : www.EasyEngineering.net
GATE SOLVED PAPER - EC ANALOG ELECTRONIC

1996 ONE MARK

Q. 143 In the circuit of the given figure, assume that the diodes are ideal and the meter
is an average indicating ammeter. The ammeter will read

(A) 0.4 2 A (B) 0.4 A


(C) 0.8 A (D) 0.4 mamp
p p
Q. 144 The circuit shown in the figure is that of

ww
w.E
asy
(C) an oscillator
En
(A) a non-inverting amplifier (B) an inverting amplifier
(D) a Schmitt trigger

1996 gin TWO MARKS

Q. 145
eer
In the circuit shown in the given figure N is a finite gain amplifier with a gain
of k , a very large input impedance, and a very low output impedance. The input

ing
impedance of the feedback amplifier with the feedback impedance Z connected
as shown will be

.ne
t
(A) Z b1 - 1 l (B) Z (1 - k)
k
(C) Z (D) Z
(k - 1) (1 - k)
Q. 146 A Darlington stage is shown in the figure. If the transconductance of Q1 is gm1 and
c
Q2 is gm2 , then the overall transconductance gmc ;T i cc E is given by
vbe

Downloaded From : www.EasyEngineering.net


Downloaded From : www.EasyEngineering.net
GATE SOLVED PAPER - EC ANALOG ELECTRONIC

(A) gm1 (B) 0.5 gm1


(C) gm2 (D) 0.5 gm2

Q. 147 Value of R in the oscillator circuit shown in the given figure, so chosen that it just
oscillates at an angular frequency of w. The value of w and the required value of
R will respectively be

ww
w.E (A) 105 rad/ sec, 2 # 10 4 W
(C) 2 # 10 4 rad/ sec, 105 W
(B) 2 # 10 4 rad/ sec, 2 # 10 4 W
(D) 105 rad/ sec, 105 W

Q. 148
asy
A zener diode in the circuit shown in the figure is has a knee current of 5 mA,

En
and a maximum allowed power dissipation of 300 mW. What are the minimum
and maximum load currents that can be drawn safely from the circuit, keeping

gin
the output voltage V0 constant at 6 V?

eer
ing
(A) 0 mA, 180 mA
(C) 10 mA, 55 mA .ne
(B) 5 mA, 110 mA
(D) 60 mA, 180 mA

*********** t

Downloaded From : www.EasyEngineering.net


Downloaded From : www.EasyEngineering.net
GATE SOLVED PAPER - EC ANALOG ELECTRONIC

SOLUTIONS

Sol. 1 Option (B) is correct.


For the given ideal op-amp, negative terminal will be also ground (at zero voltage)
and so, the collector terminal of the BJT will be at zero voltage.
i.e., VC = 0 volt
The current in 1 kW resistor is given by
I = 5 - 0 = 5 mA
1 kW

ww This current will flow completely through the BJT since, no current will flow
into the ideal op-amp ( I/P resistance of ideal op-amp is infinity). So, for BJT
we have

w.E VC = 0
VB = 0

asy IC = 5 mA
i.e.,the base collector junction is reverse biased (zero voltage) therefore, the

En
collector current (IC ) can have a value only if base-emitter is forward biased.
Hence,

&
VBE = 0.7 volts
VB - VE = 0.7 gin
&
or,
0 - Vout = 0.7
Vout =- 0.7 volt eer
Sol. 2 Option (A) is correct.
The i/p voltage of the system is given as ing
Vin = V1 + Vf = V1 + k Vout
= V1 + k A 0 V1 .ne ^Vout = A 0 V1h
= V1 ^1 + k A 0h
t
Therefore, if k is increased then input voltage is also increased so, the input
impedance increases. Now, we have
Vout = A 0 V1 = A 0Vin = A 0 Vin
^1 + k A 0h ^1 + k A 0h
Since, Vin is independent of k when seen from output mode, the output voltage
decreases with increase in k that leads to the decrease of output impedance.
Thus, input impedance increases and output impedance decreases.
Sol. 3 Option (A) is correct.
For the given circuit, we obtain the small signal model as shown in figure below :

Downloaded From : www.EasyEngineering.net


Downloaded From : www.EasyEngineering.net
GATE SOLVED PAPER - EC ANALOG ELECTRONIC

We obtain the node voltage at V1 as


V1 + V1 + gm Vi = 0
RD R + 1
L
sC
- gm Vi
& V1 =
1 + 1
RD R + 1
L
sC
Therefore, the output voltage V0 is obtained as
RL J - gm Vi N
V0 = V1 RL = K 1 O
RL + 1 RL + 1 + 1
sC sC KK RD R + 1 OO
L
L sC P
so, the transfer function is
V0 = - RD RL sCgm

ww Vi 1 + sC ^RD + RL h
Then, we have the pole at w = 1
C ^RD + RL h

w.E It gives the lower cutoff frequency of transfer function.


i.e., w0 = 1
C ^RD + RL h
or,
asy f0 = 1
2pC ^RD + RL h
= 7.97 . 8 Hz
= 1
2p # 10-6 # 20 # 103

Sol. 4
En
Option (B) is correct.

gin
eer
ing
.ne
From the circuit, we have

or,
Is = I Z + I L
I Z = Is - I L
t (1)
Since, voltage across zener diode is 5 V so, current through 100 W resistor is
obtained as
Is = 10 - 5 = 0.05 A
100
Therefore, the load current is given by
IL = 5
RL
Since, for proper operation, we must have
IZ $ Iknes
So, from Eq. (1), we write
0.05 A - 5 $ 10 mA
RL
50 mA - 5 $ 10 mA
RL

Downloaded From : www.EasyEngineering.net


Downloaded From : www.EasyEngineering.net
GATE SOLVED PAPER - EC ANALOG ELECTRONIC

40 mA $ 5
RL
40 # 10 $ 5
-3
RL
1 # RL
40 # 10-3 5
5 # RL
40 # 10-3
or, 125 W # RL
Therefore, minimum value of RL = 125 W
Now, we know that power rating of Zener diode is given by
PR = VZ IZ^maxh
IZ^maxh is maximum current through zener diode in reverse bias. Maximum
currrent through zener diode flows when load current is zero. i.e.,
IZ^maxh = Is = 10 - 5 = 0.05

ww Therefore,
100
PR = 5 # 0.05 W = 250 mW

w.E
Sol. 5 Option (C) is correct.

asy
En
gin
eer
ing
For the given ideal op-Amps we can assume
V 2- = V 2+ = V2 (ideal) .ne
V 1+ = V 1- = V1 (ideal)
So, by voltage division
V1 = Vout # 1
2
t
Vout = 2V1
and, as the I/P current in Op-amp is always zero therefore, there will be no
voltage drop across 1 KW in II op-amp
i.e., V2 = 1 V
Therefore,
V1 - V2 = V2 - ^- 2h
1 1
& V1 - 1 = 1 + 2
or, V1 = 4
Hence, Vout = 2V1 = 8 volt
Sol. 6 Option (B) is correct.
For the given circuit, we can make the truth table as below

Downloaded From : www.EasyEngineering.net


Downloaded From : www.EasyEngineering.net
GATE SOLVED PAPER - EC ANALOG ELECTRONIC

X Y Z
0 0 0
0 1 1
1 0 0
1 1 0
Logic 0 means voltage is v = 0 volt and logic 1 means voltage is 5 volt
For x = 0 , y = 0 , Transistor is at cut off mode and diode is forward biased. Since,
there is no drop across forward biased diode.
So, Z =Y=0
For x = 0 , y = 1, Again Transistor is in cutoff mode, and diode is forward biased.
with no current flowing through resistor.
So, Z =Y=1
For x = 1, y = 0 , Transistor is in saturation mode and so, z directly connected to

ww ground irrespective of any value of Y .


i.e., Z = 0 (ground)
Similarly for X = Y = 1

w.E Z = 0 (ground)
Hence, from the obtained truth table, we get
Z =XY
Sol. 7
asy
Option (D) is correct.
Given, the input voltage

EnVYZ = 100 sin wt

gin
eer
ing
.ne
For + ve half cycle
VYZ > 0
i.e., VY is a higher voltage than VZ
t
So, the diode will be in cutoff region. Therefore, there will no voltage difference
between X and W node.
i.e., VWX = 0
Now, for - ve half cycle all the four diodes will active and so, X and W
terminal is short circuited
i.e., VWX = 0
Hence, VWX = 0 for all t
Sol. 8 Option (C) is correct.
The equivalent circuit can be shown as

Downloaded From : www.EasyEngineering.net


Downloaded From : www.EasyEngineering.net
GATE SOLVED PAPER - EC ANALOG ELECTRONIC

VTh = VCC R2 = 3R2


R1 + R 2 R1 + R 2
and RTh = R 2 R1
R 2 + R1
Since, IC = bIB has b . 3 (very high) so, IB is negative in comparison to IC .
Therefore, we can write the base voltage
VB = VTh

ww So, VTh - 0.7 - IC RE = 0

^ h^ h
3R2 - 0.7 - 10-3 500 = 0

w.E or,

or,
R1 + R 2
3R 2
60 kW + R2
= 0.7 + 0.5

or,
asy 3R2 = ^60 kWh^1.2h + 1.2R2

1.8R2 = ^60 kWh # ^1.2h


or,

En R2 = 60 # 1.2 = 40 kW
Sol. 9
Hence,
Option (D) is correct.
gin 1. 8

eer
Let v > 0.7 V and diode is forward biased. By applying Kirchoff’s voltage law
10 - i # 1k - v = 0
10 - :v - 0.7 D (1000) - v = 0
500
10 - (v - 0.7) # 2 - v = 0
ing
10 - 3v + 1.4 = 0
.ne
So,
v = 11.4 = 3.8 V > 0.7
3
i = v - 0.7 = 3.8 - 0.7 = 6.2 mA
500 500
t
(Assumption is true)

Sol. 10 Option (C) is correct.


Given ib = 1 + 0.1 cos (1000pt) mA
So, IB = DC component of ib
= 1 mA
In small signal model of the transistor
bVT
rp = VT " Thermal voltage
IC
= VT = VT = VT IC = I
B
IC /b IB IB b
So, rp = 25 mV = 25 W VT = 25 mV, IB = 1 mA
1 mA
Sol. 11 Option (A) is correct.
The circuit composed of a clamper and a peak rectifier as shown.

Downloaded From : www.EasyEngineering.net


Downloaded From : www.EasyEngineering.net
GATE SOLVED PAPER - EC ANALOG ELECTRONIC

Clamper clamps the voltage to zero voltage, as shown

ww
w.E The peak rectifier adds + 1 V to peak voltage, so overall peak voltage lowers down
by - 1 volt.
So, vo = cos wt - 1

Sol. 12 asy
Option (A) is correct.

En
We put a test source between terminal 1, 2 to obtain equivalent impedance

gin
eer
ZTh = Vtest
Itest
ing
Applying KCL at top right node
Vtest + Vtest - 99I = I
.ne
9 k + 1k 100 b

Vtest + Vtest - 99I = I


10 k 100 b
test

test
t ...(i)

But Ib =- Vtest =-Vtest


9k + 1k 10k
Substituting Ib into equation (i), we have
Vtest + Vtest + 99Vtest = I
test
10 k 100 10 k
100Vtest + Vtest = I
test
10 # 103 100
2Vtest = I
test
100
ZTh = Vtest = 50 W
Itest
Sol. 13 Option (B) is correct.
First we obtain the transfer function.

Downloaded From : www.EasyEngineering.net


Downloaded From : www.EasyEngineering.net
GATE SOLVED PAPER - EC ANALOG ELECTRONIC

0 - Vi (jw) 0 - Vo (jw)
+ =0
1 +R R2
1
jwC
Vo (jw) - Vi (jw)
=
R2 1 +R
1
jw C
Vi (jw) R2

ww At w " 0 (Low frequencies),


Vo (jw) =-
R1 - j 1
wC
1 " 3, so V = 0

w.E At w " 3 (higher frequencies)


wC o

1 " 0, so V (jw) =- R2 V (jw)

asy wC o
R1 i
The filter passes high frequencies so it is a high pass filter.

En H (jw) = Vo = - R2
Vi R1 - j 1
wC
H (3) = -
gin
R
R1
2
= R
R1
2

At 3 dB frequency, gain will be


H ^ jw0h = 1 H (3) eer
2 times of maximum gain 6H (3)@

So, R2
1
= 1
2
R2
b R1 l ing
2

.ne
2
R1 + 2 2
w0 C

2R 12 = R 12 +

R 12 =1
w 2C 2
1
w02 C 2
t
w0 = 1
R1 C
Sol. 14 Option (D) is correct.
DC Analysis :

Using KVL in input loop,

Downloaded From : www.EasyEngineering.net


Downloaded From : www.EasyEngineering.net
GATE SOLVED PAPER - EC ANALOG ELECTRONIC

VC - 100IB - 0.7 = 0
VC = 100IB + 0.7 ...(i)
IC - IE = 13.7 - VC = (b + 1) IB
12k
13.7 - VC = 100I ...(ii)
B
12 # 103
Solving equation (i) and (ii),
IB = 0.01 mA
Small Signal Analysis :
Transforming given input voltage source into equivalent current source.

ww
w.E This is a shunt-shunt feedback amplifier.

asy
Given parameters,
rp = VT = 25 mV = 2.5 kW
IB 0.01 mA

En gm = =
b 100
rp 2.5 # 1000
= 0.04 s
Writing KCL at output node
gin
v0 + g v + v0 - vp = 0
RC m p
RF
v 0 : 1 + 1 D + v p :gm - 1 D = 0eer
RC RF RF
Substituting RC = 12 kW, RF = 100 kW, gm = 0.04 s
ing
v 0 (9.33 # 10-5) + v p (0.04) = 0

.ne
v 0 =- 428.72Vp ...(i)
Writing KCL at input node
vi = v p + v p + v p - vo
Rs Rs rp RF
vi = v 1 + 1 + 1 - v 0
t
p:
Rs Rs rp RF D RF
vi = v (5.1 10-4) - v 0
p #
Rs RF
Substituting Vp from equation (i)

vi = - 5.1 # 10-4 v - v 0
0
Rs 428.72 RF
vi =- 1.16 # 10-6 v 0 - 1 # 10-5 v 0 Rs = 10 kW (source resistance)
10 # 103
vi =- 1.116 # 10-5
10 # 103
Av = v 0 = 1 - 8.96
vi 10 # 103 # 1.116 # 10-5
Sol. 15 Option (A) is correct.

Downloaded From : www.EasyEngineering.net


Downloaded From : www.EasyEngineering.net
GATE SOLVED PAPER - EC ANALOG ELECTRONIC

For the parallel RLC circuit resonance frequency is,


wr = 1 = 1 = 10 M rad/s
LC 10 # 10-6 # 1 # 10-9
Thus given frequency is resonance frequency and parallel RLC circuit has
maximum impedance at resonance frequency
Gain of the amplifier is gm # (ZC RL) where ZC is impedance of parallel RLC
circuit.
At w = wr , ZC = R = 2 kW = ZC max .
Hence at this frequency (wr ), gain is
Gain w = w = gm (ZC RL) = gm (2k 2k) = gm # 103 which is maximum.
r

Therefore gain is maximum at wr = 10 M rad/ sec .

Sol. 16 Option (D) is correct.

ww The given circuit is shown below :

w.E
asy
En
From diagram we can write
Ii = Vo + Vo
gin
Transfer function
R1 sL1
eer
H (s) = Vo = sR1 L1
I1 R1 + sL1 ing
or H (jw) =
jw R 1 L 1
R 1 + jw L 1
.ne
Sol. 17
At w = 0
At w = 3
Option (C) is correct.
H (jw) = 0
H (jw) = R1 = constant . t Hence HPF.

Given circuit is shown below.

For transistor M2 ,
VGS = VG - VS = Vx - 0 = Vx
VDS = VD - VS = Vx - 0 = Vx

Downloaded From : www.EasyEngineering.net


Downloaded From : www.EasyEngineering.net
GATE SOLVED PAPER - EC ANALOG ELECTRONIC

Since VGS - VT = Vx - 1 < VDS , thus M2 is in saturation.


By assuming M1 to be in saturation we have
IDS (M ) = IDS (M )
1 2

mn C 0x m C
(4) (5 - Vx - 1) 2 = n 0x 1 (Vx - 1) 2
2 2
4 (4 - Vx ) 2 = (Vx - 1) 2
or 2 (4 - Vx ) = ! (Vx - 1)
Taking positive root, 8 - 2Vx = Vx - 1
Vx = 3 V
At Vx = 3 V for M1,VGS = 5 - 3 = 2 V < VDS . Thus our assumption is true and
Vx = 3 V .
Sol. 18 Option (D) is correct.

ww We have
Now
a = 0.98
b = a = 4.9
1-a

w.E In active region, for common emitter amplifier,


IC = bIB + (1 + b) ICO ...(1)

asy
Substituting ICO = 0.6 mA and IB = 20 mA in above eq we have,
IC = 1.01 mA
Sol. 19

En
Option (C) is correct.
In active region VBEon = 0.7 V
Emitter voltage
Emitter Current IE = E
gin
VE = VB - VBEon =- 5.7 V
V - (- 10) - 5.7 - (- 10)
= = 1 mA

Now
4.3k
IC . IE = 1 mA eer
4.3k

Applying KCL at collector


i1 = 0.5 mA ing
Since i1 = C dVC
dt
.ne
or VC = 1 # i1 dt = i1 t
C C
t ...(1)

with time, the capacitor charges and voltage across collector changes from 0
towards negative.

When saturation starts, VCE = 0.7 & VC =+ 5 V (across capacitor)

Downloaded From : www.EasyEngineering.net


Downloaded From : www.EasyEngineering.net
GATE SOLVED PAPER - EC ANALOG ELECTRONIC

Thus from (1) we get, + 5 = 0.5 mA T


5 mA
-6
or T = 5 # 5 # 10-3 = 50 m sec
0.5 # 10
Sol. 20 Option (A) is correct.
The current flows in the circuit if all the diodes are forward biased. In forward
biased there will be 0.7 V drop across each diode.
12.7 - 4 (0.7)
Thus IDC = = 1 mA
9900
Sol. 21 Option (B) is correct.
The forward resistance of each diode is
r = VT = 25 mV = 25 W
IC 1 mA
4 (r)

ww Thus Vac = Vi # e
4 (r) + 9900 o
= 100 mV cos (wt) 0.01

w.E
Sol. 22
= 1 cos (wt) mV

Option (A) is correct.

asy
The equivalent circuit of given amplifier circuit (when CE is connected, RE is
short-circuited)

En
gin
Input impedance Ri = RB || r p eer
Voltage gain AV = gm RC
ing
Now, if CE is disconnected, resistance RE appears in the circuit

.ne
t
Input impedance R in = RB || [rp + (b + 1)] RE
Input impedance increases
gm RC
Voltage gain AV = Voltage gain decreases.
1 + gm R E
Sol. 23 Option (B) is correct.
Since, emitter area of transistor Q1 is half of transistor Q2 , so current
IE = 1 IE and IB = 1 IB
1
2 2 1
2 2

The circuit is as shown below :

Downloaded From : www.EasyEngineering.net


Downloaded From : www.EasyEngineering.net
GATE SOLVED PAPER - EC ANALOG ELECTRONIC

VB =- 10 - (- 0.7) =- 9.3 V
0 - (- 9.3)
Collector current I1 = = 1 mA
(9.3 kW)
b 1 = 700 (high), So IC . IE

ww
1

Applying KCL at base we have


1 - IE = IB + IB
1 2

w.E 1 - (b 1 + 1) IB = IB + IB
1 1 2

1 = (700 + 1 + 1)
IB
2
2
+ IB 2

asy IB . 2
2
702
I 0 = IC = b 2 : IB = 715 # 2 . 2 mA
Sol. 24 En
Option (A) is correct.
2 2
702

The circuit is as shown below :


gin
eer
ing
.ne
So, 0 - Vi + 0 - Vo = 0
R1 R2
Vo =- R2
t
or
Vi R1
Sol. 25 Option (B) is correct.
By small signal equivalent circuit analysis

Input resistance seen by source vs


R in = vs = Rs + Rs || rs = (1000 W) + (93 kW || 259 W) = 1258 W
is

Downloaded From : www.EasyEngineering.net


Downloaded From : www.EasyEngineering.net
GATE SOLVED PAPER - EC ANALOG ELECTRONIC

Sol. 26 Option (B) is correct.


Cut-off frequency due to C2
fo = 1
2p (RC + RL) C2
= 1 = 271 Hz
2 # 3.14 # 1250 # 4.7 # 10-6
f
Lower cut-off frequency fL . o = 271 = 27.1 Hz
10 10
Sol. 27 Option (B) is correct.
The circuit is as shown below

ww
w.E Current I = 20 - 0 + Vi - 0 = 5 + Vi

asy
If I > 0, diode D2 conducts
4R

So, for 5 + VI > 0 & VI > - 5, D2 conducts


R R

2
En
Equivalent circuit is shown below

gin
eer
ing
Output is Vo = 0 . If I < 0 , diode D2 will be off .ne
5 + VI < 0 & V < - 5, D is off
R
The circuit is shown below
I 2
t

0 - Vi + 0 - 20 + 0 - Vo = 0
R 4R R
or Vo =- Vi - 5

At Vi =- 5 V, Vo = 0
At Vi =- 10 V, Vo = 5 V

Downloaded From : www.EasyEngineering.net


Downloaded From : www.EasyEngineering.net
GATE SOLVED PAPER - EC ANALOG ELECTRONIC

Sol. 28 Option (A) is correct.


Let diode be OFF. In this case 1 A current will flow in resistor and voltage across
resistor will be V = 1.V
Diode is off, it must be in reverse biased, therefore
Vi - 1 > 0 " Vi > 1
Thus for Vi > 1 diode is off and V = 1V
Option (B) and (C) doesn’t satisfy this condition.
Let Vi < 1. In this case diode will be on and voltage across diode will be zero and
V = Vi
Thus V = min (Vi, 1)
Sol. 29 Option (A) is correct.
The R2 decide only the frequency.
Sol. 30 Option (D) is correct.

ww For small increase in VG beyond 1 V the n - channel MOSFET goes into saturation
as VGS "+ ive and p - MOSFET is always in active region or triode region.

w.E
Sol. 31

Sol. 32
Option (C) is correct.
Option (D) is correct.

asy
The circuit is shown in fig below

En
gin
eer
ing
.ne
The voltage at non inverting terminal is 5 V because OP AMP is ideal and
inverting terminal is at 5 V.
Thus IC = 10 - 5 = 1 mA
5k
VE = IE RE = 1m # 1.4k = 1.4V
t IE = IC
= 0.6 + 1.4 = 2V
Thus the feedback is negative and output voltage is V = 2V .
Sol. 33 Option (D) is correct.
The output voltage is
hfe RC
V0 = Ar Vi .- Vi
hie
Here RC = 3 W and hie = 3 kW
Thus V0 . - 150 # 3k Vi
3k
.- 150 (A cos 20t + B sin 106 t)
Since coupling capacitor is large so low frequency signal will be filtered out, and
best approximation is

Downloaded From : www.EasyEngineering.net


Downloaded From : www.EasyEngineering.net
GATE SOLVED PAPER - EC ANALOG ELECTRONIC

V0 .- 150B sin 106 t


Sol. 34 Option (C) is correct.
For the positive half of Vi , the diode D1 is forward bias, D2 is reverse bias and the
zener diode is in breakdown state because Vi > 6.8 .
Thus output voltage is
V0 = 0.7 + 6.8 = 7.5 V
For the negative half of Vi, D2 is forward bias thus
Then V0 =- 0.7 V
Sol. 35 Option (B) is correct.
By Current mirror,
^ L h2
W
Ix = Ibias
^ L h1
W

ww Since MOSFETs are identical,


Thus W W
b L l =b L l

w.E
Sol. 36
Hence
Option (B) is correct.
2 2

Ix = Ibias

asy
The circuit is using ideal OPAMP. The non inverting terminal of OPAMP is at
ground, thus inverting terminal is also at virtual ground.

En
gin
eer
ing
Thus current will flow from -ive terminal (0 Volt) to -1 Volt source. Thus the
current I is
I =
0 - (- 1)
100k
= 1
100k .ne
The current through diode is
I = I 0 _eV - 1i
Now VT = 25 mV and I0 = 1 mA
V
t
t
I = 10-6 8e 25 # 10 - 1B = 1 5
V
Thus -3

10
or V = 0.06 V
Now V0 = I # 4k + V = 1 # 4k + 0.06 = 0.1 V
100k
Sol. 37 Option (B) is correct.
The circuit is using ideal OPAMP. The non inverting terminal of OPAMP is at
ground, thus inverting terminal is also at virtual ground.
Thus we can write
vi = -Rv
R1 + sL sR C + 1
2

2 2

or v0 =- R2
vi (R1 + sL)( sR2 C2 + 1)

Downloaded From : www.EasyEngineering.net


Downloaded From : www.EasyEngineering.net
GATE SOLVED PAPER - EC ANALOG ELECTRONIC

and from this equation it may be easily seen that this is the standard form of
T.F. of low pass filter
H (s) = K
(R1 + sL)( sR2 C2 + 1)
and form this equation it may be easily seen that this is the standard form of
T.F. of low pass filter
H (s) = 2 K
as + bs + b

ww
w.E
Sol. 38 Option (C ) is correct.

Sol. 39 asy
The current in both transistor are equal. Thus gm is decide by M1.
Option (C) is correct.

En
Let the voltage at non inverting terminal be V1, then after applying KCL at non
inverting terminal side we have

10 10
gin
15 - V1 + V0 - V1 = V1 - (- 15)
10
or V
V1 = 0
3
eer
If V0 swings from -15 to +15 V then V1 swings between -5 V to +5 V.
Sol. 40 Option (A) is correct.
ing
.ne
For the given DC values the Thevenin equivalent circuit is as follows

t
The Thevenin resistance and voltage are
VTH = 10 # 9 = 3 V
10 + 20
and total RTH = 10k # 20k = 6.67 kW
10k + 20k
Since b is very large, therefore IB is small and can be ignored
Thus IE = VTH - VBE = 3 - 0.7 = 1 mA
RE 2.3k
Sol. 41 Option (D) is correct.
The small signal model is shown in fig below

Downloaded From : www.EasyEngineering.net


Downloaded From : www.EasyEngineering.net
GATE SOLVED PAPER - EC ANALOG ELECTRONIC

IC
gm = = 1m = 1 A/V IC . IE
VT 25m 25
Vo =- gm Vp # (3k 3k )
=- 1 Vin (1.5k) Vp = Vin
25
=- 60Vin
or Am = Vo =- 60

ww
Sol. 42 Option (C) is correct.
Vin

w.E The circuit shown in (C) is correct full wave rectifier circuit.

asy
En
Sol. 43 Option (A) is correct. gin
eer
In the transconductance amplifier it is desirable to have large input resistance
and large output resistance.
Sol. 44 Option (C) is correct.
We redraw the circuit as shown in fig. ing
.ne
t
Applying voltage division rule
v+ = 0.5 V
We know that v+ = v-
Thus v- = 0.5 V
Now i = 1 - 0.5 = 0.5 mA
1k
and i = 0.5 - v0 = 0.5 mA
2k
or v0 = 0.5 - 1 =- 0.5 V
Sol. 45 Option (B) is correct.
If we assume b very large, then IB = 0 and IE = IC ; VBE = 0.7 V. We assume that

Downloaded From : www.EasyEngineering.net


Downloaded From : www.EasyEngineering.net
GATE SOLVED PAPER - EC ANALOG ELECTRONIC

BJT is in active, so applying KVL in Base-emitter loop


IE = 2 - VBE = 2 - 0.7 = 1.3 mA
RE 1k
Since b is very large, we have IE = IC , thus
IC = 1.3 mA
Now applying KVL in collector-emitter loop
10 - 10IC - VCE - IC = 0
or VCE =- 4.3 V
Now VBC = VBE - VCE
= 0.7 - (- 4.3) = 5 V
Since VBC > 0.7 V, thus transistor in saturation.
Sol. 46 Option (D) is correct.

ww Here the inverting terminal is at virtual ground and the current in resistor and
diode current is equal i.e.

w.E or
R s
IR = ID
Vi = I eV /V
D T

or
asy VD = VT 1n Vi

For the first condition


Is R

En VD = 0 - Vo1 = VT 1n 2
Is R
For the first condition
gin
VD = 0 - Vo1 = VT 1n 4

Subtracting above equation


Is R
eer
Vo1 - Vo2 = VT 1n 4 - VT 1n 2
Is R Is R ing
or Vo1 - Vo2 = VT 1n 4 = VT 1n2
2
.ne
Sol. 47 Option (D) is correct.
We have
and
Vthp = Vthp = 1 V
WP
LP
W
= N = 40mA/V2
LN
t
From figure it may be easily seen that Vas for each NMOS and PMOS is 2.5 V
mA
Thus ID = K (Vas - VT ) 2 = 40 2 (2.5 - 1) 2 = 90 m A
V
Sol. 48 Option (C) is correct.
We have VZ = 7 volt, VK = 0, RZ = 10W
Circuit can be modeled as shown in fig below

Downloaded From : www.EasyEngineering.net


Downloaded From : www.EasyEngineering.net
GATE SOLVED PAPER - EC ANALOG ELECTRONIC

Since Vi is lies between 10 to 16 V, the range of voltage across 200 kW


V200 = Vi - VZ = 3 to 9 volt
The range of current through 200 kW is
3 = 15 mA to 9 = 45 mA
200k 200k
The range of variation in output voltage
15m # RZ = 0.15 V to 45m # RZ = 0.45
Thus the range of output voltage is 7.15 Volt to 7.45 Volt
Sol. 49 Option (A) is correct.
The voltage at non-inverting terminal is
1
V+ = sC
Vi = 1 V
R+ 1
sC
1 + sCR i

ww Now V- = V+ =

Applying voltage division rule


1 V
1 + sCR i

w.E or
V+ = R1 (V0 + Vi) = o
1
R1 + R1
Vi = o
(V + Vi)
(V + Vi)
2

or asy
1 + sCR
Vo
Vi
=- 1 +
2
2
1 + sRC

En V0 = 1 - sRC
Vi 1 + sRC
Sol. 50 Option (C) is correct.
V0 = H (s) = 1 - sRC gin
Vi
H (jw) =
1 + sRC
1 - jwRC eer
1 + jwRC
+H (jw) = f =- tan - 1 wRC - tan - 1 wRC ing
Minimum value, fmin
=- 2 tan - 2 wRC
= - p (at w " 3) .ne
Sol. 51
Maximum value,

Option (D) is correct.


fmax = 0( at w = 0)
t
In the transconductance amplifier it is desirable to have large input impedance
and large output impedance.
Sol. 52 Option (C) is correct.
Sol. 53 Option (D) is correct.
The voltage at inverting terminal is
V- = V+ = 10 V
Here note that current through the capacitor is constant and that is
I = V- = 10 = 10 mA
1k 1k
Thus the voltage across capacitor at t = 1 msec is
1m 1m Im
VC = 1 # Idt = 1 # 10mdt = 10 4 #0 dt = 10 V
C 0 1m 0

Downloaded From : www.EasyEngineering.net


Downloaded From : www.EasyEngineering.net
GATE SOLVED PAPER - EC ANALOG ELECTRONIC

Sol. 54 Option (A) is correct.


In forward bias Zener diode works as normal diode.
Thus for negative cycle of input Zener diode is forward biased and it conducts
giving VR = Vin .
For positive cycle of input Zener diode is reversed biased
when 0 < Vin < 6 , Diode is OFF and VR = 0
when Vin > 6 Diode conducts and voltage across diode is 6 V. Thus voltage across
is resistor is
VR = Vin - 6
Only option (B) satisfy this condition.
Sol. 55 Option (C) is correct.
The circuit under DC condition is shown in fig below

ww
w.E
asy
En
Applying KVL we have
VCC - RC (IC + IB) - VCE = 0 ...(1)
and
gin
VCC - RB IB - VBE = 0
Substituting IC = bIB in (1) we have
...(2)

VCC - RC (bIB + IB) - VCE = 0


Solving (2) and (3) we get eer ...(3)

VCE = VCC - VCC - VBE


1+ RB ing ...(4)

Now substituting values we get


RC (1 + b)
.ne
Sol. 56
VCE = 12 -

Option (B) is correct.


1+
12 - 0.7
53
1 + (1 + 60)
= 5.95 V
t
We have b' = 110 # 60 = 66
100
Substituting b' = 66 with other values in (iv) in previous solutions
VCE = 12 - 12 - 0.7 = 5.29 V
1+ 53
1 + (1 + 66)
Thus change is = 5.29 - 59.5 # 100 =- 4.3%
5.95
Sol. 57 Option (A) is correct.
Sol. 58 Option (C) is correct.
The Zener diode is in breakdown region, thus
V+ = VZ = 6 V = Vin

Downloaded From : www.EasyEngineering.net


Downloaded From : www.EasyEngineering.net
GATE SOLVED PAPER - EC ANALOG ELECTRONIC

Rf
Vo = Vin c1 +
R1 m
We know that

or Vout = Vo = 6`1 + 12k j = 9 V


24k
The current in 12 kW branch is negligible as comparison to 10 W. Thus Current
IC . IE . = Vout = 9 = 0.9 A
RL 10
Now VCE = 15 - 9 = 6 V
The power dissipated in transistor is
P = VCE IC = 6 # 0.9 = 5.4 W
Sol. 59 Option (B) is correct.
If the unregulated voltage increase by 20%, them the unregulated voltage is 18
V, but the VZ = Vin = 6 remain same and hence Vout and IC remain same. There

ww will be change in VCE


Thus, VCE - 18 - 9 = 9 V
IC = 0.9 A

w.E Power dissipation P = VCE IC = 9 # 0.9 = 8.1 W


Thus % increase in power is

asy
8.1 - 5.4 # 100 = 50%
5.4
Sol. 60

En
Option (B) is correct.
Since the inverting terminal is at virtual ground, the current flowing through the
voltage source is
Is = Vs gin
or
Is
10k
Vs = 10 kW = R
in eer
Sol. 61 Option (D) is correct.
ing
The effect of current shunt feedback in an amplifier is to decrease the input
resistance and increase the output resistance as :
Rif = Ri
.ne
where
1 + Ab
Rof = R0 (1 + Ab)
Ri " Input resistance without feedback
t
Rif " Input resistance with feedback.
Sol. 62 Option (B) is correct.
The CE configuration has high voltage gain as well as high current gain. It
performs basic function of amplifications. The CB configuration has lowest Ri
and highest Ro . It is used as last step to match a very low impedance source and
to drain a high impedance load
Thus cascade amplifier is a multistage configuration of CE-CB
Sol. 63 Option (D) is correct.
Common mode gain
ACM =- RC
2RE
And differential mode gain

Downloaded From : www.EasyEngineering.net


Downloaded From : www.EasyEngineering.net
GATE SOLVED PAPER - EC ANALOG ELECTRONIC

ADM =- gm RC
Thus only common mode gain depends on RE and for large value of RE it
decreases.
Sol. 64 Option (C) is correct.
IE = Is `e nV - 1j = 10 - 13 c
VBE
T
0.7 - 1m = 49 mA
1 # 26 # 10- 3
e
Sol. 65 Option (C) is correct.
The circuit is as shown below

ww
w.E Writing equation for I- have
e 0 - V- = I

or asy 1M -

e0 = I- (1M) + V- ...(1)

En
Writing equation for I+ we have
0 - V+

or
1M
= I+
gin
V+ = - I+ (1M) ...(2)
Since for ideal OPAMP V+ = V- , from (1) and (2) we have
e0 = I- (1M) - I + (1M) eer
= (I- - I+) (1M) = IOS (1M)
ing
Sol. 66 Option (C) is correct. .ne
Thus if e0 has been measured, we can calculate input offset current IOS only.

t
At low frequency capacitor is open circuit and voltage acr s non-inverting terminal
is zero. At high frequency capacitor act as short circuit and all input voltage
appear at non-inverting terminal. Thus, this is high pass circuit.
The frequency is given by
w = 1 = 1 = 1000 rad/sec
RC 1 # 10 # 1 # 10 - 6
3

Sol. 67 Option (B) is correct.


The circuit under DC condition is shown in fig below

Downloaded From : www.EasyEngineering.net


Downloaded From : www.EasyEngineering.net
GATE SOLVED PAPER - EC ANALOG ELECTRONIC

Applying KVL we have


VCC - RB IB - VBE - RE IE = 0
or VCC - RB IB - VBE - RE (b + 1) IB = 0 Since IE = IB + bIB
or IB = VCC - VBE
RB + (b + 1) RE
= 20 - 0.7 = 40m A
430k + (50 + 1)1 k
Now IC = bIB = 50 # 40m = 2 mA
VC = VCC - RC IC = 20 - 2m # 2k = 16 V
Sol. 68 Option (A) is correct.
The maximum load current will be at maximum input voltage i.e.
Vmax = 30 V i.e.
Vmax - VZ = I + I

ww or
1k L Z

30 - 5.8 = I = 0.5 m
1k L

w.E
Sol. 69
or
Option (D) is correct.
IL = 24.2 - 0.5 = 23.7 mA

Sol. 70
asy
Option (B) is correct.
The small signal model is as shown below

En
gin
eer
From the figure we have
ing
Zin = 2 MW
.ne
Sol. 71
and

Option (A) is correct.


Z0 = rd RD = 20k 2k = 20 kW

The circuit in DC condition is shown below


11
t

Since the FET has high input resistance, gate current can be neglect and we get
VGS =- 2 V
Since VP < VGS < 0 , FET is operating in active region

Downloaded From : www.EasyEngineering.net


Downloaded From : www.EasyEngineering.net
GATE SOLVED PAPER - EC ANALOG ELECTRONIC

(- 2) 2
ID = IDSS c1 - VGS m = 10 c1 -
2

(- 8) m
Now = 5.625 mA
VP
Now VDS = VDD - ID RD = 20 - 5.625 m # 2 k = 8.75 V
Sol. 72 Option (B) is correct.
The transconductance is
gm = 2
VP ID IDSS

or, = 2 5.625mA # 10mA = 1.875 mS


8
The gain is A =- gm (rd RD)
So, = 1.875ms # 20 K =- 3.41
11
Sol. 73 Option (B) is correct.

ww Only one diode will be in ON conditions


When lower diode is in ON condition, then

w.E Vu = 2k Vsat = 2 10 = 8 V
2.5k
when upper diode is in ON condition
2.5

asy Vu = 2k Vsat = 2 (- 10) =- 5 V


2.5k 4
Sol. 74

En
Option (B) is correct.
An ideal OPAMP is an ideal voltage controlled voltage source.
Sol. 75 Option (C) is correct.
gin
In voltage series feed back amplifier, input impedance increases by factor (1 + Ab)

eer
and output impedance decreases by the factor (1 + Ab).
Rif = Ri (1 + Ab)
Rof = Ro
(1 + Ab)
ing
Sol. 76 Option (A) is correct.
This is a Low pass filter, because .ne
At w = 3

and at w = 0
V0 = 0
Vin
V0 = 1
Vin
t
Sol. 77 Option (D) is correct.
When IC >> ICO
IC
gm = = 1mA = 0.04 = 40 mA/V
VT 25mV
b
rp = = 100 - 3 = 2.5 kW
gm 40 # 10
Sol. 78 Option (A) is correct.
The given circuit is wein bridge oscillator. The frequency of oscillation is
2pf = 1
RC
or C = 1 = 1 = 1 m
2pRf 2p # 103 # 103 2p

Downloaded From : www.EasyEngineering.net


Downloaded From : www.EasyEngineering.net
GATE SOLVED PAPER - EC ANALOG ELECTRONIC

Sol. 79 Option (A) is correct.


The circuit is as shown below

We know that for ideal OPAMP


V- = V+

ww Applying KCL at inverting terminal


V- - Vs + V- - V0 = 0
R1 R1

w.E or 2V- - Vo = Vs
Applying KCL at non-inverting terminal
...(1)

V+
R2
asy V - Vo
+ IL + +
R2
=0

or

En
2V+ - Vo + IL R2 = 0
Since V- = V+ , from (1) and (2) we have
...(2)

or
Vs + IL R2 = 0
gin
IL =- Vs

Sol. 80 Option (D) is correct.


R2

eer
If IZ is negligible the load current is
12 - Vz = I
R L ing
as per given condition
.ne
At IL = 100 mA 12
R
100 mA # 12 - VZ # 500 mA
- 5 = 100 mA
R
t VZ = 5 V

or R = 70W
At IL = 500 mA 12 - 5 = 500 mA VZ = 5 V
R
or R = 14 W
Thus taking minimum we get
R = 14 W
Sol. 81 Option (B) is correct.
Sol. 82 Option (C) is correct.
The Thevenin equivalent is shown below

Downloaded From : www.EasyEngineering.net


Downloaded From : www.EasyEngineering.net
GATE SOLVED PAPER - EC ANALOG ELECTRONIC

VT = R1 V = 1
#5 = 1 V
R1 + R2 C 4+1
Since b is large is large, IC . IE , IB . 0 and
IE = VT - VBE = 1 - 0.7 = 3 mA
RE 300
Now VCE = 5 - 2.2kIC - 300IE

ww
Sol. 83 Option (B) is correct.
= 5 - 2.2k # 1m - 300 # 1m = 2.5 V

w.E For the different combinations the table is as follows

CE CE CC CB
Ai
Av asy High
High
High
Unity
Unity
High
Ri
En Medium High Low

Sol. 84
Ro

Option (D) is correct.


Medium

gin Low High

eer
This circuit having two diode and capacitor pair in parallel, works as voltage
doubler.
Sol. 85 Option (B) is correct.
ing
If the input is sinusoidal signal of 8 V (peak to peak) then
Vi = 4 sin wt
.ne
The output of comparator will be high when input is higher than Vref = 2 V and

t
will be low when input is lower than Vref = 2 V. Thus the waveform for input is
shown below

From fig, first crossover is at wt1 and second crossover is at wt2 where
4 sin wt1 = 2V

Downloaded From : www.EasyEngineering.net


Downloaded From : www.EasyEngineering.net
GATE SOLVED PAPER - EC ANALOG ELECTRONIC

Thus wt1 = sin - 1 1 = p


2 6
wt2 = p - = pp 5
6 6
5p p
-6
Duty Cycle = 6 =1
2p 3
Thus the output of comparators has a duty cycle of 1 .
3
Sol. 86 Option (C) is correct.
CMMR = Ad
Ac
or 20 log CMMR = 20 log Ad - 20 log Ac = 48 - 2 = 46 dB
Where Ad "Differential Voltage Gain
and AC " Common Mode Voltage Gain

ww
Sol. 87 Option (B) is correct.
The gain of amplifier is

w.E Ai =
- gm
gb + jwC
Thus the gain of a transistor amplifier falls at high frequencies due to the

Sol. 88
asy
internal capacitance that are diffusion capacitance and transition capacitance.
Option (A) is correct.

En
We have Ri = 1kW, b = 0.2, A = 50
Ri
= 1 kW

Sol. 89
Thus,

Option (A) is correct.


Rif =
(1 + Ab)
gin
11

eer
The DC equivalent circuit is shown as below. This is fixed bias circuit operating
in active region.

ing
.ne
In first case VCC - IC1 R2 - VCE1 = 0
t
or 6 - 1.5mR2 - 3 = 0
or R2 = 2kW
I
IB1 = C1 = 1.5m = 0.01 mA
b1 150
In second case IB2 will we equal to IB1 as there is no in R1.
Thus IC2 = b2 IB2 = 200 # 0.01 = 2 mA
VCE2 = VCC - IC2 R2 = 6 - 2m # 2 kW = 2 V
Sol. 90 Option (A) is correct.
The given circuit is a R - C phase shift oscillator and frequency of its oscillation
is
f = 1
2p 6 RC

Downloaded From : www.EasyEngineering.net


Downloaded From : www.EasyEngineering.net
GATE SOLVED PAPER - EC ANALOG ELECTRONIC

Sol. 91 Option (C) is correct.


If we see th figure we find that the voltage at non-inverting terminal is 3 V by the
zener diode and voltage at inverting terminal will be 3 V. Thus Vo can be get by
applying voltage division rule, i.e.
20 V = 3
20 + 40 o
or V0 = 9 V
Sol. 92 Option (B) is correct.
The circuit is as shown below

ww
w.E V+ = 8 (3) = 8 kW
1+8 3

asy 8
V+ = V- = V
3

En
Now applying KCL at inverting terminal we get
V- - 2 + V- - Vo = 0

or
1 5
gin
Vo = 6V- - 10
= 6 # 8 - 10 = 6 V
3
eer
Sol. 93 Option (C) is correct.

ing
The equivalent circuit of 3 cascade stage is as shown in fig.

.ne
t
V2 = 1k 50V1 = 40V1
1k + 0.25k
Similarly V3 = 1k 50V2 = 40V2
1k + 0.25k
or V3 = 40 # 40V1
Vo = 50V3 = 50 # 40 # 40V1
or AV = Vo = 50 # 40 # 40 = 8000
V1
or 20 log AV = 20 log 8000 = 98 dB
Sol. 94 Option (D) is correct.
If a constant current is made to flow in a capacitor, the output voltage is integration
of input current and that is sawtooth waveform as below :

Downloaded From : www.EasyEngineering.net


Downloaded From : www.EasyEngineering.net
GATE SOLVED PAPER - EC ANALOG ELECTRONIC

t
VC = 1 # idt
C 0
The time period of wave form is
T = 1 = 1 = 2 m sec
f 500
20 # 10
-3
1
6 #
Thus 3= idt
2 # 10 0
or i (2 # 10 - 3 - 0) = 6 # 10 - 6
or i = 3 mA
Thus the charging require 3 mA current source for 2 msec.
Sol. 95 Option (C) is correct.
In voltage-amplifier or voltage-series amplifier, the Ri increase and Ro decrease
because

ww Rif = Ri (1 + Ab)
Rof = Ro
(1 + Ab)

w.E
Sol. 96 Option (B) is correct.
Let x be the gain and it is 20 db, therefore

or asy20 log x = 20
x = 10

En
Since Gain band width product is 106 Hz, thus
So, bandwidth is
6 6

gin
BW = 10 = 10 = 105 Hz = 100 kHz
Gain 10
Sol. 97 Option (A) is correct.
eer
In multistage amplifier bandwidth decrease and overall gain increase. From

ing
bandwidth point of view only options (A) may be correct because lower cutoff
frequency must be increases and higher must be decreases. From following
calculation we have
We have fL = 20 Hz and fH = 1 kHz
For n stage amplifier the lower cutoff frequency is .ne
f =
Ln

The higher cutoff frequency is


fL
1
2n - 1
= 20 = 39.2 . 40 Hz
1
23 - 1
t
1
fHn = fH 2 2 - 1 = 0.5 kHz
Sol. 98 Option (A) is correct.
As per Barkhousen criterion for sustained oscillations Ab $ 1 and phase shift
must be or 2pn .
V (f)
Now from circuit A= O = 1 + R2
Vf (f) R1
V (f )
b (f) = 1 +0 = f
6 VO (f)
Thus from above equation for sustained oscillation
6 = 1 + R2
R1
or R2 = 5R1

Downloaded From : www.EasyEngineering.net


Downloaded From : www.EasyEngineering.net
GATE SOLVED PAPER - EC ANALOG ELECTRONIC

Sol. 99 Option (C) is correct.


Let the gain of OPAMP be AV then we have
20 log AV = 40 dB
or AV = 100
Let input be Vi = Vm sin wt then we have
VO = VV Vi = Vm sin wt
Now dVO = A V w cos wt
V m
dt
dVO
Slew Rate c dt m = AV Vm w = AV Vm 2pf
max

or Vm = SR = -6 1
AV V2pf 10 # 100 # 2p # 20 # 103
or VM = 79.5 mV

ww
Sol. 100 Option (A) is correct.
The circuit is shown as below

w.E
asy
En I = IZ + IL
For satisfactory operations
Vin - V0
R gin
> IZ + IL [IZ + IL = I]
When Vin = 30 V,
30 - 10 $ (10 + 1) mA
eer
or
R
20 $ 11 mA ing
or
R
R # 1818 W
.ne
when Vin = 50 V 50 - 10 $ (10 + 1) mA
R
40 $ 11 # 10 - 3
R
t
or R # 3636W Thus R # 1818W
Sol. 101 Option (D) is correct.
We have
IDSS = 10 mA and VP =- 5 V
Now VG = 0
and VS = ID RS = 1 # 2.5W = 2.5 V
Thus VGS = VG - VS = 0 - 2.5 =- 2.5 V
Now gm = 2IDSS 81 - ` - 2.5 jB = 2 mS
VP -5
AV = V0 =- gm RD
Vi
So, =- 2ms # 3k =- 6

Downloaded From : www.EasyEngineering.net


Downloaded From : www.EasyEngineering.net
GATE SOLVED PAPER - EC ANALOG ELECTRONIC

Sol. 102 Option (C) is correct.


The current gain of a BJT is
hfe = gm rp
Sol. 103 Option (A) is correct.
The ideal op-amp has following characteristic :
Ri " 3
R0 " 0
and A"3
Sol. 104 Option (C) is correct.
Both statements are correct because
(1) A stable multivibrator can be used for generating square wave, because of its
characteristic

ww (2) Bi-stable multivibrator can store binary information, and this multivibrator
also give help in all digital kind of storing.

w.E
Sol. 105 Option (B) is correct.
If fT is the frequency at which the short circuit common emitter gain attains unity
magnitude then

asy fT =
gm
2p (Cm + Cp)
= 38 # 10 - 3
2p # (10 - 14 + 4 # 10 - 13)
= 1.47 # 1010 Hz

En
If fB is bandwidth then we have
f 10
fB = T = 1.47 # 10 = 1.64 # 108 Hz

Sol. 106 Option (C) is correct.


b 90

gin
eer
If we neglect current through RB then it can be open circuit as shown in fig.

ing
.ne
t
Maximum power will dissipate in Zener diode when current through it is maximum
and it will occur at Vin = 30 V
I = Vin - Vo = 30 - 10 = 1 A
20 20
I IC + IZ = bIB + IZ Since IC = bIB
= bIZ + IZ = (b + 1) IZ since IB = IZ
or IZ = I = 1 = 0.01 A
b + 1 99 + 1
Power dissipated in zener diode is
PZ = VZ IZ = 9.5 # 0.01 = 95 mW
IC = bIZ = 99 # 0.1 = 0.99 A
VCE = Vo = 10 V
Power dissipated in transistor is
PT = VC IC = 10 # 0.99 = 9.9 W

Downloaded From : www.EasyEngineering.net


Downloaded From : www.EasyEngineering.net
GATE SOLVED PAPER - EC ANALOG ELECTRONIC

Sol. 107 Option (B) is correct.


From the it may be easily seen that the tank circuit is having 2-capacitors and
one-inductor, so it is colpits oscillator and frequency is
f = 1
2p LCeq
Ceq = C1 C2 = 2 # 2 = 1 pF
C1 + C2 4
f = 1 = 1 # 109 = 50.3 MHz
2p 10 # 10 - 6 # 10 - 12 2p 10
Sol. 108 Option (D) is correct.
The circuit is as shown below

ww
w.E Let V- be the voltage of inverting terminal, since non inverting terminal a at

asy
ground, the output voltage is
Vo = AOL V- ...(1)

En
Now applying KCL at inverting terminal we have
V- - Vs + V- - V0 = 0 ...(2)
R1 R2
From (1) and (2) we have gin
VO = A =
Vs CL
- R2
eer
R - R2 + R1
ROL
Substituting the values we have
ACL = - 10k ing
=- 1000 . - 11

Sol. 109 Option (A) is correct.


1k - 10 k +
100k
1 k 89
.ne
t
The first OPAMP stage is the differentiator and second OPAMP stage is integrator.
Thus if input is cosine term, output will be also cosine term. Only option (A) is
cosine term. Other are sine term. However we can calculate as follows. The circuit
is shown in fig

Applying KCL at inverting terminal of first OP AMP we have


V1 = - wjL = - 100 # 10 # 10 - 3 = - 1
VS R 10 10
- jVS
or V1 = = j cos 100t
10

Downloaded From : www.EasyEngineering.net


Downloaded From : www.EasyEngineering.net
GATE SOLVED PAPER - EC ANALOG ELECTRONIC

Applying KCL at inverting terminal of second OP AMP we have


VO = - 1/jwC
V1 100
=- 1 = j10
j100 # 10 # 10 - 6 # 100
or V0 = j10V2 = j10 (- j cos 100t)
V0 = 10 cos 100t
Sol. 110 Option (C) is correct.
With the addition of RE the DC abis currents and voltages remain closer to
the point where they were set by the circuit when the outside condition such as
temperature and transistor parameter b change.
Sol. 111 Option (A) is correct.
Common mode gain is

ww AC = aRC
REE

w.E
Sol. 112
Since source resistance of the current source is infinite REE = 3 , common mode
gain AC = 0
Option (D) is correct.

asy
In positive feed back it is working as OP-AMP in saturation region, and the input
applied voltage is +ve.
So,
En V0 =+ Vsat = 15 V
Sol. 113 Option (A) is correct.
At high frequency
gin
gm
Ai =- '
gbc + jw (C)
1 eer
or,

and
Ai \

Ai a
Capacitance
1 ing
frequency

.ne
Thus due to the transistor capacitance current gain of a bipolar transistor

Sol. 114
drops.
Option (C) is correct. t
As OP-AMP is ideal, the inverting terminal at virtual ground due to ground at
non-inverting terminal. Applying KCL at inverting terminal
sC (v1 sin wt - 0) + sC (V2 sin wt - 0) + sC (Vo - 0) = 0
or Vo =- (V1 + V2) sin wt
Sol. 115 Option (D) is correct.
There is R - C , series connection in parallel with parallel R - C combination.
So, it is a wein bridge oscillator because two resistors R1 and R2 is also in parallel
with them.
Sol. 116 Option (A) is correct.
The given circuit is a differentiator, so the output of triangular wave will be
square wave.
Sol. 117 Option (B) is correct.
In sampling and hold circuit the unity gain non-inverting amplifier is used.

Downloaded From : www.EasyEngineering.net


Downloaded From : www.EasyEngineering.net
GATE SOLVED PAPER - EC ANALOG ELECTRONIC

Sol. 118 Option (D) is correct.


The Thevenin equivalent is shown below

VT = R1 V = 5
# 15 = 5 V
R1 + R2 C 10 + 5
Since b is large is large, IC . IE , IB . 0 and
IE = VT - VBE
ww = 5
RE
- 0.7 = 4.3 = 10 mA

w.E
Sol. 119 Option (C) is correct.
0.430kW 0.430KW

The output voltage will be input offset voltage multiplied by open by open loop
gain. Thus
So asy V0 = 5mV # 10, 000 = 50 V
But
En V0 = ! 15 V in saturation condition
So, it can never be exceeds !15 V
So,
gin
V0 = ! Vset = ! 15V
Sol. 120

Sol. 121
Option (A) is correct.
Option (A) is correct. eer
Sol. 122 Option (A) is correct. ing
Negative feedback in amplifier reduces the gain of the system.

By drawing small signal equivalent circuit


.ne
t
by applying KCL at E2
Vp
gm1 Vp - 2
= gm2 Vp
1
rp2
2

at C2 i 0 =- gm2 Vp 2

from eq (1) and (2)


gm1 Vp + i 0 =- i
0
1
gm2 rp
2

gm1 Vp =- i 0 :1 + 1 D
1
gm2 rp 2

Downloaded From : www.EasyEngineering.net


Downloaded From : www.EasyEngineering.net
GATE SOLVED PAPER - EC ANALOG ELECTRONIC

gm2 rp
2
= b >> 1
so gm1 Vp1
=- i 0
i0 =- gm1
Vp 1

i0 = gm1 a Vp = Vi
Vi 1

Sol. 123 Option (B) is correct.


Crossover behavior is characteristic of calss B output stage. Here 2 transistor
are operated one for amplifying +ve going portion and other for -ve going
portion.
Sol. 124 Option (C) is correct.
In Voltage series feedback mode input impedance is given by
R in = Ri (1 + bv Av)

ww where bv = feedback factor ,


Av = openloop gain

w.E and
So,
Ri = Input impedance
R in = 1 # 103 (1 + 0.99 # 100) = 100 kW

asy
Similarly output impedance is given by
ROUT = R0
(1 + bv Av)
R 0 = output impedance

Thus
En
ROUT = 100
(1 + 0.99 # 100)
= 1W

Sol. 125 Option (B) is correct.


gin
Regulation = Vno - load - Vfuel - load
Vfull - load
= 30 - 25 # 100 = 20% eer
25
Output resistance = 25 = 25 W
1 ing
Sol. 126 Option (D) is correct.
.ne
Sol. 127 Option (A) is correct.
In a differential amplifier CMRR is given by
t
This is a voltage shunt feedback as the feedback samples a portion of output
voltage and convert it to current (shunt).

(1 + b) IQ R 0
CMRR = 1 ;1 + E
2 VT b
So where R 0 is the emitter resistance. So CMRR can be improved by increasing
emitter resistance.
Sol. 128 Option (C) is correct.
We know that rise time (tr ) is
tr = 0.35
fH
where fH is upper 3 dB frequency. Thus we can obtain upper 3 dB frequency it
rise time is known.
Sol. 129 Option (D) is correct.
In a BJT differential amplifier for a linear response Vid < VT .

Downloaded From : www.EasyEngineering.net


Downloaded From : www.EasyEngineering.net
GATE SOLVED PAPER - EC ANALOG ELECTRONIC

Sol. 130 Option (D) is correct.


In a shunt negative feedback amplifier.
Input impedance
R in = Ri
(1 + bA)
where Ri = input impedance of basic amplifier
b = feedback factor
A = open loop gain
So, R in < Ri
Similarly
ROUT = R0
(1 + bA)
ROUT < R 0

ww
Sol. 131
Thus input & output impedances decreases.
Option (A) is correct.

w.E
Sol. 132 Option (D) is correct.
Comparator will give an output either equal to + Vsupply or - Vsupply . So output is

Sol. 133
a square wave.
asy
Option (C) is correct.

En
In series voltage regulator the pass transistor is in common collector
configuration having voltage gain close to unity.
Sol. 134 Option (D) is correct.
gin
In bridge rectifier we do not need central tap transformer, so its less expensive

eer
and smaller in size and its PIV (Peak inverse voltage) is also greater than the two
diode circuit, so it is also suitable for higher voltage application.
Sol. 135 Option (C) is correct.
In the circuit we have ing
V2 = IS # RD
2 .ne
and V1 = IS # RD
V2 = 1
V1 2
t
V1 = 2V2
Sol. 136 Option (C) is correct.
Sol. 137 Option (C) is correct.
The equivalent circuit of given amplifier circuit (when CE is connected, RE is
short-circuited)

Input impedance Ri = RB || r p
Voltage gain AV = gm RC

Downloaded From : www.EasyEngineering.net


Downloaded From : www.EasyEngineering.net
GATE SOLVED PAPER - EC ANALOG ELECTRONIC

Now, if CE is disconnected, resistance RE appears in the circuit

Input impedance R in = RB || [rp + (b + 1)] RE


Input impedance increases
gm RC
Voltage gain AV = Voltage gain decreases.
1 + gm R E

ww
Sol. 138 Option (A) is correct.
In common emitter stage input impedance is high, so in cascaded amplifier
common emitter stage is followed by common base stage.

w.E
Sol. 139 Option (C) is correct.
We know that collect-emitter break down voltage is less than compare to collector

asy
base breakdown voltage.
BVCEO < BVCBO

En
both avalanche and zener break down. Voltage are higher than BVCEO .So BVCEO
limits the power supply.
Sol. 140 Option (C) is correct.
gin
eer
ing
.ne
If we assume consider the diode in reverse bias then Vn should be greater than VP .
VP < Vn
by calculating
VP = 10 # 4 = 5 Volt
4+4
t
Vn = 2 # 1 = 2 Volt
here VP > Vn (so diode cannot be in reverse bias mode).

apply node equation at node a


Va - 10 + Va + Va = 2
4 4 1
6Va - 10 = 8
Va = 3 Volt

Downloaded From : www.EasyEngineering.net


Downloaded From : www.EasyEngineering.net
GATE SOLVED PAPER - EC ANALOG ELECTRONIC

so current Ib = 0 - 3 + 10 - 3
4 4
Ib = 10 - 6 = 1 amp
4
Sol. 141 Option (D) is correct.
Applying node equation at terminal (2) and (3) of OP -amp

ww Va - Q Va - V0

w.E 5
+
10
=0

2Va - 4 + Va - V0 = 0

asy V0 = 3Va - 4
Va - V0 + Va - 0 = 0
100 10

En
Va - V0 + 10Va = 0
11Va = V0
Va = V0
11
gin
So V0 = 3V0 - 4
11 eer
8V0 =- 4
11 ing
Sol. 142 Option (B) is correct.
V0 =- 5.5 Volts
.ne
Circuit with diode forward resistance looks
t
So the DC current will
IDC = Vm
p (R f + RL)
Sol. 143 Option (D) is correct.
For the positive half cycle of input diode D1 will conduct & D2 will be off. In
negative half cycle of input D1 will be off & D2 conduct so output voltage wave
from across resistor (10 kW) is –

Downloaded From : www.EasyEngineering.net


Downloaded From : www.EasyEngineering.net
GATE SOLVED PAPER - EC ANALOG ELECTRONIC

Ammeter will read rms value of current


so I rms = Vm (half wave rectifier)
pR
= 4 = 0.4 mA
(10 kW) p p
Sol. 144 Option (D) is correct.
In given circuit positive feedback is applied in the op-amp., so it works as a
Schmitt trigger.

ww
Sol. 145 Option (D) is correct.
Gain with out feedback factor is given by
V0 = kVi

w.E after connecting feedback impedance Z

asy
En
gin
given input impedance is very large, so after connecting Z we have
Ii = Vi - V0
Z
Ii = Vi - kVi
eer V0 = kVi

Z
Zin = Vi = Z
ing
Sol. 146
input impedance

Option (A) is correct.


Ii (1 - k)
.ne
Sol. 147 Option (A) is correct.
t
For the circuit, In balanced condition It will oscillated at a frequency
w= 1 = 1 = 105 rad/ sec
LC 10 # 10-3 # .01 # 10-6
In this condition
R1 = R 3
R2 R4
5 =R
100 1
R = 20 kW = 2 # 10 4 W
Sol. 148 Option (C) is correct.
V0 kept constant at V0 = 6 volt
so current in 50 W resistor
I = 9-6
50 W
I = 60 m amp

Downloaded From : www.EasyEngineering.net


Downloaded From : www.EasyEngineering.net
GATE SOLVED PAPER - EC ANALOG ELECTRONIC

Maximum allowed power dissipation in zener


PZ = 300 mW
Maximum current allowed in zener
PZ = VZ (IZ ) max = 300 # 10-3
& = 6 (IZ ) max = 300 # 10-3
& = (IZ ) max = 50 m amp
Given knee current or minimum current in zener
(IZ ) min = 5 m amp
In given circuit I = IZ + I L
I L = I - IZ
(IL) min = I - (IZ ) max
= (60 - 50) m amp = 10 m amp

ww (IL) max = I - (IZ ) min


= (60 - 5) = 55 m amp

w.E ***********

asy
En
gin
eer
ing
.ne
t

Downloaded From : www.EasyEngineering.net


Downloaded From : www.EasyEngineering.net

No part of this publication may be reproduced or distributed in any form or any means, electronic, mechanical,
photocopying, or otherwise without the prior permission of the author.

ww
w.E
GATE SOLVED PAPER
Electronics & Communication
Communication System
asy
Copyright © By NODIA & COMPANY
En
gin
eer
ing
Information contained in this book has been obtained by authors, from sources believes to be reliable. However,
neither Nodia nor its authors guarantee the accuracy or completeness of any information herein, and Nodia nor its

.ne
authors shall be responsible for any error, omissions, or damages arising out of use of this information. This book
is published with the understanding that Nodia and its authors are supplying information but are not attempting
to render engineering or other professional services.

t
NODIA AND COMPANY
B-8, Dhanshree Tower Ist, Central Spine, Vidyadhar Nagar, Jaipur 302039
Ph : +91 - 141 - 2101150
www.nodia.co.in
email : enquiry@nodia.co.in

Downloaded From : www.EasyEngineering.net


Downloaded From : www.EasyEngineering.net

GATE SOLVED PAPER - EC


COMMUNICATION SYSTEM

2013 ONE MARK

Q. 1 The bit rate of a digital communication system is R kbits/s . The modulation used
is 32-QAM. The minimum bandwidth required for ISI free transmission is
(A) R/10 Hz (B) R/10 kHz
(C) R/5 Hz (D) R/5 kHz

ww
Q. 2
2013

Let U and V be two independent zero mean Gaussain random variables of


TWO MARKS

w.E variances 1 and 1 respectively. The probability P ^3V F 2U h is


(A) 4/9
4 9
(B) 1/2
(C) 2/3
asy (D) 5/9

Q. 3

En
Consider two identically distributed zero-mean random variables U and V . Let
the cumulative distribution functions of U and 2V be F ^x h and G ^x h respectively.
Then, for all values of x
(A) F ^x h - G ^x h # 0
(C) ^F (x) - G (x)h .x # 0
gin (B) F ^x h - G ^x h $ 0
(D) ^F (x) - G (x)h .x $ 0

Q. 4 eer
Let U and V be two independent and identically distributed random variables

(A) 3/4
2
(B) 1 ing
such that P ^U =+ 1h = P ^U =- 1h = 1 . The entropy H ^U + V h in bits is

(C) 3/2 (D) log 2 3


.ne
Common Data for Questions 5 and 6:
t
Bits 1 and 0 are transmitted with equal probability. At the receiver, the pdf of
the respective received signals for both bits are as shown below.

Q. 5 If the detection threshold is 1, the BER will be


(A) 1 (B) 1
2 4
(C) 1 (D) 1
8 16

Downloaded From : www.EasyEngineering.net


Downloaded From : www.EasyEngineering.net
GATE SOLVED PAPER - EC COMMUNICATION SYSTEM

Q. 6 The optimum threshold to achieve minimum bit error rate (BER) is


(A) 1 (B) 4
2 5

(C) 1 (D) 3
2

2012 ONE MARK

Q. 7 The power spectral density of a real process X (t) for positive frequencies is shown
below. The values of E [X 2 (t)] and E [X (t)] , respectively, are

ww
w.E (A) 6000/p, 0

asy
(C) 6400/p, 20/ (p 2 )
(B) 6400/p, 0
(D) 6000/p, 20/ (p 2 )

Q. 8

En
In a baseband communications link, frequencies upto 3500 Hz are used for
signaling. Using a raised cosine pulse with 75% excess bandwidth and for no

gin
inter-symbol interference, the maxi mum possible signaling rate in symbols per
second is
(A) 1750
(C) 4000
(B) 2625
(D) 5250 eer
Q. 9
ing
A source alphabet consists of N symbols with the probability of the first two
symbols being the same. A source encoder increases the probability of the first

.ne
symbol by a small amount e and decreases that of the second by e. After encoding,
the entropy of the source

Q. 10
(A) increases
(C) increases only if N = 2
(B) remains the same
(D) decreases t
Two independent random variables X and Y are uniformly distributed in the
interval 6- 1, 1@. The probability that max 6X, Y @ is less than 1/2 is
(A) 3/4 (B) 9/16
(C) 1/4 (D) 2/3

2012 TWO MARKS

Q. 11 A BPSK scheme operating over an AWGN channel with noise power


spectral density of N 0 /2, uses equiprobable signals s1 (t) = 2E sin (wc t) and
T
s2 (t) =- 2E sin (wc t) over the symbol interval (0, T). If the local oscillator in a
T
coherent receiver is ahead in phase by 45c with respect to the received signal, the
probability of error in the resulting system is
(A) Q c 2E m (B) Q c E m
N0 N0

Downloaded From : www.EasyEngineering.net


Downloaded From : www.EasyEngineering.net
GATE SOLVED PAPER - EC COMMUNICATION SYSTEM

(C) Q c E E
2N 0 m
(D) Q c
4N 0 m
Q. 12 A binary symmetric channel (BSC) has a transition probability of 1/8. If the
binary symbol X is such that P (X = 0) = 9/10, then the probability of error for
an optimum receiver will be
(A) 7/80 (B) 63/80
(C) 9/10 (D) 1/10

Q. 13 The signal m (t) as shown is applied to both a phase modulator (with k p as the
phase constant) and a frequency modulator (with k f as the frequency constant)
having the same carrier frequency.

ww
w.E
asy
The ratio k p /k f (in rad/Hz) for the same maximum phase deviation is
(A) 8p
(C) 2p
En (B) 4p
(D) p

gin
Statement for Linked Answer Question 14 and 15 :

eer
The transfer function of a compensator is given as
Gc (s) = s + a
Q. 14 Gc (s) is a lead compensator if
s+b
ing
(A) a = 1, b = 2
(C) a =- 3, b =- 1 .ne
(B) a = 3, b = 2
(D) a = 3, b = 1

Q. 15 The phase of the above lead compensator is maximum at


(A) 2 rad/s
(C) 6 rad/s
(B) 3 rad/s
(D) 1/ 3 rad/s
t
2011 ONE MARK

Q. 16 An analog signal is band-limited to 4 kHz, sampled at the Nyquist rate and


the samples are quantized into 4 levels. The quantized levels are assumed to be
independent and equally probable. If we transmit two quantized samples per
second, the information rate is
(A) 1 bit/sec (B) 2 bits/sec
(C) 3 bits/sec (D) 4 bits/sec

Q. 17 The Column -1 lists the attributes and the Column -2 lists the modulation
systems. Match the attribute to the modulation system that best meets it.

Downloaded From : www.EasyEngineering.net


Downloaded From : www.EasyEngineering.net
GATE SOLVED PAPER - EC COMMUNICATION SYSTEM

Column -1 Column -2
P. Power efficient transmission of signals 1. Conventional AM
Q. Most bandwidth efficient transmission of 2. FM
voice signals
R. Simplest receiver structure 3. VSB
S. Bandwidth efficient transmission of signals 4. SSB-SC
with significant dc component
(A) P-4, Q-2, R-1, S-3
(B) P-2, Q-4, R-1, S-3
(C) P-3, Q-2, R-1, S-4
(D) P-2, Q-4, R-3, S-1

ww 2011 TWO MARKS

w.E
Q. 18 X (t) is a stationary random process with auto-correlation function
RX (t) = exp (- pt 2). This process is passed through the system shown below.
The power spectral density of the output process Y (t) is

asy
En
gin
2 2
(A) (4p f + 1) exp (- pf ) 2

(C) (4p 2 f 2 + 1) exp (- pf ) eer


(B) (4p 2 f 2 - 1) exp (- pf 2)
(D) (4p 2 f 2 - 1) exp (- pf )

Q. 19
ing
A message signal m (t) = cos 2000pt + 4 cos 4000pt modulates the carrier

.ne
c (t) = cos 2pfc t where fc = 1 MHz to produce an AM signal. For demodulating
the generated AM signal using an envelope detector, the time constant RC of the
detector circuit should satisfy
(A) 0.5 ms < RC < 1 ms
(C) RC << 1 μs t
(B) 1 μs << RC < 0.5 ms
(D) RC >> 0.5 ms

Statement for Linked Answer Questions: 20 and 21


A four-phase and an eight-phase signal constellation are shown in the figure
below.

Downloaded From : www.EasyEngineering.net


Downloaded From : www.EasyEngineering.net
GATE SOLVED PAPER - EC COMMUNICATION SYSTEM

Q. 20 For the constraint that the minimum distance between pairs of signal points be d
for both constellations, the radii r 1 , and r 2 of the circles are
(A) r 1 = 0.707d, r2 = 2.782d (B) r 1 = 0.707d, r 2 = 1.932d
(C) r 1 = 0.707d, r 2 = 1.545d (D) r 1 = 0.707d, r 2 = 1.307d

Q. 21 Assuming high SNR and that all signals are equally probable, the additional
average transmitted signal energy required by the 8-PSK signal to achieve the
same error probability as the 4-PSK signal is
(A) 11.90 dB (B) 8.73 dB
(C) 6.79 dB (D) 5.33 dB

2010 ONE MARK

Q. 22 Suppose that the modulating signal is m (t) = 2 cos (2pfm t) and the carrier signal

ww is xC (t) = AC cos (2pfC t), which one of the following is a conventional AM signal
without over-modulation

w.E (A) x (t) = AC m (t) cos (2pfC t)


(B) x (t) = AC [1 + m (t)] cos (2pfC t)
(C) x (t) = AC cos (2pfC t) + AC m (t) cos (2pfC t)

asy 4
(D) x (t) = AC cos (2pfm t) cos (2pfC t) + AC sin (2pfm t) sin (2pfC t)

Q. 23
En
Consider an angle modulated signal

The average power of x (t) is


(A) 10 W gin
x (t) = 6 cos [2p # 106 t + 2 sin (800pt)] + 4 cos (800pt)

(B) 18 W
(C) 20 W
eer
(D) 28 W

Q. 24
filter matched to this pulse is ing
Consider the pulse shape s (t) as shown below. The impulse response h (t) of the

.ne
t

Downloaded From : www.EasyEngineering.net


Downloaded From : www.EasyEngineering.net
GATE SOLVED PAPER - EC COMMUNICATION SYSTEM

2010 TWO MARKS

Statement for linked Answer Question : 25 and 26 :


Consider a baseband binary PAM receiver shown below. The additive channel
noise n (t) is with power spectral density Sn (f ) = N 0 /2 = 10-20 W/Hz . The low-
pass filter is ideal with unity gain and cut-off frequency 1 MHz. Let Yk represent
the random variable y (tk ).
Yk = Nk , if transmitted bit bk = 0
Yk = a + Nk if transmitted bit bk = 1
Where Nk represents the noise sample value. The noise sample has a probability
density function, PNk (n) = 0.5ae- a n (This has mean zero and variance 2/a 2
). Assume transmitted bits to be equiprobable and threshold z is set to
a/2 = 10-6 V .

ww
w.E
asy
Q. 25 En
The value of the parameter a (in V - 1 ) is
(A) 1010
(C) 1.414 # 10-10 gin(B) 107
(D) 2 # 10-20

Q. 26 The probability of bit error is eer


(A) 0.5 # e-3.5
(C) 0.5 # e-7
(B) 0.5 # e-5
ing
(D) 0.5 # e-10

Q. 27 The Nyquist sampling rate for the signal


s (t) =
sin (500pt) sin (700) pt
is given by .ne
(A) 400 Hz
(C) 1200 Hz
pt # pt
(B) 600 Hz
(D) 1400 Hz
t
Q. 28 X (t) is a stationary process with the power spectral density Sx (f ) > 0 , for all f .
The process is passed through a system shown below

Let Sy (f ) be the power spectral density of Y (t). Which one of the following
statements is correct
(A) Sy (f ) > 0 for all f
(B) Sy (f ) = 0 for f > 1 kHz

Downloaded From : www.EasyEngineering.net


Downloaded From : www.EasyEngineering.net
GATE SOLVED PAPER - EC COMMUNICATION SYSTEM

(C) Sy (f ) = 0 for f = nf0, f0 = 2 kHz kHz, n any integer


(D) Sy (f ) = 0 for f = (2n + 1) f0 = 1 kHz , n any integer

2009 ONE MARK

Q. 29 For a message siganl m (t) = cos (2pfm t) and carrier of frequency fc , which of the
following represents a single side-band (SSB) signal ?
(A) cos (2pfm t) cos (2pfc t) (B) cos (2pfc t)
(C) cos [2p (fc + fm) t] (D) [1 + cos (2pfm t) cos (2pfc t)

2009 TWO MARKS

Q. 30 Consider two independent random variables X and Y with identical distributions.

ww The variables X and Y take values 0, 1 and 2 with probabilities 12 , 14 and 1


respectively. What is the conditional probability P (X + Y = 2 X - Y = 0) ?
4

w.E (A) 0
(C) 1/6
(B) 1/16
(D) 1

Q. 31

asy
A discrete random variable X takes values from 1 to 5 with probabilities as shown
in the table. A student calculates the mean X as 3.5 and her teacher calculates
the variance of X as 1.5. Which of the following statements is true ?

k En1 2 3 4 5
P (X = k) 0.1 0.2
gin 0.3 0.4 0.5
(A) Both the student and the teacher are right
(B) Both the student and the teacher are wrong
(C) The student is wrong but the teacher is right eer
(D) The student is right but the teacher is wrong
ing
A message signal given by m (t) = ( 12 ) cos w1 t - ( 12 ) sin w2 t amplitude - modulated
Q. 32

.ne
with a carrier of frequency wC to generator s (t)[ 1 + m (t)] cos wc t . What is the
power efficiency achieved by this modulation scheme ?
(A) 8.33%
(C) 20%
(B) 11.11%
(D) 25% t
Q. 33 A communication channel with AWGN operating at a signal to noise ration
SNR >> 1 and bandwidth B has capacity C1. If the SNR is doubled keeping
constant, the resulting capacity C2 is given by
(A) C2 . 2C1 (B) C2 . C1 + B
(C) C2 . C1 + 2B (D) C2 . C1 + 0.3B

Common Data For Q. 34 and 35 :


The amplitude of a random signal is uniformly distributed between -5 V and 5 V.
Q. 34 If the signal to quantization noise ratio required in uniformly quantizing the
signal is 43.5 dB, the step of the quantization is approximately
(A) 0.033 V (B) 0.05 V
(C) 0.0667 V (D) 0.10 V

Downloaded From : www.EasyEngineering.net


Downloaded From : www.EasyEngineering.net
GATE SOLVED PAPER - EC COMMUNICATION SYSTEM

Q. 35 If the positive values of the signal are uniformly quantized with a step size of 0.05
V, and the negative values are uniformly quantized with a step size of 0.1 V, the
resulting signal to quantization noise ration is approximately
(A) 46 dB (B) 43.8 dB
(C) 42 dB (D) 40 dB

2008 ONE MARK

Q. 36 Consider the amplitude modulated (AM) signalAc cos wc t + 2 cos wm t cos wc t . For
demodulating the signal using envelope detector, the minimum value of Ac should
be
(A) 2 (B) 1
(C) 0.5 (D) 0

ww 2008 TWO MARKS

w.E
Q. 37 The probability density function (pdf) of random variable is as shown below

asy
En
The corresponding commutative distribution function CDF has the form

gin
eer
ing
.ne
Q. 38
t
A memory less source emits n symbols each with a probability p. The entropy of
the source as a function of n
(A) increases as log n (B) decreases as log ( n1 )
(C) increases as n (D) increases as n log n

Q. 39 Noise with double-sided power spectral density on K over all frequencies is passed
through a RC low pass filter with 3 dB cut-off frequency of fc . The noise power
at the filter output is
(A) K (B) Kfc
(C) kpfc (D) 3

Q. 40 Consider a Binary Symmetric Channel (BSC) with probability of error being p


. To transmit a bit, say 1, we transmit a sequence of three 1s. The receiver will
interpret the received sequence to represent 1 if at least two bits are 1. The
probability that the transmitted bit will be received in error is

Downloaded From : www.EasyEngineering.net


Downloaded From : www.EasyEngineering.net
GATE SOLVED PAPER - EC COMMUNICATION SYSTEM

(A) p3 + 3p2 (1 - p) (B) p3


(C) (1 - p3) (D) p3 + p2 (1 - p)

Q. 41 Four messages band limited to W, W, 2W and 3W respectively are to be


multiplexed using Time Division Multiplexing (TDM). The minimum bandwidth
required for transmission of this TDM signal is
(A) W (B) 3W
(C) 6W (D) 7W

Q. 42 Consider the frequency modulated signal


10 cos [2p # 105 t + 5 sin (2p # 1500t) + 7.5 sin (2p # 1000t)]
with carrier frequency of 105 Hz. The modulation index is
(A) 12.5 (B) 10
(C) 7.5 (D) 5

ww
Q. 43 The signal cos wc t + 0.5 cos wm t sin wc t is
(A) FM only (B) AM only

w.E (C) both AM and FM (D) neither AM nor FM

asy
Common Data For Q. 40 to 46 :
A speed signal, band limited to 4 kHz and peak voltage varying between +5

En
V and - 5 V, is sampled at the Nyquist rate. Each sample is quantized and
represented by 8 bits.
Q. 44
gin
If the bits 0 and 1 are transmitted using bipolar pulses, the minimum bandwidth
required for distortion free transmission is
(A) 64 kHz
(C) 8 kHz eer
(B) 32 kHz
(D) 4 kHz

Q. 45
ing
Assuming the signal to be uniformly distributed between its peak to peak value,
the signal to noise ratio at the quantizer output is
(A) 16 dB (B) 32 dB
.ne
Q. 46
(C) 48 dB (D) 4 kHz
t
Assuming the signal to be uniformly distributed between its peak to peak value,
the signal to noise ratio at the quantizer output is
(A) 1024 (B) 512
(C) 256 (D) 64

2007 ONE MARK

Q. 47 If R (t) is the auto correlation function of a real, wide-sense stationary random


process, then which of the following is NOT true
(A) R (t) = R (- t)
(B) R (t) # R (0)
(C) R (t) =- R (- t)
(D) The mean square value of the process is R (0)

Q. 48 If S (f) is the power spectral density of a real, wide-sense stationary random

Downloaded From : www.EasyEngineering.net


Downloaded From : www.EasyEngineering.net
GATE SOLVED PAPER - EC COMMUNICATION SYSTEM

process, then which of the following is ALWAYS true?


(A) S (0) # S (f) (B) S (f) $ 0
#- 3 S (f) df = 0
3
(C) S (- f) =- S (f) (D)

Q. 49 If E denotes expectation, the variance of a random variable X is given by


(A) E [X2] - E2 [X] (B) E [X2] + E2 [X]
(C) E [X2] (D) E2 [X]

2007 TWO MARKS

Q. 50 A Hilbert transformer is a
(A) non-linear system (B) non-causal system
(C) time-varying system (D) low-pass system

ww
Q. 51 In delta modulation, the slope overload distortion can be reduced by
(A) decreasing the step size (B) decreasing the granular noise

w.E
Q. 52
(C) decreasing the sampling rate (D) increasing the step size

The raised cosine pulse p (t) is used for zero ISI in digital communications. The

asy
expression for p (t) with unity roll-off factor is given by
p (t) = sin 4pWt
4pWt (1 - 16W2 t2)

En
The value of p (t) at t = 1 is
(A) - 0.5
(C) 0.5
4W

gin(B) 0
(D) 3

Q. 53 eer
In the following scheme, if the spectrum M (f) of m (t) is as shown, then the
spectrum Y (f) of y (t) will be
ing
.ne
t

Q. 54 During transmission over a certain binary communication channel, bit errors


occur independently with probability p. The probability of AT MOST one bit in
error in a block of n bits is given by

Downloaded From : www.EasyEngineering.net


Downloaded From : www.EasyEngineering.net
GATE SOLVED PAPER - EC COMMUNICATION SYSTEM

(A) pn (B) 1 - pn
(C) np (1 - p) n - 1 + (1 + p) n (D) 1 - (1 - p) n

Q. 55 In a GSM system, 8 channels can co-exist in 200 kHz bandwidth using TDMA.
A GSM based cellular operator is allocated 5 MHz bandwidth. Assuming a
frequency reuse factor of 1 , i.e. a five-cell repeat pattern, the maximum number
5
of simultaneous channels that can exist in one cell is
(A) 200 (B) 40
(C) 25 (D) 5

Q. 56 In a Direct Sequence CDMA system the chip rate is 1.2288 # 106 chips per second.
If the processing gain is desired to be AT LEAST 100, the data rate
(A) must be less than or equal to 12.288 # 103 bits per sec
(B) must be greater than 12.288 # 103 bits per sec

ww (C) must be exactly equal to 12.288 # 103 bits per sec


(D) can take any value less than 122.88 # 103 bits per sec

w.E Common Data For Q. 57 and 58 :

asy
Two 4-array signal constellations are shown. It is given that f1 and f2 constitute
an orthonormal basis for the two constellation. Assume that the four symbols
in both the constellations are equiprobable. Let N0 denote the power spectral

En
density of white Gaussian noise.
2

gin
eer
ing
Q. 57
.ne
The if ratio or the average energy of Constellation 1 to the average energy of
Constellation 2 is
(A) 4a2
(C) 2
(B) 4
(D) 8
t
Q. 58 If these constellations are used for digital communications over an AWGN channel,
then which of the following statements is true ?
(A) Probability of symbol error for Constellation 1 is lower
(B) Probability of symbol error for Constellation 1 is higher
(C) Probability of symbol error is equal for both the constellations
(D) The value of N0 will determine which of the constellations has a lower
probability of symbol error

Statement for Linked Answer Question 59 and 60 :


An input to a 6-level quantizer has the probability density function f (x) as shown
in the figure. Decision boundaries of the quantizer are chosen so as to maximize the
entropy of the quantizer output. It is given that 3 consecutive decision boundaries

Downloaded From : www.EasyEngineering.net


Downloaded From : www.EasyEngineering.net
GATE SOLVED PAPER - EC COMMUNICATION SYSTEM

are’ - 1'.'0' and '1' .

Q. 59 The values of a and b are


(A) a = 1 and b = 1 (B) a = 1 and b = 3
6 12 5 40
(C) a = 1 and b = 1 (D) a = 1 and b = 1
4 16 3 24
Q. 60 Assuming that the reconstruction levels of the quantizer are the mid-points of

ww the decision boundaries, the ratio of signal power to quantization noise power is
(A) 152
9
(B) 64
3

w.E (C) 76
3
(D) 28

Q. 61
2006
asy ONE MARK

A low-pass filter having a frequency response H (jw) = A (w) e jf (w) does not produce

En
any phase distortions if
(A) A (w) = Cw3, f (w) = kw3 (B) A (w) = Cw2, f (w) = kw
(C) A (w) = Cw, f (w) = kw2
gin (D) A (w) = C, f (w) = kw- 1

2006
eer TWO MARKS

Q. 62

g (t) =
3
/ (- 1) k d (t - 0.5 # 10- 4 k) ing
A signal with bandwidth 500 Hz is first multiplied by a signal g (t) where

R =- 3

.ne
The resulting signal is then passed through an ideal lowpass filter with
bandwidth 1 kHz. The output of the lowpass filter would be
(A) d (t)
(C) 0
(B) m (t)
(D) m (t) d (t)
t
Q. 63 The minimum sampling frequency (in samples/sec) required to reconstruct the
following signal from its samples without distortion
x (t) = 5` sin 2p100t j + 7` sin 2p100t j would be
3 2

pt pt
(A) 2 # 103 (B) 4 # 103
(C) 6 # 103 (D) 8 # 103

Q. 64 The minimum step-size required for a Delta-Modulator operating at 32k samples/


sec to track the signal (here u (t) is the unit-step function)
x (t) = 125[ u (t) - u (t - 1) + (250t)[ u (t - 1) - u (t - 2)]
so that slope-overload is avoided, would be
(A) 2 - 10 (B) 2 - 8
(C) 2 - 6 (D) 2 - 4

Downloaded From : www.EasyEngineering.net


Downloaded From : www.EasyEngineering.net
GATE SOLVED PAPER - EC COMMUNICATION SYSTEM

Q. 65 A zero-mean white Gaussian noise is passes through an ideal lowpass filter of


bandwidth 10 kHz. The output is then uniformly sampled with sampling period
ts = 0.03 msec. The samples so obtained would be
(A) correlated (B) statistically independent
(C) uncorrelated (D) orthogonal

Q. 66 A source generates three symbols with probabilities 0.25, 0.25, 0.50 at a rate of
3000 symbols per second. Assuming independent generation of symbols, the most
efficient source encoder would have average bit rate is
(A) 6000 bits/sec (B) 4500 bits/sec
(C) 3000 bits/sec (D) 1500 bits/sec

Q. 67 The diagonal clipping in Amplitude Demodulation (using envelop detector) can


be avoided it RC time-constant of the envelope detector satisfies the following

ww condition, (here W is message bandwidth and w is carrier frequency both in rad/


sec)
(A) RC < 1 (B) RC > 1

w.E (C) RC <


W
1
w
W
(D) RC > 1
w
Q. 68

asy
A uniformly distributed random variable X with probability density function
fx (x) = 1 pu (x + 5) - u (x - 5)]

En10
where u (.) is the unit step function is passed through a transformation given in

gin
the figure below. The probability density function of the transformed random
variable Y would be

eer
ing
(A) fy (y) = 1 [u (y + 2.5) - u (y - 2.25)]
5 .ne
(B) fy (y) = 0.5d (y) + 0.5d (y - 1)
(C) fy (y) = 0.25d (y + 2.5) + 0.25d (y - 2.5) + 5d (y) t
(D) fy (y) = 0.25d (y + 2.5) + 0.25d (y - 2.5) + 1 [u (y + 2.5) - u (y - 2.5)]
10
Q. 69 In the following figure the minimum value of the constant "C" , which is to be
added to y1 (t) such that y1 (t) and y2 (t) are different , is

(A) 3 (B) 3
2
2
(C) 3 (D) 3
12 L

Downloaded From : www.EasyEngineering.net


Downloaded From : www.EasyEngineering.net
GATE SOLVED PAPER - EC COMMUNICATION SYSTEM

Q. 70 A message signal with bandwidth 10 kHz is Lower-Side Band SSB modulated


with carrier frequency fc1 = 106 Hz. The resulting signal is then passed through a
Narrow-Band Frequency Modulator with carrier frequency fc2 = 109 Hz.
The bandwidth of the output would be
(A) 4 # 10 4 Hz (B) 2 # 106 Hz
(C) 2 # 109 Hz (D) 2 # 1010 Hz

Common Data For Q. 71 and 72 :


Let g (t) = p (t)*( pt), where * denotes convolution & p (t) = u (t) - u (t - 1) lim
z"3
with u (t) being the unit step function
Q. 71 The impulse response of filter matched to the signal s (t) = g (t) - d (1 - 2)* g (t)
is given as :

ww (A) s (1 - t)
(C) - s (t)
(B) - s (1 - t)
(D) s (t)

w.E
Q. 72 An Amplitude Modulated signal is given as
xAM (t) = 100 [p (t) + 0.5g (t)] cos wc t

asy
in the interval 0 # t # 1. One set of possible values of modulating signal and
modulation index would be
(A) t, 0.5 (B) t, 1.0
(C) t, 2.0
En (D) t2, 0.5

Common Data For Q. 73 and 74 : gin


eer
The following two question refer to wide sense stationary stochastic process
Q. 73

ing
It is desired to generate a stochastic process (as voltage process) with power
spectral density S (w) = 16/ (16 + w2) by driving a Linear-Time-Invariant system
by zero mean white noise (As voltage process) with power spectral density being

.ne
constant equal to 1. The system which can perform the desired task could be
(A) first order lowpass R-L filter
(B) first order highpass R-C filter
(C) tuned L-C filter
(D) series R-L-C filter
t
Q. 74 The parameters of the system obtained in previous Q would be
(A) first order R-L lowpass filter would have R = 4W L = 1H
(B) first order R-C highpass filter would have R = 4W C = 0.25F
(C) tuned L-C filter would have L = 4H C = 4F
(D) series R-L-C lowpass filter would have R = 1W , L = 4H , C = 4F

Common Data For Q. 75 an 76 :


Consider the following Amplitude Modulated (AM) signal, where fm < B
XAM (t) = 10 (1 + 0.5 sin 2pfm t) cos 2pfc t
Q. 75 The average side-band power for the AM signal given above is

Downloaded From : www.EasyEngineering.net


Downloaded From : www.EasyEngineering.net
GATE SOLVED PAPER - EC COMMUNICATION SYSTEM

(A) 25 (B) 12.5


(C) 6.25 (D) 3.125

Q. 76 The AM signal gets added to a noise with Power Spectral Density Sn (f) given in
the figure below. The ratio of average sideband power to mean noise power would
be :

ww (A) 25
8N0 B
(C) 25
(B) 25
4N0 B
(D) 25

w.E 2N0 B

2005
N0 B

ONE MARK

Q. 77
Group 1 asy
Find the correct match between group 1 and group 2.
Group 2

En
P. {1 + km (t) A sin (wc t)}
Q. km (t) A sin (wc t)
W. Phase modulation
X. Frequency modulation
R. A sin {wc t + km (t)}
t
S. A sin ;wc t + k # m (t) dt E gin
Y. Amplitude modulation
Z. DSB-SC modulation
-3
(A) P - Z, Q - Y, R - X, S - W
(B) P - W, Q - X, R - Y, S - Z eer
(C) P - X, Q - W, R - Z, S - Y
(D) P - Y, Q - Z, R - W, S - X
ing
Q. 78 .ne
Which of the following analog modulation scheme requires the minimum
transmitted power and minimum channel bandwidth ?
(A) VSB
(C) SSB
(B) DSB-SC
(D) AM
t
2005 TWO MARKS

Q. 79 A device with input X (t) and output y (t) is characterized by: Y (t) = x2 (t). An
FM signal with frequency deviation of 90 kHz and modulating signal bandwidth
of 5 kHz is applied to this device. The bandwidth of the output signal is
(A) 370 kHz (B) 190 kHz
(C) 380 kHz (D) 95 kHz

Q. 80 A signal as shown in the figure is applied to a matched filter. Which of the


following does represent the output of this matched filter ?

Downloaded From : www.EasyEngineering.net


Downloaded From : www.EasyEngineering.net
GATE SOLVED PAPER - EC COMMUNICATION SYSTEM

ww
w.E
Q. 81
asy
Noise with uniform power spectral density of N0 W/Hz is passed though a filter
H (w) = 2 exp (- jwtd ) followed by an ideal pass filter of bandwidth B Hz. The

(A) 2N0 B En
output noise power in Watts is
(B) 4N0 B
(C) 8N0 B
gin (D) 16N0 B

An output of a communication channel is a random variable v with the probability

eer
Q. 82
density function as shown in the figure. The mean square value of v is

ing
.ne
(A) 4
(C) 8
(B) 6
(D) 9
t
Q. 83 A carrier is phase modulated (PM) with frequency deviation of 10 kHz by a
single tone frequency of 1 kHz. If the single tone frequency is increased to 2 kHz,
assuming that phase deviation remains unchanged, the bandwidth of the PM
signal is
(A) 21 kHz (B) 22 kHz
(C) 42 kHz (D) 44 kHz

Common Data For Q. 84 and 85 :


Asymmetric three-level midtread quantizer is to be designed assuming equiprobable
occurrence of all quantization levels.

Downloaded From : www.EasyEngineering.net


Downloaded From : www.EasyEngineering.net
GATE SOLVED PAPER - EC COMMUNICATION SYSTEM

Q. 84 If the probability density function is divide into three regions as shown in the
figure, the value of a in the figure is
(A) 1 (B) 2
3 3
(C) 1 (D) 1
2 4
Q. 85 The quantization noise power for the quantization region between - a and + a in
the figure is

ww (A) 4

(C)
81
5
(B) 1
9
(D) 2

w.E 2004
81 81

ONE MARK

Q. 86
asy
In a PCM system, if the code word length is increased from 6 to 8 bits, the signal
to quantization noise ratio improves by the factor
(A) 8
6 En (B) 12

(C) 16
gin (D) 8

Q. 87

eer
An AM signal is detected using an envelop detector. The carrier frequency and
modulating signal frequency are 1 MHz and 2 kHz respectively. An appropriate
value for the time constant of the envelop detector is
(A) 500m sec
(C) 0.2m sec ing
(B) 20m sec
(D) 1m sec

Q. 88
.ne
An AM signal and a narrow-band FM signal with identical carriers, modulating

(A) broadband FM
(C) DSB-SC
t
signals and modulation indices of 0.1 are added together. The resultant signal can
be closely approximated by
(B) SSB with carrier
(D) SSB without carrier

Q. 89 In the output of a DM speech encoder, the consecutive pulses are of opposite


polarity during time interval t1 # t # t2 . This indicates that during this interval
(A) the input to the modulator is essentially constant
(B) the modulator is going through slope overload
(C) the accumulator is in saturation
(D) the speech signal is being sampled at the Nyquist rate

Q. 90 The distribution function Fx (x) of a random variable x is shown in the figure. The
probability that X = 1 is

Downloaded From : www.EasyEngineering.net


Downloaded From : www.EasyEngineering.net
GATE SOLVED PAPER - EC COMMUNICATION SYSTEM

(A) zero (B) 0.25


(C) 0.55 (D) 0.30

2004 TWO MARKS

Q. 91 A 1 mW video signal having a bandwidth of 100 MHz is transmitted to a receiver


through cable that has 40 dB loss. If the effective one-side noise spectral density
at the receiver is 10 - 20 Watt/Hz, then the signal-to-noise ratio at the receiver is

ww (A) 50 dB
(C) 40 dB
(B) 30 dB
(D) 60 dB

w.E
Q. 92 Consider the signal x (t) shown in Fig. Let h (t) denote the impulse response of
the filter matched to x (t), with h (t) being non-zero only in the interval 0 to 4 sec.
The slope of h (t) in the interval 3 < t < 4 sec is

asy
En
gin
(A) 1 sec - 1 eer
(B) - 1 sec - 1
2
(C) - 1 sec - 1
2
(D) 1 sec - 1 ing
Q. 93
represented as shown in the figure. .ne
A source produces binary data at the rate of 10 kbps. The binary symbols are

t
The source output is transmitted using two modulation schemes, namely Binary
PSK (BPSK) and Quadrature PSK (QPSK). Let B1 and B2 be the bandwidth
requirements of the above rectangular pulses is 10 kHz, B1 and B2 are

(A) B1 = 20 kHz, B2 = 20 kHz (B) B1 = 10 kHz, B2 = 20 kHz


(C) B1 = 20 khz, B2 = 10 kHz (D) B1 = 10 kHz, B2 = 10 kHz

Q. 94 A 100 MHz carrier of 1 V amplitude and a 1 MHz modulating signal of 1 V


amplitude are fed to a balanced modulator. The ourput of the modulator is
passed through an ideal high-pass filter with cut-off frequency of 100 MHz. The
output of the filter is added with 100 MHz signal of 1 V amplitude and 90c phase

Downloaded From : www.EasyEngineering.net


Downloaded From : www.EasyEngineering.net
GATE SOLVED PAPER - EC COMMUNICATION SYSTEM

shift as shown in the figure. The envelope of the resultant signal is

(A) constant (B) 1 + sin (2p # 106 t)

(C) 5 - sin (2p - 106 t) (D) 5 + cos (2p # 106 t)


4 4
Q. 95 Two sinusoidal signals of same amplitude and frequencies 10 kHz and 10.1 kHz
are added together. The combined signal is given to an ideal frequency detector.
The output of the detector is
(A) 0.1 kHz sinusoid (B) 20.1 kHz sinusoid

ww
Q. 96
(C) a linear function of time (D) a constant

Consider a binary digital communication system with equally likely 0’s and 1’s.

w.E When binary 0 is transmitted the detector input can lie between the levels - 0.25
V and + 0.25 V with equl probability : when binary 1 is transmitted, the voltage
at the detector can have any value between 0 and 1 V with equal probability. If

asy
the detector has a threshold of 0.2 V (i.e., if the received signal is greater than 0.2
V, the bit is taken as 1), the average bit error probability is
(A) 0.15
(C) 0.05
En (B) 0.2
(D) 0.5

Q. 97
follows : gin
A random variable X with uniform density in the interval 0 to 1 is quantized as

If 0 # X # 0.3 ,
If 0.3 < X # 1,
xq = 0
xq = 0.7 eer
where xq is the quantized value of X.
ing
The root-mean square value of the quantization noise is
(A) 0.573
(C) 2.205
(B) 0.198
(D) 0.266
.ne
Q. 98
item from Group 1 with the most appropriate item in Group 2.
Group 1 Group 2
t
Choose the current one from among the alternative A, B, C, D after matching an

1. FM P. Slope overload
2. DM Q. m-law
3. PSK R. Envelope detector
4. PCM S. Hilbert transform
T. Hilbert transform
U. Matched filter
(A) 1 - T, 2 - P, 3 - U, 4 - S (B) 1 - S, 2 - U, 3 - P, 4 - T
(C) 1 - S, 2 - P, 3 - U, 4 - Q (D) 1 - U, 2 - R, 3 - S, 4 - Q

Q. 99 Three analog signals, having bandwidths 1200 Hz, 600 Hz and 600 Hz, are sampled
at their respective Nyquist rates, encoded with 12 bit words, and time division
multiplexed. The bit rate for the multiplexed. The bit rate for the multiplexed
signal is
(A) 115.2 kbps (B) 28.8 kbps

Downloaded From : www.EasyEngineering.net


Downloaded From : www.EasyEngineering.net
GATE SOLVED PAPER - EC COMMUNICATION SYSTEM

(C) 57.6 kbps (D) 38.4 kbps

Q. 100 Consider a system shown in the figure. Let X (f) and Y (f) and denote the Fourier
transforms of x (t) and y (t) respectively. The ideal HPF has the cutoff frequency
10 kHz.

ww
w.E The positive frequencies where Y (f) has spectral peaks are
(A) 1 kHz and 24 kHz
(C) 1 kHz and 14 kHz
(B) 2 kHz and 24 kHz
(D) 2 kHz and 14 kHz

2003
asy ONE MARK

Q. 101
En
The input to a coherent detector is DSB-SC signal plus noise. The noise at the
detector output is
(A) the in-phase component
gin
(B) the quadrature - component

Q. 102
(C) zero

eer
(D) the envelope

The noise at the input to an ideal frequency detector is white. The detector is

ing
operating above threshold. The power spectral density of the noise at the output
is
(A) raised - cosine
(C) parabolic
(B) flat
(D) Gaussian .ne
Q. 103

PSK by.
(A) 6 dB (B) 3 dB
t
At a given probability of error, binary coherent FSK is inferior to binary coherent

(C) 2 dB (D) 0 dB

2003 TWO MARKS

Q. 104 Let X and Y be two statistically independent random variables uniformly


distributed in the ranges (- 1, 1) and (- 2, 1) respectively. Let Z = X + Y . Then
the probability that (z #- 1) is
(A) zero (B) 1
6
(C) 1 (D) 1
3 12

Downloaded From : www.EasyEngineering.net


Downloaded From : www.EasyEngineering.net
GATE SOLVED PAPER - EC COMMUNICATION SYSTEM

Common Data For Q. 105 and 106 :


X (t) is a random process with a constant mean value of 2 and the auto correlation
function Rxx (t) = 4 (e - 0.2 t + 1).
Q. 105 Let X be the Gaussian random variable obtained by sampling the process at
t = ti and let
Q (a) = # - 1 e dy
3 x2
2

a 2p
The probability that 6x # 1@ is
(A) 1 - Q (0.5) (B) Q (0.5)
(C) Q c 1 (D) 1 - Q c 1 m
2 2m 2 2
Q. 106 Let Y and Z be the random variable obtained by sampling X (t) at t = 2 and
t = 4 respectively. Let W = Y - Z . The variance of W is

ww (A) 13.36
(C) 2.64
(B) 9.36
(D) 8.00

w.E
Q. 107 A sinusoidal signal with peak-to-peak amplitude of 1.536 V is quantized into 128
levels using a mid-rise uniform quantizer. The quantization-noise power is
(A) 0.768 V (B) 48 # 10 - 6 V2

asy
(B) 12 # 10 - 6 V2 (D) 3.072 V

Q. 108

En
Let x (t) = 2 cos (800p) + cos (1400pt). x (t) is sampled with the rectangular pulse
train shown in the figure. The only spectral components (in kHz) present in the

gin
sampled signal in the frequency range 2.5 kHz to 3.5 kHz are

eer
ing
(A) 2.7, 3.4
(C) 2.6, 2.7, 3.3, 3.4, 3.6
(B) 3.3, 3.6
(D) 2.7, 3.3
.ne
Q. 109
t
A DSB-SC signal is to be generated with a carrier frequency fc = 1 MHz using a
non-linear device with the input-output characteristic V0 = a0 vi + a1 vi3 where a0
and a1 are constants. The output of the non-linear device can be filtered by an
appropriate band-pass filter.
Let Vi = Aci cos (2pfi ct) + m (t) is the message signal. Then the value of fci (in
MHz) is
(A) 1.0 (B) 0.333
(B) 0.5 (D) 3.0

Common Data For Q. 110 and 111 :


Let m (t) = cos [(4p # 103) t] be the message signal &
c (t) = 5 cos [(2p # 106 t)] be the carrier.

Q. 110 c (t) and m (t) are used to generate an AM signal. The modulation index of the
Total sideband power
generated AM signal is 0.5. Then the quantity is
Carrier power

Downloaded From : www.EasyEngineering.net


Downloaded From : www.EasyEngineering.net
GATE SOLVED PAPER - EC COMMUNICATION SYSTEM

(A) 1 (B) 1
2 4
(C) 1 (D) 1
3 8

Q. 111 c (t) and m (t) are used to generated an FM signal. If the peak frequency deviation
of the generated FM signal is three times the transmission bandwidth of the AM
signal, then the coefficient of the term cos [2p (1008 # 103 t)] in the FM signal (in
terms of the Bessel coefficients) is
(A) 5J4 (3) (B) 5 J8 (3)
2
5
(C) J8 (4) (D) 5J4 (6)
2
Q. 112 Choose the correct one from among the alternative A, B, C, D after matching an
item in Group 1 with most appropriate item in Group 2.

ww Group 1
P. Ring modulator
Group 2
1. Clock recovery

w.E Q. VCO
R. Foster-Seely discriminator
S. Mixer
2. Demodulation of FM
3. Frequency conversion
4. Summing the two inputs

asy
(A) P - 1; Q - 3; R - 2; S - 4
5. Generation of FM
6. Generation of DSB-Sc
(B) P - 6; Q = 5; R - 2; S - 3

En
(C) P - 6; Q - 1; R - 3; S - 2 (D) P - 5; Q - 6; R - 1; S - 3

Q. 113

gin
A superheterodyne receiver is to operate in the frequency range 550 kHz - 1650
kHz, with the intermediate frequency of 450 kHz. Let R = Cmax /Cmin denote the

eer
required capacitance ratio of the local oscillator and I denote the image frequency
(in kHz) of the incoming signal. If the receiver is tuned to 700 kHz, then
(A) R = 4.41, I = 1600
(C) R = 3.0, I = 600
ing
(B) R = 2.10, I - 1150
(D) R = 9.0, I = 1150

Q. 114

.ne
If Eb , the energy per bit of a binary digital signal, is 10 - 5 watt-sec and the one-
sided power spectral density of the white noise, N0 = 10 - 6 W/Hz, then the output
SNR of the matched filter is
(A) 26 dB
(C) 20 dB
(B) 10 dB
(D) 13 dB
t
Q. 115 The input to a linear delta modulator having a step-size 3= 0.628 is a sine wave
with frequency fm and peak amplitude Em . If the sampling frequency fx = 40 kHz,
the combination of the sine-wave frequency and the peak amplitude, where slope
overload will take place is
Em fm
(A) 0.3 V 8 kHz
(B) 1.5 V 4 kHz
(C) 1.5 V 2 kHz
(D) 3.0 V 1 kHz
Q. 116 If S represents the carrier synchronization at the receiver and r represents the
bandwidth efficiency, then the correct statement for the coherent binary PSK is
(A) r = 0.5, S is required (B) r = 1.0, S is required
(C) r = 0.5, S is not required (D) r = 1.0, S is not required

Downloaded From : www.EasyEngineering.net


Downloaded From : www.EasyEngineering.net
GATE SOLVED PAPER - EC COMMUNICATION SYSTEM

Q. 117 A signal is sampled at 8 kHz and is quantized using 8 - bit uniform quantizer.
Assuming SNRq for a sinusoidal signal, the correct statement for PCM signal
with a bit rate of R is
(A) R = 32 kbps, SNRq = 25.8 dB
(B) R = 64 kbps, SNRq = 49.8 dB
(C) R = 64 kbps, SNRq = 55.8 dB
(D) R = 32 kbps, SNRq = 49.8 dB

2002 ONE MARK

Q. 118 A 2 MHz sinusoidal carrier amplitude modulated by symmetrical square wave of


period 100 m sec . Which of the following frequencies will NOT be present in the
modulated signal ?

ww (A) 990 kHz


(C) 1020 kHz
(B) 1010 kHz
(D) 1030 kHz

w.E
Q. 119 Consider a sample signal y (t) = 5 # 10 - 6 # (t)
where x (t) = 10 cos (8p # 103) t and Ts = 100m sec.
+3
/ d (t - nTs)
n =- 3

asy
When y (t) is passed through an ideal lowpass filter with a cutoff frequency of 5
KHz, the output of the filter is

En
(A) 5 # 10 - 6 cos (8p # 103) t
(C) 5 # 10 - 1 cos (8p # 103) t
(b) 5 # 10 - 5 cos (8p # 103) t
(D) 10 cos (8p # 103) t

Q. 120

in a coherent binary FSK system are gin


For a bit-rate of 8 Kbps, the best possible values of the transmitted frequencies

(A) 16 kHz and 20 kHz


(C) 20 kHz and 40 kHz eer
(C) 20 kHz and 32 kHz
(D) 32 kHz and 40 kHz

Q. 121
ing
The line-of-sight communication requires the transmit and receive antennas to

the receiver antenna should be


(A) horizontally polarized
.ne
face each other. If the transmit antenna is vertically polarized, for best reception

(B) vertically polarized


(C) at 45c with respect to horizontal polarization
t
(D) at 45c with respect to vertical polarization

2002 TWO MARKS

Q. 122 An angle-modulated signal is given by


s (t) = cos 2p (2 # 106 t + 30 sin 150t + 40 cos 150t).
The maximum frequency and phase deviations of s (t) are
(A) 10.5 kHz, 140p rad (B) 6 kHz, 80p rad
(C) 10.5 kHz, 100p rad (D) 7.5 kHz, 100p rad

Q. 123 In the figure m (t) = 2 sin 2pt , s (t) = cos 200pt and n (t) = sin 199pt .
t t
The output y (t) will be

Downloaded From : www.EasyEngineering.net


Downloaded From : www.EasyEngineering.net
GATE SOLVED PAPER - EC COMMUNICATION SYSTEM

(A) sin 2pt (B) sin 2pt + sin pt cos 3pt


t t t
(C) sin 2p t + sin 0 . 5 p t cos 1.5pt (D) sin 2p t + sin pt cos 0.75pt
t t t t
Q. 124 A signal x (t) = 100 cos (24p # 103) t is ideally sampled with a sampling period of
50m sec ana then passed through an ideal lowpass filter with cutoff frequency of
15 kHz. Which of the following frequencies is/are present at the filter output ?
(A) 12 kHz only (B) 8 kHz only
(C) 12 kHz and 9 kHz (D) 12 kHz and 8 kHz

ww
Q. 125 If the variance ax2 of d (n) = x (n) - x (n - 1) is one-tenth the variance ax2 of stationary
zero-mean discrete-time signal x (n), then the normalized autocorrelation function
Rxx (k)
at k = 1 is

w.E (A)ax20.95
(C) 0.10
(B) 0.90
(D) 0.05

2001 asy ONE MARK

Q. 126
En
A bandlimited signal is sampled at the Nyquist rate. The signal can be recovered
by passing the samples through
(A) an RC filter
(B) an envelope detector gin
(C) a PLL
eer
(D) an ideal low-pass filter with the appropriate bandwidth

Q. 127 The PDF of a Gaussian random variable X is given by ing


(x - 4)

.ne
2

px (x) = 1 e - 18 . The probability of the event {X = 4} is


3 2p
(A) 1 1

(C) 0
2
(B)
3 2p
(D) 1
4
t
2001 TWO MARKS

Q. 128 A video transmission system transmits 625 picture frames per second. Each frame
consists of a 400 # 400 pixel grid with 64 intensity levels per pixel. The data rate
of the system is
(A) 16 Mbps (B) 100 Mbps
(C) 600 Mbps (D) 6.4 Gbps

Q. 129 The Nyquist sampling interval, for the signal sin c (700t) + sin c (500t) is
(A) 1 sec (B) p sec
350 350
(C) 1 sec (D) p sec
700 175

Downloaded From : www.EasyEngineering.net


Downloaded From : www.EasyEngineering.net
GATE SOLVED PAPER - EC COMMUNICATION SYSTEM

Q. 130 During transmission over a communication channel, bit errors occur independently
with probability p. If a block of n bits is transmitted, the probability of at most
one bit error is equal to
(A) 1 - (1 - p) n (B) p + (n - 1)( 1 - p)
(C) np (1 - p) n - 1 (D) (1 - p) n + np (1 - p) n - 1

Q. 131 The PSD and the power of a signal g (t) are, respectively, Sg (w) and Pg . The PSD
and the power of the signal ag (t) are, respectively,
(A) a2 Sg (w) and a2 Pg (B) a2 Sg (w) and aPg
(C) aSg (w) and a2 Pg (D) aSg (w) and aPs

2000 ONE MARK

Q. 132 The amplitude modulated waveform s (t) = Ac [1 + Ka m (t)] cos wc t is fed to an

ww ideal envelope detector. The maximum magnitude of K0 m (t) is greater than 1.


Which of the following could be the detector output ?
(B) Ac2 [1 + Ka m (t)] 2

w.E (A) Ac m (t)


(C) [Ac (1 + Ka m (t)] (D) Ac [1 + Ka m (t)] 2

Q. 133

asy
The frequency range for satellite communication is
(A) 1 KHz to 100 KHz
(C) 10 MHz to 30 MHz
(B) 100 KHz to 10 KHz
(D) 1 GHz to 30 GHz

En
Q. 134
2000
gin TWO MARKS

In a digital communication system employing Frequency Shift Keying (FSK), the

eer
0 and 1 bit are represented by sine waves of 10 KHz and 25 KHz respectively.
These waveforms will be orthogonal for a bit interval of
(A) 45m sec
(C) 50m sec
(B) 200m sec
(D) 250m sec ing
Q. 135
the output signal in the figure is .ne
A message m (t) bandlimited to the frequency fm has a power of Pm . The power of

t
(A) Pm cos q (B) Pm
2 4
2 2
(C) Pm sin q (D) Pm cos q
4 4
Q. 136 The Hilbert transform of cos w1 t + sin w2 t is
(A) sin w1 t - cos w2 t (B) sin w1 t + cos w2 t
(C) cos w1 t - sin w2 t (D) sin w1 t + sin w2 t

Q. 137 In a FM system, a carrier of 100 MHz modulated by a sinusoidal signal of 5


KHz. The bandwidth by Carson’s approximation is 1 MHz. If y (t) = (modulated
waveform) 3 , than by using Carson’s approximation, the bandwidth of y (t) around
300 MHz and the and the spacing of spectral components are, respectively.

Downloaded From : www.EasyEngineering.net


Downloaded From : www.EasyEngineering.net
GATE SOLVED PAPER - EC COMMUNICATION SYSTEM

(A) 3 MHz, 5 KHz (B) 1 MHz, 15 KHz


(C) 3 MHz, 15 KHz (D) 1 MHz, 5 KHz

1999 ONE MARK

Q. 138 The input to a channel is a bandpass signal. It is obtained by linearly modulating


a sinusoidal carrier with a single-tone signal. The output of the channel due to
this input is given by
y (t) = (1/100) cos (100t - 10-6) cos (106 t - 1.56)
The group delay (tg) and the phase delay (t p) in seconds, of the channel are
(A) tg = 10-6, t p = 1.56 (B) tg = 1.56, t p = 10-6
(C) tg = 108, t p = 1.56 # 10-6 (D) tg = 108, t p = 1.56

Q. 139 A modulated signal is given by s (t) = m1 (t) cos (2pfc t) + m2 (t) sin (2pfc t)

ww where the baseband signal m1 (t) and m2 (t) have bandwidths of 10 kHz, and
15 kHz, respectively. The bandwidth of the modulated signal, in kHz, is

w.E (A) 10
(C) 25
(B) 15
(D) 30

Q. 140

asy
A modulated signal is given by s (t) = e-at cos [(wc + Dw) t] u (t),
where a wc and Dw are positive constants, and wc >> Dw . The complex envelope
of s (t) is given by

En
(A) exp (- at) exp [j (wc + Dw) t] u (t)
(B) exp (- at) exp (jDwt) u (t)
(C) exp (jDwt) u (t)
gin
(D) exp [jwc + Dw) t]
eer
Q. 141
1999
ing
The Nyquist sampling frequency (in Hz) of a signal given by
TWO MARKS

6 # 10 4 sin c2 (400t) * 106 sin c3 (100t) is


(A) 200 (B) 300 .ne
Q. 142
(C) 500 (D) 1000
t
The peak-to-peak input to an 8-bit PCM coder is 2 volts. The signal power-to-
quantization noise power ratio (in dB) for an input of 0.5 cos (wm t) is
(A) 47.8 (B) 49.8
(C) 95.6 (D) 99.6

Q. 143 The input to a matched filter is given by


"10 sin (2p # 10 t) 0 < 1 < 10 sec
6 -4
s (t) = 0
otherwise
The peak amplitude of the filter output is
(A) 10 volts (B) 5 volts
(C) 10 millivolts (D) 5 millivolts

Q. 144 Four independent messages have bandwidths of 100 Hz, 200 Hz and 400 Hz ,
respectively. Each is sampled at the Nyquist rate, and the samples are time
division multiplexed (TDM) and transmitted. The transmitted sample rate (in

Downloaded From : www.EasyEngineering.net


Downloaded From : www.EasyEngineering.net
GATE SOLVED PAPER - EC COMMUNICATION SYSTEM

Hz) is
(A) 1600 (B) 800
(C) 400 (D) 200

1998 ONE MARK

Q. 145 The amplitude spectrum of a Gaussian pulse is


(A) uniform (B) a sine function
(C) Gaussian (D) an impulse function

Q. 146 The ACF of a rectangular pulse of duration T is


(A) a rectangular pulse of duration T
(B) a rectangular pulse of duration 2T

ww (C) a triangular pulse of duration T


(D) a triangular pulse of duration 2T

w.E
Q. 147 The image channel selectivity of superheterodyne receiver depends upon
(A) IF amplifiers only
(B) RF and IF amplifiers only

asy
(C) Preselector, RF and IF amplifiers
(D) Preselector, and RF amplifiers only

Q. 148
En
In a PCM system with uniform quantisation, increasing the number of bits from

(A) 9 gin
8 to 9 will reduce the quantisation noise power by a factor of
(B) 8

Q. 149
(C) 4

Flat top sampling of low pass signals


(D) 2
eer
(A) gives rise to aperture effect
(C) leads to aliasing ing
(B) implies oversampling
(D) introduces delay distortion

Q. 150
.ne
A DSB-SC signal is generated using the carrier cos (we t + q) and modulating
signal x (t). The envelope of the DSB-SC signal is
(A) x (t)
(C) only positive portion of x (t)
(B) x (t)
(D) x (t) cos q
t
Q. 151 Quadrature multiplexing is
(A) the same as FDM
(B) the same as TDM
(C) a combination of FDM and TDM
(D) quite different from FDM and TDM

Q. 152 The Fourier transform of a voltage signal x (t) is X (f). The unit of X (f) is
(A) volt (B) volt-sec
(C) volt/sec (D) volt 2

Q. 153 Compression in PCM refers to relative compression of


(A) higher signal amplitudes (B) lower signal amplitudes
(C) lower signal frequencies (D) higher signal frequencies

Downloaded From : www.EasyEngineering.net


Downloaded From : www.EasyEngineering.net
GATE SOLVED PAPER - EC COMMUNICATION SYSTEM

Q. 154 For a give data rate, the bandwidth B p of a BPSK signal and the bandwidth B 0
of the OOK signal are related as
(A) B p = B 0 (B) B p = B 0
4 2
(C) B p = B 0 (D) B p = 2B 0

Q. 155 The spectral density of a real valued random process has


(A) an even symmetry (B) an odd symmetry
(C) a conjugate symmetry (D) no symmetry

Q. 156 The probability density function of the envelope of narrow band Gaussian noise is
(A) Poisson (B) Gaussian
(C) Rayleigh (D) Rician

ww
Q. 157
1997 ONE MARK

The line code that has zero dc component for pulse transmission of random

w.E binary data is


(A) Non-return to zero (NRZ)

asy
(B) Return to zero (RZ)
(C) Alternate Mark Inversion (AM)

Q. 158 En
(D) None of the above
2
A probability density function is given by p (x) = Ke-x /2 - 3 < x < 3. The value
of K should be
(A) 1 gin (B) 2

(C) 1
2p
(D) 1 eer
p

Q. 159
2 p p 2

ing
A deterministic signal has the power spectrum given in the figure is, The minimum

.ne
sampling rate needed to completely represent this signal is

(A) 1 kHz (B) 2 kHz


(C) 3 kHz (D) None of these

Q. 160 A communication channel has first order low pass transfer function. The channel
is used to transmit pulses at a symbol rate greater than the half-power frequency
of the low pass function. Which of the network shown in the figure is can be used
to equalise the received pulses?

Downloaded From : www.EasyEngineering.net


Downloaded From : www.EasyEngineering.net
GATE SOLVED PAPER - EC COMMUNICATION SYSTEM

Q. 161 The power spectral density of a deterministic signal is given by [sin (f) /f 2] where

ww f is frequency. The auto correlation function of this signal in the time domain is
(A) a rectangular pulse (B) a delta function

w.E (C) a sine pulse

1996
(D) a triangular pulse

ONE MARK

Q. 162
asy
A rectangular pulse of duration T is applied to a filter matched to this input. The
out put of the filter is a

En
(A) rectangular pulse of duration T
(B) rectangular pulse of duration 2T
(C) triangular pulse
gin
Q. 163
(D) sine function

eer
The image channel rejection in a superheterodyne receiver comes from
(A) IF stages only
(C) detector and RF stages only ing
(B) RF stages only
(D) detector RF and IF stages

1996
.ne TWO MARKS

Q. 164
t
The number of bits in a binary PCM system is increased from n to n + 1. As a
result, the signal to quantization noise ratio will improve by a factor
(A) n + 1 (B) 2(n + 1)/n
n
(C) 22 (n + 1)/n (D) which is independent of n

Q. 165 The auto correlation function of an energy signal has


(A) no symmetry (B) conjugate symmetry
(C) odd symmetry (D) even symmetry

***********

Downloaded From : www.EasyEngineering.net


Downloaded From : www.EasyEngineering.net
GATE SOLVED PAPER - EC COMMUNICATION SYSTEM

SOLUTIONS

Sol. 1 Option (B) is correct.


In ideal Nyquist Channel, bandwidth required for ISI (Inter Symbol reference)
free transmission is
W = Rb
2
Here, the used modulation is 32 - QAM (Quantum Amplitude modulation
i.e., q = 32
or 2v = 32

ww v = 5 bits
So, the signaling rate (sampling rate) is
Rb = R

w.E 5
(R " given bit rate)

Hence, for ISI free transmission, minimum bandwidth is

Sol. 2 asy W = Rb = R kHz


Option (B) is correct.
2 10

i.e., En
Given, random variables U and V with mean zero and variances 1 and 1
U =V=0
4 9

su2 = 1
4
sv2 = 1
gin
and

so, P ^U $ 0h = 1
9
eer
and P ^V $ 0h =
2
1
2 ing
The distribution is shown in the figure below
.ne
t

fu ^u h = 1 e -u 2
2s
2p su2
u

fv ^v h = 1 e -v 2
2s
2p sv2
v

We can express the distribution in standard form by assuming


X = u - 0 = u = 2U
su Y2
and Y = v - 0 = v = 3V
sv Y3

Downloaded From : www.EasyEngineering.net


Downloaded From : www.EasyEngineering.net
GATE SOLVED PAPER - EC COMMUNICATION SYSTEM

for which we have


X = 2U = 0
Y = 2V = 0
and X2 = 4U2 = 1
also, Y2 = 9V2 = 1
Therefore, X - Y is also a normal random variable with
X-Y = 0
Hence,
P ^X - Y $ 0h = P ^X - Y # 0h = 1
2
or, we can say
P ^2U - 3V # 0h = 1
2
Thus, P ^3V $ 2U h = 1
2

ww
Sol. 3 Option (C) is correct.
The mean of random variables U and V are both zero

w.E i.e., U =V=0


Also, the random variables are identical
i.e., fU ^u h = fV ^v h
or,
asy FU ^u h = FV ^v h
i.e., their cdf are also same. So,
FU ^u h = F2V ^2v h
En
i.e., the cdf of random variable 2V will be also same but for any instant

Therefore,
2V $ U

G ^x h = F ^x h
gin
but, x G ^x h $ xF ^x h
or, 6F ^x h - G ^x h@ x # 0 eer
Sol. 4 Option (C) is correct.
P ^U =+ 1h = P ^U =- 1h = 1
ing
Given,
2
.ne
where U is a random variable which is identical to V i.e.,
P ^V =+ 1h = P ^V =- 1h = 1
2
So, random variable U and V can have following values
U =+ 1, - 1; V =+ 1, - 1
t
Therefore the random variable U + V can have the following values,
- 2 When U = V =- 1
U + V = *0 When U = 1,V = 1 or u =- 1, v = 1
2 When U = V = 1
Hence, we obtain the probabilities for U + V as follows
U+V P ^U + V h
-2 1 1=1
2#2 4
0 1 1 1 1 1
b2 # 2l+b2 # 2l = 2
2 1 1=1
2#2 4
Therefore, the entropy of the ^U + V h is obtained as

Downloaded From : www.EasyEngineering.net


Downloaded From : www.EasyEngineering.net
GATE SOLVED PAPER - EC COMMUNICATION SYSTEM

H ^U + V h = / P ^U + V h log 2 ' 1
P ^U + V h 1
= 1 log 2 4 + 1 log 2 2 + 1 log 2 4
4 2 4
2
= + + 1 2
4 2 4
=3
2
Sol. 5 Option (D) is correct.
For the shown received signal, we conclude that if 0 is the transmitted signal then
the received signal will be also zero as the threshold is 1 and the pdf of bit 0 is
not crossing 1. Again, we can observe that there is an error when bit 1 is received
as it crosses the threshold. The probability of error is given by the area enclosed
by the 1 bit pdf (shown by shaded region)

ww
w.E
asy
P (error when bit 1 received) = 1 # 1 # 0.25 = 1

En 2
P b received 1 l = 1
8
or

gin
transmitted 0 8
Since, the 1 and 0 transmission is equiprobable:
i.e., P ^ 0 h = P ^1 h = 1
Hence bit error rate (BER) is
2
eer
transmitted 1 ing
BER = P b received 0 l P ^0 h + P b received 1 l P ^1 h
transmitted 0
= 0+1 #1
1
8 2
.ne
Sol. 6 Option (B) is correct.
=
16
t
The optimum threshold is the threshold value for transmission as obtained at the
intersection of two pdf. From the shown pdf. We obtain at the intersection
(transmitted, received) = b 4 , 1 l
5 5
we can obtain the intersection by solving the two linear eqs
x+y = 1 pdf of received bit 0
y = 0. 5 x pdf of received bit 1
2
Hence for threshold = 4 , we have
5
BER = P b received 1 l P ^0 h + P b received 0 l P ^1 h
transmitted 0 transmitted 1
= b # # l# +b # # 1l# 1
1 1 1 1 1 4
2 5 2 2 2 5 5 2
= 1 <(BER for threshold = 1)
20

Downloaded From : www.EasyEngineering.net


Downloaded From : www.EasyEngineering.net
GATE SOLVED PAPER - EC COMMUNICATION SYSTEM

Hence, optimum threshold is 4


5
Sol. 7 Option (A) is correct.
The mean square value of a stationary process equals the total area under the
graph of power spectral density, that is
#S
3
E [X 2 (t)] = X (f ) df
-3

E [X 2 (t)] = 1 #S
3
or, (w) dw
2p -3
X

E [X 2 (t)] = 2 # 1 # SX (w) dw
3
or, (Since the PSD is even)
2p 0
= 1 [area under the triangle + integration of delta function]
p
= 1 ;2 b 1 # 1 # 103 # 6 l + 400E

ww p 2
= 1 66000 + 400@ = 6400
p p

w.E E [X (t)] is the absolute value of mean of signal X (t) which is also equal to value
of X (w) at (w = 0).

asy
From given PSD
SX (w) w = 0 = 0

En
SX (w) = X (w) 2 = 0

X (w) 2w = 0 = 0

X (w) w = 0 = 0 gin
Sol. 8 Option (C) is correct. eer
For raised cosine spectrum transmission bandwidth is given as
BT = W (1 + a) ing a " Roll of factor
BT = Rb (1 + a)
2 .ne
Rb " Maximum signaling rate

3500 = Rb (1 + 0.75)
2
Rb = 3500 # 2 = 4000
1.75
t
Sol. 9 Option (D) is correct.
Entropy function of a discrete memory less system is given as
N-1
H = / P log b P1 l
k
k
k=0
where Pk is probability of symbol Sk .
For first two symbols probability is same, so
N-1
H = P1 log b 1 l + P2 log b 1 l + / Pk log b 1 l
P1 P2 k=3
Pk
N-1
=-e P1 log P1 + P2 log P2 + / P log P o
k k
k=3
N-1
=-e 2P log P + / P log P o
k k (P1 = P2 = P)
k=3

Downloaded From : www.EasyEngineering.net


Downloaded From : www.EasyEngineering.net
GATE SOLVED PAPER - EC COMMUNICATION SYSTEM

Now, P1 = P + e, P2 = P - e
N-1
So, Hl =-=(P + e) log (P + e) + (P - e) log (P - e) + / P log P G
k k
k=3

By comparing, Hl < H , Entropy of source decreases.


Sol. 10 Option (B) is correct.
Probability density function of uniformly distributed variables X and Y is shown
as

ww P &[max (x, y)] < 1 0


2

w.E Since X and Y are independent.


P &[max (x, y)] < 1 0 = P b X < 1 l P bY < 1 l
2 2 2

asyP b X < 1 l = shaded area = 3


2 4
Similarly for Y :
En P bY < 1 l = 3
2 4
So
2
gin
P &[max (x, y)] < 1 0 = 3 # 3 = 9
4 4 16

Alternate Method: eer


ing
From the given data since random variables X and Y lies in the interval [- 1, 1]
as from the figure X , Y lies in the region of the square ABCD .

.ne
t

Probability for max 6X, Y @ < 1/2 : The points for max 6X, Y @ < 1/2 will be inside
the region of square AEFG .
So, P &max 6X, Y @ < 1 0 = Area of 4AEFG
2 Area of square ABCD
3 3
= 9
2 #2
=
2#2 16
Sol. 11 Option (B) is correct.

Downloaded From : www.EasyEngineering.net


Downloaded From : www.EasyEngineering.net
GATE SOLVED PAPER - EC COMMUNICATION SYSTEM

In a coherent binary PSK system, the pair of signals s1 (t) and s2 (t) used to
represent binary system 1 and 0 respectively.
s1 (t) = 2E sin wc t
T
s2 (t) =- 2E sin w t
c
T
where 0 # t # T , E is the transmitted energy per bit.
General function of local oscillator
f1 (t) = 2 sin (w t), 0 # t < T
c
T
But here local oscillator is ahead with 45c. so,
f1 (t) = 2 sin (w t + 45c)
c
T
The coordinates of message points are

ww s11 =
0
T
# s (t) f (t) dt
1 1

w.E 2E sin w t
T
2 sin (w t + 45c) dt
= #
0 T c
T c

2E T
= # sin (w t) sin (w t + 45c) dt
asy
c c
T 0

2E 2 T
1 [sin 45c + sin (2w t + 45c)] dt
=
T T # 2 c

En
0

= 1
T
1 dt + 1 E Tsin (2w t + 45c) dt
T
E # 2 T #0 c

gin
0
1 444444 42
0
444444 43
= E
2
Similarly, s21 =- E
2 eer
Signal space diagram
ing
.ne
t
Now here the two message points are s11 and s21 .
The error at the receiver will be considered.
When : (i) s11 is transmitted and s21 received
(ii) s21 is transmitted and s11 received
So, probability for the 1st case will be as :
P b s21 received l = P (X < 0) (as shown in diagram)
s11 transmitted
= P _ E/2 + N < 0i
= P _N < - E/2 i
Taking the Gaussian distribution as shown below :

Downloaded From : www.EasyEngineering.net


Downloaded From : www.EasyEngineering.net
GATE SOLVED PAPER - EC COMMUNICATION SYSTEM

Mean of the Gaussian distribution = E/2


Variance = N 0
2
Putting it in the probability function :
`x + E/2 j
2

E = 0
1
P bN < -
2l # e- 2N 0 /2 dx
-3
2p N 0
2
1 e- `x + NE/2 j dx
2

ww
0
= #-3 pN 0
0

x + E/2

w.E Taking,
N 0 /2
=t

dx = N 0 dt
2

asy
So, P _N < - E/2 i = #
3

E/N 0
1 e- t2 dt Q
2p
2

c
E
N0 m

En
where Q is error function.
Since symbols are equiprobable in the 2 nd case
So,
P b s11 received l = Q c E m gin
s21 transmitted
So the average probability of error
N0
eer
= 1 ;P b s21 received l + P b s11 received lE
2 s11 transmitted
= 1 =Q c E +Q
s21 transmitted
E =Q
ing E

Option ( ) is correct.
2 N0 m c N 0 mG c
.ne
N0 m

t
Sol. 12

Sol. 13 Option (B) is correct.


General equation of FM and PM waves are given by
t
fFM (t) = Ac cos ;wc t + 2pk f # m (t) dtE
0

fPM (t) = Ac cos [wc t + k p m (t)]


For same maximum phase deviation.
t
k p [m (t)] max = 2pk f ; # m (t) dtE
0 max
k p # 2 = 2pk f [x (t)] max
t
where, x (t) = # m (t) dt
0

Downloaded From : www.EasyEngineering.net


Downloaded From : www.EasyEngineering.net
GATE SOLVED PAPER - EC COMMUNICATION SYSTEM

[x (t)] max = 4
So, k p # 2 = 2p k f # 4
kp
= 4p
kf

Sol. 14 Option (A) is correct.


jw + a
GC (s) = s + a =
ww Phase lead angle
s+b jw + b
f = tan-1 a w k - tan-1 a w k

w.E f = tan-1 K a
KK
a
Jw - wN
w 2
OO
b

b O = tan-1 w (b - a)
c ab + w 2 m

asy
For phase-lead compensation f > 0
L
1+
ab P

En b-a > 0
b >a

gin
Note: For phase lead compensator zero is nearer to the origin as compared to
pole, so option (C) can not be true.
Sol. 15 Option (A) is correct.
eer
f = tan-1 a w k - tan-1 a w k

df
dw
=
1/a
a

-
1 +a k 1 +awk
w
1/b ing
b

=0

.ne
2 2
a b
1 + w2 = 1 + 1 w2
a ab2 b b a2
1 - 1 = w2 1 - 1
a b ab b a b l
t
w = ab = 1 # 2 = 2 rad/ sec
Sol. 16 Option (D) is correct.
Quantized 4 level require 2 bit representation i.e. for one sample 2 bit are
required. Since 2 sample per second are transmitted we require 4 bit to be
transmitted per second.
Sol. 17 Option (B) is correct.
In FM the amplitude is constant and power is efficient transmitted. No variation
in power.
There is most bandwidth efficient transmission in SSB- SC. because we transmit
only one side band.
Simple Diode in Non linear region ( Square law ) is used in conventional AM that
is simplest receiver structure.

Downloaded From : www.EasyEngineering.net


Downloaded From : www.EasyEngineering.net
GATE SOLVED PAPER - EC COMMUNICATION SYSTEM

In VSB dc. component exists.


Sol. 18 Option (A) is correct.
We have Sx (f) = F {Rx (t)} = F {exp (- pt2)}
2
= e- pf
The given circuit can be simplified as

Power spectral density of output is


Sy (f) = G (f) 2 Sx (f)
2
= j2pf - 1 2 e- pf
2
= ( (2pf) 2 + 1) 2 e- pf

ww
Sol. 19
or
Option (B) is correct.
Sy (f) = (4p2 f 2 + 1) e- pf
2

w.E Highest frequency component in m (t) is fm = 4000p/2p = 2000 Hz


Carrier frequency fC = 1 MHz

asy
For Envelope detector condition
1/fC << RC << 1/fm

Sol. 20
En
Option (D) is correct.
1 μs << RC << 0.5 ms

Four phase signal constellation is shown below


gin
eer
ing
.ne
Now d2 = r 12 + r 12
d2 = 2r 12
t
r1 = d/ 2 = 0.707d

q = 2p = 2p = p
M 8 4
Applying Cooine law we have

Downloaded From : www.EasyEngineering.net


Downloaded From : www.EasyEngineering.net
GATE SOLVED PAPER - EC COMMUNICATION SYSTEM

d2 = r 22 + r 22 - 2r 22 cos p
4
= 2r 22 - 2r 22 1/ 2 = (2 - 2 ) r 22
or r2 = d = 1.3065d
2- 2
Sol. 21 Option (D) is correct.
Here Pe for 4 PSK and 8 PSK is same because Pe depends on d . Since Pe is same,
d is same for 4 PSK and 8 PSK.

ww
w.E Additional Power SNR
= (SNR) 2 - (SNR) 1

asy = 10 log b ES2 l - 10 log b ES1 l


No
E
= 10 log b S2 l
No

En ES1
= 10 log a r2 k & 20 log a r2 k = 20 log 1.3065d
2

Additional SNR = 5.33 dB ginr1 r1 0.707d

Sol. 22 Option (C) is correct.


Conventional AM signal is given by eer
ing
x (t) = AC [1 + mm (t)] cos (2pfC t)
Where m < 1, for no over modulation.
In option (C)
x (t) = AC :1 + 1 m (t)D cos (2pfC t).ne
Sol. 23
4
Option (B) is correct.
4
t
Thus m = 1 < 1 and this is a conventional AM-signal without over-modulation

(6) 2
Power P = = 18 W
2
Sol. 24 Option (C) is correct.
Impulse response of the matched filter is given by
h (t) = S (T - t)

Downloaded From : www.EasyEngineering.net


Downloaded From : www.EasyEngineering.net
GATE SOLVED PAPER - EC COMMUNICATION SYSTEM

Sol. 25 Option (B) is correct.


Let response of LPF filters
1, f < 1 MHz
H (f ) = *
0, elsewhere
Noise variance (power) is given as
H (f ) No df = 22 (given)
fo 2
P = s2 = #0
a

ww #0 2 # 10-20 df = 22
1 # 106

w.E 2 # 10 # 10 = 22
-20 6
a
a = 1014
2

Sol. 26
or
Option (D) is correct. asy a = 107

En
Probability of error is given by
Pe = 1 [P (0/1) + P (1/0)]
2
P (0/1) =
a/2
gin
#- 3 0.5e- a n - a dn = 0.5e-10
where a = 2 # 10-6 V and a = 107 V - 1
eer
P (1/0) = #a/2 0.5e
3 -a n
dn = 0.5e -10

ing
Sol. 27 Option (C) is correct.
Pe = 0.5e-10
.ne
S (t) = sin c (500t) sin c (700t)
S (f ) is convolution of two signals whose spectrum covers f 1 = 250 Hz and
f 2 = 350 Hz . So convolution extends
t
f = 25 + 350 = 600 Hz
Nyquist sampling rate
N = 2f = 2 # 600 = 1200 Hz
Sol. 28 Option (D) is correct.
For the given system, output is written as
y (t) = d [x (t) + x (t - 0.5)]
dt
dx (t) dx (t - 0.5)
y (t) = +
dt dt
Taking Laplace on both sides of above equation
Y (s) = sX (s) + se-0.5s X (s)
Y (s)
H (s) = = s (1 + e-0.5s)
X (s)

Downloaded From : www.EasyEngineering.net


Downloaded From : www.EasyEngineering.net
GATE SOLVED PAPER - EC COMMUNICATION SYSTEM

H (f ) = jf (1 + e-0.5 # 2pf ) = jf (1 + e- pf )
Power spectral density of output
SY (f ) = H (f ) 2 SX (f ) = f 2 (1 + e- pf ) 2 SX (f )
For SY (f ) = 0 , 1 + e- pf = 0
f = (2n + 1) f0
or f0 = 1 KHz
Sol. 29 Option (C) is correct.
cos (2pfm t) cos (2pfc t) $ DSB suppressed carrier
cos (2pfc t) $ Carrier Only
cos [2p (fc + fm) t] $ USB Only
[1 + cos (2pfm t) cos (2pfc t)] $ USB with carrier
Sol. 30 Option (C) is correct.

ww We have
p (X = 0) = p (Y = 0) = 1

w.E 2
p (X = 1) = p (Y = 1) = 1
4

Let
asy p (X = 2) = p (Y = 2) =

X+Y = 2 $ A
1
4

and
En X-Y = 0 $ B
Now
P (X + Y = 2 X - Y = 0) = gin P (A + B)
P (B)

eer
Event P (A + B) happen when X + Y = 2 and X - Y = 0 . It is only the case
when X = 1 and Y = 1.
Thus P (A + B) = 1 # 1 = 1
4 4 16 ing
Now event P (B) happen when
.ne
X - Y = 0 It occurs when X = Y , i.e.
X = 0 and Y = 0 or
X = 1 and Y = 1 or
t
X = 2 and Y = 2
Thus P (B) = 1 # 1 + 1 # 1 + 1 # 1 = 6
2 2 4 4 4 4 16
P (A + B) 1/16
Now = =1
P (B) 6/16 6
Sol. 31 Option (B) is correct.
The mean is
X = Sxi pi (x)
= 1 # 0.1 + 2 # 0.2 + 3 # 0.4 + 4 # 0.2 + 5 # 0.1
= 0.1 + 0.4 + 1.2 + 0.8 + 0.5 = 3.0
X = Sxi2 pi (x)
2

= 1 # 0.1 + 4 # 0.2 + 9 # 0.4 + 16 # 0.2 + 25 # 0.1

Downloaded From : www.EasyEngineering.net


Downloaded From : www.EasyEngineering.net
GATE SOLVED PAPER - EC COMMUNICATION SYSTEM

= 0.1 + 0.8 + 3.6 + 3.2 + 2.5 = 10.2


Variance sx2 = X2 - ^X h2
= 10.2 - (3) 2 = 1.2
Sol. 32 Option (C) is correct.
m (t) = 1 cos w1 t - 1 sin w2 t
2 2
sAM (t) = [1 + m (t)] cos wc t
m (t) max
Modulation index =
Vc
2 2
m = `1j +`1j = 1
2 2 2
` 2j
1 2
2
h = m # 100% = 1 2 # 100% = 20%

ww ` 2j +2
2
m +2
Sol. 33 Option (B) is correct.
C1 = B log2 `1 + S j
w.E We have

. B log2 ` S j
N
N
As S >> 1
N

asy
If we double the S ratio then
N

En C2 . B log2 ` 2S j
N

. B + C1 gin
. B log2 2 + B log2 S
N

Sol. 34 Option (C) is correct.


eer
We have
or
SNR = 1.76 + 6n
43.5 = 1.76 + 6n
ing
6n = 43.5 + 1.76
6n = 41.74 $ n . 7 .ne
No. of quantization level is

Step size required is


27 = 128 t
5 - (- 5)
= VH - VL = = 10
128 128 128
= .078125
. .0667
Sol. 35 Option (B) is correct.
For positive values step size
s+ = 0.05 V
For negative value step size
s- = 0 . 1 V
No. of quantization in + ive is
= 5 = 5 = 100
s+ 0.05

Downloaded From : www.EasyEngineering.net


Downloaded From : www.EasyEngineering.net
GATE SOLVED PAPER - EC COMMUNICATION SYSTEM

Thus 2n + = 100 $ n+ = 7
No. of quantization in - ve
Q1 = 5 = 5 = 50
s- 0. 1
2n = 50 $ n - = 6
-
Thus
S
` N j+ = 1.76 + 6n = 1.76 + 42 = 43.76 dB
+

S
` N j- = 1.76 + 6n = 1.76 + 36 = 37.76 dB
-

S
Best ` N j0 = 43.76 dB
Sol. 36 Option (A) is correct.
We have xAM (t) = Ac cos wc + 2 cos wm t cos wc t

ww = AC c1 + 2 cos wm t m cos wc t
Ac
For demodulation by envelope demodulator modulation index must be less than

w.E or equal to 1.
Thus 2 #1
Ac

asy Ac $ 2
Hence minimum value of Ac = 2
Sol. 37
En
Option (A) is correct.

gin
CDF is the integration of PDF. Plot in option (A) is the integration of plot given
in question.
Sol. 38 Option (A) is correct.
The entropy is
eer
ing
m
H = / pi log2 1 bits
i=1
pi
p1 = p2 = ... = pn = 1
Since
n
H = / 1 log n = log n
n
.ne
Sol. 39 Option (C) is correct.
PSD of noise is
i=1
n

N0 = K
t ...(1)
2
The 3-dB cut off frequency is
fc = 1 ...(2)
2pRC
Output noise power is
= N0 = c N0 m 1 = Kpfc
4RC 2 2RC
Sol. 40 Option (D) is correct.
At receiving end if we get two zero or three zero then its error.
Let p be the probability of 1 bit error, the probability that transmitted bit error
is
= Three zero + two zero and single one

Downloaded From : www.EasyEngineering.net


Downloaded From : www.EasyEngineering.net
GATE SOLVED PAPER - EC COMMUNICATION SYSTEM

= 3 C3 p3 + 3C2 p2 (1 - p)
= p3 + p2 (1 - p)
Sol. 41 Option (D) is correct.
Bandwidth of TDM is
= 1 (sum of Nyquist Rate)
2
= 1 [2W + 2W + 4W + 6W] = 7W
2
Sol. 42 Option (B) is correct.
We have qi = 2p105 t + 5 sin (2p1500t) + 7.5 sin (2p1000t)
wi = dqi = 2p105 + 10p1500 cos (2p1500t) + 15p1000 cos (2p1000t)
dt
Maximum frequency deviation is

ww 3wmax = 2p (5 # 1500 + 7.5 # 1000)


3 fmax = 15000

w.E Option (C) is correct.


3f
Modulation index is = max = 15000 = 10
fm 1500

asy
Sol. 43

Sol. 44 Option (B) is correct.

En
fm = 4 KHz
fs = 2fm = 8 kHz
Bit Rate
The minimum transmission bandwidth is gin
Rb = nfs = 8 # 8 = 64 kbps

BW = Rb = 32 kHz
2
eer
Sol. 45 Option (C) is correct.
S0
c N m = 1.76 + 6n dB ing
0

= 1.76 + 6 # 8 = 49.76 dB
.ne We have n = 8

Sol. 46 Option (B) is correct.


As Noise \ 12
L
t
To reduce quantization noise by factor 4, quantization level must be two times
i.e. 2L .
Now L = 2n = 28 = 256
Thus 2L = 512
Sol. 47 Option (C) is correct.
Autocorrelation is even function.
Sol. 48 Option (B) is correct.
Power spectral density is non negative. Thus it is always zero or greater than zero.
Sol. 49 Option (A) is correct.
The variance of a random variable x is given by
E [X2] - E2 [X]

Downloaded From : www.EasyEngineering.net


Downloaded From : www.EasyEngineering.net
GATE SOLVED PAPER - EC COMMUNICATION SYSTEM

Sol. 50 Option (A) is correct.


A Hilbert transformer is a non-linear system.
Sol. 51 Option (D) is correct.
Slope overload distortion can be reduced by increasing the step size
3 $ slope of x (t)
Ts
Sol. 52 Option (C) is correct.
sin (4pWt)
We have p (t) =
4pWt (1 - 16W2 t2)
at t = 1 it is 0 form. Thus applying L' Hospital rule
4W 0
4pW cos (4pWt)
p( ) =
1
4W

4pW [1 - 48W2 t2]

ww =
cos (4pWt)
1 - 48W2 t2
= cos p = 0.5
1-3

w.E
Sol. 53 Option (B) is correct.
The block diagram is as shown below

asy
En
gin
Here M1 (f) = Mt (f) eer
Y1 (f) = M (f) c e
j 2p B
+ e -j2pB
2 m
ing
.ne
j 2p B
Y2 (f) = M1 (f) c e - e -j2pB
2 m

Y (f) = Y1 (f) + Y2 (f)


All waveform is shown below
t

Downloaded From : www.EasyEngineering.net


Downloaded From : www.EasyEngineering.net
GATE SOLVED PAPER - EC COMMUNICATION SYSTEM

Sol. 54 Option (C) is correct.


By Binomial distribution the probability of error is
pe = n Cr pr (1 - p) n - r
Probability of at most one error
= Probability of no error + Probability of one error
= n C0 p0 (1 - p) n - 0 + n C1 p1 (1 - p) n - 1
= n (1 - p) n + np (1 - p) n - 1
Sol. 55 Option (B) is correct.

ww Bandwidth allocated for 1 Channel = 5 M Hz


Average bandwidth for 1 Channel 5 = 1 MHz
5

w.E Total Number of Simultaneously Channel = 1M # 8 = 40 Channel


200k
Sol. 56

Chip Rate
asy
Option (A) is correct.
RC = 1.2288 # 106 chips/sec
Rb = RC
Data Rate
En G
Since the processing gain G must be at least 100, thus for Gmin we get
6
gin
Rb max = RC = 1.2288 # 10 = 12.288 # 103 bps
Gmin 100
Sol. 57 Option (B) is correct.
Energy of constellation 1 is eer
2
= (0) + (- 2
2 a) + (-
Eg1
2 a) 2 + ( 2 a) 2 + (- 2 2 a) 2
ing
= 2a2 + 2a2 + 2a2 + 8a2 = 16a2
.ne
Energy of constellation 2 is
Eg2 = a2 + a2 + a2 + a2 = 4a2
E
Eg2 4a
2
Ratio = g1 = 16a2 = 4
t
Sol. 58 Option (A) is correct.
Noise Power is same for both which is N0 .
2
Thus probability of error will be lower for the constellation 1 as it has higher
signal energy.
Sol. 59 Option (A) is correct.
Area under the pdf curve must be unity
Thus 2a + 4a + 4b = 1
2a + 8b = 1 ...(1)
For maximum entropy three region must by equivaprobable thus
2a = 4b = 4b ...(2)
From (1) and (2) we get

Downloaded From : www.EasyEngineering.net


Downloaded From : www.EasyEngineering.net
GATE SOLVED PAPER - EC COMMUNICATION SYSTEM

b = 1 and a = 1
12 6
Sol. 60 Option (*) is correct.
Sol. 61 Option (B) is correct.
A LPF will not produce phase distortion if phase varies linearly with frequency.
f (w) \ w
i.e. f (w) = kw
Sol. 62 Option (B) is correct.
Let m (t) is a low pass signal, whose frequency spectra is shown below

ww
w.E Fourier transform of g (t)

asy
3
1
G (t) = / d (f - 20 # 103 k)
0.5 # 10-4 k =- 3
Spectrum of G (f ) is shown below

En
gin
eer
ing
Now when m (t) is sampled with above signal the spectrum of sampled signal will
look like.
.ne
t
When sampled signal is passed through a LP filter of BW 1 kHz, only m (t) will
remain.
Sol. 63 Option (C) is correct.
The highest frequency signal in x (t) is 1000 # 3 = 3 kHz if expression is expanded.
Thus minimum frequency requirement is
f = 2 # 3 # 103 = 6 # 103 Hz
Sol. 64 Option (B) is correct.
We have
x (t) = 125t [u (t) - u (t - 1)] + (250 - 125t) [u (t - 1) - u (t - 2)]

Downloaded From : www.EasyEngineering.net


Downloaded From : www.EasyEngineering.net
GATE SOLVED PAPER - EC COMMUNICATION SYSTEM

The slope of expression x (t) is 125 and sampling frequency fs is 32 # 1000


samples/sec.
Let 3 be the step size, then to avoid slope overload
3 $ slope x (t)
Ts
3 fc $ slope x (t)
3# 32000 $ 125
3 $ 125
32000
3 = 2- 8
Sol. 65 Option (A) is correct.
The sampling frequency is
1 = 33 kHz

ww fs =
0.03m
Since fs $ 2fm , the signal can be recovered and are correlated.

w.E
Sol. 66 Option (B) is correct.
We have p1 = 0.25 , p2 = 0.25 and p3 = 0.5
3

asyH = / p1 log2 1 bits/symbol


i=1
p1
= p1 log2 1 + p2 log2 1 + p3 log2 1

En p1 p2 p3
= 0.25 log2 1 + 0.25 log2 1 + 0.5 log2 1

gin
0.25 0.25 0.5
= 0.25 log2 4 + 0.25 log2 4 + 0.5 log2 2

2
Rb = 3000 symbol/sec
2
eer
= 0.5 + 0.5 + 1 = 3 bits/symbol

Average bit rate = Rb H


= 3 # 3000 = 4500 bits/sec
ing
Option (A) is correct.
2
.ne
t
Sol. 67

The diagonal clipping in AM using envelop detector can be avoided if


1 << RC < 1
wc W
But from 1 $ Wm sin Wt
RC 1 + m cos Wt
We can say that RC depends on W , thus
RC < 1
W
Sol. 68 Option (B) is correct.
Sol. 69 Option (B) is correct.
When 3 /2 is added to y (t) then signal will move to next quantization level.
Otherwise if they have step size less than 3 then they will be on the same
2
quantization level.
Sol. 70 Option (C) is correct.
After the SSB modulation the frequency of signal will be fc - fm i.e.

Downloaded From : www.EasyEngineering.net


Downloaded From : www.EasyEngineering.net
GATE SOLVED PAPER - EC COMMUNICATION SYSTEM

1000 - 10 kHz . 1000 kHz


The bandwidth of FM is
BW = 2 (b + 1) 3 f
For NBFMb << 1, thus
BWNBFM . 2 3 f = 2 (109 - 106) . 2 # 109
Sol. 71 Option (A) is correct.
We have p (t) = u (t) - u (t - 1)
g (t) = p (t)* p (t)
s (t) = g (t) - d (t - 2)* g (t) = g (t) - g (t - 2)
All signal are shown in figure below :

ww
w.E The impulse response of matched filter is

asyh (t) = s (T - t) = s (1 - t)
Here T is the time where output SNR is maximum.
Sol. 72
En
Option (A) is correct.
We have
where gin
xAM (t) = 10 [P (t) + 0.5g (t)] cos wc t
p (t) = u (t) - u (t - 1)
and
eer
g (t) = r (t) - 2r (t - 1) + r (t - 2)
For desired interval 0 # t # 1, p (t) = 1 and g (t) = t , Thus we have,
xAM (t) = 100 (1 - 0.5t) cos wc t
Hence modulation index is 0.5 ing
Sol. 73 Option (A) is correct.
We know that SYY (w) = H (w) 2 .SXX (w) .ne
Now SYY (w) = 16 2 and SXX (w) = 1 white noise

Thus
16 + w
16 = H (w) 2
t
16 + w2
or H (w) = 4
16 + w2
or H (s) = 4
4+s
which is a first order low pass RL filter.
Sol. 74 Option (A) is correct.
We have R = 4
R + sL 4+s
R
or = 4 L
+s R
L
4+s
Comparing we get L = 1 H and R = 4W
Sol. 75 Option (C) is correct.

Downloaded From : www.EasyEngineering.net


Downloaded From : www.EasyEngineering.net
GATE SOLVED PAPER - EC COMMUNICATION SYSTEM

We have xAM (t) = 10 (1 + 0.5 sin 2pfm t) cos 2pfc t


The modulation index is 0.5
(10) 2
Carrier power Pc = = 50
2
(10) 2
Side band power Ps = = 50
2
2 (0.5) 2 (50)
Side band power Ps = m Pc = = 6.25
2 2
Sol. 76 Option (B) is correct.
Mean noise power = Area under the PSD curve
= 4 ; 1 # B # No E = BNo
2 2
The ratio of average sideband power to mean noise power is

ww Side Band Power = 6.25 = 25


Noise Power N0 B 4No B

w.E
Sol. 77 Option (D) is correct.
{1 + km (t)} A sin (wc t) $ Amplitude modulation
dm (t) Asin (wc t) $ DSB-SC modulation

asy
A sin {cos t + km (t)} $ Phase Modulation
A sin [wct + k] t- 3 m (t) dt $ Frequency Modulation
Sol. 78
En
Option (C) is correct.
VSB $ fm + fc
DSB - SC $ 2fm gin
SSB $ fm
AM $ 2fm eer
Sol. 79 Option (A) is correct. ing
Thus SSB has minimum bandwidth and it require minimum power.

Let x (t) be the input signal where


x (t) = cos (cos t + b1 cos wm t) .ne
Here
y (t) = x2 (t) = 1 +
2
b = 2b1 and b1 =
cos (2wc t + 2b1 cos wm t)

3f
2
= 90 = 18
t
fm 5
BW = 2 (b + 1) fm = 2 (2 # 18 + 1) # 5 = 370 kHz
Sol. 80 Option (C) is correct.
The transfer function of matched filter is
h (t) = x (t - t) = x (2 - t)
The output of matched filter is the convolution of x (t) and h (t) as shown below

Downloaded From : www.EasyEngineering.net


Downloaded From : www.EasyEngineering.net
GATE SOLVED PAPER - EC COMMUNICATION SYSTEM

Sol. 81 Option (B) is correct.


We have H (f) = 2e - jwt d

H (f) = 2
G0 (f) = H (f) 2 Gi (f)

ww The noise power is


= 4No W/Hz
= 4No # B

w.E
Sol. 82 Option (C) is correct.
As the area under pdf curve must be unity
1 (4 # k) = 1 $ k = 1

asy 2
Now mean square value is
2

En
+3
sv2 = #- 3 v2 p (v) dv

gin
4
= #0v2 ` v j dv as p (v) = 1 v
8 8
4 3
v
= # c m dv = 8

Sol. 83 Option (D) is correct.


0 8
eer
The phase deviation is
3f
= 10 = 10
ing
b =
fm 1
If phase deviation remain same and modulating frequency is changed .ne
Sol. 84
BW = 2 (b + 1) fm' = 2 (10 + 1) 2 = 44 kHz
Option (B) is correct.
t
As the area under pdf curve must be unity and all three region are equivaprobable.
Thus are under each region must be 13 .
2a # 1 = 1 $ a = 2
4 3 3
Sol. 85 Option (A) is correct.
+a 3 a 3
x $ 1 dx = 1 ; x E = a
a 2
Nq = #- a x2 p (x) dx = 2 #0 4 2 3 0 6
Substituting a = 2 we have
3
Nq = 4
81
Sol. 86 Option (C) is correct.
When word length is 6
S
` N jN = 6 = 2 = 2
2 #6 12

Downloaded From : www.EasyEngineering.net


Downloaded From : www.EasyEngineering.net
GATE SOLVED PAPER - EC COMMUNICATION SYSTEM

When word length is 8


S
` N jN = 8 = 2 = 2
2 #8 16

^ N hN = 8
S 16
Now = 212 = 2 4 = 16
^ N hN = 6
S
2
Thus it improves by a factor of 16.
Sol. 87 Option (B) is correct.
Carrier frequency fc = 1 # 106 Hz
Modulating frequency
fm = 2 # 103 Hz
For an envelope detector
2pfc > 1 > 2pfm
ww Rc
1 < RC < 1
2pfc 2pfm

w.E 1 < RC < 1


2pfc 2pfm

asy 1 < RC <


2p10 6
1
2 # 103
1.59 # 10 - 7 < RC < 7.96 # 10 - 5

En
so, 20 msec sec best lies in this interval.
Sol. 88 Option (B) is correct.
gin
SAM (t) = Ac [1 + 0.1 cos wm t] cos wm t

eer
sNBFM (t) = Ac cos [wc t + 0.1 sin wm t]
s (t) = SAM (t) + SNB fm (t)

ing
= Ac [1 + 0.1 cos wm t] cos wc t + Ac cos (wc t + 0.1 sin wm t)
= Ac cos wc t + Ac 0.1 cos wm t cos wc t

+ Ac cos wc t cos (0.1 sin wm t) - Ac sin wc t. sin (0.1 sin wm t) .ne


As
so,
0.1 sin wm t ,+ 0.1 to - 0.1
cos (0.1 sin wm t) . 1
As when q is small cos q . 1 and sin q , q, thus
t
sin (0.1 sin wm t) = 0.1 sincos wc t cos wm t + Ac cos wc t
- Ac 0.1 sin wm t sin wc t
= 2Ac cos wc t + 0.1Ac cos (wc + wm) t
1 44 2 44 3 1 4444 4 2 4444 43
cosec USB

Thus it is SSB with carrier.


Sol. 89 Option (A) is correct.
Consecutive pulses are of same polarity when modulator is in slope overload.
Consecutive pulses are of opposite polarity when the input is constant.
Sol. 90 Option (D) is correct.
F (x1 # X < x2) = p (X = x2) - P (X = x1)
or P (X = 1) = P (X = 1+) - P (X = 1 -)

Downloaded From : www.EasyEngineering.net


Downloaded From : www.EasyEngineering.net
GATE SOLVED PAPER - EC COMMUNICATION SYSTEM

= 0.55 - 0.25 = 0.30


Sol. 91 Option (A) is correct.
The SNR at transmitter is
SNRtr = Ptr
NB
10 - 3 = 109
- 20
10 # 100 # 106
In dB SNRtr = 10 log 109 = 90 dB
Cable Loss = 40 db
At receiver after cable loss we have
SNRRc = 90 - 40 = 50 dB
Sol. 92 Option (B) is correct.

ww The impulse response of matched filter is


h (t) = x (T - t)
Since here T = 4 , thus

w.E h (t) = x (4 - t)
The graph of h (t) is as shown below.

asy
En
gin
eer
From graph it may be easily seen that slope between 3 < t < 4 is - 1.
Sol. 93 Option (C) is correct.
The required bandwidth of M array PSK is ing
BW = 2Rb
n
.ne
where 2n = M and Rb is bit rate
For BPSK,
Thus
M = 2 = 2n $ n = 1
B1 = 2Rb = 2 # 10 = 20 kHz
1
t
For QPSK, M = 4 = 2n $ n = 2
Thus B2 = 2Rb = 10 kHz
2
Sol. 94 Option (C) is correct.
We have fc = 100 MHz = 100 # 106 and fm = 1 MHz
= 1 # 106
The output of balanced modulator is
VBM (t) = [cos wc t][ cos wc t]
= 1 [cos (wc + wm) t + cos (wc - wm) t]
2
If VBM (t) is passed through HPF of cut off frequency fH = 100 # 106 , then only
(wc + wm) passes and output of HPF is

Downloaded From : www.EasyEngineering.net


Downloaded From : www.EasyEngineering.net
GATE SOLVED PAPER - EC COMMUNICATION SYSTEM

VHP (t) = 1 cos (wc + wm) t


2
Now V0 (t) = VHP (t) + sin (2p # 100 # 106) t
= 1 cos [2p100 # 106 + 2p # 1 # 106 t] + sin (2p # 100 # 106) t
2
= 1 cos [2p108 + 2p106 t] + sin (2p108) t
2
1
= [cos (2p108 t) t cos (2p106 t)] - sin [2p108 t sin (2p106 t) + sin 2p108 t]
2
= 1 cos (2p106 t) cos 2p108 t + `1 - 1 sin 2p106 t j sin 2p108 t
2 2
This signal is in form
= A cos 2p108 t + B sin 2p108 t
The envelope of this signal is

ww =
=
A2 + B2
1 2 1 6 2
` 2 cos (2p10 t)j + `1 - 2 sin (2p10 t j
6

w.E = 1 cos2 (2p106 t) + 1 + 1 sin2 (2p106 t) - sin (2p106 t)


4 4
=
asy 1 + 1 - sin (2p106 t) =
4
5 - sin (2p106 t)
4

Sol. 95
En
Option (A) is correct.
s (t) = A cos [2p10 # 103 t] + A cos [2p10.1 # 103 t]
Here T1 = 1
10 # 103 gin
= 100m sec

and T2 = 1
10.1 # 103
= 99m sec
eer
Period of added signal will be LCM [T1, T2]
Thus T = LCM [100, 99] = 9900m sec ing
Thus frequency f = 1 = 0.1 kHz
9900m
.ne
Sol. 96 Option (A) is correct.
The pdf of transmission of 0 and 1 will be as shown below : t
Probability of error of 1
P (0 # X # 0.2) = 0.2
Probability of error of 0 :
P (0.2 # X # 0.25) = 0.05 # 2 = 0.1
P (0 # X # 0.2) + P (0.2 # X # 0.25)
Average error =
2
= 0.2 + 0.1 = 0.15
0

Downloaded From : www.EasyEngineering.net


Downloaded From : www.EasyEngineering.net
GATE SOLVED PAPER - EC COMMUNICATION SYSTEM

Sol. 97 Option (B) is correct.


The square mean value is
#- 3 (x - xq) 2 f (x) dx
3
s2 =
1
= #0 (x - xq) 2 f (x) dx
0. 3 0. 1
= #0 (x - 0) 2 f (x) dx + #0.3 (x - 0.7) 2 f (x) dx
3 0. 3 3 2 1
= ; x E + ; x + 0.49x - 14 x E
3 0 3 2 0. 3
or s2 = 0.039
RMS = s2 = 0.039 = 0.198
Sol. 98 Option (C) is correct.

ww FM $ Capture effect
DM $ Slope over load

w.E
Sol. 99 Option (C) is correct.
PSK $ Matched filter
PCM $ m - law

asy
Since fs = 2fm , the signal frequency and sampling frequency are as follows
fm1 = 1200 Hz $ 2400 samples per sec

En fm2 = 600 Hz $ 1200 samples per sec

gin
fm3 = 600 Hz $ 1200 samples per sec
Thus by time division multiplexing total 4800 samples per second will be sent.

Thus bit rate Rb = 4800 # 12 = 57.6 kbps eer


Since each sample require 12 bit, total 4800 # 12 bits per second will be sent

Sol. 100 Option (B) is correct.


ing
The input signal X (f) has the peak at 1 kHz and - 1 kHz. After balanced modulator
the output will have peak at fc ! 1 kHz i.e. :
10 ! 1 $ 11 and 9 kHz .ne
10 ! (- 1) $ 9 and 11 kHz
t
9 kHz will be filtered out by HPF of 10 kHz. Thus 11 kHz will remain. After
passing through 13 kHz balanced modulator signal will have 13 ! 11 kHz signal
i.e. 2 and 24 kHz.
Thus peak of Y (f) are at 2 kHz and 24 kHz.
Sol. 101 Option (A) is correct.
The input is a coherent detector is DSB - SC signal plus noise. The noise at the
detector output is the in-phase component as the quadrature component nq (t) of
the noise n (t) is completely rejected by the detector.
Sol. 102 Option (C) is correct.
The noise at the input to an ideal frequency detector is white. The PSD of noise
at the output is parabolic
Sol. 103 Option (B) is correct.
Pe = 1 erfc c Ed
2h m
We have
2

Downloaded From : www.EasyEngineering.net


Downloaded From : www.EasyEngineering.net
GATE SOLVED PAPER - EC COMMUNICATION SYSTEM

Since Pe of Binary FSK is 3 dB inferior to binary PSK


Sol. 104 Option (D) is correct.
The pdf of Z will be convolution of pdf of X and pdf of Y as shown below.
z
Now p [Z # z] = #- 3 fZ (z) dz
-2
p [Z #- 2] = #- 3fZ (z) dz
= Area [z #- 2]
= 1 # 1 #1 = 1
2 6 12

ww
w.E
asy
En
gin
eer
Sol. 105 Option (D) is correct.
We have RXX (t) = 4 (e - 0.2 t + 1)
ing
RXX (0) = 4 (e - 0.2 0 + 1) = 8 = s2
.ne
or
mean
Now
s =2 2
m =0
P (x # 1) = Fx (1)
t Given

X-m
= 1 - Qc
s m
at x = 1

= 1 - Qc 1 - 0 m = 1 - Qc 1 m
2 2 2 2
Sol. 106 Option (C) is correct.
W = Y-Z
E [W2] = E [Y - Z] 2
= E [Y2] + E [Z2] - 2E [YZ]
= sw2
We have E [X2 (t)] = Rx (10)
= 4 [e - 0.2 0 + 1] = 4 [1 + 1] = 8
E [Y2] = E [X2 (2)] = 8

Downloaded From : www.EasyEngineering.net


Downloaded From : www.EasyEngineering.net
GATE SOLVED PAPER - EC COMMUNICATION SYSTEM

E [Z2] = E [X2 (4)] = 8


E [YZ] = RXX (2) = 4 [e-0.2 (4 - 2) + 1] = 6.68
E [W2] = sw2 = 8 + 8 - 2 # 6.68 = 2.64
Sol. 107 Option (C) is correct.
2mp
Step size d = = 1.536 = 0.012 V
L 128
2 (0.012) 2
Quantization Noise power = d =
12 12
= 12 # 10-6 V2
Sol. 108 Option (D) is correct.
The frequency of pulse train is
f 1- 3 = 1 k Hz

ww 10
The Fourier Series coefficient of given pulse train is
Cn = 1 #
-T /2

w.E
o

Ae-jnw t dt o

To -T /2 o

-To /6
= 1 # Ae-jhw t dt
o

To

asy =
-To /6

A
To (- jhwo)
[e-jw t] --TT //66
o o
o

En = A
(- j2pn)
(e-jw t - e jhw T /6)
o o o

gin
= A (e jhp/3 - e-jhp/3)
j2pn
or Cn = A sin ` np j
pn 3
eer
From Cn it may be easily seen that 1, 2, 4, 5, 7, harmonics are present and 0, 3, 6, 9,..

ing
are absent. Thus p (t) has 1 kHz, 2 kHz, 4 kHz, 5 kHz, 7 kHz,... frequency
component and 3 kHz, 6 kHz.. are absent.
The signal x (t) has the frequency components 0.4 kHz and 0.7 kHz. The
sampled signal of x (t) i.e. x (t)* p (t) will have .ne
1 ! 0.4 and
2 ! 0.4 and
4 ! 0.4 and
1 ! 0.7 kHz
2 ! 0.7 kHz
4 ! 0.7 kHz
t
Thus in range of 2.5 kHz to 3.5 kHz the frequency present is
2 + 0.7 = 2.7 kHz
4 - 0.7 = 3.3 kHz
Sol. 109 Option (C) is correct.
vi = Ac1 cos (2pfc t) + m (t)
v0 = ao vi + avi3
v0 = a0 [Ac' cos (2pfc' t) + m (t)] + a1 [Ac' cos (2pfc' t) + m (t)] 3
= a0 Ac' cos (2pfc' t) + a0 m (t) + a1 [(Ac' cos 2pfc' t) 3
+ (Ac' cos (2pfc') t) 2 m (t) + 3Ac' cos (2pfc' t) m2 (t) + m3 (t)]
= a0 Ac' cos (2pfc' t) + a0 m (t) + a1 (Ac' cos 2fc' t) 3
1 + cos (4pfc' t)
+ 3a1 Ac'2 ; Em (t)
2

Downloaded From : www.EasyEngineering.net


Downloaded From : www.EasyEngineering.net
GATE SOLVED PAPER - EC COMMUNICATION SYSTEM

= 3a1 Ac' cos (2pfc' t) m2 (t) + m3 (t)


The term 3a1 Ac' ( cos 42pf t ) m (t) is a DSB-SC signal having carrier frequency 1. MHz.
'
c

Thus 2fc' = 1 MHz or fc' = 0.5 MHz


Sol. 110 Option (D) is correct.
2
PT = Pc c1 + a m
2
2 P (0.5) 2
Psb = Pc a = c
2 2
or Psb = 1
Pc 8
Sol. 111 Option (D) is correct.
AM Band width = 2fm

ww Peak frequency deviation = 3 (2fm) = 6fm


6f
Modulation index b = m = 6
fm

w.E The FM signal is represented in terms of Bessel function as

xFM (t) = Ac
3
/ Jn (b) cos (wc - nwn) t
asy n =- 3
wc + nwm = 2p (1008 # 103)
2p106 + n4p # 103 = 2p (1008 # 103), n = 4

En
Thus coefficient = 5J4 (6)
Sol. 112 Option (B) is correct.
gin
Ring modulation $ Generation of DSB - SC
VCO $ Generation of FM
Foster seely discriminator $ Demodulation of fmeer
Sol. 113 Option (A) is correct.
mixer $ frequency conversion
ing
fmax = 1650 + 450 = 2100 kHz
fmin = 550 + 450 = 1000 kHz
.ne
or f = 1
2p LC
frequency is minimum, capacitance will be maximum
t
f2
R = Cmax = max = (2.1) 2
Cmin 2
fmin
or R = 4.41
fi = fc + 2fIF = 700 + 2 (455) = 1600 kHz
Sol. 114 Option (D) is correct.
Eb = 10 - 6 watt-sec
No = 10 - 5 W/Hz
(SNR) matched filler = Eo
= 106 = .05
N
2
o
2 # 10 - 5
(SNR)dB = 10 log 10 (0.05) = 13 dB
Sol. 115 Option (B) is correct.

Downloaded From : www.EasyEngineering.net


Downloaded From : www.EasyEngineering.net
GATE SOLVED PAPER - EC COMMUNICATION SYSTEM

3 fs
For slopeoverload to take place Em $
2pfm
This is satisfied with Em = 1.5 V and fm = 4 kHz
Sol. 116 Option (A) is correct.
If s " carrier synchronization at receiver
r " represents bandwidth efficiency
then for coherent binary PSK r = 0.5 and s is required.
Sol. 117 Option (B) is correct.
Bit Rate = 8k # 8 = 64 kbps
(SNR)q = 1.76 + 6.02n dB
= 1.76 + 6.02 # 8 = 49.8 dB
Sol. 118 Option (C) is correct.

ww The frequency of message signal is


fc = 1000 kHz

w.E 1 The frequency of message signal is


fm = 1
100 # 10 - 6
= 10 kHz

asy
Here message signal is symmetrical square wave whose FS has only odd
harmonics i.e. 10 kHz, 30 kHz 50 kHz. Modulated signal contain fc ! fm

En
frequency component. Thus modulated signal has
fc ! fm = (1000 ! 10) kH = 1010 kHz, 990 kHz

gin
fc ! 3fm = (1000 ! 10) kH = 1030 kHz, 970 kHz

Sol. 119 Option (C) is correct.


eer
Thus, there is no 1020 kHz component in modulated signal.

We have -6
y (t) = 5 # 10 x (t)
+3
/ d (t - nTs)
n =- 3 ing
.ne
3
x (t) = 10 cos (8p # 10 ) t
Ts = 100m sec
The cut off fc of LPF is 5 kHz
We know that for the output of filter
=
x (t) y (t)
t
Ts
10 cos (8p # 103) t # 5 # 10 - 6
=
100 # 10 - 6
= 5 # 10 - 1 cos (8p # 103) t
Sol. 120 Option (C) is correct.
Transmitted frequencies in coherent BFSK should be integral of bit rate 8 kHz.
Sol. 121 Option (B) is correct.
For best reception, if transmitting waves are vertically polarized, then receiver
should also be vertically polarized i.e. transmitter and receiver must be in same
polarization.
Sol. 122 Option (D) is correct.
s (t) = cos 2p (2 # 106 t + 30 sin 150t + 40 cos 150t)

Downloaded From : www.EasyEngineering.net


Downloaded From : www.EasyEngineering.net
GATE SOLVED PAPER - EC COMMUNICATION SYSTEM

= cos {4p106 t + 100p sin (150t + q)}


Angle modulated signal is
s (t) = A cos {wc t + b sin (wm t + q)}
Comparing with angle modulated signal we get
Phase deviations b = 100p
Frequency deviations
3 f = bfm = 100p # 150 = 7.5 kHz
2p
Sol. 123 Option (*) is correct.
We have m (t) s (t) = y1 (t)
2 sin (2pt) cos (200pt)
=
t

ww =
sin (202pt) - sin (198pt)
t
y1 (t) + n (t) = y2 (t) = sin 202pt - sin 198pt + sin 199pt

w.E y2 (t) s (t) = u (t)


t t

asy =
[sin 202pt - sin 198pt + sin 199pt] cos 200pt
t
= 1 [sin (402pt) + sin (2pt) - {sin (398pt) - sin (2pt)} + sin (399pt) - sin (pt)]
2
En
After filtering y (t) =
gin
sin (2pt) + sin (2pt) - sin (pt)
2t
=
2t
eer
sin (2pt) + 2 sin (0.5t) cos (1.5pt)

= sin 2pt + sin 0.5pt cos 1.5pt

Sol. 124 Option (B) is correct.


2t t
ing
The signal frequency is
3
fm = 24p10 = 12 kHz .ne
2p
Ts = 50m sec " fs = 1 = 1 # 106 = 20 kHz
Ts 50 t
After sampling signal will have fs ! fm frequency component i.e. 32 and 12 kHz
At filter output only 8 kHz will be present as cutoff frequency is 15 kHz.
Sol. 125 Option (A) is correct.
d (n) = x (n) - x (n - 1)
E [d (n)] 2 = E [x (n) - x (n - 1)] 2
or E [d (n)] 2 = E [x (n)] 2 + E [x (n - 1)] 2 - 2E [x (n) x (n - 1)]
or sd2 = sx2 + sx2 - 2Rxx (1) as k = 1
2
As we have been given sd2 = sx , therefore
10
sx2 = s2 + s2 - 2R (1)
x x xx
10
or 2Rxx (1) = 19 sx2
10

Downloaded From : www.EasyEngineering.net


Downloaded From : www.EasyEngineering.net
GATE SOLVED PAPER - EC COMMUNICATION SYSTEM

or Rxx = 19 = 0.95
sx2 20
Sol. 126 Option (A) is correct.
An ideal low - pass filter with appropriate bandwidth fm is used to recover the
signal which is sampled at nyquist rate 2fm .
Sol. 127 Option (A) is correct.
For any PDF the probability at mean is 1 . Here given PDF is Gaussian random
2
variable and X = 4 is mean.
Sol. 128 Option (C) is correct.
We require 6 bit for 64 intensity levels because 64 = 26
Data Rate = Frames per second # pixels per frame # bits per pixel
= 625 # 400 # 400 # 6 = 600 Mbps sec

ww
Sol. 129 Option (C) is correct.
We have
sin (700pt) sin (500pt)

w.E sin c (700t) + sin c (500t) =


700pt
+
500pt
Here the maximum frequency component is 2pfm = 700p i.e. fm = 350 Hz

asy
Thus Nyquist rate fs = 2fm
= 2 (350) = 700 Hz

En
Thus sampling interval = 1 sec
700
Sol. 130 Option (D) is correct.
Probability of error = p gin
Probability of no error = q = (1 - p)
Probability for at most one bit error eer
ing
= Probability of no bit error
+ probability of 1 bit error

Sol. 131 Option (A) is correct.


= (1 - p) n + np (1 - p) n - 1
.ne
If g (t)
FT

then PSD of g (t) is


G (w)

Sg (w) = G (w) 2
t
and power is
Pg = 1 #- 3Sg (w) dw
3

2p
FT
Now ag (t) aG (w)
PSD of ag (t) is
Sag (w) = a (G (w)) 2
= a2 G (w) 2
or Sag (w) = a2 Sg (w)
Similarly Pag = a2 Pg
Sol. 132 Option (C) is correct.
The envelope of the input signal is [1 + ka m (t)] that will be output of envelope

Downloaded From : www.EasyEngineering.net


Downloaded From : www.EasyEngineering.net
GATE SOLVED PAPER - EC COMMUNICATION SYSTEM

detector.
Sol. 133 Option (D) is correct.
Frequency Range for satellite communication is 1 GHz to 30 GHz,
Sol. 134 Option (B) is correct.
Waveform will be orthogonal when each bit contains integer number of cycles of
carrier.
Bit rate Rb = HCF (f1, f2)
= HCF (10k, 25k)
= 5 kHz
Thus bit interval is Tb = 1 = 1 = 0.2 msec = 200 msec
Rb 5k
Sol. 135 Option (D) is correct.

ww We have
The input to LPF is
Pm = m2 (t)

w.E x (t) = m (t) cos wo t cos (wo t + q)

=
m (t)
2
[cos (2wo t + q) + cos q]

asy =
m (t) cos (2wo t + q) m (t) cos q
2
+
2

En
The output of filter will be
m (t) cos q

Power of output signal is


y (t) =
2
gin
eer
2
Py = y2 (t) = 1 m2 (t) cos2 q = Pm cos q
4 4
Sol. 136 Option (A) is correct.
ing
Hilbert transformer always adds - 90c to the positive frequency component and
90c to the negative frequency component.
Hilbert Trans form .ne
Thus
cos wt " sin wt
sin wt " cos wt
cos w1 t + sin w2 t " sin w1 t - cos w2 t
t
Sol. 137 Option (A) is correct.
We have x (t) = Ac cos {wc t + b sin wm t}
y (t) = {x (t)} 3

= Ac2 cos (3wc t + 3b sin wm t) + 3 cos (wc t + b sin wm t)


Thus the fundamental frequency doesn’t change but BW is three times.
BW = 2 (3 f') = 2 (3 f # 3) = 3 MHz
Sol. 138 Option (C) is correct.
Sol. 139 Option (C) is correct.
This is Quadrature modulated signal. In QAM, two signals having bandwidth.
B 1 & B 2 can be transmitted simultaneous over a bandwidth of (B 1 + B 2) Hz

Downloaded From : www.EasyEngineering.net


Downloaded From : www.EasyEngineering.net
GATE SOLVED PAPER - EC COMMUNICATION SYSTEM

so B.W. = (15 + 10) = 25 kHz


Sol. 140 Option (B) is correct.
A modulated signal can be expressed in terms of its in-phase and quadrature
component as
S (t) = S1 (t) cos (2pfc t) - SQ (t) sin (2pfc t)
Here S (t) = [e-at cpsDwt cos wc t - eat sin Dwt sin wc t] m (t)
= [e-at cos Dwt] cos 2pfc t - [e-at sin Dwt] sin 2pfc t
= S1 (t) cos 2pfc t - SQ (t) sin 2pfc t
Complex envelope of s (t) is
S (t) = S1 (t) + jSQ (t)
= e-at cos Dwt + je-at sin Dwt
= e-at [cos Dwt + j sin Dwt]

ww
Sol. 141 Option (B) is correct.
= exp (- at) exp (jDwt) m (t)

w.E Given function

Let
g (t) = 6 # 10 4 sin c2 (400t) ) 106 sin c3 (100t)
g1 (t) = 6 # 10 4 sin c2 (400t)

asy g2 (t) = (106) sin c3 (100t)


We know that g1 (t) ) g2 (t) ? G1 (w) G2 (w) occupies minimum of Bandwidth of

En
G1 (w) or G2 (w)
Band width of G1 (w) = 2 # 400 = 800 rad/ sec or = 400 Hz

gin
Band width of G2 (w) = 3 # 100 = 300 rad/ sec or 150 Hz

Sol. 142
Sampling frequency
Option (B) is correct.
= 2 # 150 = 300 Hz

eer
SNR (dB) = 1.8 + 6n ing
For a sinusoidal input SNR (dB) is PCM is obtained by following formulae.
n is no. of bits
Here
So,
n =8
SNR (dB) = 1.8 + 6 # 8 = 49.8 .ne
Sol. 143 Option (D) is correct.
We know that matched filter output is given by
g 0 (t) =
t
# g (l) g (T - t + l) dl at t = T
3
0 0
-3

6g 0 (t)@max = # g (l) g (l) dl = # g (t) dt


3 3 2
-3 -3
1 # 10-4
= #
0
[10 sin (2p # 106) 2] dt

[g 0 (t)] max = 1 # 100 # 10-4 = 5 mV


2
Sol. 144 Option (B) is correct.
Sampling rate must be equal to twice of maximum frequency.
f s = 2 # 400 = 800 Hz
Sol. 145 Option (C) is correct.
The amplitude spectrum of a gaussian pulse is also gaussian as shown in the fig.
-y 2
fY (y) = 1 exp c
2p 2 m

Downloaded From : www.EasyEngineering.net


Downloaded From : www.EasyEngineering.net
GATE SOLVED PAPER - EC COMMUNICATION SYSTEM

Sol. 146 Option (C) is correct.


Let the rectangular pulse is given as

ww
w.E
asy
Auto correlation function is given by

En T/2
Rxx (t) = 1 # x (t) x (t - t) dt
T -T/2

gin
When x (t) is shifted to right (t > 0), x (t - t) will be shown as dotted line.

eer
ing
.ne
Rxx (t) = 1
T
2
#T
- +t
2
T
2
+t

A2 dt
2
= A :T + T - tD = A :T - tD
t
T 2 2 T 2
(t) can be negative or positive, so generalizing above equations
2
Rxx (t) = A :T - t D
T 2
Rxx (t) is a regular pulse of duration T .

Downloaded From : www.EasyEngineering.net


Downloaded From : www.EasyEngineering.net
GATE SOLVED PAPER - EC COMMUNICATION SYSTEM

Sol. 147 Option (B) is correct.


Selectivity refers to select a desired frequency while rejecting all others. In super
heterodyne receiver selective is obtained partially by RF amplifier and mainly by
IF amplifier.
Sol. 148 Option (C) is correct.
In PCM, SNR a 22n
so if bit increased from 8 to 9
(SNR) 1 2#8
= 22 # 9 = 22 = 1
(SNR) 2 2 4
so SNR will increased by a factor of 4
Sol. 149 Option (A) is correct.
In flat top sampling an amplitude distortion is produced while reconstructing
original signal x (t) from sampled signal s (t). High frequency of x (t) are mostly

ww
Sol. 150
attenuated. This effect is known as aperture effect.
Option (A) is correct.

w.E Carrier C (t) = cos (we t + q)


Modulating signal = x (t)

asy
DSB - SC modulated signal = x (t) c (t) = x (t) cos (we t + q)
envelope = x (t)
Sol. 151

En
Option (D) is correct.
In Quadrature multiplexing two baseband signals can transmitted or modulated

Sol. 152 Option (A) is correct. gin


using I 4 phase & Quadrature carriers and its quite different form FDM & TDM.

eer
Fourier transform perform a conversion from time domain to frequency domain
for analysis purposes. Units remain same.
Sol. 153 Option (A) is correct.
ing
In PCM, SNR is depends an step size (i.e. signal amplitude) SNR can be improved

.ne
by using smaller steps for smaller amplitude. This is obtained by compressing the
signal.
Sol. 154 Option (C) is correct.
t
Band width is same for BPSK and APSK(OOK) which is equal to twice of signal
Bandwidth.
Sol. 155 Option (A) is correct.
The spectral density of a real value random process symmetric about vertical axis
so it has an even symmetry.
Sol. 156 Option (A) is correct.
Sol. 157 Option (C) is correct.
It is one of the advantage of bipolar signalling (AMI) that its spectrum has a dc
null for binary data transmission PSD of bipolar signalling is

Downloaded From : www.EasyEngineering.net


Downloaded From : www.EasyEngineering.net
GATE SOLVED PAPER - EC COMMUNICATION SYSTEM

Sol. 158 Option (A) is correct.


Probability Density function (PDF) of a random variable x defined as
Px (x) = 1 e-x /2
2

2p

ww
Sol. 159
so here

Option (C) is correct.


K = 1
2p

w.E Here the highest frequency component in the spectrum is 1.5 kHz
[at 2 kHz is not included in the spectrum]

Sol. 160
asy
Minimum sampling freq. = 1.5 # 2 = 3 kHz
Option (B) is correct.

Sol. 161
En
We need a high pass filter for receiving the pulses.
Option (D) is correct.

gin
Power spectral density function of a signal g (t) is fourier transform of its auto
correlation function
Rg (t)
F
Sg (w)
here Sg (w) = sin c2 (f) eer
so Rg (t) is a triangular pulse.
f [triang.] = sin c2 (f) ing
Sol. 162 Option (C) is correct.
For a signal g (t), its matched filter response given as .ne
h (t) = g (T - t)
so here g (t) is a rectangular pulse of duration T . t

output of matched filter


y (t) = g (t) ) h (t)

Downloaded From : www.EasyEngineering.net


Downloaded From : www.EasyEngineering.net
GATE SOLVED PAPER - EC COMMUNICATION SYSTEM

if we shift g (- t) for convolution y (t) increases first linearly then decreases to


zero.

ww
Sol. 163 Option (C) is correct.

w.E The difference between incoming signal frequency (fc) and its image frequency (fc)
is 2I f (which is large enough). The RF filter may provide poor selectivity against

asy
adjacent channels separated by a small frequency differences but it can provide
reasonable selectivity against a station separated by 2I f . So it provides adequate
suppression of image channel.
Sol. 164
En
Option (C) is correct.
In PCM SNR is given by
SNR = 3 22n
2 gin
eer
if no. of bits is increased from n to (n + 1) SNR will increase by a factor of
22 (n + 1)/n
Sol. 165 Option (D) is correct.
ing
The auto correlation of energy signal is an even function.
auto correlation function is gives as
.ne
t
3
R (t) = # x (t) x (t + t) dt
-3
3
put R (- t) = # x (t) x (t - t) dt
-3
Let t-t = a
dt = da
3
R (- t) = # x (a + t) x (a) da
-3
Changing variable a " t
3
R (- t) = # x (t) x (t + t) dt = R (t)
-3
R (- t) = R (t) even function

***********

Downloaded From : www.EasyEngineering.net


Downloaded From : www.EasyEngineering.net

No part of this publication may be reproduced or distributed in any form or any means, electronic, mechanical,
photocopying, or otherwise without the prior permission of the author.

ww
w.E
GATE SOLVED PAPER
Electronics & Communication
Control System
asy
Copyright © By NODIA & COMPANY
En
gin
eer
ing
Information contained in this book has been obtained by authors, from sources believes to be reliable. However,
neither Nodia nor its authors guarantee the accuracy or completeness of any information herein, and Nodia nor its

.ne
authors shall be responsible for any error, omissions, or damages arising out of use of this information. This book
is published with the understanding that Nodia and its authors are supplying information but are not attempting
to render engineering or other professional services.

t
NODIA AND COMPANY
B-8, Dhanshree Tower Ist, Central Spine, Vidyadhar Nagar, Jaipur 302039
Ph : +91 - 141 - 2101150
www.nodia.co.in
email : enquiry@nodia.co.in

Downloaded From : www.EasyEngineering.net


Downloaded From : www.EasyEngineering.net

GATE SOLVED PAPER - EC


CONTROL SYSTEM

2013 ONE MARK

Q. 1 The Bode plot of a transfer function G ^s h is shown in the figure below.

ww
w.E
asy
En
The gain _20 log G ^s h i is 32 dB and - 8 dB at 1 rad/s and 10 rad/s respectively.

(A) 39.8
gin
The phase is negative for all w. Then G ^s h is
(B) 392.8
s
32 eer
s
32
(C)
s
(D) 2
s
ing
2013

.ne TWO MARKS

Y ^s h
Q. 2
this system is
t
The signal flow graph for a system is given below. The transfer function
U ^s h
for

(A) s+1 (B) s+1


5s2 + 6s + 2 s 2 + 6s + 2
(C) 2 s + 1 (D) 2 1
s + 4s + 2 5s + 6s + 2

Statement for Linked Answer Questions 3 and 4:


The state diagram of a system is shown below. A system is described by the state-
o = AX + Bu ; y = CX + Du
variable equations X

Downloaded From : www.EasyEngineering.net


Downloaded From : www.EasyEngineering.net
GATE SOLVED PAPER - EC CONTROL SYSTEM

Q. 3 The state-variable equations of the system shown in the figure above are
o = >- 1 0 H X + >- 1H u
X Xo = >- 1 0 H X + >- 1H u
(A) 1 -1 1 (B) -1 -1 1
y = 61 - 1@ X + u y = 6- 1 - 1@ X + u
o = >- 1 0 H X + >- 1H u
X Xo = >- 1 - 1H X + >- 1H u
(C) -1 -1 1 (D) 0 -1 1
y = 6- 1 - 1@ X - u y = 61 - 1@ X - u
The state transition matrix eAt of the system shown in the figure above is

ww
Q. 4

e-t 0 e-t 0
(A) > -t -tH (B) > H
te e - te e-t
-t

w.E e-t 0
(C) > -t -tH
e e
(D) >
e-t - te-t
0 e-t
H

Q. 5
asy
The open-loop transfer function of a dc motor is given as V ^^shh = 1 +1010s . When
ws
a

connected in feedback as shown below, the approximate value of Ka that will

En
reduce the time constant of the closed loop system by one hundred times as
compared to that of the open-loop system is

gin
eer
ing
(A) 1
(C) 10
(B) 5
(D) 100 .ne
2012

(s2 + 9) (s + 2)
t ONE MARK

Q. 6 A system with transfer function G (s) =


(s + 1) (s + 3) (s + 4)
is excited by sin (wt). The steady-state output of the system is zero at
(A) w = 1 rad/s (B) w = 2 rad/s
(C) w = 3 rad/s (D) w = 4 rad/s

2012 TWO MARKS

Q. 7 The feedback system shown below oscillates at 2 rad/s when

Downloaded From : www.EasyEngineering.net


Downloaded From : www.EasyEngineering.net
GATE SOLVED PAPER - EC CONTROL SYSTEM

(A) K = 2 and a = 0.75 (B) K = 3 and a = 0.75


(C) K = 4 and a = 0.5 (D) K = 2 and a = 0.5

Q. 8 The state variable description of an LTI system is given by


Jxo1N J 0 a1 0NJx1N J0N Jx1N
K O K OK O K O K O
Kxo2O = K 0 0 a2OKx2O + K0O u y = _1 0 0iKx2O
Kxo O Ka 0 0OKx 3O K 1O Kx 3O
3 3
L P L PL P L P L P
where y is the output and u is the input. The system is controllable for
(A) a1 ! 0, a2 = 0, a 3 ! 0 (B) a1 = 0, a2 ! 0, a 3 ! 0
(C) a1 = 0, a 3 ! 0, a 3 = 0 (D) a1 ! 0, a2 ! 0, a 3 = 0

2011 ONE MARK

ww
Q. 9 The root locus plot for a system is given below. The open loop transfer function
corresponding to this plot is given by

w.E
asy
En
gin
(A) G ^s h H ^s h = k
s (s + 1)
(s + 2) (s + 3)
eer
(B) G ^s h H ^s h = k
(s + 1)
s (s + 2) (s + 3) 2
(C) G ^s h H ^s h = k 1
s (s - 1) (s + 2) (s + 3) ing
(D) G ^s h H ^s h = k
(s + 1)
s (s + 2) (s + 3)
Q. 10
positive frequency has the form .ne
For the transfer function G (jw) = 5 + jw , the corresponding Nyquist plot for

Downloaded From : www.EasyEngineering.net


Downloaded From : www.EasyEngineering.net
GATE SOLVED PAPER - EC CONTROL SYSTEM

2011 TWO MARKS

Q. 11 The block diagram of a system with one input u and two outputs y1 and y2 is
given below.

A state space model of the above system in terms of the state vector x and the
output vector y = [y1 y2]T is

ww (A) xo = [2] x + [1] u ; y = [1 2] x


1
(B) xo = [- 2] x + [1] u; y = > H x
2

w.E (C) xo = >


-2 0
0 - 2H
1
x + > H u ; y = 81 2B x
1
2 0
asy 1 1
(D) xo = > H x + > H u ; y = > H x
0 2 1 2

En
Common Data For Q. 12 and 13

gin
The input-output transfer function of a plant H (s) = 100
s (s + 10) 2
.

eer
The plant is placed in a unity negative feedback configuration as shown in the
figure below.

ing
.ne
Q. 12
(A) 0 dB (B) 20 dB
t
The gain margin of the system under closed loop unity negative feedback is

(C) 26 dB (D) 46 dB

Q. 13 The signal flow graph that DOES NOT model the plant transfer function H (s) is

Downloaded From : www.EasyEngineering.net


Downloaded From : www.EasyEngineering.net
GATE SOLVED PAPER - EC CONTROL SYSTEM

2010 ONE MARK

Q. 14 The transfer function Y (s) /R (s) of the system shown is

(A) 0 (B) 1
s+1
(C) 2 (D) 2
s+1 s+3
Y (s)
= s has an output y (t) = cos a2t - p k
ww
Q. 15 A system with transfer function
X (s) s + p
p
for the input signal x (t) = p cos a2t - k. Then, the system parameter p is
2
3

w.E (A) 3
(C) 1
(B) 2/ 3
(D) 3 /2

Q. 16

asy
For the asymptotic Bode magnitude plot shown below, the system transfer
function can be

En
gin
eer
(A) 10s + 1
0.1s + 1 ing
(B) 100s + 1
0.1s + 1
(C) 100s
10s + 1
(D) 0.1s + 1
10s + 1 .ne
2010
t TWO MARKS

Common Data For Q. 17 and 18


The signal flow graph of a system is shown below:

Q. 17 The state variable representation of the system can be


1 1 0 -1 1 0
xo = >
- 1 0H
x +> Hu
(A) x = >- 1 0H x + >2H u
o
(B) 2
yo = [0 0.5] x yo = 80 0.5B x

Downloaded From : www.EasyEngineering.net


Downloaded From : www.EasyEngineering.net
GATE SOLVED PAPER - EC CONTROL SYSTEM

1 1 0 -1 1 0
xo = > H x +> Hu xo = > H x +> Hu
(C) -1 0 2 (D) -1 0 2
y = 80.5 0.5B x
o y = 80.5
o 0.5B x
Q. 18 The transfer function of the system is
(A) s2+ 1 (B) s2- 1
s +1 s +1
(C) 2 s + 1 (D) 2 s - 1
s +s+1 s +s+1
Q. 19 A unity negative feedback closed loop system has a plant with the transfer
function G (s) = s + 21s + 2 and a controller Gc (s) in the feed forward path. For a unit
2

set input, the transfer function of the controller that gives minimum steady state
error is
(A) Gc (s) = s + 1 (B) Gc (s) = s + 2
s+2 s+1

ww (C) Gc (s) =
(s + 1) (s + 4)
(s + 2) (s + 3)
(D) Gc (s) = 1 + 2 + 3s
s

w.E 2009 ONE MARK

Q. 20

asy
The magnitude plot of a rational transfer function G (s) with real coefficients is
shown below. Which of the following compensators has such a magnitude plot ?

En
gin
eer
(A) Lead compensator
(C) PID compensator ing
(B) Lag compensator
(D) Lead-lag compensator

Q. 21 Consider the system


.ne
dx = Ax + Bu with A = =1 0G and B = =p G
dt 0 1 q
t
where p and q are arbitrary real numbers. Which of the following statements
about the controllability of the system is true ?
(A) The system is completely state controllable for any nonzero values of p and
q
(B) Only p = 0 and q = 0 result in controllability
(C) The system is uncontrollable for all values of p and q
(D) We cannot conclude about controllability from the given data

2009 TWO MARKS

Q. 22 The feedback configuration and the pole-zero locations of


2
G (s) = s2 - 2s + 2
s + 2s + 2
are shown below. The root locus for negative values of k , i.e. for - 3 < k < 0 , has

Downloaded From : www.EasyEngineering.net


Downloaded From : www.EasyEngineering.net
GATE SOLVED PAPER - EC CONTROL SYSTEM

breakaway/break-in points and angle of departure at pole P (with respect to the


positive real axis) equal to

(A) ! 2 and 0c (B) ! 2 and 45c


(C) ! 3 and 0c (D) ! 3 and 45c

Q. 23 The unit step response of an under-damped second order system has steady state
value of -2. Which one of the following transfer functions has theses properties ?

ww (A) 2 - 2.24
s + 2.59s + 1.12
- 2.24
(B) 2 - 3.82
s + 1.91s + 1.91
- 382

w.E (C)
s2 - 2.59s + 1.12
(D)
s2 - 1.91s + 1.91

asy
Common Data For Q. 24 and 25 :
The Nyquist plot of a stable transfer function G (s) is shown in the figure are

En
interested in the stability of the closed loop system in the feedback configuration
shown.

gin
eer
ing
.ne
Q. 24 Which of the following statements is true ?
(A) G (s) is an all-pass filter
(B) G (s) has a zero in the right-half plane
t
(C) G (s) is the impedance of a passive network
(D) G (s) is marginally stable

Q. 25 The gain and phase margins of G (s) for closed loop stability are
(A) 6 dB and 180c (B) 3 dB and 180c
(C) 6 dB and 90c (D) 3 dB and 90c

2008 ONE MARKS

Q. 26 Step responses of a set of three second-order underdamped systems all have the
same percentage overshoot. Which of the following diagrams represents the poles
of the three systems ?

Downloaded From : www.EasyEngineering.net


Downloaded From : www.EasyEngineering.net
GATE SOLVED PAPER - EC CONTROL SYSTEM

ww
Q. 27 The pole-zero given below correspond to a

w.E
asy
(A) Law pass filter
En (B) High pass filter
(C) Band filter
gin (D) Notch filter

2008

A signal flow graph of a system is given below eer TWO MARKS

ing
Q. 28

.ne
t
The set of equalities that corresponds to this signal flow graph is
Jx1N R b - g 0 VJx1N R0 0 V
K O S WK O S W u1
(A) d Kx2O = S g a 0 WKx2O + S0 1 We o
dt K O S u2
x3 S- a b 0 WWKx3O SS1 0 WW
L P L P
Jx1N RT0 a g XVJx1N TR1 0 XV
K O S WK O S W u1
(B) d Kx2O = S0 - a - g WKx2O + S0 1 We o
dt K O S u2
x3 S0 b - b WWKx3O SS0 0 WW
L P TR L P
Jx1N - a b 0 VXJx1N RT1 0 VX
K O S WK O S W u1
(C) d Kx2O = S- b - g 0 WKx2O + S0 1 We o
dt K O S u2
x3 S a g 0 WWKx3O SS0 0 WW
L P TR L P
Jx1N - a 0 b XVJx1N TR1 0XV
K O S WK O S W u1
(D) d Kx2O = S g 0 a WKx2O + S0 1 We o
dt K O S u2
x3 S- b 0 - a WWKx3O SS0 0 WW
L P T XL P T X

Downloaded From : www.EasyEngineering.net


Downloaded From : www.EasyEngineering.net
GATE SOLVED PAPER - EC CONTROL SYSTEM

Q. 29 Group I lists a set of four transfer functions. Group II gives a list of possible step
response y (t). Match the step responses with the corresponding transfer functions.

ww
w.E
asy
En
(A) P - 3, Q - 1, R - 4, S - 2
(B) P - 3, Q - 2, R - 4, S - 1
(C) P - 2, Q - 1, R - 4, S - 2 gin
(D) P - 3, Q - 4, R - 1, S - 2
eer
Q. 30 A certain system has transfer function
G (s) = 2 s + 8
s + as - 4
ing
.ne
where a is a parameter. Consider the standard negative unity feedback
configuration as shown below

t
Which of the following statements is true?
(A) The closed loop systems is never stable for any value of a
(B) For some positive value of a, the closed loop system is stable, but not for
all positive values.
(C) For all positive values of a, the closed loop system is stable.
(D) The closed loop system stable for all values of a, both positive and
negative.

Q. 31 The number of open right half plane of


G (s) = 5 10 is
s + 2s + 3s + 6s2 + 5s + 3
4 3

(A) 0 (B) 1
(C) 2 (D) 3

Downloaded From : www.EasyEngineering.net


Downloaded From : www.EasyEngineering.net
GATE SOLVED PAPER - EC CONTROL SYSTEM

Q. 32 The magnitude of frequency responses of an underdamped second order system


is 5 at 0 rad/sec and peaks to 10 at 5 2 rad/sec. The transfer function of the
system is 3
(A) 2 500 (B) 2 375
s + 10s + 100 s + 5s + 75
(C) 2 720 (D) 2 1125
s + 12s + 144 s + 25s + 225
Q. 33 Group I gives two possible choices for the impedance Z in the diagram. The
circuit elements in Z satisfy the conditions R2 C2 > R1 C1. The transfer functions
V0 represents a kind of controller.
Vi

ww
w.E Match the impedances in Group I with the type of controllers in Group II

asy
En
(A) Q - 1, R - 2 gin (B) Q - 1, R - 3
(C) Q - 2, R - 3
eer
(D) Q - 3, R - 2

2007
ing ONE MARK

s-5
Q. 34

, then It is
.ne
If the closed-loop transfer function of a control system is given as T (s)
(s + 2)( s + 3)

(A) an unstable system


(B) an uncontrollable system
(C) a minimum phase system
t
(D) a non-minimum phase system

2007 TWO MARKS

Q. 35 A control system with PD controller is shown in the figure. If the velocity error
constant KV = 1000 and the damping ratio z = 0.5 , then the value of KP and KD
are

(A) KP = 100, KD = 0.09 (B) KP = 100, KD = 0.9


(C) KP = 10, KD = 0.09 (D) KP = 10, KD = 0.9

Downloaded From : www.EasyEngineering.net


Downloaded From : www.EasyEngineering.net
GATE SOLVED PAPER - EC CONTROL SYSTEM

Q. 36 The transfer function of a plant is T (s) = 5


(s + 5)( s2 + s + 1)
The second-order approximation of T (s) using dominant pole concept is
(A) 1 (B) 5
(s + 5)( s + 1) (s + 5)( s + 1)
(C) 2 5 (D) 2 1
s +s+1 s +s+1
Q. 37 The open-loop transfer function of a plant is given as G (s) = s 1- 1 . If the plant is
2

operated in a unity feedback configuration, then the lead compensator that an


stabilize this control system is
10 (s - 1) 10 (s + 4)
(A) (B)
s+2 s+2
10 (s + 2) 2 (s + 2)
(C) (D)
s + 10 s + 10

ww
Q. 38 A unity feedback control system has an open-loop transfer function
G (s) = K

w.E 2
s (s + 7s + 12)
The gain K for which s = 1 + j1 will lie on the root locus of this system is
(A) 4 (B) 5.5
(C) 6.5
asy (D) 10

Q. 39

En
The asymptotic Bode plot of a transfer function is as shown in the figure. The
transfer function G (s) corresponding to this Bode plot is

gin
eer
ing
1 1 .ne
(A)

(C)
(s + 1)( s + 20)
100
s (s + 1)( s + 20)
(B)

(D) 100 t
s (s + 1)( s + 20)

s (s + 1)( 1 + 0.05s)

Q. 40 The state space representation of a separately excited DC servo motor dynamics


is given as
dw
-1 1 w 0
> H = =- 1 - 10G=ia G + =10Gu
dt
dio
dt

where w is the speed of the motor, ia is the armature current and u is the
w (s)
armature voltage. The transfer function of the motor is
U (s)
(A) 2 10 (B) 2 1
s + 11s + 11 s + 11s + 11
(C) 2 10s + 10 (D) 2 1
s + 11s + 11 s + s + 11

Downloaded From : www.EasyEngineering.net


Downloaded From : www.EasyEngineering.net
GATE SOLVED PAPER - EC CONTROL SYSTEM

Statement for linked Answer Question 41 and 42 :


Consider a linear system whose state space representation is x (t) = Ax (t). If
1
the initial state vector of the system is x (0) = = G, then the system response is
- 2
e-2x 1
x (t) = > -2tH . If the itial state vector of the system changes to x (0) = =
- 2G
, then
- 2e
e-t
the system response becomes x (t) = > -tH
-e
Q. 41 The eigenvalue and eigenvector pairs (li vi) for the system are
1 1 1 1
(A) e- 1 = Go and e- 2 = Go (B) e- 1, = Go and e2, = Go
-1 -2 -1 -2
1 1 1 1
(C) e- 1, = Go and e- 2, = Go (D) e- 2 = Go and e1, = Go
-1 -2 -1 -2
Q. 42 The system matrix A is

ww (A) =
0 1
- 1 1G
2 1
(B) =
1 1
- 1 - 2G
0 1

w.E (C) =
- 1 - 1G
(D) =
- 2 - 3G

Q. 43
2006

asy
The open-loop function of a unity-gain feedback control system is given by
ONE MARK

En
G (s) = K
(s + 1)( s + 2)

(A) 0 gin
The gain margin of the system in dB is given by
(B) 1
(C) 20 (D) 3
eer
2006

1 ing
and G2 (s) = 2 s
TWO MARKS

Q. 44 Consider two transfer functions G1 (s) = 2


s + as + b
.ne
s + as + b
The 3-dB bandwidths of their frequency responses are, respectively
.

Q. 45
(A) a2 - 4b , a2 + 4b
(C) a2 - 4b , a2 - 4b
(B) a2 + 4b , a2 - 4b
(D) t
a 2 + 4b , a 2 + 4b

The Nyquist plot of G (jw) H (jw)for a closed loop control system, passes through
(- 1, j0) point in the GH plane. The gain margin of the system in dB is equal to
(A) infinite (B) greater than zero
(C) less than zero (D) zero

Q. 46 The positive values of K and a so that the system shown in the figures below
oscillates at a frequency of 2 rad/sec respectively are

(A) 1, 0.75 (B) 2, 0.75


(C) 1, 1 (D) 2, 2

Downloaded From : www.EasyEngineering.net


Downloaded From : www.EasyEngineering.net
GATE SOLVED PAPER - EC CONTROL SYSTEM

Q. 47 The transfer function of a phase lead compensator is given by Gc (s) = 1 + 3Ts


1 + Ts
where T > 0 The maximum phase shift provide by
such a compensator is
(A) p (B) p
2 3
(C) p (D) p
4 6

Q. 48 A linear system is described by the following state equation


0 1
Xo (t) = AX (t) + BU (t), A = =
- 1 0G
The state transition matrix of the system is
cos t sin t - cos t sin t
(A) =
- sin t cos t G
(B) =
- sin t - cos t G

ww (C) =
- cos t - sin t
- sin t cos t G (D) =
cos t - sin t
cos t sin t G

w.E Statement for Linked Answer Questions 49 and 50:


Consider a unity - gain feedback control system whose open - loop transfer

asy
function is : G (s) = as +
s2
1

The value of a so that the system has a phase - margin equal to p is approximately
Q. 49
equal to
En 4
(A) 2.40
(C) 0.84 gin (B) 1.40
(D) 0.74

Q. 50
eer
With the value of a set for a phase - margin of p , the value of unit - impulse
4
response of the open - loop system at t = 1 second is equal to
(A) 3.40
(C) 1.84
(B) 2.40
(D) 1.74ing
.ne
Q. 51
2005

t ONE MARK

Which one of the following polar diagrams corresponds to a lag network ?

Q. 52 A linear system is equivalently represented by two sets of state equations :


Xo = AX + BU and Wo = CW + DU

Downloaded From : www.EasyEngineering.net


Downloaded From : www.EasyEngineering.net
GATE SOLVED PAPER - EC CONTROL SYSTEM

The eigenvalues of the representations are also computed as [l] and [m]. Which
one of the following statements is true ?
(A) [l] = [m] and X = W (B) [l] = [m] and X ! W
(C) [l] ! [m] and X = W (D) [l] = [m] and X ! W

Q. 53 Despite the presence of negative feedback, control systems still have problems of
instability because the
(A) Components used have non- linearities
(B) Dynamic equations of the subsystem are not known exactly.
(C) Mathematical analysis involves approximations.
(D) System has large negative phase angle at high frequencies.

2005 TWO MARKS

ww
Q. 54 The polar diagram of a conditionally stable system for open loop gain K = 1 is
shown in the figure. The open loop transfer function of the system is known to be
stable. The closed loop system is stable for

w.E
asy
En
gin
eer
(A) K < 5 and 1 < K < 1
2
(C) K < 1 and 5 < K
8 8
ing
(B) K < 1 and 1 < K < 5
2
(D) K > 1 and 5 > K

Q. 55
8 8
.ne
In the derivation of expression for peak percent overshoot
Mp = exp e
- px
1 - x2
o # 100%
Which one of the following conditions is NOT required ?
t
(A) System is linear and time invariant
(B) The system transfer function has a pair of complex conjugate poles and no
zeroes.
(C) There is no transportation delay in the system.
(D) The system has zero initial conditions.

Q. 56 A ramp input applied to an unity feedback system results in 5% steady state


error. The type number and zero frequency gain of the system are respectively
(A) 1 and 20 (B) 0 and 20
(C) 0 and 1 (D) 1 and 1
20 20
2
Q. 57 A double integrator plant G (s) = K/s , H (s) = 1 is to be compensated to achieve
the damping ratio z = 0.5 and an undamped natural frequency, wn = 5 rad/sec

Downloaded From : www.EasyEngineering.net


Downloaded From : www.EasyEngineering.net
GATE SOLVED PAPER - EC CONTROL SYSTEM

which one of the following compensator Ge (s) will be suitable ?


(A) s + 3 (B) s + 99
s + 99 s+3
(C) s - 6 (D) s - 6
s + 8.33 s
K (1 - s)
Q. 58 An unity feedback system is given as G (s) = . Indicate the correct root
s (s + 3)
locus diagram.

ww
w.E
asy
Statement for Linked Answer Question 59 and 60
The open loop transfer function of a unity feedback system is given by

En
G (s) = 3e
-2s

s (s + 2)
Q. 59
(A) 0.632 and 1.26 gin
The gain and phase crossover frequencies in rad/sec are, respectively

(B) 0.632 and 0.485


(C) 0.485 and 0.632 eer
(D) 1.26 and 0.632
ing
Q. 60
(A) -7.09 dB and 87.5c .ne
Based on the above results, the gain and phase margins of the system will be

(B) 7.09 dB and 87.5c


(C) 7.09 dB and - 87.5c
(D) - 7.09 and - 87.5c
t
2004 ONE MARK

Q. 61 The gain margin for the system with open-loop transfer function
2 (1 + s)
G (s) H (s) = , is
s2
(A) 3 (B) 0
(C) 1 (D) - 3

Q. 62 Given G (s) H (s) = s (s + 1)(


K
s + 3)
.The point of intersection of the asymptotes of the
root loci with the real axis is
(A) - 4 (B) 1.33
(C) - 1.33 (D) 4

Downloaded From : www.EasyEngineering.net


Downloaded From : www.EasyEngineering.net
GATE SOLVED PAPER - EC CONTROL SYSTEM

2004 TWO MARKS

Q. 63 Consider the Bode magnitude plot shown in the fig. The transfer function H (s) is

(s + 10) 10 (s + 1)
(A) (B)
(s + 1)( s + 100) (s + 10)( s + 100)
102 (s + 1) 103 (s + 100)
(C) (D)
(s + 10)( s + 100) (s + 1)( s + 10)

ww
Q. 64 A causal system having the transfer function H (s) = 1/ (s + 2) is excited with
10u (t). The time at which the output reaches 99% of its steady state value is
(A) 2.7 sec (B) 2.5 sec

w.E (C) 2.3 sec (D) 2.1 sec

A system has poles at 0.1 Hz, 1 Hz and 80 Hz; zeros at 5 Hz, 100 Hz and 200 Hz.

asy
Q. 65
The approximate phase of the system response at 20 Hz is
(A) - 90c (B) 0c
(C) 90c
En (D) - 180c

Q. 66

gin
Consider the signal flow graph shown in Fig. The gain x5 is
x1

eer
(A)
1 - (be + cf + dg)
abcd
(B)
ing
bedg
1 - (be + cf + dg)
(C) abcd
1 - (be + cf + dg) + bedg
(D)
.ne
1 - (be + cf + dg) + bedg
abcd
Q. 67 If A = =

(A) 1 =
-2 2
1 - 3G
, then sin At is

sin (- 4t) + 2 sin (- t) - 2 sin (- 4t) + 2 sin (- t)


t
G
3 - sin (- 4t) + sin (- t) 2 sin (- 4t) + sin (- t)
sin (- 2t) sin (2t)
(B) =
sin (t) sin (- 3t)G
sin (4t) + 2 sin (t) 2 sin (- 4t) - 2 sin (- t)
(C) 1 =
3 sin (- 4t) + sin (t)
- 2 sin (4t) + sin (t) G
cos (- t) + 2 cos (t) 2 cos (- 4t) + 2 cos (- t)
(D) 1 = G
3 - cos (- 4t) + cos (- t) - 2 cos (- 4t) + cos (t)
Q. 68 The open-loop transfer function of a unity feedback system is
G (s) = K
s (s2 + s + 2)( s + 3)
The range of K for which the system is stable is
(A) 21 > K > 0 (B) 13 > K > 0
4

Downloaded From : www.EasyEngineering.net


Downloaded From : www.EasyEngineering.net
GATE SOLVED PAPER - EC CONTROL SYSTEM

(C) 21 < K < 3 (D) - 6 < K < 3


4
Q. 69 For the polynomial P (s) = s2 + s 4 + 2s3 + 2s2 + 3s + 15 the number of roots which
lie in the right half of the s -plane is
(A) 4 (B) 2
(C) 3 (D) 1

Q. 70 The state variable equations of a system are : xo1 =- 3x1 - x2 = u, xo2 = 2x1 and
y = x1 + u . The system is
(A) controllable but not observable
(B) observable but not controllable
(C) neither controllable nor observable
(D) controllable and observable

ww
Q. 71
1 0
Given A = = G, the state transition matrix eAt is given by
0 1

w.E e
0 e-t
(A) > -t
0
e-t 0
H
et 0
(B) = t G
0 e
0 et
(C) >
0 e-t
asy
H (D) = t G
e 0

2003
En ONE MARK

Q. 72

gin
Fig. shows the Nyquist plot of the open-loop transfer function G (s) H (s) of a
system. If G (s) H (s) has one right-hand pole, the closed-loop system is

eer
ing
(A) always stable
.ne
(B) unstable with one closed-loop right hand pole
(C) unstable with two closed-loop right hand poles
(D) unstable with three closed-loop right hand poles
t
Q. 73 A PD controller is used to compensate a system. Compared to the uncompensated
system, the compensated system has
(A) a higher type number (B) reduced damping
(C) higher noise amplification (D) larger transient overshoot

2003 TWO MARKS

Q. 74 The signal flow graph of a system is shown in Fig. below. The transfer function
C (s)/ R (s) of the system is

Downloaded From : www.EasyEngineering.net


Downloaded From : www.EasyEngineering.net
GATE SOLVED PAPER - EC CONTROL SYSTEM

(A) 6 (B) 6s
s2 + 29s + 6 s2 + 29s + 6
s (s + 2) s (s + 27)
(C) 2
(D) 2
s + 29s + 6 s + 29s + 6
Q. 75 The root locus of system G (s) H (s) = K has the break-away point
s (s + 2)( s + 3)
located at
(A) (- 0.5, 0) (B) (- 2.548, 0)
(C) (- 4, 0) (D) (- 0.784, 0)

Q. 76 The approximate Bode magnitude plot of a minimum phase system is shown in


Fig. below. The transfer function of the system is

ww
w.E
(A) 108
asy
(s + 0.1) 3
(s + 10) 2 (s + 100)
(B) 10 7 (s + 0.1) 3
(s + 10)( s + 100)

(C)
(s + 0.1)
En
2

(s + 10) 2 (s + 100)
(D)
(s + 0.1) 3
(s + 10)( s + 100) 2
Q. 77

C (s)
= 2 4
gin
A second-order system has the transfer function

R (s) s + 4s + 4
eer
With r (t) as the unit-step function, the response c (t) of the system is
represented by
ing
.ne
t

Q. 78 The gain margin and the phase margin of feedback system with
G (s) H (s) = 8 are
(s + 100) 3

Downloaded From : www.EasyEngineering.net


Downloaded From : www.EasyEngineering.net
GATE SOLVED PAPER - EC CONTROL SYSTEM

(A) dB, 0c (B) 3, 3


(C) 3, 0c (D) 88.5 dB, 3

Q. 79 The zero-input response of a system given by the state-space equation


xo1 1 0 x1 x1 (0) 1
=xo G = =1 1G=x G and =x (0)G = =0 G is
2 2 2

tet et
(A) = G (B) = G
t t
et t
(C) = t G (D) = t G
te te

2002 ONE MARK

Q. 80 Consider a system with transfer function G (s) = 2s + 6 . Its damping ratio

ww will be 0.5 when the value of k is


(A) 2
6
(B) 3
ks + s + 6

w.E (C) 1
6
(D) 6

Q. 81

asy
Which of the following points is NOT on the root locus of a system with the open-
loop transfer function G (s) H (s) =
(A) s =- j 3
k
s (s + 1)( s + 3)
(B) s =- 1.5
(C) s =- 3
En (D) s =- 3

Q. 82

G (s) H (s) =
(1 - s) gin
The phase margin of a system with the open - loop transfer function

(A) 0c
(1 + s)( 2 + s)
(B) 63.4c eer
Q. 83
(C) 90c (D) 3
ing
The transfer function Y (s)/ U (s) of system described by the state equation
xo (t) =- 2x (t) + 2u (t) and y (t) = 0.5x (t) is
(A) 0.5 (B) 1 .ne
(C) 0.5
(s - 2)

(s + 2)
(D)
(s - 2)
1
(s + 2)
t
2002 TWO MARKS

Q. 84 The system shown in the figure remains stable when


(A) k < - 1 (B) - 1 < k < 3
(C) 1 < k < 3 (D) k > 3

Q. 85 The transfer function of a system is G (s) = (s + 1)(100


s + 100)
. For a unit - step input to
the system the approximate settling time for 2% criterion is

(A)100 sec (B) 4 sec


(C) 1 sec (D) 0.01 sec

Downloaded From : www.EasyEngineering.net


Downloaded From : www.EasyEngineering.net
GATE SOLVED PAPER - EC CONTROL SYSTEM

Q. 86 The characteristic polynomial of a system is


q (s) = 2s5 + s 4 + 4s3 + 2s2 + 2s + 1
The system is
(A) stable
(B) marginally stable
(C) unstable
(D) oscillatory

Q. 87 The system with the open loop transfer function G (s) H (s) = 1 has a
2
gain margin of s (s + s + 1)
(A) - 6 db (B) 0 db
(C) 35 db (D) 6 db

ww 2001 ONE MARK

w.E
Q. 88 The Nyquist plot for the open-loop transfer function G (s) of a unity negative
feedback system is shown in the figure, if G (s) has no pole in the right-half of s -
plane, the number of roots of the system characteristic equation in the right-half
of s -plane is
(A) 0
asy (B) 1

Q. 89
(C) 2

En (D) 3

The equivalent of the block diagram in the figure is given is

gin
eer
ing
.ne
t
Q. 90 The root-locus diagram for a closed-loop feedback system is shown in the figure.
The system is overdamped.

Downloaded From : www.EasyEngineering.net


Downloaded From : www.EasyEngineering.net
GATE SOLVED PAPER - EC CONTROL SYSTEM

(A) only if 0 # k # 1 (B) only if 1 < k < 5


(C) only if k > 5 (D) if 0 # k < 1 or k > 5

Q. 91 If the characteristic equation of a closed - loop system is s2 + 2s + 2 = 0 , then the


system is
(A) overdamped (B) critically damped
(C) underdamped (D) undamped

2001 TWO MARK

Q. 92 An electrical system and its signal-flow graph representations are shown the
figure (A) and (B) respectively. The values of G2 and H , respectively are

ww
w.E
asy
(A) En
Z3 (s)
,
- Z3 (s)
(B)
- Z3 (s)
,
- Z3 (s)

(C)
Z3 (s)
, gin
Z1 (s) + Z3 (s) + Z4 (s) Z1 (s) + Z3 (s)
Z3 (s)
Z2 (s) + Z3 (s) + Z4 (s) Z1 (s) + Z3 (s)
(D)
Z2 (s) - Z3 (s) + Z4 (s) Z1 (s) + Z3 (s)
- Z3 (s)
,
Z3 (s)
Z2 (s) - Z3 (s) + Z4 (s) Z1 (s) + Z3 (s)
Q. 93

transfer function 2 s + 4
eer
The open-loop DC gain of a unity negative feedback system with closed-loop

(A) 4
13
s + 7s + 13
is
(B) 4
9 ing
(C) 4 (D) 13
.ne
Q. 94 The feedback control system in the figure is stable
t
(A) for all K $ 0 (B) only if K $ 0
(C) only if 0 # K < 1 (D) only if 0 # K # 1

2000 ONE MARK

Q. 95 An amplifier with resistive negative feedback has tow left half plane poles in its
open-loop transfer function. The amplifier
(A) will always be unstable at high frequency
(B) will be stable for all frequency
(C) may be unstable, depending on the feedback factor
(D) will oscillate at low frequency.

Downloaded From : www.EasyEngineering.net


Downloaded From : www.EasyEngineering.net
GATE SOLVED PAPER - EC CONTROL SYSTEM

2000 TWO MARKS

Q. 96 A system described by the transfer function H (s) = 3 1 is stable.


2
The constraints on a and k are. s + a s + ks + 3
(A) a > 0, ak < 3 (B) a > 0, ak > 3
(C) a < 0, ak > 3 (D) a > 0, ak < 3

1999 ONE MARK

Q. 97 For a second order system with the closed-loop transfer function


T (s) = 2 9
s + 4s + 9
the settling time for 2-percent band, in seconds, is
(A) 1.5 (B) 2.0

ww
Q. 98
(C) 3.0 (D) 4.0

The gain margin (in dB) of a system a having the loop transfer function

w.E (A) 0
G (s) H (s) = 2
s (s + 1)
is
(B) 3
(C) 6
asy (D) 3

Q. 99

X =>
0 1
En
The system modeled described by the state equations is

2 - 3H
0
x + > Hu

Y = 81 1B x
1
gin
(A) controllable and observable
(C) observable, but not controllable eer
(B) controllable, but not observable
(D) neither controllable nor observable

Q. 100
ing
The phase margin (in degrees) of a system having the loop transfer function
G (s) H (s) = 2 3 is
(A) 45c
s (s + 1)
(B) - 30c .ne
(C) 60c (D) 30c
t
1999 TWO MARKS

Q. 101 An amplifier is assumed to have a single-pole high-frequency transfer function.


The rise time of its output response to a step function input is 35 n sec . The upper
3 dB frequency (in MHz) for the amplifier to as sinusoidal input is approximately
at
(A) 4.55 (B) 10
(C) 20 (D) 28.6

Q. 102 If the closed - loop transfer function T (s) of a unity negative feedback system is
given by
T (s) = n an - 1 s + an
s + a1 sn - 1 + .... + an - 1 s + an
then the steady state error for a unit ramp input is

Downloaded From : www.EasyEngineering.net


Downloaded From : www.EasyEngineering.net
GATE SOLVED PAPER - EC CONTROL SYSTEM

(A) an (B) an
an - 1 an - 2
(C) an - 2 (D) zero
an
Q. 103 Consider the points s1 =- 3 + j4 and s2 =- 3 - j2 in the s-plane. Then, for a
system with the open-loop transfer function
G (s) H (s) = K 4
(s + 1)
(A) s1 is on the root locus, but not s2
(B) s2 is on the root locus, but not s1
(C) both s1 and s2 are on the root locus
(D) neither s1 nor s2 is on the root locus

Q. 104 For the system described by the state equation

ww R 0 1 0V
S W
R0V
S W
xo = S 0 0 1W x + S0W u
S0.5 1 2W S1W

w.E T X T X
If the control signal u is given by u = [- 0.5 - 3 - 5] x + v , then the eigen values
of the closed-loop system will be

asy
(A) 0, - 1, - 2
(C) - 1, - 1, - 2
(B) 0, - 1, - 3
(D) 0, - 1, - 1

1998 En ONE MARK

Q. 105
gin
The number of roots of s3 + 5s2 + 7s + 3 = 0 in the left half of the s -plane is
(A) zero
(C) two
eer
(B) one
(D) three

Q. 106

(A) Ks (B) K ing


The transfer function of a tachometer is of the form

(C) K
(s + 1)
(D)
s
K
s (s + 1)
.ne
Q. 107

G (s) = K .
s (s + 1)
t
Consider a unity feedback control system with open-loop transfer function

The steady state error of the system due to unit step input is
(A) zero (B) K
(C) 1/K (D) infinite

Q. 108 The transfer function of a zero-order-hold system is


(A) (1/s) (1 + e-sT ) (B) (1/s) (1 - e-sT )
(C) 1 - (1/s) e-sT (D) 1 + (1/s) e-sT

Q. 109 In the Bode-plot of a unity feedback control system, the value of phase of G (jw)
at the gain cross over frequency is - 125c. The phase margin of the system is
(A) - 125c (B) - 55c
(C) 55c (D) 125c

Downloaded From : www.EasyEngineering.net


Downloaded From : www.EasyEngineering.net
GATE SOLVED PAPER - EC CONTROL SYSTEM

Q. 110 Consider a feedback control system with loop transfer function


K (1 + 0.5s)
G (s) H (s) =
s (1 + s) (1 + 2s)
The type of the closed loop system is
(A) zero (B) one
(C) two (D) three

Q. 111 The transfer function of a phase lead controller is 1 + 3Ts . The maximum value
1 + Ts
of phase provided by this controller is
(A) 90c (B) 60c
(C) 45c (D) 30c

Q. 112 The Nyquist plot of a phase transfer function g (jw) H (jw) of a system encloses the
(–1, 0) point. The gain margin of the system is

ww (A) less than zero


(C) greater than zero
(B) zero
(D) infinity

w.E
Q. 113

of the system is
2
The transfer function of a system is 2s + 26s + 5 . The characteristic equation
(s + 1) (s + 2)

asy
(A) 2s2 + 6s + 5 = 0
(C) 2s2 + 6s + 5 + (s + 1) 2 (s + 2) = 0
(B) (s + 1) 2 (s + 2) = 0
(D) 2s2 + 6s + 5 - (s + 1) 2 (s + 2) = 0

Q. 114
En
In a synchro error detector, the output voltage is proportional to [w (t)] n, where w (t)
is the rotor velocity and n equals
(A) –2
(C) 1 gin(B) –1
(D) 2

eer
Q. 115
1997

In the signal flow graph of the figure is y/x equals ing ONE MARK

.ne
(A) 3 (B) 5
2
t
(C) 2 (D) None of the above

Q. 116 A certain linear time invariant system has the state and the output equations
given below
Xo1 1 - 1 X1 0
> o H = >0 1 H>X H + >1H u
X2 2

dy
y = 81 1B: X1 D, If X1 (0) = 1, X2 (0) =- 1, u (0) = 0, then is
X2 dt t=0
(A) 1 (B) –1
(C) 0 (D) None of the above

***********

Downloaded From : www.EasyEngineering.net


Downloaded From : www.EasyEngineering.net
GATE SOLVED PAPER - EC CONTROL SYSTEM

SOLUTIONS

Sol. 1 Option (B) is correct.


From the given plot, we obtain the slope as
20 log G2 - 20 log G1
Slope =
log w2 - log w1
From the figure
20 log G2 =- 8 dB
20 log G1 = 32 dB
and w1 = 1 rad/s

ww So, the slope is


w2 = 10 rad/s

w.E Slope = - 8 - 32
log 10 - log 1
=- 40 dB/decade

at w = 1
asy
Therefore, the transfer function can be given as
G ^s h = k2
S

En
G ^ jwh = k 2 = k
w
In decibel,
20 log G ^ jwh = 20 log k = 32 gin
or,
32

k = 10 = 39.8
Hence, the Transfer function is
20

eer
G ^s h = k2 = 392.8
s s ing
Sol. 2 Option (A) is correct.
For the given SFG, we have two forward paths
Pk1 = ^1 h^s-1h^s-1h^1 h = s-2
.ne
Pk2 = ^1 h^s-1h^1 h^1 h = s-1
t
since, all the loops are touching to the paths Pk1 and Pk2 so,
Dk 1 = Dk 2 = 1
Now, we have
D = 1 - (sum of individual loops)
+ (sum of product of nontouching loops)
Here, the loops are
L1 = ^- 4h^1 h =- 4
L2 = ^- 4h^s-1h = 4s-1
L 3 = ^- 2h^s-1h^s-1h =- 2s-2
L 4 = ^- 2h^s-1h^1 h =- 2s-1
As all the loop L1, L2, L 3 and L 4 are touching to each other so,
D = 1 - ^L1 + L2 + L 3 + L 4h
= 1 - ^- 4 - 4s-1 - 2s-2 - 2s-1h

Downloaded From : www.EasyEngineering.net


Downloaded From : www.EasyEngineering.net
GATE SOLVED PAPER - EC CONTROL SYSTEM

= 5 + 6s1 + 2s2
From Mason’s gain formulae
Y ^s h s-2 + s-1
= SPk Dk = = 2s+1
U ^s h D 5 + 6s-1 + 2s-2 5s + 6s + 2
Sol. 3 Option (A) is correct.
For the shown state diagram we can denote the states x1 , x2 as below

So, from the state diagram, we obtain

ww xo1 =- x1 - u
xo2 =- x2 + ^1 h^- 1h^1 h^- 1h u + ^- 1h^1 h^- 1h x1
xo2 =- x2 + x1 + u

w.E and y = ^- 1h^1 h x2 + ^- 1h^1 h^- 1h x1 + ^1 h^- 1h^1 h^- 1h^1 h u


= x1 - x 2 + u

asy
Hence, in matrix form we can write the state variable equations
xo1 - 1 0 x1 -1
> o H = > 1 - 1H >x H + > 1 H u
x2
En x1
2

y = 81 - 1B > H + u
and
x2
gin
which can be written in more general form as

Xo = >
-1 0
1 - 1H
-1
X +> H
1 eer
Sol. 4 Option (A) is correct.
y = 81 - 1B X + u
ing
From the obtained state-variable equations
We have .ne
A =>
-1 0
1 - 1H
S+1 0
t
SI - A = >
- 1 S + 1H
So,

1 >S + 1 0 H
and ^SI - Ah-1 =
^S + 1h2 1 S + 1
R 1 V
S 0 W
S+1
=S 1 1 W
W
S
S^S + 1h2 S + 1W
T X
Hence, the state transition matrix is obtained as
eAt = L-1 ^SI - Ah-1
ZR V_
]S 1 0 Wb -1
S+1
= L-1 [S 1 W` = >e 0
H
S 1 W te e-t
-t
]S^S + 1h2 S + 1Wb
\T Xa

Downloaded From : www.EasyEngineering.net


Downloaded From : www.EasyEngineering.net
GATE SOLVED PAPER - EC CONTROL SYSTEM

Sol. 5 Option (C) is correct.


Given, open loop transfer function
G ^s h = 10Ka = Ka 1
1 + 10s s + 10
By taking inverse Laplace transform, we have
g ^ t h = e- t
1
10

Comparing with standard form of transfer function, Ae-t/t , we get the open
loop time constant,
tol = 10
Now, we obtain the closed loop transfer function for the given system as
G ^s h
H ^s h = = 10Ka = Ka
1 + G ^s h 1 + 10s + 10Ka s + ^Ka + 101 h
Taking inverse Laplace transform, we get
h ^ t h = ka .e-^k + ht
1
a 10

ww So, the time constant of closed loop system is obtained as


tcl = 1 1
ka + 10

w.E or, tcl = 1


ka
(approximately)
Now, given that ka reduces open loop time constant by a factor of 100. i.e.,

asy tcl = tol


1
100
= 10
or,
Hence, En ka 100
ka = 10
Sol. 6 Option (C) is correct.
(s2 + 9) (s + 2) gin
G (s) =
(s + 1) (s + 3) (s + 4)
(- w2 + 9) (jw + 2) eer
=
(jw + 1) (jw + 3) (jw + 4)
The steady state output will be zero if ing
G (jw) = 0
-w 2 + 9 = 0 & w = 3 rad/s .ne
Sol. 7 Option (A) is correct.
Y (s) =
K (s + 1)
s + as2 + 2s + 1
3
t
[R (s) - Y (s)]
K (s + 1) K (s + 1)
Y (s) ;1 + 3 2 E = 3 R (s)
s + as + 2s + 1 s + as2 + 2s + 1
Y (s) [s3 + as2 + s (2 + k) + (1 + k)] = K (s + 1) R (s)
Y (s) K (s + 1)
Transfer Function, H (s) = = 3
R (s) s + as + s (2 + k) + (1 + k)
2

Routh Table :

Downloaded From : www.EasyEngineering.net


Downloaded From : www.EasyEngineering.net
GATE SOLVED PAPER - EC CONTROL SYSTEM

a (2 + K) - (1 + K)
For oscillation, =0
a
a = K+1
K+2
Auxiliary equation A (s) = as2 + (k + 1) = 0
s2 =- k + 1 = - k + 1 (k + 2) =- (k + 2)
a (k + 1)
s = j k+2
jw = j k + 2
w = k+2 = 2 (Oscillation frequency)
k =2
and a = 2 + 1 = 3 = 0.75
2+2 4

ww
Sol. 8 Option (D) is correct.
General form of state equations are given as

w.E xo = Ax + Bu

For the given problem


yo = Cx + Du

asy R 0 a 0V
S 1 W
A = S 0 0 a2W, B = S0W
R0V
S W

En Sa

S
3 0 0W
RT 0 a 0VXR0V R 0VT X
1 WS W S W
S 1W

AB = S 0 0 a2WS0W = Sa2W
Sa
3
RT 0
gin
0 0WS 1W S 0W
0XT aX1 a2VWTRS0XVW RSa1 a2VW
S
A2 B = Sa2 a 3
S 0 a a
0
eer
0WS0W = S 0W
0WS 1W S 0W
T
3 1
XT X T
ing X
For controllability it is necessary that following matrix has a tank of n = 3 .
R0 0 a a V
U = 6B : AB : A2 B@ = S0 a2
S
S1 0 .ne
1 2W
0W
0W

Sol. 9
So, a2 ! 0
a1 a 2 ! 0 & a1 ! 0
Option (B) is correct.
T X
ta 3 may be zero or not.

For given plot root locus exists from - 3 to 3, So there must be odd number of
poles and zeros. There is a double pole at s =- 3
Now poles = 0, - 2, - 3, - 3
zeros =- 1
k (s + 1)
Thus transfer function G (s) H (s) =
s (s + 2) (s + 3) 2
Sol. 10 Option (A) is correct.
We have G (jw) = 5 + jw
Here s = 5 . Thus G (jw) is a straight line parallel to jw axis.
Sol. 11 Option (B) is correct.
dy
Here x = y1 and xo = 1
dx

Downloaded From : www.EasyEngineering.net


Downloaded From : www.EasyEngineering.net
GATE SOLVED PAPER - EC CONTROL SYSTEM

y1 x 1
y = > H = > H = > Hx
y2 2x 2
Now y1 = 1 u
s+2
y1 (s + 2) = u
yo1 + 2y1 = u
xo + 2x = u
xo =- 2x + u
xo = [- 2] x + [1] u
Drawing SFG as shown below

ww
w.E Thus xo1 = [- 2] x1 + [1] u
y1 = x1 ; y2 = 2x1

asy y1 1
y = > H = > H x1
y2 2

Sol. 12
Here
En
Option (C) is correct.
x1 = x

We have G (s) H (s) =


gin
100
s (s + 10) 2
Now G (jw) H (jw) = 100
jw (jw + 10) 2
eer
Thus ing
If wp is phase cross over frequency +G (jw) H (jw) = 180c
- 180c = 100 tan-1 0 - tan-1 3 - 2 tan-1 a
wp
10 k
or - 180c =- 90 - 2 tan-1 (0.1wp)
.ne
or
or
or
45c = tan-1 (0.1wp)
tan 45c 0.1wp = 1
wp = 10 rad/se
t
Now G (jw) H (jw) = 100
w (w2 + 100)
At w = wp
G (jw) H (jw) = 100 = 1
10 (100 + 100) 20
Gain Margin =- 20 log 10 G (jw) H (jw)
=- 20 log 10 b 1 l
20
= 26 dB
Sol. 13 Option (D) is correct.
From option (D) TF = H (s)
= 100 ! 100
s (s2 + 100) s (s + 10) 2

Downloaded From : www.EasyEngineering.net


Downloaded From : www.EasyEngineering.net
GATE SOLVED PAPER - EC CONTROL SYSTEM

Sol. 14 Option (B) is correct.


From the given block diagram

H (s) = Y (s) - E (s) $ 1


s+1
E (s) = R (s) - H (s)
E (s)
= R (s) - Y (s) +
(s + 1)

ww E (s) :1 - 1
s + 1D
= R (s) - Y (s)

w.E sE (s)
(s + 1)
= R (s) - Y (s)
E (s)
...(1)

From (1) and (2)asy Y (s) =


s+1
sY (s) = R (s) - Y (s)
...(2)

Transfer function En (s + 1) Y (s) = R (s)

Y (s)
= 1
R (s) s + 1 gin
Sol. 15 Option (B) is correct.
Transfer function is given as eer
H (s) =
Y (s)
= s
X (s) s + p ing
H (jw) =
jw
jw + p
.ne
Amplitude Response
H (jw) = w
w +p2
2
t
Phase Response qh (w) = 90c - tan-1 a w k
p
Input x (t) = p cos a2t - p k
2
Output y (t) = H (jw) x (t - qh) = cos a2t - p k
3
H (jw) = p = w
w +p2
2

1 = 2 , (w = 2 rad/ sec)
p 4+p2
or 4p 2 = 4 + p 2 & 3p 2 = 4
or p = 2/ 3
Alternative :
qh = 9- p - a- p kC = p
3 2 6

Downloaded From : www.EasyEngineering.net


Downloaded From : www.EasyEngineering.net
GATE SOLVED PAPER - EC CONTROL SYSTEM

So, p = p - tan-1 a w k
6 2 p
tan-1 a w k =p-p =p
p 2 6 3
w = tan a p k = 3
p 3
2 = 3, (w = 2 rad/ sec)
p
or p = 2/ 3
Sol. 16 Option (A) is correct.
Initial slope is zero, so K = 1
At corner frequency w 1 = 0.5 rad/ sec , slope increases by + 20 dB/decade, so
there is a zero in the transfer function at w 1

ww At corner frequency w 2 = 10 rad/ sec , slope decreases by - 20 dB/decade and


becomes zero, so there is a pole in transfer function at w 2

w.E Transfer function H (s) =


K a1 + s k
s
w1
a1 + w 2 k

asy =
1 a1 + s k (1 + 10s)
0.1
s
=

Sol. 17
En
Option (D) is correct.
a1 + 0.1 k
(1 + 0.1s)

gin
Assign output of each integrator by a state variable

eer
ing
xo1 =- x1 + x2
xo2 =- x1 + 2u .ne
y = 0.5x1 + 0.5x2
State variable representation

xo = >
-1 1 0
t
-1 0 H x + > Hu
2
yo = [0.5 0.5] x
Sol. 18 Option (C) is correct.
By masson’s gain formula

Transfer function
H (s) =
Y (s)
=
/ PK DK
U (s) D

Downloaded From : www.EasyEngineering.net


Downloaded From : www.EasyEngineering.net
GATE SOLVED PAPER - EC CONTROL SYSTEM

Forward path given


P1 (abcdef ) = 2 # 1 # 1 # 0.5 = 12
s s s
1
P2 (abcdef ) = 2 # # 1 # 0.5
3
Loop gain L1 (cdc) =- 1
s
L2 (bcdb) = 1 # 1 # - 1 = -21
s s s
D = 1 - [L1 + L2] = 1 - :- 1 - 12 D = 1 + 1 + 12
s s s s
D1 = 1, D2 = 2
Y (s)
So, H (s) = = P1 D 1 + P2 D 2
U (s) D

ww = s
1 :1+1:1
2

1
s
1+ + 2 1
= 2
(s
(1 + s)
+ s + 1)

w.E
Sol. 19 Option (D) is correct.
s s

Steady state error is given as

asy
eSS = lim
s"0
sR (s)
1 + G (s) GC (s)

En
R (s) = 1
s
(unit step unit)

eSS = lim
s"0
1
gin
1 + G (s) GC (s)
= lim
s"0
1+ 2
1
GC (s)
s + 2s + 2 eer
eSS will be minimum if lim GC (s) is maximum
In option (D)
s"0
ing
lim GC (s) = lim 1 + 2 + 3s = 3
s"0 s"0 s .ne
Sol. 20
So, eSS = lim 1 = 0 (minimum)

Option (C) is correct.


s"0 3
t
This compensator is roughly equivalent to combining lead and lad compensators
in the same design and it is referred also as PID compensator.
Sol. 21 Option (C) is correct.
1 0 p
Here A == G and B = = G
0 1 q
1 0 p p
AB = =
1G=q G =q G
=
0
p q
S = 8B AB B = =
q pG
S = pq - pq = 0
Since S is singular, system is completely uncontrollable for all values of p and q .

Downloaded From : www.EasyEngineering.net


Downloaded From : www.EasyEngineering.net
GATE SOLVED PAPER - EC CONTROL SYSTEM

Sol. 22 Option (B) is correct.


The characteristic equation is
1 + G (s) H (s) = 0
K (s2 - 2s + 2)
or 1+ =0
s 2 + 2s + 2
or s2 + 2s + 2 + K (s2 - 2s + 2) = 0
2
or K =- s2 + 2s + 2
s - 2s + 2
For break away & break in point differentiating above w.r.t. s we have
2 2
dK =- (s - 2s + 2)( 2s + 2) - (s + 2s + 2)( 2s - 2) = 0
ds (s2 - 2s + 2) 2
Thus (s2 - 2s + 2)( 2s + 2) - (s2 + 2s + 2)( 2s - 2) = 0
or s =! 2

ww Let qd be the angle of departure at pole P , then

w.E
asy
En
- qd - qp1 + qz1 + qz2 = 180c

gin
- qd = 180c - (- qp1 + qz1 + q2)
= 180c - (90c + 180 - 45c) =- 45c
Sol. 23 Option (B) is correct.
For under-damped second order response eer
T (s) = 2 kwn2
s + 2xwn s + wn2 ing where x < 1

Thus (A) or (B) may be correct


For option (A) .ne
wn = 1.12 and 2xwn = 2.59 " x = 1.12

Sol. 24
For option (B)
Option (B) is correct. t
wn = 1.91 and 2xwn = 1.51 " x = 0.69

The plot has one encirclement of origin in clockwise direction. Thus G (s) has a
zero is in RHP.
Sol. 25 Option (C) is correct.
The Nyzuist plot intersect the real axis ate - 0.5. Thus
G. M. =- 20 log x =- 20 log 0.5 = 6.020 dB
And its phase margin is 90c.
Sol. 26 Option (C) is correct.
Transfer function for the given pole zero plot is:
(s + Z1)( s + Z2)
(s + P1)( s + P2)
From the plot Re (P1 and P2 )>(Z1 and Z2 )
So, these are two lead compensator.
Hence both high pass filters and the system is high pass filter.

Downloaded From : www.EasyEngineering.net


Downloaded From : www.EasyEngineering.net
GATE SOLVED PAPER - EC CONTROL SYSTEM

Sol. 27 Option (C) is correct.


Percent overshoot depends only on damping ratio, x .
2
Mp = e- xp 1 - x
If Mp is same then x is also same and we get
x = cos q
Thus q = constant
The option (C) only have same angle.
Sol. 28 Option (C) is correct.
We labeled the given SFG as below :

ww
w.E From this SFG we have
xo1 =- gx1 + bx3 + m1

asy
xo2 = gx1 + ax3
xo3 =- bx1 - ax3 + u2
R V R VR V R V
Thus En
Sx1 W S- g 0 b WSx1 W S0 1 W u1
Sx2 W = S g 0 a WSx2 W + S0 0 We o
SSx WW SS- b 0 - a WWSSx WW SS1 0 WW u2
Sol. 29
3
T X T
Option (D) is correct.
3

gin
XT X T X

P = 2 25
s + 25
2xwn = 0, x = 0 " Undamped
eer Graph 3

Q= 2 62
s + 20s + 62
2xwn = 20, x > 1 " Overdamped
ing
Graph 4

R= 62
s + 12s + 62
2
2xwn = 12, x = 1 " Critically
.ne
Graph 1

Sol. 30
S= 72
s2 + 7s + 72

Option (C) is correct.


2xwn = 7, x < 1 " underdamped Graph 2
t
The characteristic equation of closed lop transfer function is
1 + G (s) H (s) = 0
1+ 2 s+8 =0
s + as - 4
or s 2 + as - 4 + s + 8 = 0
or s2 + (a + 1) s + 4 = 0
This will be stable if (a + 1) > 0 " a > - 1. Thus system is stable for all
positive value of a.
Sol. 31 Option (C) is correct.
The characteristic equation is
1 + G (s) = 0

Downloaded From : www.EasyEngineering.net


Downloaded From : www.EasyEngineering.net
GATE SOLVED PAPER - EC CONTROL SYSTEM

or s5 + 2s 4 + 3s3 + 6s2 + 5s + 3 = 0
Substituting s = z1 we have
3z5 + 5z 4 + 6z3 + 3z2 + 2z + 1 = 0
The routh table is shown below. As there are two sign change in first column,
there are two RHS poles.

z5 3 6 2
4 5 3 1
z
z3 21
5
7
5

z2 4
3 3
z1 - 74
z0 1

ww
Sol. 32 Option (A) is correct.

w.E For underdamped second order system the transfer function is


T (s) =Kwn2
s + 2xwn s + wn2
2

asy
It peaks at resonant frequency. Therefore
Resonant frequency wr = wn 1 - 2x2

En
and peak at this frequency
mr = 5

gin
2x 1 - x2

We have wr = 5 2 , and mr = 10 . Only options (A) satisfy these values.

wn = 10, x =
3
1 eer
where
2
wr = 10 1 - 2` 1 j = 5 2
4
ing
and mr = 1 5
22 1- 4 1
= 10
3 .ne
Hence satisfied

Sol. 33 Option (B) is correct.


t
The given circuit is a inverting amplifier and transfer function is
Vo = - Z = - Z (sC1 R1 + 1)
Vi R
sC R + 1
1
R1
1 1

(sC2 R2 + 1)
For Q , Z =
sC2
Vo (sC2 R2 + 1) (sC1 R1 + 1)
=- # PID Controller
Vi sC2 R1
For R, Z = R 2
(sC2 R2 + 1)
Vo R2 (sC1 R1 + 1)
=- #
Vi (sC2 R2 + 1) R1
Since R2 C2 > R1 C1, it is lag compensator.

Sol. 34 Option (D) is correct.


In a minimum phase system, all the poles as well as zeros are on the left half of

Downloaded From : www.EasyEngineering.net


Downloaded From : www.EasyEngineering.net
GATE SOLVED PAPER - EC CONTROL SYSTEM

the s -plane. In given system as there is right half zero (s = 5), the system is a
non-minimum phase system.

Sol. 35 Option (B) is correct.


We have Kv = lim sG (s) H (s)
s"0

(Kp + KD s) 100
or 1000 = lim s = Kp
s"0 s (s + 100)
Now characteristics equations is
1 + G (s) H (s) = 0
(Kp + KD s) 100
1000 = lims " 0 s = Kp
s (s + 100)
Now characteristics equation is

ww or 1+
1 + G (s) H (s) = 0
(100 + KD s) 100
s (s + 10)
=0 Kp = 100

w.E or s2 + (10 + 100KD) s + 10 4 = 0


Comparing with s2 + 2xwn + wn2 = 0 we get

or asy
2xwn = 10 + 100KD
KD = 0.9
Sol. 36
En
Option (D) is correct.
5
We have T (s) =

= gin
(s + 5)( s2 + s + 1)
5 = 2 1
5`1 + s j (s2 + s + 1)
5
s +s+1
eer
In given transfer function denominator is (s + 5)[( s + 0.5) 2 + 43 ]. We can see
easily that pole at s =- 0.5 ! j
have approximated it.
2
3
ing
is dominant then pole at s =- 5 . Thus we

Sol. 37 Option (A) is correct.


.ne
G (s) = 1 = 1
s2 - 1 (s + 1)( s - 1)
t
The lead compensator C (s) should first stabilize the plant i.e. remove 1
(s - 1)
term. From only options (A), C (s) can remove this term

1 10 (s - 1)
Thus G (s) C (s) = #
(s + 1)( s - 1) (s + 2)
= 10 Only option (A) satisfies.
(s + 1)( s + 2)
Sol. 38 Option (D) is correct.
For ufb system the characteristics equation is
1 + G (s) = 0
or 1+ K =0
s (s2 + 7s + 12)
or s (s2 + 7s + 12) + K = 0
Point s =- 1 + j lie on root locus if it satisfy above equation i.e
(- 1 + j)[( - 1 + j) 2 + 7 (- 1 + j) + 12) + K] = 0

Downloaded From : www.EasyEngineering.net


Downloaded From : www.EasyEngineering.net
GATE SOLVED PAPER - EC CONTROL SYSTEM

or K =+ 10
Sol. 39 Option (D) is correct.
At every corner frequency there is change of -20 db/decade in slope which indicate
pole at every corner frequency. Thus
G (s) = K
s (1 + s)`1 + s j
20
Bode plot is in (1 + sT) form
20 log K = 60 dB = 1000
w w = 0. 1
Thus K =5
Hence G (s) = 100
s (s + 1)( 1 + .05s)
Sol. 40 Option (A) is correct.

ww We have
dw
-1 1 w 0
> di H = =- 1 - 10G=in G + =10Gu
dt
dt
a

w.E or

and
dw =- w + i
dt n

dia =- w - 10i + 10u


...(1)

...(2)

asy dt a

Taking Laplace transform (i) we get

or En
sw (s) =- w (s) = Ia (s)
(s + 1) w (s) = Ia (s) ...(3)
Taking Laplace transform (ii) we get
gin
sIa (s) =- w (s) - 10Ia (s) + 10U (s)
or w (s) = (- 10 - s) Ia (s) + 10U (s)
eer
= (- 10 - s)( s + 1) w (s) + 10U (s) From (3)
or w (s) =- [s2 + 11s + 10] w (s) + 10U (s)
(s2 + 11s + 11) w (s) = 10U (s)
ing
or
or
w (s)
U (s)
= 2 10
(s + 11s + 11)
.ne
Sol. 41 Option (A) is correct.
We have xo (t) = Ax (t)
p q
t
A ==
r sG
Let

1 e-2t
For initial state vector x (0) = = G the system response is x (t) = > H
-2 - 2e-2t
d
e-2t p q 1
> d (- 2e-2t)H ==
r s G=- 2G
dt
Thus
dt
t=0

- 2e-2 (0) p q 1
or > 4e-2 (0) H = =r s G=- 2G
-2 p - 2q
= 4 G = =r - 2s G

We get p - 2q =- 2 and r - 2s = 4 ...(i)

Downloaded From : www.EasyEngineering.net


Downloaded From : www.EasyEngineering.net
GATE SOLVED PAPER - EC CONTROL SYSTEM

1 e-t
For initial state vector x (0) = = G the system response is x (t) = > -tH
-1 -e
d
e-t p q 1
> d (- e-t)H ==
r s G=- 1G
dt
Thus
dt
t=0

- (0)
-e p q 1
> e- (0) H = =r s G=- 1G
-1 p-q
= 1 G = =r - s G

We get p - q =- 1 and r - s = 1 ...(2)


Solving (1) and (2) set of equations we get
p q 0 1
=r s G = =- 2 - 3G

ww The characteristic equation


lI - A = 0

w.E l -1
2 l+3
=0

or
or asy l (l + 3) + 2 = 0
l =- 1, - 2

En
Thus Eigen values are - 1 and - 2
Eigen vectors for l1 =- 1
(l1 I - A) X1 = 0
gin
or
l1 - 1 x11
= 2 l + 3G=x G = 0
1 21
eer
- 1 - 1 x11
= 2 2 G=x G = 0
21
ing
or
or
- x11 - x21 = 0
x11 + x21 = 0 .ne
We have only one independent equation x11 =- x21.
Let x11 = K , then x21 =- K , the Eigen vector will be
x11 K 1
=x G = =- K G = K =- 1G
t
21

Now Eigen vector for l2 =- 2


(l2 I - A) X2 = 0
l2 - 1 x12
or = 2 l + 3G=x G = 0
2 22
- 2 - 1 x11
or = 2 1 G=x G = 0
21

or - x11 - x21 = 0
or x11 + x21 = 0
We have only one independent equation x11 =- x21.
Let x11 = K, then x21 =- K , the Eigen vector will be
x12 K 1
=x G = =- 2K G = K =- 2G
22

Downloaded From : www.EasyEngineering.net


Downloaded From : www.EasyEngineering.net
GATE SOLVED PAPER - EC CONTROL SYSTEM

Sol. 42 Option (D) is correct.


As shown in previous solution the system matrix is
0 1
A ==
- 2 - 3G
Sol. 43 Option (D) is correct.
Given system is 2nd order and for 2nd order system G.M. is infinite.
Sol. 44 Option (D) is correct.
Sol. 45 Option (D) is correct.
If the Nyquist polt of G (jw) H (jw) for a closed loop system pass through (- 1, j0)
point, the gain margin is 1 and in dB
GM =- 20 log 1

ww
Sol. 46 Option (B) is correct.
= 0 dB

w.E The characteristics equation is


1 + G (s) H (s) = 0

1+
asyK (s + 1)
s + as2 + 2s + 1
3
=0

En
s3 + as2 + (2 + K) s + K + 1 = 0

a (2 + K) - (K + 1)
a
=0 gin
The Routh Table is shown below. For system to be oscillatory stable

or a = K+1
K+2 eer ...(1)

Then we have
as2 + K + 1 = 0 ing
At 2 rad/sec we have
s = jw " s2 =- w2 =- 4 ,
.ne
Thus - 4a + K + 1 = 0
Solving (i) and (ii) we get K = 2 and a = 0.75 .
t ...(2)

s3 1 2+K
s 2
a 1+K
(1 + K) a - (1 + K)
s1 a

s0 1+K

Sol. 47 Option (D) is correct.


The transfer function of given compensator is
Gc (s) = 1 + 3Ts T>0
1 + Ts
Comparing with
Gc (s) = 1 + aTs we get a = 3
1 + Ts
The maximum phase sift is

Downloaded From : www.EasyEngineering.net


Downloaded From : www.EasyEngineering.net
GATE SOLVED PAPER - EC CONTROL SYSTEM

fmax = tan-1 a - 1 = tan-1 3 - 1 = tan-1 1


2 a 2 3 3
or fmax = p
6
Sol. 48 Option (A) is correct.
s 0 0 1 s -1
(sI - A) = = G - = G ==
0 s -1 0 1 sG
1 =s - 1G = >s + 1
s 1

H
2
s2 + 1
(sI - A) -1 = -1 s
s2 + 1 1 s s +12
s2 + 1

cos t sin t
f (t) = eAt = L-1 [(sI - A)] -1 = =
- sin t cos t G
Sol. 49 Option (C) is correct.
G (s) = as + 1

ww We have
s 2

+G (jw) = tan-1 (wa) - p

w.E Since PM is p i.e. 45c, thus


4
p = p + +G (jw ) w " Gain cross over Frequency
4 g g

or
asy
p = p + tan-1 (w a) - p
4 g

or
4
En
p = tan-1 (w a)
g

or awg = 1
At gain crossover frequency G (jwg) = 1 gin
Thus
1+ a
wg2
2
wg2
=1
eer
or 1 + 1 = wg2
1 ing (as awg = 1)
or wg = (2)

.ne
4

Sol. 50 Option (C) is correct.


For a = 0.84 we have
G (s) = 0.84s2 + 1
s
Due to ufb system H (s) = 1 and due to unit impulse response R (s) = 1, thus
t
C (s) = G (s) R (s) = G (s)
= 0.84s2 + 1 = 12 + 0.84
s s s
Taking inverse Laplace transform
c (t) = (t + 0.84) u (t)
At t = 1, c (1 sec) = 1 + 0.84 = 1.84
Sol. 51 Option (D) is correct.
The transfer function of a lag network is
T (s) = 1 + sT b > 1; T > 0
1 + sbT

T (jw) = 1 + w2 T2
1 + w2 b2 T2

Downloaded From : www.EasyEngineering.net


Downloaded From : www.EasyEngineering.net
GATE SOLVED PAPER - EC CONTROL SYSTEM

and +T (jw) = tan-1 (wT) - tan-1 (wbT)


At w = 0 , T (jw) = 1
At w = 0 , +T (jw) =- tan-1 0 = 0
At w = 3 , T (jw) = 1
b
At w = 3 , +T (jw) = 0
Sol. 52 Option (C) is correct.
We have Xo = AX + BU where l is set of Eigen values
and Wo = CW + DU where m is set of Eigen values
If a liner system is equivalently represented by two sets of state equations, then
for both sets, states will be same but their sets of Eigne values will not be same

ww i.e.
X = W but l ! m

w.E
Sol. 53 Option (A) is correct.
Despite the presence of negative feedback, control systems still have problems of
instability because components used have nonlinearity. There are always some

Sol. 54 asy
variation as compared to ideal characteristics.
Option (B) is correct.
Sol. 55

En
Option (C) is correct.
The peak percent overshoot is determined for LTI second order closed loop system

T (s) = wn2 gin


with zero initial condition. It’s transfer function is

s2 + 2xwn s + wn2
eer
Transfer function has a pair of complex conjugate poles and zeroes.
Sol. 56 Option (A) is correct.
For ramp input we have R (s) = 12 ing
Now ess = lim sE (s)
s
.ne
t
s"0

R (s) 1
= lim s = lim
s"0 1 + G (s) s " 0 s + sG (s)
or ess = lim 1 = 5% = 1 Finite
s " 0 sG (s) 20
But kv = 1 = lim sG (s) = 20
ess s"0

kv is finite for type 1 system having ramp input.


Sol. 57 Option (A) is correct.
Sol. 58 Option (C) is correct.
Any point on real axis of s - is part of root locus if number of OL poles and zeros
to right of that point is even. Thus (B) and (C) are possible option.
The characteristics equation is
1 + G (s) H (s) = 0
K (1 - s)
or 1+ =0
s (s + 3)

Downloaded From : www.EasyEngineering.net


Downloaded From : www.EasyEngineering.net
GATE SOLVED PAPER - EC CONTROL SYSTEM

2
or K = s + 3s
1-s
For break away & break in point
dK = (1 - s)( 2s + 3) + s2 + 3s = 0
ds
or - s2 + 2s + 3 = 0
which gives s = 3 , - 1
Here - 1 must be the break away point and 3 must be the break in point.
Sol. 59 Option (D) is correct.
-2s
G (s) = 3e
s (s + 2)
-2jw
or G (jw) = 3e
jw (jw + 2)

ww G (jw) = 3
w w2 + 4

w.E Let at frequency wg the gain is 1. Thus


3
wg (wg2 + 4)
=1

or
or
asy
wg4 + 4wg2 - 9 = 0
wg2 = 1.606
or
En
wg = 1.26 rad/sec
Now
2
gin
+G (jw) =- 2w - p - tan-1 w

Let at frequency wf we have +GH =- 180c


2

w
- p =- 2wf - p - tan-1 f
2 2 eer
or
w
2wf + tan-1 f = p
2 2 ing
or
w
2
w 3
2wf + c f - 1 ` f j m = p
3 2 2 .ne
or
5wf wf3
2
-
24
5wf
2
=p
2
.p
2
t
or wf = 0.63 rad
Sol. 60 Option (D) is correct.
The gain at phase crossover frequency wf is
G (jwg) = 3 = 3
wf (w2f + 4)
1

0.63 (0.632 + 4) 2

or G (jwg) = 2.27
G.M. =- 20 log G (jwg)
- 20 log 2.26 =- 7.08 dB
Since G.M. is negative system is unstable.
The phase at gain cross over frequency is
w
+G (jwg) =- 2wg - p - tan-1 g
2 2

Downloaded From : www.EasyEngineering.net


Downloaded From : www.EasyEngineering.net
GATE SOLVED PAPER - EC CONTROL SYSTEM

=- 2 # 1.26 - p - tan-1 1.26


2 2
or =- 4.65 rad or - 266.5c
PM = 180c + +G (jwg) = 180c - 266.5c =- 86.5c
Sol. 61 Option (D) is correct.
The open loop transfer function is
2 (1 + s)
G (s) H (s) =
s2
Substituting s = jw we have
2 (1 + jw)
G (jw) H (jw) = ...(1)
- w2
+G (jw) H (jw) =- 180c + tan-1 w
The frequency at which phase becomes - 180c, is called phase crossover

ww frequency.
Thus - 180 =- 180c + tan-1 wf

w.E or
or
tan-1 wf = 0
wf = 0

asy
The gain at wf = 0 is

G (jw) H (jw) = 2 1 +
w2
w2 = 3

En
Thus gain margin is = 1
3
= 0 and in dB this is - 3 .
Sol. 62 Option (C) is correct.
gin
Centroid is the point where all asymptotes intersects.
s =
eer
SReal of Open Loop Pole - SReal Part of Open Loop Pole
SNo.of Open Loop Pole - SNo.of Open Loop zero
= - 1 - 3 =- 1.33
3
ing
Option (C) is correct.

.ne
Sol. 63

The given bode plot is shown below

t
At w = 1 change in slope is +20 dB " 1 zero at w = 1
At w = 10 change in slope is - 20 dB " 1 poles at w = 10
At w = 100 change in slope is - 20 dB " 1 poles at w = 100
K (s + 1)
Thus T (s) = s
( 10 + 1)( 100
s
+ 1)
Now 20 log10 K =- 20 " K = 0.1
0.1 (s + 1) 100 (s + 1)
Thus T (s) = s =
( 10 + 1)( 100 + 1) (s + 10)( s + 100)
s

Sol. 64 Option (C) is correct.

Downloaded From : www.EasyEngineering.net


Downloaded From : www.EasyEngineering.net
GATE SOLVED PAPER - EC CONTROL SYSTEM

We have r (t) = 10u (t)


or R (s) = 10
s
Now H (s) = 1
s+2
C (s) = H (s) $ R (s) = 1 $ 10 10
s + 2 s s (s + 2)
or C (s) = 5 - 5
s s+2
c (t) = 5 [1 - e-2t]
The steady state value of c (t) is 5. It will reach 99% of steady state value
reaches at t , where
5 [1 - e-2t] = 0.99 # 5

ww or

or
1 - e-2t = 0.99
e-2t = 0.1
- 2t = ln 0.1

w.E
Sol. 65
or
Option (A) is correct.
t = 2.3 sec

asy
Approximate (comparable to 90c) phase shift are
Due to pole at 0.01 Hz " - 90c

En
Due to pole at 80 Hz " - 90c
Due to pole at 80 Hz " 0
Due to zero at 5 Hz " 90c
Due to zero at 100 Hz " 0 gin
Due to zero at 200 Hz " 0
Thus approximate total - 90c phase shift is provided.
eer
Sol. 66 Option (C) is correct.
Mason Gain Formula
ing
T (s) =
Spk 3 k
3
.ne
In given SFG there is only one forward path and 3 possible loop.
p1 = abcd
31 = 1
t
3= 1 - (sum of indivudual loops) - (Sum of two non touching loops)
= 1 - (L1 + L2 + L3) + (L1 L3)
Non touching loop are L1 and L3 where
L1 L2 = bedg
C (s) p1 3 1
Thus =
R (s) 1 - (be + cf + dg) + bedg
= abcd
1 - (be + cf + dg) + bedg
Sol. 67 Option (A) is correct.
-2 2
A ==
1 - 3G
We have

Characteristic equation is

Downloaded From : www.EasyEngineering.net


Downloaded From : www.EasyEngineering.net
GATE SOLVED PAPER - EC CONTROL SYSTEM

[lI - A] = 0
l + 2 -2
or =0
-1 l + 3
or (l + 2)( l + 3) - 2 = 0
or l2 + 5l + 4 = 0
Thus l1 =- 4 and l2 =- 1
Eigen values are - 4 and - 1.
Eigen vectors for l1 =- 4
(l1 I - A) X1 = 0
l1 + 2 - 2 x11
or = 1 l + 3G=x G = 0
1 21
- 2 - 2 x11
=- 1 - 1G=x G = 0

ww or
21

- 2x11 - 2x21 = 0
x11 + x21 = 0

w.E or
We have only one independent equation x11 =- x21.
Let x21 = K , then x11 =- K , the Eigen vector will be

asy x11 -K -1
=x G = = K G = K = 1 G
21

En
Now Eigen vector for l2 =- 1
(l2 I - A) X2 = 0

or
l2 + 2 - 2 x12
= - 1 l + 3G=x G = 0
2 22 gin
or
1 - 2 x12
=- 1 2 G=x G = 0
22 eer
We have only one independent equation x12 = 2x22
ing
Let x22 = K , then x12 = 2K . Thus Eigen vector will be
x12 2K 2
=x G = = K G = K =1 G
22 .ne
Digonalizing matrix

M ==
x11 x12 -1 2
x21 x22 G = 1 1G
=
t
1 -2
M-1 = ` - 1 j=
1 - 1G
Now
3 -
Now Diagonal matrix of sin At is D where
sin (l1 t) 0 sin (- 4t) 0
D == G ==
0 sin (l2 t) 0 sin (l2 t)G
Now matrix B = sin At = MDM-1
- 1 2 sin (- 4t) 0 1 -2
=-` 1 j= G=
3 1 1 0 sin (- t) - 1 - 1G
G=
- sin (- 4t) - 2 sin (- t) 2 sin (- 4t) - 2 sin (- t)
=-` 1 j=
3 sin (- 4t) + 2 sin (t) - 2 sin (- 4t) - sin (- t)G
- sin (- 4t) - 2 sin (- t) 2 sin (- 4t) - 2 sin (- t)
=-` 1 j=
3 sin (- 4t) - sin (- t) - 2 sin (- 4t) + 2 sin (- t)G

Downloaded From : www.EasyEngineering.net


Downloaded From : www.EasyEngineering.net
GATE SOLVED PAPER - EC CONTROL SYSTEM

sin (- 4t) + 2 sin (- t) - 2 sin (- 4t) + 2 sin (- t)


= ` 1 j= Gs
3 - sin (- 4t + sin (- t) 2 sin (- 4t) + sin (- t)

Sol. 68 Option (A) is correct.


For ufb system the characteristic equation is
1 + G (s) = 0
1 + G (s)
1+ K =0
s (s2 + 2s + 2)( s + 3)
s 4 + 4s3 + 5s2 + 6s + K = 0
The routh table is shown below. For system to be stable,
(21 - 4K)
0 < K and 0 <
2/7
21

ww This gives

s4 1
0<K<
4

5 K

w.E s3
s 2
4
7
6
K
0

s1
0
2
21 - 4K
7/2

K
asy 0

Sol. 69
s

Option (B) is correct. En


We have
The routh table is shown below.
gin
P (s) = s5 + s 4 + 2s3 + 3s + 15

eer
If e " 0+ then 2e +e 12 is positive and -15e2-e +2412e - 144 is negative. Thus there are two
2

1 2 3 ing
sign change in first column. Hence system has 2 root on RHS of plane.

s5
s4 1 2 15
.ne
s3
s2
s1
2
e
e
2e + 12

-15e - 24e - 144


2e + 12
- 12
15
0
0 t
s0 0

Sol. 70 Option (D) is correct.


x1 - 3 - 1 x1 1
We have =x G = = 2 0 G=x G + =0 Gu
2 2
x1 1
and Y = [1 0]= G + = Gu
x2 2
-3 -1 1
A ==
2 0G
Here , B = = G and C = [1 0]
0
The controllability matrix is
QC = [B AB ]
1 -3
==
0 2G

Downloaded From : www.EasyEngineering.net


Downloaded From : www.EasyEngineering.net
GATE SOLVED PAPER - EC CONTROL SYSTEM

det QC ! 0 Thus controllable


The observability matrix is
Q0 = [CT AT CT ]
1 -3
==
0 - 1G
!0

det Q0 ! 0 Thus observable


Sol. 71 Option (B) is correct.
s 0 1 0 s-1 0
(sI - A) = = G - = G = =
0 s 0 1 0 s - 1G
(s - 1) 0 1
0
1 = G > H
s-1
(sI - A) -1 = =
(s - 1) 2 0 (s - 1) 0 1
s-1

eAt = L-1 [(sI - A)] -1

ww et 0
= = tG
0 e

w.E
Sol. 72 Option (A) is correct.
Z = P-N

asy
N " Net encirclement of (- 1 + j0) by Nyquist plot,
P " Number of open loop poles in right hand side of s - plane

En
Z " Number of closed loop poles in right hand side of s - plane
Here N = 1 and P = 1
Thus Z =0
gin
Hence there are no roots on RH of s -plane and system is always stable.
Sol. 73 Option (C) is correct.
eer
PD Controller may accentuate noise at higher frequency. It does not effect the type

Sol. 74 Option (D) is correct. ing


of system and it increases the damping. It also reduce the maximum overshoot.

Mason Gain Formula


Spk 3 k .ne
T (s) =
3
In given SFG there is only forward path and 3 possible loop.
p1 = 1
t
31 = 1 + 3 + 24 = s + 27
s s s
L1 = - 2 , L2 = - 24 and L3 = - 3
s s s
where L1 and L3 are non-touching
C (s) p1 3 1
This =
R (s) 1 - (loop gain) + pair of non - touching loops
^ s +s27 h ^
s + 27
h
= = s

1 - ^ -s3 - 24s - s2 h + -s2 . -s3 1 + 29s + s62

s (s + 27)
= 2
s + 29s + 6
Sol. 75 Option (D) is correct.
We have

Downloaded From : www.EasyEngineering.net


Downloaded From : www.EasyEngineering.net
GATE SOLVED PAPER - EC CONTROL SYSTEM

1 + G (s) H (s) = 0
or 1+ K =0
s (s + 2)( s + 3)
or K =- s (s2 + 5s2 + 6s)
dK =- (3s2 + 10s + 6) = 0
ds
which gives s = - 10 ! 100 - 72 =- 0.784, - 2.548
6
The location of poles on s - plane is

ww
Sol. 76
Since breakpoint must lie on root locus so s =- 0.748 is possible.
Option (A) is correct.

w.E The given bode plot is shown below

asy
En
gin
At w = 0.1 change in slope is + 60 dB " 3 zeroes at w = 0.1
At w = 10 change in slope is - 40 dB " 2 poles at w = 10
eer
At w = 100 change in slope is - 20 dB " 1 poles at w = 100

Thus T (s) = s
K ( 0s.1 + 1) 3
( 10 + 1) 2 ( 100
s
+ 1)
ing
Now 20 log10 K = 20
K = 10
.ne
Sol. 77
or

Thus T (s) =

Option (B) is correct.


10 ( 0s.1 + 1) 3
( 10s + 1) 2 ( 100
s
+ 1)
= t
108 (s + 0.1) 3
(s + 10) 2 (s + 100)

The characteristics equation is


s2 + 4s + 4 = 0
Comparing with
s2 + 2xwn + wn2 = 0
we get 2xwn = 4 and wn2 = 4
Thus x =1 Critically damped
ts = 4 = 4 = 2
xwn 1#2
Sol. 78 Option (B) is correct.
Sol. 79 Option (C) is correct.
We have

Downloaded From : www.EasyEngineering.net


Downloaded From : www.EasyEngineering.net
GATE SOLVED PAPER - EC CONTROL SYSTEM

xo1 1 0 x1 x1 (0) 1
=xo G = =1 1G=x2 G
and =
x2 (0)G =0 G
=
2
1 0
A ==
1 1G
s 0 1 0 s-1 0
(sI - A) = = G -= G= =
0 s 1 1 - 1 s - 1G
1 >(s - 1)
1
0 0
H > H
s-1
(sI - A) -1 = = +1
(s - 1) 2 + 1 (s - 1)
1
(s - 1) 2 s-1

et 0
L-1 [(sI - A) -1] = eAt = = t t G
te e
et 0 1 et
x (t) = eAt # [x (t0)] = = t t G= G = = t G
te e 0 te

ww
Sol. 80 Option (C) is correct.
The characteristics equation is
ks2 + s + 6 = 0

w.E or s2 + 1 s + 6 = 0
K K
Comparing with s2 + 2xwn s + wn2 = 0 we have
we get
asy2xwn = 1 and wn2 = 6
K K
or
En
2 # 0.5 # 6 Kw = 1
K
Given x = 0.5

or
K
6 = 2 & K =1
K
1
6
gin
Sol. 81 Option (B) is correct.
eer
Any point on real axis lies on the root locus if total number of poles and zeros

ing
to the right of that point is odd. Here s =- 1.5 does not lie on real axis because
there are total two poles and zeros (0 and - 1) to the right of s =- 1.5 .
Sol. 82 Option (D) is correct.

.ne
From the expression of OLTF it may be easily see that the maximum magnitude

Sol. 83
is infinite.
Option (D) is correct.
t
is 0.5 and does not become 1 at any frequency. Thus gain cross over frequency
does not exist. When gain cross over frequency does not exist, the phase margin

We have xo (t) =- 2x (t) + 2u (t) ...(i)


Taking Laplace transform we get
sX (s) =- 2X (s) + 2U (s)
or (s + 2) X (s) = 2U (s)
2U (s)
or X (s) =
(s + 2)
Now y (t) = 0.5x (t)
Y (s) = 0.5X (s)
0.5 # 2U (s)
or Y (s) =
s+2
Y (s) 1
or =
U (s) (s + 2)

Downloaded From : www.EasyEngineering.net


Downloaded From : www.EasyEngineering.net
GATE SOLVED PAPER - EC CONTROL SYSTEM

Sol. 84 Option (D) is correct.


From Mason gain formula we can write transfer function as
Y (s) K
K
= s
=
R (s) 1 - ( s + -sK ) s - 3 (3 - K)
3

For system to be stable (3 - K) < 0 i.e. K > 3


Sol. 85 Option (B) is correct.
The characteristics equation is
(s + 1)( s + 100) = 0
s2 + 101s + 100 = 0
Comparing with s2 + 2xwn + wn2 = 0 we get
2xwn = 101 and wn2 = 100
Thus x = 101 Overdamped
20

ww For overdamped system settling time can be determined by the dominant pole
of the closed loop system. In given system dominant pole consideration is at

w.E s =- 1. Thus
1 =1
T
and Ts = 4 = 4 sec
T
Sol. 86
asy
Option (B) is correct.
Routh table is shown below. Here all element in 3rd row are zero, so system is
marginal stable.
En
s5
s4
2
1
4
2
2
1 gin
s3 0 0 0
eer
s2
s1 ing
s0
.ne
Sol. 87 Option (B) is correct.
The open loop transfer function is
G (s) H (s) = 2
1
s (s + s + 1)
t
Substituting s = jw we have
G (jw) H (jw) = 1
jw (- w2 + jw + 1)
+G (jw) H (jw) =- p - tan-1 w 2
2 (1 - w )
The frequency at which phase becomes - 180c, is called phase crossover
frequency.
wf
Thus - 180 =- 90 - tan-1
1 - wf2
wf
or - 90 =- tan-1
1 - w2f
or 1 - w2f = 0

Downloaded From : www.EasyEngineering.net


Downloaded From : www.EasyEngineering.net
GATE SOLVED PAPER - EC CONTROL SYSTEM

wf = 1 rad/sec
The gain margin at this frequency wf = 1 is
GM =- 20 log10 G (jwf) H (jwf)
= 20 log10 (wf (1 - w2f) 2 + w2f =- 20 log 1 = 0
Sol. 88 Option (A) is correct.
Z = P-N
N " Net encirclement of (- 1 + j0) by Nyquist plot,
P " Number of open loop poles in right had side of s - plane
Z " Number of closed loop poles in right hand side of s - plane
Here N = 0 (1 encirclement in CW direction and other in CCW)
and P = 0
Thus Z = 0

ww
Sol. 89
Hence there are no roots on RH of s - plane.
Option (D) is correct.

w.E Take off point is moved after G2 as shown below

asy
En
Option (D) is correct.

gin
Sol. 90

If roots of characteristics equation lie on negative axis at different positions (i.e.


unequal), then system response is over damped.

eer
From the root locus diagram we see that for 0 < K < 1, the roots are on imaginary
axis and for 1 < K < 5 roots are on complex plain. For K > 5 roots are again on
imaginary axis.
Thus system is over damped for 0 # K < 1 and K > 5 . ing
Sol. 91 Option (C) is correct.
The characteristics equation is .ne
s2 + 2s + 2 = 0
Comparing with s2 + 2xwn + wn2 = 0 we get
2xwn = 2 and wn2 = 2
t
wn =
2
and x = 1
2
Since x < 1 thus system is under damped
Sol. 92 Option (C) is correct.
From SFG we have
I1 (s) = G1 Vi (s) + HI2 (s) ...(1)
I2 (s) = G2 I1 (s) ...(2)
V0 (s) = G3 I2 (s) ...(3)
Now applying KVL in given block diagram we have
Vi (s) = I1 (s) Z1 (s) + [I1 (s) - I2 (s)] Z3 (s) ...(4)

Downloaded From : www.EasyEngineering.net


Downloaded From : www.EasyEngineering.net
GATE SOLVED PAPER - EC CONTROL SYSTEM

0 = [I2 (s) - I1 (s)] Z3 (s) + I2 (s) Z2 (s) + I2 (s) Z4 (s) ...(5)


From (4) we have
or Vi (s) = I1 (s)[ Z1 (s) + Z3 (S)] - I2 (s) Z3 (S)
1 Z3 (s)
or I1 (s) = Vi + I2 ...(6)
Z1 (s) + Z3 (s) Z1 (s) + Z3 (s)
From (5) we have
I1 (s) Z3 (S) = I2 (s)[ Z2 (s) + Z3 (s) + Z4 (s)] ...(7)
I1 (s) Z3 (s)
or Is (s) =
Z3 (s) + Z2 (s) + Z4 (s)
Comparing (2) and (7) we have
Z3 (s)
G2 =
Z3 (s) + Z2 (s) + Z4 (s)
Comparing (1) and (6) we have

ww H =
Z3 (s)
Z1 (s) + Z3 (s)

w.E
Sol. 93 Option (B) is correct.
For unity negative feedback system the closed loop transfer function is
G (s) s+4

or
CLTF =
asy =
1 + G (s) s2 + 7s + 13
1 + G (s) 2
,

= s + 7s + 13
G (s) " OL Gain

En
G (s) s+4
1 = s2 + 7s + 13 - 1 = s2 + 6s + 9
or

or
G (s)
G (s) = 2 s
s+4
+ 4
s+4
gin
s + 6s + 9
For DC gain s = 0 , thus
eer
Thus G (0) = 4
9
ing
Sol. 94 Option (C) is correct.
From the Block diagram transfer function is
G (s) .ne
Where
T (s) =

G (s) =
1 + G (s) H (s)
K (s - 2)
(s + 2)
t
and H (s) = (s - 2)
The Characteristic equation is
1 + G (s) H (s) = 0
K (s - 2)
1+ (s - 2) = 0
(s + 2) 2
or (s + 2) 2 + K (s - 2) 2 = 0
or (1 + K) s2 + 4 (1 - K) s + 4K + 4 = 0
Routh Table is shown below. For System to be stable 1 + k > 0 , and 4 + 4k > 0
and 4 - 4k > 0 . This gives - 1 < K < 1
As per question for 0 # K < 1

Downloaded From : www.EasyEngineering.net


Downloaded From : www.EasyEngineering.net
GATE SOLVED PAPER - EC CONTROL SYSTEM

s2 1+k 4 + 4k

s1 4 - 4k 0

s0 4 + 4k

Sol. 95 Option (B) is correct.


It is stable at all frequencies because for resistive network feedback factor is
always less than unity. Hence overall gain decreases.
Sol. 96 Option (B) is correct.
The characteristics equation is s2 + as2 + ks + 3 = 0
The Routh Table is shown below
For system to be stable a > 0 and aK - 3 > 0
a

ww Thus a > 0 and aK > 3

w.E s3
s2 a
1 K
3
s1
s0 asy aK - 3
a

3
0

Sol. 97
En
Option (B) is correct.
Closed loop transfer function is given as
T (s) = 2 9 gin
s + 4s + 9

eer
by comparing with standard form we get natural freq.
wA2 = 9
wn = 3
ing
Damping factor
2xwn = 4
4 = 2/3
x =
2#3 .ne
xwn
4
3 # 2/3 2
=4 =2
t
For second order system the setting time for 2-percent band is given by
ts = 4 =

Sol. 98 Option (D) is correct.


Given loop transfer function is
G (s) H (s) = 2
s (s + 1)

G (jw) H (jw) = 2
jw (jw + 1)
Phase cross over frequency can be calculated as
f (w) at w = w =- 180c
p

So here f (w) =- 90c - tan-1 (w)


- 90c - tan-1 (wp) =- 180c
tan-1 (wp) = 90c
wp = 3

Downloaded From : www.EasyEngineering.net


Downloaded From : www.EasyEngineering.net
GATE SOLVED PAPER - EC CONTROL SYSTEM

Gain margin
20 log 10 = 1
G (jw) H (jw) G
at w = wp

G.M. = 20 log 10 e 1
G (jw) H (jwp) o

G (jwp) H (jwp) = 2 =0
wp w2p + 1
so G.M. = 20 log 10 b 1 l = 3
0
Sol. 99 Option (A) is correct.
0 1 0
Here A == G, B = = G and C = [1 1]
2 -3 1
The controllability matrix is

ww det QC ! 0
QC = [B AB ] = =
0 1
1 - 3G
Thus controllable

w.E The observability matrix is

Q0 = [CT AT CT ] = =
1 2
1 - 2G
asy
det Q0 ! 0
!0

Thus observable
Sol. 100 Option (D) is correct.
EnG (s) H (s) = 2 3
we have

or G (jw) H (jw) =
s (s + 1)
2 3 gin
Gain cross over frequency
jw (jw + 1)
eer
G (jw) H (jw) at w = w = 1

2 3
g

ing
or
w w2 + 1
=1

12 = w2 (w2 + 1) .ne
w4 + w2 - 12 = 0
(w2 + 4) (w2 - 3) = 0
w2 = 3 and w2 =- 4
t
which gives w1, w2 = ! 3
wg = 3
f (w) at w = w =- 90 - tan-1 (wg)
g

=- 90 - tan-1 3
=- 90 - 60 =- 150
Phase margin = 180 + f (w) at w = w
g

= 180 - 150 = 30c


Sol. 101 Option (B) is correct.
Sol. 102 Option (C) is correct.
Closed-loop transfer function is given by
T (s) = n an - 1 s + an
s + a1 sn - 1 + ... + an - 1 s + an

Downloaded From : www.EasyEngineering.net


Downloaded From : www.EasyEngineering.net
GATE SOLVED PAPER - EC CONTROL SYSTEM

an - 1 s + an
n n-1
= s + a 1s + ...an - 2 s2
a n - 1 s + an
1+ n
s + a1 sn - 1 + ...an - 2 s2

Thus G (s) H (s) = an - 1 s + an


sn + a1 sn - 1 + ....an - 2 s2
For unity feed back H (s) = 1
Thus G (s) = an - 1 s + an
sn + a1 sn - 1 + ....an - 2 s2
Steady state error is given by
E (s) = lim R (s) 1
s"0 1 + G (s) H (s)
for unity feed back H (s) = 1

ww Here input

so
R (s) = 12 (unit Ramp)

E (s) = lim 12
s
1

w.E s " 0 s 1 + G (s)


n
= lim 12 s +
s"0 s
a1 sn - 1 + .... + an - 2 s2
s + a1 sn - 1 + . + a n
n

asy = an - 2
an
Sol. 103

Sol. 104 En
Option (B) is correct.
Option (A) is correct.
Sol. 105 Option (A) is correct.
Applying Routh’s criteria gin
s3 + 5s2 + 7s + 3 = 0
eer
s
s2
3 1
5
7
3 ing
s 1 7#5-3
5 = 32
5 0
.ne
s0 3

t
There is no sign change in the first column. Thus there is no root lying in the
left-half plane.
Sol. 106 Option (A) is correct.
Techometer acts like a differentiator so its transfer function is of the form ks .
Sol. 107 Option (A) is correct.
Open loop transfer function is
G (s) = K
s (s + 1)
Steady state error
sR (s)
E (s) = lim
s " 0 1 + G (s) H (s)

Where R (s) = input H (s) = 1 (unity feedback)


R (s) = 1
s

Downloaded From : www.EasyEngineering.net


Downloaded From : www.EasyEngineering.net
GATE SOLVED PAPER - EC CONTROL SYSTEM

s1 s (s + 1)
so E (s) = lim s = lim 2 =0
s"0
1+ K s"0 s + s + K
s (s + 1)
Sol. 108 Option (B) is correct.
Fig given below shows a unit impulse input given to a zero-order hold circuit
which holds the input signal for a duration T & therefore, the output is a unit
step function till duration T .

ww h (t) = u (t) - u (t - T)
Taking Laplace transform we have

w.E H (s) = 1 - 1 e-sT = 1 61 - e-sT @


s s s
Sol. 109

asy
Option (C) is correct.
Phase margin = 180c + qg where qg = value of phase at gain crossover
frequency.
Here En
qg =- 125c

Sol. 110
so P.M = 180c - 125c = 55c
Option (B) is correct. gin
Open loop transfer function is given by
G (s) H (s) =
K (1 + 0.5s) eer
s (1 + s) (1 + 2s)
Close looped system is of type 1. ing
in OLTF. .ne
It must be noted that type of the system is defined as no. of poles lies at origin
lying

Sol. 111 Option (D) is correct.


Transfer function of the phase lead controller is
T.F = 1 + 3Ts =
1 + (3Tw) j
t
1+s 1 + (Tw) j
Phase is
f (w) = tan-1 (3Tw) - tan-1 (Tw)
f (w) = tan-1 ; 3Tw - T w
2 2E
1 + 3T w
f (w) = tan ; 2Tw2 2 E
-1
1 + 3T w
For maximum value of phase
df (w)
=0
dw
or 1 = 3T 2 w2
Tw = 1
3

Downloaded From : www.EasyEngineering.net


Downloaded From : www.EasyEngineering.net
GATE SOLVED PAPER - EC CONTROL SYSTEM

So maximum phase is
fmax = tan-1 ; 2Tw2 2 E at Tw = 1
1 + 3T w 3
1
2 3 1 = 30c
= tan-1 > H = tan ;
3E
-1
1 + 3 # 13
Sol. 112 Option (A) is correct.
G (jw) H (jw) enclose the (- 1, 0) point so here G (jwp) H (jwp) > 1
wp = Phase cross over frequency
Gain Margin = 20 log 10 1
G (jwp) H (jwp)
so gain margin will be less than zero.
Sol. 113 Option (B) is correct.
The denominator of Transfer function is called the characteristic equation of the

ww system. so here characteristic equation is


(s + 1) 2 (s + 2) = 0

w.E
Sol. 114 Option (C) is correct.
In synchro error detector, output voltage is proportional to [w (t)], where w (t) is
the rotor velocity so here n = 1
Sol. 115
asy
Option (C) is correct.
By masson’s gain formulae
y
x
=En/ Dk Pk
D
Forward path gain
gin
P1 = 5 # 2 # 1 = 10
D = 1 - (2 # - 2) = 1 + 4 = 5
D1 = 1
eer
so gain
y
x
Option (C) is correct.
= 10 # 1 = 2
5
ing
.ne
Sol. 116

By given matrix equations we can have


Xo1 = dx1 = x1 - x2 + 0
dt
Xo2 = dx2 = 0 + x2 + m
dt
t
x1
y = [1 1] > H = x1 + x2
x2
dy
= dx1 + dx2
dt dt dt
dy
= x1 + m
dt
dy
= x1 (0) + m (0)
dt t = 0
= 1+0 = 0

***********

Downloaded From : www.EasyEngineering.net


Downloaded From : www.EasyEngineering.net

No part of this publication may be reproduced or distributed in any form or any means, electronic, mechanical,
photocopying, or otherwise without the prior permission of the author.

ww
w.E
GATE SOLVED PAPER
Electronics & Communication
Digital Circuits
asy
Copyright © By NODIA & COMPANY
En
gin
eer
ing
Information contained in this book has been obtained by authors, from sources believes to be reliable. However,
neither Nodia nor its authors guarantee the accuracy or completeness of any information herein, and Nodia nor its

.ne
authors shall be responsible for any error, omissions, or damages arising out of use of this information. This book
is published with the understanding that Nodia and its authors are supplying information but are not attempting
to render engineering or other professional services.

t
NODIA AND COMPANY
B-8, Dhanshree Tower Ist, Central Spine, Vidyadhar Nagar, Jaipur 302039
Ph : +91 - 141 - 2101150
www.nodia.co.in
email : enquiry@nodia.co.in

Downloaded From : www.EasyEngineering.net


Downloaded From : www.EasyEngineering.net

GATE SOLVED PAPER - EC


DIGITAL CIRCUITS

2013 ONE MARK

Q. 1 A bulb in a staircase has two switches, one switch being at the ground floor
and the other one at the first floor. The bulb can be turned ON and also can
be turned OFF by any one of the switches irrespective of the state of the other
switch. The logic of switching of the bulb resembles
(A) and AND gate (B) an OR gate

ww
Q. 2
(C) an XOR gate (D) a NAND gate

For 8085 microprocessor, the following program is executed.

w.E MVI A, 05H;


MVI B, 05H;
PTR: ADD B;

asy
DCR B;
JNZ PTR;
ADI 03H;
HLT;
En
At the end of program, accumulator contains
(A) 17H
(C) 23H gin (B) 20H
(D) 05H

2013 eer TWO MARKS

Q. 3
ing
There are four chips each of 1024 bytes connected to a 16 bit address bus as shown

.ne
in the figure below, RAMs 1, 2, 3 and 4 respectively are mappped to addresses

Downloaded From : www.EasyEngineering.net


Downloaded From : www.EasyEngineering.net
GATE SOLVED PAPER - EC DIGITAL CIRCUITS

(A) 0C00H-0FFFH, 1C00H-1FFFH, 2C00H-2FFFH, 3C00H-3FFFH


(B) 1800H-1FFFH, 2800H-2FFFH, 3800H-3FFFH, 4800H-4FFFH
(C) 0500H-08FFH, 1500H-18FFH, 3500H-38FFH, 5500H-58FFH
(D) 0800H-0BFFH, 1800H-1BFFH, 2800H-2BFFH, 3800H-3BFFH

2012 ONE MARK

Q. 4 Consider the given circuit

ww In this circuit, the race around

w.E (A) does not occur


(B) occur when CLK = 0

asy
(C) occur when CLK = 1 and A = B = 1
(D) occur when CLK = 1 and A = B = 0

Q. 5

En
The output Y of a 2-bit comparator is logic 1 whenever the 2-bit input A is
greater than the 2-bit input B . The number of combinations for which the output
is logic 1, is
(A) 4 gin(B) 6

Q. 6
(C) 8

In the circuit shown


(D) 10
eer
ing
.ne
t
(A) Y = A B + C (B) Y = (A + B) C
(C) Y = (A + B ) C (D) Y = AB + C

Q. 7 In the sum of products function f (X, Y, Z) = / (2, 3, 4, 5), the prime implicants
are
(A) XY, XY (B) XY, X Y Z , XY Z
(C) XY Z , XYZ, XY (D) XY Z , XYZ, XY Z , XY Z

Downloaded From : www.EasyEngineering.net


Downloaded From : www.EasyEngineering.net
GATE SOLVED PAPER - EC DIGITAL CIRCUITS

2012 TWO MARKS

Q. 8 The state transition diagram for the logic circuit shown is

ww
w.E
asy
Q. 9
2011

En
The output Y in the circuit below is always ‘1’ when
ONE MARK

gin
eer
ing
(A) two or more of the inputs P, Q, R are ‘0’ .ne
(B) two or more of the inputs P, Q, R are ‘1’
(C) any odd number of the inputs P, Q, R is ‘0’
(D) any odd number of the inputs P, Q, R is ‘1’
t
Q. 10 When the output Y in the circuit below is “1”, it implies that data has

(A) changed from “0” to “1”


(B) changed from “1” to “0”
(C) changed in either direction
(D) not changed

Downloaded From : www.EasyEngineering.net


Downloaded From : www.EasyEngineering.net
GATE SOLVED PAPER - EC DIGITAL CIRCUITS

Q. 11 The logic function implemented by the circuit below is (ground implies a logic
“0”)

(A) F = AND ^P, Q h (B) F = OR ^P, Q h


(C) F = XNOR ^P, Q h (D) F = XOR ^P, Q h

ww 2011 TWO MARKS

w.E
Q. 12 The output of a 3-stage Johnson (twisted ring) counter is fed to a digital-to
analog (D/A) converter as shown in the figure below. Assume all states of the

asy
counter to be unset initially. The waveform which represents the D/A converter
output Vo is

En
gin
eer
ing
.ne
t

Q. 13 Two D flip-flops are connected as a synchronous counter that goes through the
following QB QA sequence 00 " 11 " 01 " 10 " 00 " ....
The connections to the inputs DA and DB are
(A) DA = QB, DB = QA
(B) DA = Q A, DB = Q B
(C) DA = (QA Q B + Q A QB), DB = QA
(D) DA = (QA QB + Q A Q B), DB = Q B

Downloaded From : www.EasyEngineering.net


Downloaded From : www.EasyEngineering.net
GATE SOLVED PAPER - EC DIGITAL CIRCUITS

Q. 14 An 8085 assembly language program is given below. Assume that the carry flag is
initially unset. The content of the accumulator after the execution of the program
is

(A) 8CH (B) 64H


(C) 23H (D) 15H

ww 2010 ONE MARK

w.E
Q. 15 Match the logic gates in Column A with their equivalents in Column B

asy
En
gin
eer
ing
(A) P-2, Q-4, R-1, S-3
(C) P-2, Q-4, R-3, S-1 .ne
(B) P-4, Q-2, R-1, S-3
(D) P-4, Q-2, R-3, S-1

Q. 16
t
In the circuit shown, the device connected Y5 can have address in the range

(A) 2000 - 20FF (B) 2D00 - 2DFF


(C) 2E00 - 2EFF (D) FD00 - FDFF

Downloaded From : www.EasyEngineering.net


Downloaded From : www.EasyEngineering.net
GATE SOLVED PAPER - EC DIGITAL CIRCUITS

Q. 17 For the output F to be 1 in the logic circuit shown, the input combination should
be

(A) A = 1, B = 1, C = 0 (B) A = 1, B = 0, C = 0
(C) A = 0, B = 1, C = 0 (D) A = 0, B = 0, C = 1

2010 TWO MARKS

Q. 18 Assuming that the flip-flop are in reset condition initially, the count sequence

ww observed at QA , in the circuit shown is

w.E
asy
En
(A) 0010111...
(C) 0101111... gin (B) 0001011...
(D) 0110100....

Q. 19
eer
The Boolean function realized by the logic circuit shown is

ing
.ne
(A) F = Sm (0, 1, 3, 5, 9, 10, 14)
t
(B) F = Sm (2, 3, 5, 7, 8, 12, 13)
(C) F = Sm (1, 2, 4, 5, 11, 14, 15) (D) F = Sm (2, 3, 5, 7, 8, 9, 12)

Q. 20 For the 8085 assembly language program given below, the content of the
accumulator after the execution of the program is

(A) 00H (B) 45H


(C) 67H (D) E7H

Downloaded From : www.EasyEngineering.net


Downloaded From : www.EasyEngineering.net
GATE SOLVED PAPER - EC DIGITAL CIRCUITS

2009 ONE MARK

Q. 21 The full form of the abbreviations TTL and CMOS in reference to logic families
are
(A) Triple Transistor Logic and Chip Metal Oxide Semiconductor
(B) Tristate Transistor Logic and Chip Metal Oxide Semiconductor
(C) Transistor Transistor Logic and Complementary Metal Oxide
Semiconductor
(D) Tristate Transistor Logic and Complementary Metal Oxide Silicon

Q. 22 In a microprocessor, the service routine for a certain interrupt starts from a fixed
location of memory which cannot be externally set, but the interrupt can be
delayed or rejected Such an interrupt is
(A) non-maskable and non-vectored

ww (B) maskable and non-vectored


(C) non-maskable and vectored

w.E (D) maskable and vectored

2009 TWO MARKS

Q. 23
(A) Y = Z
asy
If X = 1 in logic equation 6X + Z {Y + (Z + XY )}@{X + X (X + Y)} = 1, then
(B) Y = Z
(C) Z = 1
En (D) Z = 0

Q. 24

gin
What are the minimum number of 2- to -1 multiplexers required to generate a
2- input AND gate and a 2- input Ex-OR gate
(A) 1 and 2
(C) 1 and 1 eer
(B) 1 and 3
(D) 2 and 2

Q. 25
ing
What are the counting states (Q1, Q2) for the counter shown in the figure below

.ne
t
(A) 11, 10, 00, 11, 10,... (B) 01, 10, 11, 00, 01...
(C) 00, 11, 01, 10, 00... (D) 01, 10, 00, 01, 10...

Statement for Linked Answer Question 26 & 27 :


Two products are sold from a vending machine, which has two push buttons P1
and P2 .
When a buttons is pressed, the price of the corresponding product is displayed in
a 7 - segment display. If no buttons are pressed, '0' is displayed signifying ‘Rs 0’.
If only P1 is pressed, ‘2’ is displayed, signifying ‘Rs. 2’
If only P2 is pressed ‘5’ is displayed, signifying ‘Rs. 5’
If both P1 and P2 are pressed, 'E' is displayed, signifying ‘Error’

Downloaded From : www.EasyEngineering.net


Downloaded From : www.EasyEngineering.net
GATE SOLVED PAPER - EC DIGITAL CIRCUITS

The names of the segments in the 7 - segment display, and the glow of the display
for ‘0’, ‘2’, ‘5’ and ‘E’ are shown below.

Consider
(1) push buttons pressed/not pressed in equivalent to logic 1/0 respectively.
(2) a segment glowing/not glowing in the display is equivalent to logic 1/0
respectively.

Q. 26 If segments a to g are considered as functions of P1 and P2 , then which of the

ww following is correct
(A) g = P 1 + P2, d = c + e
(C) g = P1 + P2, e = b + c
(B) g = P1 + P2, d = c + e
(D) g = P1 + P2, e = b + c

w.E
Q. 27 What are the minimum numbers of NOT gates and 2 - input OR gates required
to design the logic of the driver for this 7 - Segment display

asy
(A) 3 NOT and 4 OR
(C) 1 NOT and 3 OR
(B) 2 NOT and 4 OR
(D) 2 NOT and 3 OR

Q. 28
En
Refer to the NAND and NOR latches shown in the figure. The inputs (P1, P2) for
both latches are first made (0, 1) and then, after a few seconds, made (1, 1). The

gin
corresponding stable outputs (Q1, Q2) are

eer
ing
.ne
(A) NAND: first (0, 1) then (0, 1) NOR: first (1, 0) then (0, 0)

t
(B) NAND : first (1, 0) then (1, 0) NOR : first (1, 0) then (1, 0)
(C) NAND : first (1, 0) then (1, 0) NOR : first (1, 0) then (0, 0)
(D) NAND : first (1, 0) then (1, 1) NOR : first (0, 1) then (0, 1)

2008 TWO MARKS

Q. 29 The logic function implemented by the following circuit at the terminal OUT is

(A) P NOR Q (B) P NAND Q


(C) P OR Q (D) P AND Q

Downloaded From : www.EasyEngineering.net


Downloaded From : www.EasyEngineering.net
GATE SOLVED PAPER - EC DIGITAL CIRCUITS

Q. 30 The two numbers represented in signed 2’s complement form are P + 11101101
and Q = 11100110 . If Q is subtracted from P , the value obtained in signed 2’s
complement is
(A) 1000001111 (B) 00000111
(C) 11111001 (D) 111111001

Q. 31 Which of the following Boolean Expressions correctly represents the relation


between P, Q, R and M1

ww (A) M1 = (P OR Q) XOR R

w.E (B) M1 = (P AND Q) X OR R


(C) M1 = (P NOR Q) X OR R
(D) M1 = (P XOR Q) XOR R

Q. 32 asy
For the circuit shown in the figure, D has a transition from 0 to 1 after CLK

En
changes from 1 to 0. Assume gate delays to be negligible
Which of the following statements is true

gin
eer
ing
(A) Q goes to 1 at the CLK transition and stays at 1 .ne
(B) Q goes to 0 at the CLK transition and stays 0
t
(C) Q goes to 1 at the CLK tradition and goes to 0 when D goes to 1
(D) Q goes to 0 at the CLK transition and goes to 1 when D goes to 1

Q. 33 For each of the positive edge-triggered J - K flip flop used in the following
figure, the propagation delay is 3 t .

Which of the following wave forms correctly represents the output at Q1 ?

Downloaded From : www.EasyEngineering.net


Downloaded From : www.EasyEngineering.net
GATE SOLVED PAPER - EC DIGITAL CIRCUITS

ww Statement For Linked Answer Question 34 & 35 :


In the following circuit, the comparators output is logic “1” if V1 > V2 and is logic

w.E
3
"0" otherwise. The D/A conversion is done as per the relation VDAC = / 2n - 1 bn
Volts, where b3 (MSB), b1, b2 and b0 (LSB) are the counter outputs. Then =counter
0

starts from the clear state.

asy
En
gin
eer
ing
The stable reading of the LED displays is .ne
t
Q. 34
(A) 06 (B) 07
(C) 12 (D) 13

Q. 35 The magnitude of the error between VDAC and Vin at steady state in volts is
(A) 0.2 (B) 0.3
(C) 0.5 (D) 1.0

Q. 36 For the circuit shown in the following, I0 - I3 are inputs to the 4:1 multiplexers,
R(MSB) and S are control bits. The output Z can be represented by

Downloaded From : www.EasyEngineering.net


Downloaded From : www.EasyEngineering.net
GATE SOLVED PAPER - EC DIGITAL CIRCUITS

(A) PQ + PQS + QRS


(B) PQ + PQR + PQS
(C) PQR + PQR + PARS + QRS
(D) PQR + PQRS + PQRS + QRS

Q. 37 An 8085 executes the following instructions


2710 LXI H, 30A0 H
2713 DAD H
2414 PCHL
All address and constants are in Hex. Let PC be the contents of the program
counter and HL be the contents of the HL register pair just after executing
PCHL. Which of the following statements is correct ?
PC = 2715H PC = 30A0H
(A) (B)
HL = 30A0H HL = 2715H

ww (C)
PC = 6140H
HL = 6140H
(D)
PC = 6140H
HL = 2715H

w.E 2007 ONE MARK

Q. 38
asy
X = 01110 and Y = 11001 are two 5-bit binary numbers represented in two’s
complement format. The sum of X and Y represented in two’s complement

(A) 100111 En
format using 6 bits is
(B) 0010000
(C) 000111
gin(D) 101001

Q. 39
gates. The minimum number of gates required is
(A) 2 (B) 3 eer
The Boolean function Y = AB + CD is to be realized using only 2 - input NAND

(C) 4 (D) 5
ing
2007
.ne TWO MARKS

Q. 40 In the following circuit, X is given by


t

(A) X = ABC + ABC + ABC + ABC (B) X = ABC + ABC + ABC + ABC
(C) X = AB + BC + AC (D) X = AB + BC + AC

Q. 41 The Boolean expression Y = ABC D + ABCD + ABC D + ABC D can be


minimized to
(A) Y = ABC D + ABC + AC D (B) Y = ABC D + BCD + ABC D
(C) Y = ABCD + BC D + ABC D (D) Y = ABCD + BC D + ABC D

Downloaded From : www.EasyEngineering.net


Downloaded From : www.EasyEngineering.net
GATE SOLVED PAPER - EC DIGITAL CIRCUITS

Q. 42 The circuit diagram of a standard TTL NOT gate is shown in the figure. Vi = 25
V, the modes of operation of the transistors will be

ww (A) Q1: revere active; Q2: normal active; Q3: saturation; Q4: cut-off
(B) Q1: revere active; Q2: saturation; Q3: saturation; Q4: cut-off

w.E (C) Q1: normal active; Q2: cut-off; Q3: cut-off; Q4: saturation
(D) Q1: saturation; Q2: saturation; Q3: saturation; Q4: normal active
Q. 43

asy
The following binary values were applied to the X and Y inputs of NAND latch
shown in the figure in the sequence indicated below :

En
X = 0,Y = 1; X = 0, Y = 0; X = 1; Y = 1
The corresponding stable P, Q output will be.

gin
eer
(A) P = 1, Q = 0; ing
P = 1, Q = 0; P = 1, Q = 0 or P = 0, Q = 1
(B) P = 1, Q = 0;
.ne
P = 0, Q = 1; or P = 0, Q = 1; P = 0, Q = 1

Q. 44
(C) P = 1, Q = 0;
(D) P = 1, Q = 0;
P = 1, Q = 1; P = 1, Q = 0 or P = 0, Q = 1
P = 1, Q = 1; P = 1, Q = 1
t
An 8255 chip is interfaced to an 8085 microprocessor system as an I/O mapped
I/O as show in the figure. The address lines A0 and A1 of the 8085 are used by
the 8255 chip to decode internally its thee ports and the Control register. The
address lines A3 to A7 as well as the IO/M signal are used for address decoding.
The range of addresses for which the 8255 chip would get selected is

(A) F8H - FBH (B) F8GH - FCH


(C) F8H - FFH (D) F0H - F7H

Downloaded From : www.EasyEngineering.net


Downloaded From : www.EasyEngineering.net
GATE SOLVED PAPER - EC DIGITAL CIRCUITS

Statement for Linked Answer Question 45 and 46 :


In the Digital-to-Analog converter circuit shown in the figure below,
VR = 10V and R = 10kW

Q. 45 The current is

ww (A) 31.25mA
(C) 125mA
(B) 62.5mA
(D) 250mA

w.E
Q. 46 The voltage V0 is
(A) - 0.781 V (B) - 1.562 V

asy
(C) - 3.125 V (D) - 6.250 V

En
Statement for Linked Answer Questions 47 & 48 :
An 8085 assembly language program is given below.
Line 1: MVI A, B5H
2: MVI B, OEH gin
3:
4:
XRI 69H
ADD B eer
5:
6:
ANI 9BH
CPI 9FH
ing
7:
8:
STA 3010H
HLT
.ne
Q. 47
will be
(A) C3H
(C) DCH
(B) EAH
(D) 69H
t
The contents of the accumulator just execution of the ADD instruction in line 4

Q. 48 After execution of line 7 of the program, the status of the CY and Z flags will be
(A) CY = 0, Z = 0 (B) CY = 0, Z = 1
(C) CY = 1, Z = 0 (D) CY = 1, Z = 1

Q. 49 For the circuit shown, the counter state (Q1 Q0) follows the sequence

Downloaded From : www.EasyEngineering.net


Downloaded From : www.EasyEngineering.net
GATE SOLVED PAPER - EC DIGITAL CIRCUITS

(A) 00, 01, 10, 11, 00 (B) 00, 01, 10, 00, 01
(C) 00, 01, 11, 00, 01 (D) 00, 10, 11, 00, 10

2006 ONE MARK

Q. 50 The number of product terms in the minimized sum-of-product expression


obtained through the following K - map is (where, "d" denotes don’t care states)

ww (A) 2
(C) 4
(B) 3
(D) 5

w.E
Q. 51
2006 TWO MARKS

An I/O peripheral device shown in Fig. (b) below is to be interfaced to an 8085

asy
microprocessor. To select the I/O device in the I/O address range D4 H - D7 H,
its chip-select (CS ) should be connected to the output of the decoder shown in
as below :
En
gin
eer
ing
(A) output 7 (B) output 5 .ne
Q. 52
(C) output 2 (D) output 0
t
For the circuit shown in figures below, two 4 - bit parallel - in serial - out shift
registers loaded with the data shown are used to feed the data to a full adder.
Initially, all the flip - flops are in clear state. After applying two clock pulse, the
output of the full-adder should be

(A) S = 0, C0 = 0 (B) S = 0, C0 = 1
(C) S = 1, C0 = 0 (D) S = 1, C0 = 1

Downloaded From : www.EasyEngineering.net


Downloaded From : www.EasyEngineering.net
GATE SOLVED PAPER - EC DIGITAL CIRCUITS

Q. 53 A new Binary Coded Pentary (BCP) number system is proposed in which every
digit of a base-5 number is represented by its corresponding 3-bit binary code. For
example, the base-5 number 24 will be represented by its BCP code 010100. In
this numbering system, the BCP code 10001001101 corresponds of the following
number is base-5 system
(A) 423 (B) 1324
(C) 2201 (D) 4231

Q. 54 A 4 - bit DAC is connected to a free - running 3 - big UP counter, as shown in


the following figure. Which of the following waveforms will be observed at V0 ?

ww
w.E In the figure shown above, the ground has been shown by the symbol 4

asy
En
gin
Q. 55 eer
Following is the segment of a 8085 assembly language program
LXI SP, EFFF H
CALL 3000 H ing
:
:
.ne
:
3000 H LXI H, 3CF4
PUSH PSW
SPHL
t
POP PSW
RET
On completion of RET execution, the contents of SP is
(A) 3CF0 H (B) 3CF8 H
(C) EFFD H (D) EFFF H

Q. 56 Two D - flip - flops, as shown below, are to be connected as a synchronous


counter that goes through the sequence 00 " 01 " 11 " 10 " 00 " ...
The inputs D0 and D1 respectively should be connected as,

Downloaded From : www.EasyEngineering.net


Downloaded From : www.EasyEngineering.net
GATE SOLVED PAPER - EC DIGITAL CIRCUITS

(A) Q 1 and Q0 (B) Q 0 and Q1


(C) Q1 Q0 and Q 1 Q0 (D) Q 1 Q 0 and Q1 Q0

Q. 57 The point P in the following figure is stuck at 1. The output f will be

(A) ABC (B) A


(C) ABC (D) A

2005 ONE MARK

ww
Q. 58 Decimal 43 in Hexadecimal and BCD number system is respectively
(A) B2, 0100 011 (B) 2B, 0100 0011

w.E
Q. 59
(C) 2B, 0011 0100 (D) B2, 0100 0100

The Boolean function f implemented in the figure using two input multiplexes is

asy
En
gin
eer
(A) ABC + ABC
(C) ABC + ABC
(B) ABC + ABC
(D) ABC + ABCing
2005
.ne TWO MARKS

Q. 60
t
The transistors used in a portion of the TTL gate show in the figure have b = 100
. The base emitter voltage of is 0.7 V for a transistor in active region and 0.75
V for a transistor in saturation. If the sink current I = 1 A and the output is at
logic 0, then the current IR will be equal to

(A) 0.65 mA (B) 0.70 mA


(C) 0.75 mA (D) 1.00 mA

Downloaded From : www.EasyEngineering.net


Downloaded From : www.EasyEngineering.net
GATE SOLVED PAPER - EC DIGITAL CIRCUITS

Q. 61 The Boolean expression for the truth table shown is

(A) B (A + C)( A + C ) (B) B (A + C )( A + C)


(C) B (A + C )( A + C) (D) B (A + C)( A + C )

ww
Q. 62 The present output Qn of an edge triggered JK flip-flop is logic 0. If J = 1, then
Qn + 1
(A) Cannot be determined (B) Will be logic 0

w.E
Q. 63
(C) will be logic 1 (D) will rave around

The given figure shows a ripple counter using positive edge triggered flip-flops. If

asy
the present state of the counter is Q2 Q1 Q0 = 001 then is next state Q2 Q1 Q will be

En
gin
(A) 010
eer
(B) 111

Q. 64
(C) 100 (D) 101

ing
What memory address range is NOT represents by chip # 1 and chip # 2 in the

.ne
figure A0 to A15 in this figure are the address lines and CS means chip select.

(A) 0100 - 02FF (B) 1500 - 16FF


(C) F900 - FAFF (D) F800 - F9FF

Downloaded From : www.EasyEngineering.net


Downloaded From : www.EasyEngineering.net
GATE SOLVED PAPER - EC DIGITAL CIRCUITS

Statement For Linked Answer Questions 65 & 66 :


Consider an 8085 microprocessor system.
Q. 65 The following program starts at location 0100H.
LXI SP, OOFF
LXI H, 0701
MVI A, 20H
SUB M
The content of accumulator when the program counter reaches 0109 H is
(A) 20 H (B) 02 H
(C) 00 H (D) FF H

Q. 66 If in addition following code exists from 019H onwards,


ORI 40 H

ww ADD M
What will be the result in the accumulator after the last instruction is executed ?
(A) 40 H (B) 20 H

w.E (C) 60 H (D) 42 H

Q. 67
2004
asy
A master - slave flip flop has the characteristic that
ONE MARK

En
(A) change in the output immediately reflected in the output
(B) change in the output occurs when the state of the master is affected

gin
(C) change in the output occurs when the state of the slave is affected
(D) both the master and the slave states are affected at the same time

Q. 68 eer
The range of signed decimal numbers that can be represented by 6-bits 1’s
complement number is
(A) -31 to +31 (B) -63 to +63 ing
Q. 69
(C) -64 to +63 (D) -32 to +31
.ne
A digital system is required to amplify a binary-encoded audio signal. The user

t
should be able to control the gain of the amplifier from minimum to a maximum
in 100 increments. The minimum number of bits required to encode, in straight
binary, is
(A) 8 (B) 6
(C) 5 (D) 7

Q. 70 Choose the correct one from among the alternatives A, B, C, D after matching an
item from Group 1 most appropriate item in Group 2.
Group 1 Group 2
P. Shift register 1. Frequency division
Q. Counter 2. Addressing in memory chips
R. Decoder 3. Serial to parallel data conversion
(A) P - 3, Q - 2, R - 1 (B) P - 3, Q - 1, R - 2
(C) P - 2, Q - 1, R - 3 (D) P - 1, Q - 2, R - 2

Q. 71 The figure the internal schematic of a TTL AND-OR-OR-Invert (AOI) gate. For
the inputs shown in the figure, the output Y is

Downloaded From : www.EasyEngineering.net


Downloaded From : www.EasyEngineering.net
GATE SOLVED PAPER - EC DIGITAL CIRCUITS

(A) 0 (B) 1
(C) AB (D) AB

2004 TWO MARKS

Q. 72 11001, 1001, 111001 correspond to the 2’s complement representation of which


one of the following sets of number
(A) 25,9, and 57 respectively (B) -6, -6, and -6 respectively
(C) -7, -7 and -7 respectively (D) -25, -9 and -57 respectively

ww
Q. 73 In the modulo-6 ripple counter shown in figure, the output of the 2- input gate is
used to clear the J-K flip-flop
The 2-input gate is

w.E
asy
En
(A) a NAND gate gin (B) a NOR gate
(C) an OR gate
eer
(D) a AND gare

Q. 74
multiplexers is
(A) 1 (B) 2
ing
The minimum number of 2- to -1 multiplexers required to realize a 4- to -1

(C) 3 (D) 4
.ne
Q. 75 The Boolean expression AC + BC is equivalent to
(A) AC + BC + AC
(C) AC + BC + BC + ABC
(B) BC + AC + BC + ACBt
(D) ABC + ABC + ABC + ABC

Q. 76 A Boolean function f of two variables x and y is defined as follows :


f (0, 0) = f (0, 1) = f (1, 1) = 1; f (1, 0) = 0
Assuming complements of x and y are not available, a minimum cost solution
for realizing f using only 2-input NOR gates and 2- input OR gates (each
having unit cost) would have a total cost of
(A) 1 unit (B) 4 unit
(C) 3 unit (D) 2 unit

Q. 77 The 8255 Programmable Peripheral Interface is used as described below.


(i) An A/D converter is interface to a microprocessor through an 8255.
The conversion is initiated by a signal from the 8255 on Port C. A signal on Port
C causes data to be stobed into Port A.

Downloaded From : www.EasyEngineering.net


Downloaded From : www.EasyEngineering.net
GATE SOLVED PAPER - EC DIGITAL CIRCUITS

(ii) Two computers exchange data using a pair of 8255s. Port A works as a
bidirectional data port supported by appropriate handshaking signals.
The appropriate modes of operation of the 8255 for (i) and (ii) would be
(A) Mode 0 for (i) and Mode 1 for (ii)
(B) Mode 1 for (i) and Mode 2 for (ii)
(C) Mode for (i) and Mode 0 for (ii)
(D) Mode 2 for (i) and Mode 1 for (ii)

Q. 78 The number of memory cycles required to execute the following 8085 instructions
(i) LDA 3000 H
(ii) LXI D, FOF1H
would be
(A) 2 for (i) and 2 for (ii) (B) 4 for (i) and 3 for (ii)

ww
Q. 79
(C) 3 for (i) and 3 for (ii) (D) 3 for (i) and 4 for (ii)

Consider the sequence of 8085 instructions given below

w.E LXI H, 9258


MOV A, M
CMA
MOV M, A
asy
Which one of the following is performed by this sequence ?
(A) Contents of location 9258 are moved to the accumulator

En
(B) Contents of location 9258 are compared with the contents of the
accumulator

gin
(C) Contents of location 8529 are complemented and stored in location 8529

Q. 80 eer
(D) Contents of location 5892 are complemented and stored in location 5892

It is desired to multiply the numbers 0AH by 0BH and store the result in the

ing
accumulator. The numbers are available in registers B and C respectively. A part
of the 8085 program for this purpose is given below :

LOOP
MVI A, 00H
------
.ne
------
-----
HLT
END
t
The sequence of instructions to complete the program would be
(A) JNX LOOP, ADD B, DCR C
(B) ADD B, JNZ LOOP, DCR C
(C) DCR C, JNZ LOOP, ADD B
(D) ADD B, DCR C, JNZ LOOP

2003 ONE MARK

Q. 81 The number of distinct Boolean expressions of 4 variables is


(A) 16 (B) 256
(C) 1023 (D) 65536

Downloaded From : www.EasyEngineering.net


Downloaded From : www.EasyEngineering.net
GATE SOLVED PAPER - EC DIGITAL CIRCUITS

Q. 82 The minimum number of comparators required to build an 8-bits flash ADC is


(A) 8 (B) 63
(C) 255 (D) 256

Q. 83 The output of the 74 series of GATE of TTL gates is taken from a BJT in
(A) totem pole and common collector configuration
(B) either totem pole or open collector configuration
(C) common base configuration
(D) common collector configuration

Q. 84 Without any additional circuitry, an 8:1 MUX can be used to obtain


(A) some but not all Boolean functions of 3 variables
(B) all functions of 3 variables but non of 4 variables

ww (C) all functions of 3 variables and some but not all of 4 variables
(D) all functions of 4 variables

w.E
Q. 85 A 0 to 6 counter consists of 3 flip flops and a combination circuit of 2 input gate
(s). The common circuit consists of
(A) one AND gate

asy
(B) one OR gate
(C) one AND gate and one OR gate

En
(D) two AND gates

2003
gin TWO MARKS

Q. 86

Y, Z with Y = P 5 Q 5 R and Z = RQ + PR + QP eer


The circuit in the figure has 4 boxes each described by inputs P, Q, R and outputs

The circuit acts as a


ing
.ne
t
(A) 4 bit adder giving P + Q
(B) 4 bit subtractor giving P - Q
(C) 4 bit subtractor giving Q-P
(D) 4 bit adder giving P + Q + R

Q. 87 If the function W, X, Y and Z are as follows


W = R + PQ + RS X = PQRS + PQRS + PQRS
Y = RS + PR + PQ + P .Q Z = R + S + PQ + P .Q .R + PQ .S
Then,
(A) W = Z, X = Z (B) W = Z, X = Y
(C) W = Y (D) W = Y = Z

Downloaded From : www.EasyEngineering.net


Downloaded From : www.EasyEngineering.net
GATE SOLVED PAPER - EC DIGITAL CIRCUITS

Q. 88 A 4 bit ripple counter and a bit synchronous counter are made using flip flops
having a propagation delay of 10 ns each. If the worst case delay in the ripple
counter and the synchronous counter be R and S respectively, then
(A) R = 10 ns, S = 40 ns (B) R = 40 ns, S = 10 ns
(C) R = 10 ns S = 30 ns (D) R = 30 ns, S = 10 ns

Q. 89 In the circuit shown in the figure, A is parallel-in, parallel-out 4 bit register,


which loads at the rising edge of the clock C . The input lines are connected to a
4 bit bus, W . Its output acts at input to a 16 # 4 ROM whose output is floating
when the input to a partial table of the contents of the ROM is as follows

Data 0011 1111 0100 1010 1011 1000 0010 1000

Address 0 2 4 6 8 10 11 14

ww The clock to the register is shown, and the data on the W bus at time t1 is 0110.
The data on the bus at time t2 is

w.E
asy
En
gin
eer
ing
.ne
t
(A) 1111 (B) 1011
(C) 1000 (D) 0010

Q. 90 The DTL, TTL, ECL and CMOS famil GATE of digital ICs are compared in the
following 4 columns

(P) (Q) (R) (S)


Fanout is minimum DTL DTL TTL CMOS
Power consumption is minimum TTL CMOS ECL DTL
Propagation delay is minimum CMOS ECL TTL TTL

Downloaded From : www.EasyEngineering.net


Downloaded From : www.EasyEngineering.net
GATE SOLVED PAPER - EC DIGITAL CIRCUITS

The correct column is


(A) P (B) Q
(C) R (D) S

Q. 91 The circuit shown in figure converts

ww (A) BCD to binary code


(C) Excess -3 to gray code
(B) Binary to excess - 3 code
(D) Gray to Binary code

w.E
Q. 92 In an 8085 microprocessor, the instruction CMP B has been executed while the
content of the accumulator is less than that of register B . As a result

asy
(A) Carry flag will be set but Zero flag will be reset
(B) Carry flag will be rest but Zero flag will be set

En
(C) Both Carry flag and Zero flag will be rest
(D) Both Carry flag and Zero flag will be set

Q. 93
gin
The circuit shown in the figure is a 4 bit DAC

eer
ing
.ne
t
The input bits 0 and 1 are represented by 0 and 5 V respectively. The OP AMP
is ideal, but all the resistance and the 5 v inputs have a tolerance of !10%. The
specification (rounded to nearest multiple of 5%) for the tolerance of the DAC is
(A) !35% (B) !20%
(C) !10% (D) !5%

2002 ONE MARK

Q. 94 4 - bit 2’s complement representation of a decimal number is 1000. The number is


(A) +8 (B) 0
(C) -7 (D) -8

Q. 95 The number of comparators required in a 3-bit comparators type ADC


(A) 2 (B) 3
(C) 7 (D) 8

Downloaded From : www.EasyEngineering.net


Downloaded From : www.EasyEngineering.net
GATE SOLVED PAPER - EC DIGITAL CIRCUITS

Q. 96 If the input to the digital circuit (in the figure) consisting of a cascade of 20 XOR
- gates is X , then the output Y is equal to

(A) 0 (B) 1
(C) X (D) X

2002 TWO MARKS

Q. 97 The gates G1 and G2 in the figure have propagation delays of 10 ns and 20 ns

ww respectively. If the input V1, makes an output change from logic 0 to 1 at time
t = t0 , then the output waveform V0 is

w.E
asy
En
gin
Q. 98 eer
If the input X3, X2, X1, X0 to the ROM in the figure are 8 4 2 1 BCD numbers, then
the outputs Y3, Y2, Y1, Y0 are
ing
.ne
t

(A) gray code numbers (B) 2 4 2 1 BCD numbers


(C) excess - 3 code numbers (D) none of the above

Q. 99 Consider the following assembly language program


MVI B, 87H
MOV A, B
START : JMP NEXT
MVI B, 00H
XRA B

Downloaded From : www.EasyEngineering.net


Downloaded From : www.EasyEngineering.net
GATE SOLVED PAPER - EC DIGITAL CIRCUITS

OUT PORT1
HLT
NEXT : XRA B
JP START
OUT PORT2
HTL
The execution of above program in an 8085 microprocessor will result in
(A) an output of 87H at PORT1
(B) an output of 87H at PORT2
(C) infinite looping of the program execution with accumulator data remaining
at 00H
(D) infinite looping of the program execution with accumulator data alternating
between 00H and 87H

ww
Q. 100 The circuit in the figure has two CMOS NOR gates. This circuit functions as a:

w.E
asy
(A) flip-flop
En
(C) Monostable multivibrator
(B) Schmitt trigger
(D) astable multivibrator

2001
gin ONE MARKS

Q. 101 The 2’s complement representation of -17 is


eer
(A) 101110
(C) 111110
(B) 101111

ing
(D) 110001

Q. 102

.ne
For the ring oscillator shown in the figure, the propagation delay of each inverter
is 100 pico sec. What is the fundamental frequency of the oscillator output

t
(A) 10 MHz (B) 100 MHz
(C) 1 GHz (D) 2 GHz

Q. 103 Ab 8085 microprocessor based system uses a 4K # 8 bit RAM whose starting
address is AA00H. The address of the last byte in this RAM is
(A) OFFFH (B) 1000H
(C) B9FFH (D) BA00H

2001 TWO MARKS

Q. 104 In the TTL circuit in the figure, S2 and S0 are select lines and X7 and X0 are input
lines. S0 and X0 are LSBs. The output Y is

Downloaded From : www.EasyEngineering.net


Downloaded From : www.EasyEngineering.net
GATE SOLVED PAPER - EC DIGITAL CIRCUITS

(A) indeterminate (B) A 5 B


(C) A 5 B (D) C (A 5 B ) + C (A 5 B)

Q. 105 The digital block in the figure is realized using two positive edge triggered D-flip-
flop. Assume that for t < t0, Q1 = Q2 = 0 . The circuit in the digital block is given
by

ww
w.E
asy
En
gin
eer
ing
.ne
t
Q. 106 In the DRAM cell in the figure, the Vt of the NMOSFET is 1 V. For the following
three combinations of WL and BL voltages.

(A) 5 V; 3 V; 7 V
(B) 4 V; 3 V; 4 V
(C) 5 V; 5 V; 5 V
(D) 4 V; 4 V; 4 V

Downloaded From : www.EasyEngineering.net


Downloaded From : www.EasyEngineering.net
GATE SOLVED PAPER - EC DIGITAL CIRCUITS

Q. 107 In the figure, the LED

(A) emits light when both S1 and S2 are closed


(B) emits light when both S1 and S2 are open
(C) emits light when only of S1 and S2 is closed
(D) does not emit light, irrespective of the switch positions.

ww 2000 ONE MARKS

w.E
Q. 108 An 8 bit successive approximation analog to digital communication has full scale
reading of 2.55 V and its conversion time for an analog input of 1 V is 20 ms. The

(A) 10 ms
asy
conversion time for a 2 V input will be
(B) 20 ms

Q. 109
(C) 40 ms

En (D) 50 ms

The number of comparator in a 4-bit flash ADC is


(A) 4
(C) 15 gin (B) 5
(D) 16

Q. 110
eer
For the logic circuit shown in the figure, the required input condition (A, B, C) to
make the output (X) = 1 is

ing
.ne
(A) 1,0,1
(B) 0,0,1
t
(C) 1,1,1
(D) 0,1,1

Q. 111 The number of hardware interrupts (which require an external signal to interrupt)
present in an 8085 microprocessor are
(A) 1 (B) 4
(C) 5 (D) 13

Q. 112 In the microprocessor, the RST6 instruction transfer the program execution to
the following location :
(A)30 H (B) 24 H
(C) 48 H (D) 60 H

Downloaded From : www.EasyEngineering.net


Downloaded From : www.EasyEngineering.net
GATE SOLVED PAPER - EC DIGITAL CIRCUITS

2000 TWO MARKS

Q. 113 The contents of register (B) and accumulator (A) of 8085 microprocessor are 49J
are 3AH respectively. The contents of A and status of carry (CY) and sign (S)
after execution SUB B instructions are
(A) A = F1, CY = 1, S = 1 (B) A = 0F, CY = 1, S = 1
(C) A = F0, CY = 0, S = 0 (D) A = 1F, CY = 1, S = 1

Q. 114 For the logic circuit shown in the figure, the simplified Boolean expression for the
output Y is

ww (A) A + B + C (B) A

w.E
Q. 115
(C) B (D) C

For the 4 bit DAC shown in the figure, the output voltage V0 is

asy
En
gin
eer
(A) 10 V
(C) 4 V
(B) 5 V
(D) 8 V ing
Q. 116

X and Y are the inputs and Z is the inputs. The circuit is .ne
A sequential circuit using D flip-flop and logic gates is shown in the figure, where

(A) S - R Flip-Flop with inputs X = R and Y = S


(B) S - R Flip-Flop with inputs X = S and Y = R
(C) J - K Flip-Flop with inputs X = J and Y = K
(D) J - K Flip-Flop with input X = K and Y = J

Q. 117 In the figure, the J and K inputs of all the four Flip-Flips are made high. The
frequency of the signal at output Y is

Downloaded From : www.EasyEngineering.net


Downloaded From : www.EasyEngineering.net
GATE SOLVED PAPER - EC DIGITAL CIRCUITS

(A) 0.833 kHz (B) 1.0 kHz


(C) 0.91 kHz (D) 0.77 kHz

1999 ONE MARK

Q. 118 The logical expression y = A + AB is equivalent to

ww (A) y = AB
(C) y = A + B
(B) y = AB
(D) y = A + B

w.E
Q. 119 A Darlington emitter follower circuit is sometimes used in the output stage of a
TTL gate in order to
(A) increase its IOL

asy
(B) reduce its IOH
(C) increase its speed of operation

En
(D) reduce power dissipation

Q. 120
in order to gin
Commercially available ECL gears use two ground lines and one negative supply

(A) reduce power dissipation


(B) increase fan-out
eer
(C) reduce loading effect
ing
(D) eliminate the effect of power line glitches or the biasing circuit

Q. 121

6.6 volts, the digital output of the ADC will be .ne


The resolution of a 4-bit counting ADC is 0.5 volts. For an analog input of

(A) 1011
(C) 1100
(B) 1101
(D) 1110 t
1999 TWO MARKS

Q. 122 The minimized form of the logical expression


(ABC + ABC + ABC + ABC ) is
(A) AC + BC + AB (B) AC + BC + AB
(C) AC + BC + AB (D) AC + BC + AB

Q. 123 For a binary half-subtractor having two inputs A and B, the correct set of logical
expressions for the outputs D (= A minus B) and X (= borrow) are
(A) D = AB + AB, X = AB
(B) D = AB + AB + AB , X = AB
(C) D = AB + AB , X = AB
(D) D = AB + AB , X = AB

Downloaded From : www.EasyEngineering.net


Downloaded From : www.EasyEngineering.net
GATE SOLVED PAPER - EC DIGITAL CIRCUITS

Q. 124 If CS = A15 A14 A13 is used as the chip select logic of a 4 K RAM in an 8085 system,
then its memory range will be
(A) 3000 H - 3 FFF H
(B) 7000 H - 7 FFF H
(C) 5000 H - 5 FFF H and 6000 H - 6 FFF H
(D) 6000 H - 6 FFF H and 7000 H - 7 FFF H

Q. 125 The ripple counter shown in the given figure is works as a

ww
w.E (A) mod-3 up counter
(C) mod-3 down counter
(B) mod-5 up counter
(D) mod-5 down counter

1998
asy ONE MARK

Q. 126
En
The minimum number of 2-input NAND gates required to implement of Boolean

(A) two
gin
function Z = ABC , assuming that A, B and C are available, is
(B) three

Q. 127
(C) five

The noise margin of a TTL gate is about


(D) six

eer
(A) 0.2 V
(C) 0.6 V
(B) 0.4 V
(D) 0.8 V ing
Q. 128

101010. ..., the output x and y will be .ne


In the figure is A = 1 and B = 1, the input B is now replaced by a sequence

t
(A) fixed at 0 and 1, respectively
(B) x = 1010.....while y = 0101. ....
(C) x = 1010.....and y = 1010......
(D) fixed at 1 and 0, respectively

Q. 129 An equivalent 2’s complement representation of the 2’s complement number 1101
is
(A) 110100 (B) 01101
(C) 110111 (D) 111101

Downloaded From : www.EasyEngineering.net


Downloaded From : www.EasyEngineering.net
GATE SOLVED PAPER - EC DIGITAL CIRCUITS

Q. 130 The threshold voltage for each transistor in the figure is 2 V. For this circuit to
work as an inverter, Vi must take the values

(A) - 5 V and 0 V (B) - 5 V and 5 V


(C) - 0 V and 3 V (D) 3 V and 5 V

Q. 131 An I/O processor control the flow of information between

ww (A) cache memory and I/O devices


(B) main memory and I/O devices

w.E (C) two I/O devices


(D) cache and main memories

Q. 132

asy
Two 2’s complement number having sign bits x and y are added and the sign bit
of the result is z . Then, the occurrence of overflow is indicated by the Boolean
function
(A) xyz
En (B) x y z

Q. 133
(C) x yz + xyz
gin (D) xy + yz + zx

The advantage of using a dual slope ADC in a digital voltmeter is that


(A) its conversion time is small
(B) its accuracy is high eer
(C) it gives output in BCD format
(D) it does not require a ing
Q. 134
.ne
For the identity AB + AC + BC = AB + AC , the dual form is
(A) (A + B) (A + C) (B + C) = (A + B) (A + C)
(B) (A + B ) (A + C ) (B + C ) = (A + B ) (A + C )
(C) (A + B) (A + C) (B + C) = (A + B ) (A + C )
t
(D) AB + AC + BC = AB + AC

Q. 135 An instruction used to set the carry Flag in a computer can be classified as
(A) data transfer (B) arithmetic
(C) logical (D) program control

Q. 136 The figure is shows a mod-K counter, here K is equal to

Downloaded From : www.EasyEngineering.net


Downloaded From : www.EasyEngineering.net
GATE SOLVED PAPER - EC DIGITAL CIRCUITS

(A) 1 (B) 2
(C) 3 (D) 4

Q. 137 The current I through resistance r in the circuit shown in the figure is

(A) - V (B) V
12R 12R
(C) V (D) V
6R 3T

ww
Q. 138 The K -map for a Boolean function is shown in the figure is the number of
essential prime implicates for this function is

w.E
asy
En
gin
(A) 4 (B) 5
eer
(C) 6 (D) 8
ing
Q. 139
1997

Each cell of a static Random Access Memory contains .ne ONE MARK

(A) 6 MOS transistors


(B) 4 MOS transistors and 2 capacitors
(C) 2 MOS transistors and 4 capacitors
t
(D) 1 MOS transistors and 1 capacitors

Q. 140 A 2 bit binary multiplier can be implemented using


(A) 2 inputs ANSs only
(B) 2 input XORs and 4 input AND gates only
(C) Two 2 inputs NORs and one XNO gate
(D) XOR gates and shift registers

Q. 141 In standard TTL, the ‘totem pole’ stage refers to


(A) the multi-emitter input stage
(B) the phase splitter
(C) the output buffer
(D) open collector output stage

Downloaded From : www.EasyEngineering.net


Downloaded From : www.EasyEngineering.net
GATE SOLVED PAPER - EC DIGITAL CIRCUITS

Q. 142 The inverter 74 ALSO4 has the following specifications


IOH max =- 0.4 A, IOL max = 8 mA, IIH max = 20 mA, IIL max =- 0.1 mA
The fan out based on the above will be
(A) 10 (B) 20
(C) 60 (D) 100

Q. 143 The output of the logic gate in the figure is

(A) 0 (B) 1
(C) A (D) F

ww
Q. 144 In an 8085 mP system, the RST instruction will cause an interrupt
(A) only if an interrupt service routine is not being executed

w.E (B) only if a bit in the interrupt mask is made 0


(C) only if interrupts have been enabled by an EI instruction
(D) None of the above

Q. 145
asy
The decoding circuit shown in the figure is has been used to generate the active

En
low chip select signal for a microprocessor peripheral. (The address lines are
designated as AO to A7 for I/O address)

gin
eer
ing
.ne
The peripheral will correspond to I/O address in the range

Q. 146
(A) 60 H to 63 H
(C) 30 H to 33 H
(B) A4 to A 7H
(D) 70 H to 73 H

The following instructions have been executed by an 8085 mP


t
ADDRESS (HEX) INSTRUCTION
6010 LXI H, 8 A 79 H
6013 MOV A, L
6015 ADDH
6016 DAA
6017 MOV H, A
6018 PCHL
From which address will the next instruction be fetched ?
(A) 6019 (B) 6379
(C) 6979 (D) None of the above

Downloaded From : www.EasyEngineering.net


Downloaded From : www.EasyEngineering.net
GATE SOLVED PAPER - EC DIGITAL CIRCUITS

Q. 147 A signed integer has been stored in a byte using the 2’s complement format. We
wish to store the same integer in a 16 bit word. We should
(A) copy the original byte to the less significant byte of the word and fill the
more significant with zeros
(B) copy the original byte to the more significant byte of the word and fill the
less significant byte with zeros
(C) copy the original byte to the less significant byte of the word and make
each fit of the more significant byte equal to the most significant bit of the
original byte
(D) copy the original byte to the less significant byte as well as the more
significant byte of the word

1997 TWO MARKS

ww
Q. 148 For the NMOS logic gate shown in the figure is the logic function implemented is

w.E
asy
En
gin
(A) ABCDE eer
(B) (AB + C ) : (D + E )

Q. 149
(C) A : (B + C) + D : E
ing
(D) (A + B ) : C + D : E

In a J–K flip-flop we have J = Q and K = 1. Assuming the flip flop was initially

.ne
cleared and then clocked for 6 pulses, the sequence at the Q output will be

t
(A) 010000 (B) 011001
(C) 010010 (D) 010101

Q. 150 The gate delay of an NMOS inverter is dominated by charge time rather than
discharge time because
(A) the driver transistor has larger threshold voltage than the load transistor
(B) the driver transistor has larger leakage currents compared to the load
transistor
(C) the load transistor has a smaller W/L ratio compared to the driver
transistor

Downloaded From : www.EasyEngineering.net


Downloaded From : www.EasyEngineering.net
GATE SOLVED PAPER - EC DIGITAL CIRCUITS

(D) none of the above

Q. 151 The boolean function A + BC is a reduced form of


(A) AB + BC (B) (A + B) : (A + C)
(C) AB + ABC (D) (A + C) : B

1996 ONE MARK

Q. 152 Schottky clamping is resorted in TTl gates


(A) to reduce propagation delay (B) to increase noise margins
(C) to increase packing density (D) to increase fan-out

Q. 153 A pulse train can be delayed by a finite number of clock periods using
(A) a serial-in serial-out shift register

ww (B) a serial-in parallel-out shift register


(C) a parallel-in serial-out shift register

w.E
Q. 154
(D) a parallel-in parallel-out shift register

A 12-bit ADC is operating with a 1 m sec clock period and the total conversion

asy
time is seen to be 14 m sec . The ADC must be of the
(A) flash type (B) counting type

En
(C) intergrating type (D) successive approximation type

Q. 155

gin
The total number of memory accesses involved (inclusive of the op-code fetch)
when an 8085 processor executes the instruction LDA 2003 is
(A) 1 (B) 2
(C) 3
eer
(D) 4

1996 ing TWO MARKS

Q. 156
.ne
A dynamic RAM cell which hold 5 V has to be refreshed every 20 m sec, so that
the stored voltage does not fall by more than 0.5 V. If the cell has a constant

(C) 4 # 10-12 F
(B) 4 # 10-9 F
(D) 4 # 10-15 F
t
discharge current of 1 pA, the storage capacitance of the cell is
(A) 4 # 10-6 F

Q. 157 A 10-bit ADC with a full scale output voltage of 10.24 V is designed to have
a !LSB/2 accuracy. If the ADC is calibrated at 25c C and the operating
temperature ranges from 0c C to 25c C , then the maximum net temperature
coefficient of the ADC should not exceed
(A) ! 200 mV/cC (B) ! 400 mV/cC
(C) ! 600 mV/cC (D) ! 800 mV/cC

Q. 158 A memory system of size 26 K bytes is required to be designed using memory


chips which have 12 address lines and 4 data lines each. The number of such chips
required to design the memory system is
(A) 2 (B) 4
(C) 8 (D) 13

Downloaded From : www.EasyEngineering.net


Downloaded From : www.EasyEngineering.net
GATE SOLVED PAPER - EC DIGITAL CIRCUITS

Q. 159 The following sequence of instructions are executed by an 8085 microprocessor:


1000 LXI SP, 27 FF
1003 CALL 1006
1006 POP H
The contents of the stack pointer (SP) and the HL, register pair on completion of
execution of these instruction are
(A) SP = 27 FF, HL = 1003 (B) SP = 27 FD, HL = 1003
(C) SP = 27 FF, HL = 1006 (D) SP = 27 FD, HL = 1006

***********

ww
w.E
asy
En
gin
eer
ing
.ne
t

Downloaded From : www.EasyEngineering.net


Downloaded From : www.EasyEngineering.net
GATE SOLVED PAPER - EC DIGITAL CIRCUITS

SOLUTIONS

Sol. 1 Option (C) is correct.


Let A denotes the position of switch at ground floor and B denotes the position
of switch at upper floor. The switch can be either in up position or down position.
Following are the truth table given for different combinations of A and B
A B Y(Bulb)
up(1) up(1) OFF(0)
Down(0) Down(0) OFF(0)

ww up(1)
Down(0)
Down(0)
up(1) ON(1)
ON(1)

w.E When the switches A and B are both up or both down, output will be zero (i.e.
Bulb will be OFF). Any of the switch changes its position leads to the ON state
of bulb. Hence, from the truth table, we get

asy
i.e., the XOR gate
Y = A5B

Sol. 2
En
Option (A) is correct.

MVI A, 0.5H;
MVI B, 0.5H;
gin
The program is being executed as follows
A = 05H
B = 05H

eer
At the next instruction, a loop is being introduced in which for the instruction
“DCR B” if the result is zero then it exits from loop so, the loop is executed five
times as follows :
Content in B
ing
Output of ADD B (Stored value at A)
05 05 + 05
.ne
04
03
02
05 + 05 + 04
05 + 05 + 04 + 03
05 + 05 + 04 + 03 + 02
t
01 05 + 05 + 04 + 03 + 02 + 01
00 System is out of loop
i.e., A = 05 + 05 + 04 + 03 + 02 + 01 = 144
At this stage, the 8085 microprocessor exits from the loop and reads the next
instruction. i.e., the accumulator is being added to 03 H. Hence, we obtain
A = A + 03 H = 14 + 03 = 17 H
Sol. 3 Option (D) is correct.
For chip-1, we have the following conclusions:
it is enable when (i) S1 S 0 = 0 0
and (ii) Input = 1
For S1 S 0 = 0 0
We have A13 = A12 = 0

Downloaded From : www.EasyEngineering.net


Downloaded From : www.EasyEngineering.net
GATE SOLVED PAPER - EC DIGITAL CIRCUITS

and for I/p = 1we obtain


A10 = 1 or A10 = 0
A11 = 1
A14 = 1 or A14 = 0
A15 = 1 or A15 = 0
Since, A 0 - A 9 can have any value 0 or 1
Therefore, we have the address range as
A15 A14 A13 A12 A11 A10 A 9 A 8 A7 A 6 A5 A 4 A 3 A2 A1 A 0
From 0 0 0 0 1 0 0 0 0 0 0 0 0 0 0 0
to 0 0 0 0 1 0 1 1 1 1 1 1 1 1 1 1
In Hexadecimal & 0800 H to 0BFFH

ww Similarly, for chip 2, we obtain the range as follows

so,
E = 1 for S1 S 0 = 0 1
A13 = 0 and A12 = 1

w.E and also the I/P = 1 for


A10 = 0 , A11 = 1, A14 = 0 , A15 = 0

A15 asy
so, the fixed I/ps are
A14 A13 A12 A11 A10
0
En 0
Therefore, the address range is
0 1 1 0

gin
A15 A14 A13 A12 A11 A10 A 9 A 8 A7 A 6 A5 A 4 A 3 A2 A1 A 0
From
to
0
0
0
0
0
0
1
1
1
1
0
0
0
1
0
1
0
1 eer
0
1
0
1
0
1
0
1
0
1
0
1
0
1

ing
In hexadecimal it is from 1800 H to 1BFFH . There is no need to obtain rest of
address ranged as only (D) is matching to two results.
Sol. 4 Option (A) is correct.
The given circuit is .ne
t
Condition for the race-around
It occurs when the output of the circuit (Y1, Y2) oscillates between ‘0’ and ‘1’
checking it from the options.
1. Option (A): When CLK = 0
Output of the NAND gate will be A1 = B1 = 0 = 1. Due to these input to the next
NAND gate, Y2 = Y1 : 1 = Y1 and Y1 = Y2 : 1 = Y2 .
If Y1 = 0 , Y2 = Y1 = 1 and it will remain the same and doesn’t oscillate.
If Y2 = 0 , Y1 = Y2 = 1 and it will also remain the same for the clock period. So,
it won’t oscillate for CLK = 0 .
So, here race around doesn’t occur for the condition CLK = 0 .

Downloaded From : www.EasyEngineering.net


Downloaded From : www.EasyEngineering.net
GATE SOLVED PAPER - EC DIGITAL CIRCUITS

2. Option (C): When CLK = 1, A = B = 1


A1 = B1 = 0 and so Y1 = Y2 = 1
And it will remain same for the clock period. So race around doesn’t occur for
the condition.
3. Option (D): When CLK = 1, A = B = 0
So, A1 = B1 = 1
And again as described for Option (B) race around doesn’t occur for the
condition.
Sol. 5 Option (B ) is correct.

ww Y = 1, when A > B

w.E a1 a0 b1 b0
A = a1 a 0, B = b1 b 0

Y
0
1
1
0 asy
0
0
0
0
1
1
1
1
0
1
0
0 En 1
0
1
1
1
1
1
1
0
1
1
0
1
1 gin
Total combination = 6 eer
Sol. 6 Option (A) is correct.
Parallel connection of MOS & OR operation ing
Series connection of MOS & AND operation
.ne
The pull-up network acts as an inverter. From pull down network we write

Sol. 7
Y = (A + B) C = (A + B) + C = A B + C
Option (A) is correct.
Prime implicants are the terms that we get by solving K-map
t

F = XY + XY
1prime
44 2 44 3
implicants

Sol. 8 Option (D) is correct.


Let Qn + 1 is next state and Qn is the present state. From the given below figure.
D = Y = AX 0 + AX1
Qn + 1 = D = AX 0 + AX1
Qn + 1 = A Qn + AQn X 0 = Q , X1 = Q

Downloaded From : www.EasyEngineering.net


Downloaded From : www.EasyEngineering.net
GATE SOLVED PAPER - EC DIGITAL CIRCUITS

If A = 0, Qn + 1 = Qn (toggle of previous state)


If A = 1, Qn + 1 = Qn
So state diagram is

Sol. 9 Option (B) is correct.


The given circuit is shown below:

ww
w.E (PQ QR ) PR = (PQ + QR PR ) = PQ + QR + PR = PQ + QR + PR

Sol. 10
asy
If any two or more inputs are ‘1’ then output y will be 1.
Option (A) is correct.

En
For the output to be high, both inputs to AND gate should be high.
The D-Flip Flop output is the same, after a delay.
Let initial input be 0;
gin (Consider Option A)
then Q = 1 (For 1 D-Flip Flop). This is given as input to 2nd FF.
st

eer
Let the second input be 1. Now, considering after 1 time interval; The output of
1st Flip Flop is 1 and 2nd FF is also 1. Thus Output = 1.
Sol. 11 Option (D) is correct.
F = S1 S 0 I 0 + S1 S 0 I1 + S1 S 0 I 2 + S1 S 0 I 3 ing
I0 = I3 = 0
.ne
Sol. 12
F = PQ + PQ = XOR (P, Q)
Option (A) is correct.
All the states of the counter are initially unset.
t
( S1 = P, S 0 = Q )

State Initially are shown below in table :


Q2 Q1 Q0
0 0 0 0
1 0 0 4
1 1 0 6
1 1 1 7

Downloaded From : www.EasyEngineering.net


Downloaded From : www.EasyEngineering.net
GATE SOLVED PAPER - EC DIGITAL CIRCUITS

0 1 1 3
0 0 1 1
0 0 0 0
Sol. 13 Option (D) is correct.
The sequence is QB QA
00 " 11 " 01 " 10 " 00 " ...
QB QA QB (t + 1) QA (t + 1)
0 0 1 1
1 1 0 1
0 1 1 0
1 0 0 0
QB ^t + 1h

ww
w.E QB ^t + 1h = Q A

asy
En
Sol. 14 Option (C) is correct.
gin
DA = Q A Q B + QA QB

Initially Carry Flag, C = 0


MVI A, 07 H ; A = 0000 0111 eer
RLC
MVO B, A ; B = A = 0000 1110 ing
; Rotate left without carry. A = 0000 1110

RLC
RLC
; A = 0001 1100
; A = 0011 1000 .ne
ADD B ; A = 0011 1000
;
;
+ 0000 1110
0100 0110
t
RRC ; Rotate Right with out carry, A = 0010 0011
Thus A = 23 H
Sol. 15 Option ( ) is correct.

Downloaded From : www.EasyEngineering.net


Downloaded From : www.EasyEngineering.net
GATE SOLVED PAPER - EC DIGITAL CIRCUITS

Sol. 16 Option (B) is correct.


Since G2 is active low input, output of NAND gate must be 0
G2 = A15 : A14 A13 A12 A11 = 0
So, A15 A14 A13 A12 A11 = 00101
To select Y5 Decoder input
ABC = A 8 A 9 A10 = 101
Address range
A15 A14 A13 A12 A11 A10 A 9 A 8 ...............A 0
0011101........A 0
S2
S D
^2D00 - 2DFF h
Sol. 17 Option (A) (B) (C) are correct.
In the circuit F = (A 5 B) 9 (A 9 B) 9 C

ww For two variables


So,
A5B = A9B
(A 5 B) 9 (A 9 B) = 0 (always)

w.E So, F = 1 when C = 1 or C = 0


F = 09C = 0$C+1$C = C

Sol. 18 Option (D) is correct.

asy
Let QA (n), QB (n), QC (n) are present states and QA (n + 1), QB (n + 1), QC (n + 1)
are next states of flop-flops.
In the circuit
En
QA (n + 1) = QB (n) 9 QC (n)
QB (n + 1) QA (n)
gin
QC (n + 1) QB (n)
Initially all flip-flops are reset
eer
1st clock pulse
QA = 0 9 0 = 1 ing
QB = 0
QC = 0 .ne
2 nd
clock pulse
QA = 0 9 0 = 1
QB = 1
t
QC = 0
3 rd clock pulse
QA = 1 9 0 = 0
QB = 1
QC = 1
4 th clock pulse
QA = 1 9 1 = 1
QB = 0
QC = 1
So, sequence QA = 01101.......
Sol. 19 Option (D) is correct.

Downloaded From : www.EasyEngineering.net


Downloaded From : www.EasyEngineering.net
GATE SOLVED PAPER - EC DIGITAL CIRCUITS

Output of the MUX can be written as


F = I 0 S 0 S1 + I1 S 0 S1 + I 2 S 0 S1 + I 3 S 0 S1
Here, I 0 = C, I1 = D, I2 = C , I 3 = CD
and S 0 = A, S1 = B
So, F = C A B + D A B + C A B + C DA B
Writing all SOP terms
F = A B C D + A B C D + A BCD + A B C D + A B C D + A B C D + ABC D
1 44 2
m
44 3 1 44 2
m
44 3 S
m
14 2
m
4 3 14 2
7 m
4 3 1 44 2
m
44 3 Sm 12
3 2 5 9 8

F = / m (2, 3, 5, 7, 8, 9, 12)

Sol. 20 Option (C) is correct.


By executing instruction one by one
MVI A, 45 H & MOV 45 H into accumulator, A = 45 H

ww STC & Set carry, C = 1


CMC & Complement carry flag, C = 0
RAR & Rotate accumulator right through carry

w.E
asy
En
A = 00100010
XRA B & XOR A and B
gin
A = A 5 B = 00100010 5 01000101 = 01100111 = 674
Sol. 21 Option (C) is correct.
TTL " Transistor - Transistor logic eer
Sol. 22
CMOS " Complementary Metal Oxide Semi-conductor
Option (D) is correct. ing
.ne
Vectored interrupts : Vectored interrupts are those interrupts in which program
control transferred to a fixed memory location.

Sol. 23
t
Maskable interrupts : Maskable interrupts are those interrupts which can be
rejected or delayed by microprocessor if it is performing some critical task.
Option (D) is correct.
We have 6X + Z {Y + (Z + XY )}@[X + Z (X + Y)] = 1
Substituting X = 1 and X = 0 we get
[1 + Z {Y + (Z + 1Y )}][ 0 + Z (1 + Y)] = 1
or [1][ Z (1)] = 1 1 + A = 1 and 0 + A = A
or Z =1)Z=0
Sol. 24 Option (A) is correct.
The AND gate implementation by 2:1 mux is as follows

Y = AI 0 + AI1 = AB

Downloaded From : www.EasyEngineering.net


Downloaded From : www.EasyEngineering.net
GATE SOLVED PAPER - EC DIGITAL CIRCUITS

The EX - OR gate implementation by 2:1 mux is as follows

Y = BI0 + BI1 = AB + BA
Sol. 25 Option (A) is correct.
The given circuit is as follows.

ww
w.E The truth table is as shown below. Sequence is 00, 11, 10, 00 ...

CLK J1 asy K1 Q1 J2 K2 Q2
1
2
1
1 En1
1
0
1
1
1
1
1
0
1
3
4
0
1
0
1
1
0 gin 0
1
1
1
0
0

Sol. 26 Option (B) is correct. eer


The given situation is as follows
ing
.ne
The truth table is as shown below
t
P1 P2 a b c d e f g
0 0 1 1 1 1 1 1 0
0 1 1 0 1 1 0 1 1
1 0 1 1 0 1 1 0 1
1 1 1 0 0 1 1 1 1
From truth table we can write
a =1
b = P 1 P 2 + P1 P 2 = P 2 1 NOT Gate
c = P1 P2 + P1 P2 = P1 1 NOT Gate
d = 1 = c+e

Downloaded From : www.EasyEngineering.net


Downloaded From : www.EasyEngineering.net
GATE SOLVED PAPER - EC DIGITAL CIRCUITS

and c = P1 P2 = P1 + P2 1 OR GATE
f = P1 P2 = P1 + P2 1 OR GATE
g = P1 P2 = P1 + P2 1 OR GATE
Thus we have g = P1 + P2 and d = 1 = c + e . It may be observed easily from
figure that
Led g does not glow only when both P1 and P2 are 0. Thus
g = P1 + P2
LED d is 1 all condition and also it depends on
d = c+e
Sol. 27 Option (D) is correct.
As shown in previous solution 2 NOT gates and 3-OR gates are required.

ww Option (C) is correct.


Sol. 28

For the NAND latche the stable states are as follows

w.E
asy
En
For the NOR latche the stable states are as follows

gin
eer
Sol. 29 Option (D) is correct.
ing
From the figure shown below it may be easily seen upper MOSFET are shorted

.ne
and connected to Vdd thus OUT is 1 only when the node S is 0,

t
Since the lower MOSFETs are shorted to ground, node S is 0 only when input P
and Q are 1. This is the function of AND gate.
Sol. 30 Option (B) is correct.
MSB of both number are 1, thus both are negative number. Now we get
11101101 = (- 19) 10
and 11100110 = (- 26) 10
P - Q = (- 19) - (- 26) = 7
Thus 7 signed two’s complements form is
(7) 10 = 00000111

Downloaded From : www.EasyEngineering.net


Downloaded From : www.EasyEngineering.net
GATE SOLVED PAPER - EC DIGITAL CIRCUITS

Sol. 31 Option (D) is correct.


The circuit is as shown below

X = PQ
Y = (P + Q)
So Z = PQ (P + Q)
= (P + Q )( P + Q) = PQ + PQ = P 5 Q

ww
Sol. 32
and M1 = Z 5 R = (P 5 Q) 5 R
Option (A) is correct.
The circuit is as shown below

w.E
asy
En
gin
The truth table is shown below. When CLK make transition Q goes to 1 and

Sol. 33
when D goes to 1, Q goes to 0
Option (B) is correct. eer
ing
Since the input to both JK flip-flop is 11, the output will change every time with
clock pulse. The input to clock is

.ne
t
The output Q0 of first FF occurs after time 3 T and it is as shown below

The output Q1 of second FF occurs after time 3 T when it gets input (i.e. after
3 T from t1) and it is as shown below

Sol. 34 Option (D) is correct.


We have

Downloaded From : www.EasyEngineering.net


Downloaded From : www.EasyEngineering.net
GATE SOLVED PAPER - EC DIGITAL CIRCUITS

3
VDAC = / 2n - 1bn = 2- 1b0 + 20 b1 + 21b2 + 22 b3
n=0
or VDAC = 0.5b0 + b1 + 2b2 + 4b3
The counter outputs will increase by 1 from 0000 till Vth > VDAC . The output of
counter and VDAC is as shown below

Clock b3 b3 b2 b0 VDAC
1 0001 0
2 0010 0.5
3 0011 1
4 0100 1.5
5 0101 2

ww 6
7
0110
0111
2.5
3

w.E 8
9
10
1000
1001
1010
3.5
4
4.5
11
asy1011 5
12
13
En
1100
1101
5.5
6
14 1110 6.5
gin
and when VADC = 6.5 V (at 1101), the output of AND is zero and the counter
stops. The stable output of LED display is 13.
eer
Sol. 35 Option (B) is correct.
The VADC - Vin at steady state is
ing
Sol. 36 Option (A) is correct.
= 6.5 - 6.2 = 0.3V
.ne
Z = I0 RS + I1 RS + I2 RS + I3 RS
= (P + Q ) RS + PRS + PQRS + PRS
= PRS + QRS + PRS + PQRS + PRS
t
The k - Map is as shown below

Z = PQ + PQS + QRS
Sol. 37 Option (C) is correct.
2710H LXI H, 30A0H ; Load 16 bit data 30A0 in HL pair
2713H DAD H ; 6140H " HL

Downloaded From : www.EasyEngineering.net


Downloaded From : www.EasyEngineering.net
GATE SOLVED PAPER - EC DIGITAL CIRCUITS

2714H PCHL ; Copy the contents 6140H of HL in PC


Thus after execution above instruction contests of PC and HL are same and that
is 6140H
Sol. 38 Option (C) is correct.
MSB of Y is 1, thus it is negative number and X is positive number
Now we have X = 01110 = (14) 10
and Y = 11001 = (- 7) 10
X + Y = (14) + (- 7) = 7
In signed two’s complements from 7 is
(7) 10 = 000111
Sol. 39 Option (B) is correct.
Y = AB + CD = AB .CD

ww
Sol. 40
This is SOP form and we require only 3 NAND gate
Option (A) is correct.

w.E The circuit is as shown below

asy
En
gin
and
Y = AB + AB

eer
X = YC + YC = (AB + AB ) C + (AB + AB ) C
= (AB + AB) C + (AB + AB ) C
= ABC + ABC + ABC + ABC ing
Sol. 41 Option (D) is correct.
Y = ABCD + ABCD + ABC D + ABC D .ne
= ABCD + ABC D + ABC D + ABC D
= ABCD + ABC D + BC D (A + A)
= ABCD + ABC D + BC D
t A+A = 1
Sol. 42 Option (B) is correct.
In given TTL NOT gate when Vi = 2.5 (HIGH), then
Q1 " Reverse active
Q2 " Saturation
Q3 " Saturation
Q4 " cut - off region
Sol. 43 Option (C) is correct.
For X = 0, Y = 1 P = 1, Q = 0
For X = 0, Y = 0 P = 1, Q = 1
For X = 1, Y = 1 P = 1, Q = 0 or P = 0, Q = 1

Downloaded From : www.EasyEngineering.net


Downloaded From : www.EasyEngineering.net
GATE SOLVED PAPER - EC DIGITAL CIRCUITS

Sol. 44 Option (C) is correct.


Chip 8255 will be selected if bits A3 to A7 are 1. Bit A0 to A2 can be 0 or.
1. Thus address range is
11111000 F8H
11111111 FFH
Sol. 45 Option (B) is correct.
Since the inverting terminal is at virtual ground the resistor network can be
reduced as follows

ww
w.E
asy
En
The current from voltage source is
I = VR = 10 = 1 mA
R 10k
gin
This current will be divide as shown below

eer
ing
.ne
-3
t
Now i = I = 1 # 10 = 62.5 m A
16 16
Sol. 46 Option (C) is correct.
The net current in inverting terminal of OP - amp is
I- = 1 + 1 = 5I
4 16 16
So that V0 =- R # 5I =- 3.125
16
Sol. 47 Option (B) is correct.
Line
1 : MVI A, B5H ; Move B5H to A
2 : MVI B, 0EH ; Move 0EH to B
3 : XRI 69H ; [A] XOR 69H and store in A
; Contents of A is CDH

Downloaded From : www.EasyEngineering.net


Downloaded From : www.EasyEngineering.net
GATE SOLVED PAPER - EC DIGITAL CIRCUITS

4 : ADDB ; Add the contents of A to contents of B and


; store in A, contents of A is EAH
5 : ANI 9BH ; [a] AND 9BH, and store in A,
; Contents of A is 8 AH
6 : CPI 9FH ; Compare 9FH with the contents of A
; Since 8 AH < 9BH, CY = 1
7 : STA 3010 H ; Store the contents of A to location 3010 H
8 : HLT ; Stop
Thus the contents of accumulator after execution of ADD instruction is EAH.
Sol. 48 Option (C) is correct.
The CY = 1 and Z = 0
Sol. 49 Option (A) is correct.
For this circuit the counter state (Q1, Q0) follows the sequence 00, 01, 10, 00 ... as

ww shown below

Clock D1 D0 Q1 Q0 Q1 NOR Q0

w.E 1st 01
00
10
1
0
2nd
3rd
10
00 asy 01
00
0
0

En
gin
eer
Sol. 50 Option (A) is correct.
ing
As shown below there are 2 terms in the minimized sum of product expression.

1 0 0 1 .ne
0
0
1
d
0
0
0
d
0
0
1
1
t
Sol. 51 Option (B) is correct.
The output is taken from the 5th line.
Sol. 52 Option (D) is correct.
After applying two clock poles, the outputs of the full adder is S = 1, C0 = 1
A B Ci S Co
1st 1 0 0 0 1
2nd 1 1 1 1 1
Sol. 53 Option (D) is correct.
SSSS
100010011001
4 2 3 1

Sol. 54 Option (B) is correct.

Downloaded From : www.EasyEngineering.net


Downloaded From : www.EasyEngineering.net
GATE SOLVED PAPER - EC DIGITAL CIRCUITS

In this the diode D2 is connected to the ground. The following table shows the
state of counter and D/A converter

Q2 Q1 Q0 D3 = Q2 D2 = 0 D1 = Q1 D0 = Q0 Vo
000 0 0 0 0 0
001 0 0 0 1 1
010 0 0 1 0 2
011 0 0 1 1 3
100 1 0 0 0 8
101 1 0 0 1 9
110 1 0 1 0 10
111 1 0 1 1 11

ww 000
001
0
0
0
0
0
0
0
1
0
1

w.E
Sol. 55
Thus option (B) is correct
Option (B) is correct.
LXI, EFFF H
asy
CALL 3000 H
:
; Load SP with data EFFH
; Jump to location 3000 H

:
En
:
3000H LXI H, 3CF4
PUSH PSW gin
; Load HL with data 3CF4H
; Store contnets of PSW to Stack
POP PSW
PRE eer
; Restore contents of PSW from stack
; stop
Before instruction SPHL the contents of SP is 3CF4H.
After execution of POP PSW, SP + 2 " SP ing
After execution of RET, SP + 2 " SP
Thus the contents of SP will be 3CF4H + 4 = 3CF8H .ne
Sol. 56 Option (A) is correct.
t
The inputs D0 and D1 respectively should be connected as Q1 and Q0
where Q0 " D1 and Q1 " D0
Sol. 57 Option (D) is correct.
If the point P is stuck at 1, then output f is equal to A

Sol. 58 Option (B) is correct.


Dividing 43 by 16 we get

g
2
16 43
32
11

Downloaded From : www.EasyEngineering.net


Downloaded From : www.EasyEngineering.net
GATE SOLVED PAPER - EC DIGITAL CIRCUITS

11 in decimal is equivalent is B in hexamal.


Thus 4310 * 2B16
Now 410 * 01002
310 * 00112
Thus 4310 * 01000011BCD
Sol. 59 Option (A) is correct.
The diagram is as shown in fig

ww
w.E f' = BC + BC
f = f' A + f ' 0

Sol. 60
asy
Option (C) is correct.
= f'A = ABC + ABC

En
The circuit is as shown below

gin
eer
ing
.ne
If output is at logic 0, the we have V0 = 0 which signifies BJT Q3 is in saturation
and applying KVL we have

or
VBE3 = IR # 1k
0.75 = IR # 1k
t
or IR = 0.75 mA
Sol. 61 Option (A) is correct.
We have f = ABC + ABC
= B (AC + AC ) = B (A + C)( A + C )
Sol. 62 Option (C) is correct.
Characteristic equation for a jk flip-flop is written as
Qn + 1 = JQ n + K Qn
Where Qn is the present output
Qn + 1 is next output
So, Qn + 1 = 10 + K : 0 Qn = 0
Qn + 1 = 1

Downloaded From : www.EasyEngineering.net


Downloaded From : www.EasyEngineering.net
GATE SOLVED PAPER - EC DIGITAL CIRCUITS

Sol. 63 Option (C) is correct.


Since T2 T1 T0 is at 111, at every clock Q2 Q1 Q0 will be changes. Ir present state is
011, the next state will be 100.
Sol. 64 Option (D) is correct.
Sol. 65 Option (C) is correct.
0100H LXI SP, 00FF ; Load SP with 00FFG
0103H LXI H, 0701 ; Load HL with 0107H
0106H MVI A, 20H ; Move A with 20 H
0108 H SUB M ; Subtract the contents of memory
; location whose address is stored in HL
; from the A and store in A
0109H ORI 40H ; 40H OR [A] and store in A
010BH ADD M ; Add the contents of memeory location

ww ; whose address is stored in HL to A


; and store in A

w.E
Sol. 66
HL contains 0107H and contents of 0107H is 20H
Thus after execution of SUB the data of A is 20H - 20H = 00
Option (C) is correct.

asy
Before ORI instruction the contents of A is 00H. On execution the ORI 40H the
contents of A will be 40H

En00H = 00000000
40H = 01000000
ORI 01000000
gin
After ADD instruction the contents of memory location whose address is stored
in HL will be added to and will be stored in A
40H + 20 H = 60 H
eer
Sol. 67 Option (C) is correct.
ing
A master slave D-flip flop is shown in the figure.

.ne
t
In the circuit we can see that output of flip-flop call be triggered only by transition
of clock from 1 to 0 or when state of slave latch is affected.
Sol. 68 Option (A) is correct.
The range of signed decimal numbers that can be represented by n - bits 1’s
complement number is - (2n - 1 - 1) to + (2n - 1 - 1).
Thus for n = 6 we have
Range =- (26 - 1 - 1) to + (26 - 1 - 1)
=- 31 to + 31
Sol. 69 Option (D) is correct.
The minimum number of bit require to encode 100 increment is
2n $ 100

Downloaded From : www.EasyEngineering.net


Downloaded From : www.EasyEngineering.net
GATE SOLVED PAPER - EC DIGITAL CIRCUITS

or n $7
Sol. 70 Option (B) is correct.
Shift Register " Serial to parallel data conversion
Counter " Frequency division
Decoder " Addressing in memory chips.
Sol. 71 Option (A) is correct.
For the TTL family if terminal is floating, then it is at logic 1.
Thus Y = (AB + 1) = AB .0 = 0
Sol. 72 Option (C) is correct.
11001 1001 111001
00110 0110 000110
+1 +1 +1

ww 00111
7
0111
7
000111
7
Thus 2’s complement of 11001, 1001 and 111001 is 7. So the number given in the

w.E
Sol. 73
question are 2’s complement correspond to -7.
Option (C) is correct.

asy
In the modulo - 6 ripple counter at the end of sixth pulse (i.e. after 101 or at 110)
all states must be cleared. Thus when CB is 11 the all states must be cleared. The
input to 2-input gate is C and B and the desired output should be low since the
CLEAR is active low
En
Thus when C and B are 0, 0, then output must be 0. In all other case the output

Sol. 74 Option (C) is correct.


gin
must be 1. OR gate can implement this functions.

required.
3 2 eer
Number of MUX is 4 = 2 and 2 = 1. Thus the total number 3 multiplexers is

Sol. 75 Option (D) is correct.


ing
AC + BC = AC1 + BC 1 = AC (B + B ) + BC (A + A)
= ACB + ACB + BC A + BC A .ne
Sol. 76 Option (D) is correct.
We have
or
t
f (x, y) = xy + xy + xy = x (y + y) + xy = x + xy
f (x, y) = x + y
Here compliments are not available, so to get x we use NOR gate. Thus desired
circuit require 1 unit OR and 1 unit NOR gate giving total cost 2 unit.
Sol. 77 Option (D) is correct.
For 8255, various modes are described as following.
Mode 1 : Input or output with hand shake
In this mode following actions are executed
1. Two port (A & B) function as 8 - bit input output ports.
2. Each port uses three lines from C as a hand shake signal
3. Input & output data are latched.
Form (ii) the mode is 1.
Mode 2 : Bi-directional data transfer
This mode is used to transfer data between two computer. In this mode port A

Downloaded From : www.EasyEngineering.net


Downloaded From : www.EasyEngineering.net
GATE SOLVED PAPER - EC DIGITAL CIRCUITS

can be configured as bidirectional port. Port A uses five signal from port C as
hand shake signal.
For (1), mode is 2
Sol. 78 Option (B) is correct.
LDA 16 bit & Load accumulator directly this instruction copies data byte from
memory location (specified within the instruction) the accumulator.
It takes 4 memory cycle-as following.
1. in instruction fetch
2. in reading 16 bit address
1. in copying data from memory to accumulator
LXI D, (F0F1) 4 & It copies 16 bit data into register pair D and E.
It takes 3 memory cycles.

ww
Sol. 79 Option (A) is correct.
LXI H, 9258H
MOV A, M
; 9258H " HL
; (9258H) " A

w.E CMa
MOV M, A
; A"A
; A"M

Sol. 80
asy
This program complement
Option (D) is correct.
the data of memory location 9258H.

En
MVI A, 00H
LOOP ADD B
; Clear accumulator
; Add the contents of B to A
DCR C
JNZ LOOP
HLT
gin
; Decrement C
; If C is not zero jump to loop

END
eer
This instruction set add the contents of B to accumulator to contents of C times.
Sol. 81 Option (D) is correct.
ing
The number of distinct boolean expression of n variable is 22n . Thus
22 = 216 = 65536
4

.ne
Sol. 82 Option (C) is correct.
t
In the flash analog to digital converter, the no. of comparators is equal to 2n - 1,
where n is no. of bit.s
So, 2n - 1 = 28 - 1 = 255
Sol. 83 Option (B) is correct.
When output of the 74 series gate of TTL gates is taken from BJT then the
configuration is either totem pole or open collector configuration .
Sol. 84 Option (D) is correct.
A 2n: 1 MUX can implement all logic functions of (n + 1) variable without andy
additional circuitry. Here n = 3 . Thus a 8 : 1 MUX can implement all logic
functions of 4 variable.
Sol. 85 Option (D) is correct.
Counter must be reset when it count 111. This can be implemented by following
circuitry

Downloaded From : www.EasyEngineering.net


Downloaded From : www.EasyEngineering.net
GATE SOLVED PAPER - EC DIGITAL CIRCUITS

Sol. 86 Option (B) is correct.


We have Y = P5Q5R
Z = RQ + PR + QP
Here every block is a full subtractor giving P - Q - R where R is borrow. Thus
circuit acts as a 4 bit subtractor giving P - Q .
Sol. 87 Option (A) is correct.
W = R + PQ + RS
X = PQRS + PQRS + PQRS
Y = RS + PR + PQ + PQ = RS + PR $ PQ $ PQ

ww = RS + (P + R )( P + Q)( P + Q)
= RS + (P + PQ + PR + QR )( P + Q)

w.E = RS + PQ + QR (P + P ) + QR = RS + PQ + QR
Z = R + S + PQ + PQR + PQS = R + S + PQ $ PQR $ PQS

asy
= R + S + (P + Q )( P + Q + R)( P + Q + S)
= R + S + PQ + PQ + PQS + PR + PQR

En + PRS + PQ + PQS + PQR + QRS


= R + S + PQ + PQS + PR + PQR + PRS + PQS + PQR + QRS

gin
= R + S + PQ (1 + S) + PR (1 + P ) + PRS + PQS + PQR + QRS
= R + S + PQ + PR + PRS + PQS + PQR + QRS
= R + S + PQ + PR (1 + Q ) + PQS + QRS
eer
= R + S + PQ + PR + PQS + QRS
Thus W = Z and X = Z
ing
Sol. 88 Option (B) is correct.
Propagation delay of flip flop is
.ne
tpd = 10 nsec
Propagation delay of 4 bit ripple counter
R = 4tpd = 40 ns
t
and in synchronous counter all flip-flop are given clock simultaneously, so
S = tpd = 10 ns
Sol. 89 Option (C) is correct.
After t = t1, at first rising edge of clock, the output of shift register is 0110, which
in input to address line of ROM. At 0110 is applied to register. So at this time
data stroed in ROM at 1010 (10), 1000 will be on bus.
When W has the data 0110 and it is 6 in decimal, and it’s data value at that add
is 1010
then 1010 i.e. 10 is acting as odd, at time t2 and data at that movement is 1000.
Sol. 90 Option (B) is correct.
The DTL has minimum fan out and CMOS has minimum power consumption.
Propagation delay is minimum in ECL.

Downloaded From : www.EasyEngineering.net


Downloaded From : www.EasyEngineering.net
GATE SOLVED PAPER - EC DIGITAL CIRCUITS

Sol. 91 Option (D) is correct.


Let input be 1010; output will be 1101
Let input be 0110; output will be 0100
Thus it convert gray to Binary code.
Sol. 92 Option (A) is correct.
CMP B & Compare the accumulator content with context of Register B
If A < R CY is set and zero flag will be reset.
Sol. 93 Option (A) is correct.
Vo =- V1 :R bo + R b1 + R b2 + R b 3D
R 2R 4R 4R

Exact value when V1 = 5 , for maximum output


VoExact =- 5 :1 + 1 + 1 + 1 D =- 9.375
2 4 8

ww Maximum Vout due to tolerance


Vo max =- 5.5 :110 + 110 + 110 + 110 D

w.E Tolerance
90
=- 12.604
2 # 90 4 # 90 8 # 90

= 34.44% = 35%
Sol. 94
asy
Option (D) is correct.
If the 4- bit 2’s complement representation of a decimal number is 1000, then the
number is -8
En
Sol. 95 Option (C) is correct.
gin
In the comparator type ADC, the no. of comparators is equal to 2n - 1, where n is
no. of bit.s
So, 23 - 1 = 7
eer
Sol. 96 Option (B) is correct.
Output of 1 st XOR = = X $ 1 + X $ 1 = X ing
Output of 2 nd XOR = X X + XX = 1
So after 4,6,8,...20 XOR output will be 1.
.ne
Sol. 97 Option (B) is correct.
They have prorogation delay as respectively,
G1 " 10 nsec
t
G2 " 20 nsec
For abrupt change in Vi from 0 to 1 at time t = t0 we have to assume the output
of NOR then we can say that option (B) is correct waveform.

Sol. 98 Option (B) is correct.


Let X3 X2 X1 X0 be 1001 then Y3 Y2 Y1 Y0 will be 1111.

Downloaded From : www.EasyEngineering.net


Downloaded From : www.EasyEngineering.net
GATE SOLVED PAPER - EC DIGITAL CIRCUITS

Let X3 X2 X1 X0 be 1000 then Y3 Y2 Y1 Y0 will be 1110


Let X3 X2 X1 X0 be 0110 then Y3 Y2 Y1 Y0 will be 1100
So this converts 2-4-2-1 BCD numbers.
Sol. 99 Option (B) is correct.
MVI B, 87H ; B = 87
MOV A, B ; A = B = 87
START : JMP NEXT ; Jump to next
XRA B ; A 5 B " A,
; A = 00, B = 87
JP START ; Since A = 00 is positive
; so jump to START
JMP NEXT ;Jump to NEXT ; unconditionally
NEXT : XRA ; B ; A 5 B " A, A = 87 ,

ww JP START
OUT PORT2
; B = 87 H
; will not jump as D7 , of A is 1
; A = 87 " PORT2

w.E
Sol. 100 Option (C) is correct.
The circuit is as shown below

asy
En
gin
The circuit shown is monostable multivibrator as it requires an external triggering
and it has one stable and one quasistable state.
eer
Sol. 101 Option (B) is correct.
The two’s compliment representation of 17 is
17 = 010001
ing
Its 1’s complement is 101110
.ne
So 2’s compliment is

+
101110
1
t
101111
Sol. 102 Option (C) is correct.
The propagation delay of each inverter is tpd then The fundamental frequency of
oscillator output is
f = 1 = 1 = 1 GHz
2ntpd 2 # 5 # 100 # 10 - 12
Sol. 103 Option (C) is correct.
4K # 8 bit means 102410 location of byte are present
Now 102410 * 1000H
It starting address is AA00H then address of last byte is
AA00H + 1000H - 0001H = B9FFH

Downloaded From : www.EasyEngineering.net


Downloaded From : www.EasyEngineering.net
GATE SOLVED PAPER - EC DIGITAL CIRCUITS

Sol. 104 Option (D) is correct.


Y = I0 + I3 + I5 + I6 = C BA + C AB + CBA + CBA
= C (BA + AB) + C (AB + BA)
or Y = C (A 5 B ) + C (A 5 B)
Sol. 105 Option (C) is correct.
The output of options (C) satisfy the given conditions

ww
w.E
Sol. 106 Option (B) is correct.
Sol. 107

asy
Option (D) is correct.
For the LED to glow it must be forward biased. Thus output of NAND must be
LOW for LED to emit light. So both input to NAND must be HIGH. If any one

En
or both switch are closed, output of AND will be LOW. If both switch are open,
output of XOR will be LOW. So there can’t be both input HIGH to NAND. So
LED doesn’t emit light.
gin
Sol. 108 Option (B) is correct.

eer
Conversion time of successive approximate analog to digital converters is
independent of input voltage. It depends upon the number of bits only. Thus it
remains unchanged.
ing
Option (C) is correct.

.ne
Sol. 109

In the flash analog to digital converter, the no. of comparators is equal to 2n - 1,


where n is no. of bits.

Sol. 110
So, 2 4 - 1 = 15
Option (D) is correct.
t
As the output of AND is X = 1, the all input of this AND must be 1. Thus
AB + AB = 1 ...(1)
BC + BC = 1 ...(2)
C =1 ...(3)
From (2) and (3), if C = 1, then B = 1
If B = 1, then from (1) A = 0 . Thus A = 0, B = 1 and C = 1
Sol. 111 Option (C) is correct.
Interrupt is a process of data transfer by which an external device can inform
the processor that it is ready for communication. 8085 microprocessor have five
interrupts namely TRAP, INTR, RST 7.5, RST 6.5 and RST 5.5

Downloaded From : www.EasyEngineering.net


Downloaded From : www.EasyEngineering.net
GATE SOLVED PAPER - EC DIGITAL CIRCUITS

Sol. 112 Option (A) is correct.


For any RST instruction, location of program transfer is obtained in following
way.
RST x & (x ) 8) 10 " convert in hexadecimal
So for RST 6 & (6 ) 8) 10 = (48) 10 = (30) H
Sol. 113 Option (A) is correct.
Accumulator contains A = 49 H
Register B = 3 AH
SUB B = A minus B
A = 49 H = 01001001
B = 3 AH = 00111010
2’s complement of (- B) = 11000110

ww A - B = A + (- B)
010 010 01

w.E & +11 0 0 0 11 0


0 0 0 0 1111
Carry = 1

asy
so here outputA = 0 F
Carry CY = 1

En Sign flag S = 1
Sol. 114 Option (C) is correct.
The circuit is as shown below : gin
eer
ing
.ne
Sol. 115
Y = B + (B + C ) = B (B + C ) = B
Option (B) is correct.
t
The circuit is as shown below

The voltage at non-inverting terminal is


V+ = 1 + 1 = 5
8 2 8

Downloaded From : www.EasyEngineering.net


Downloaded From : www.EasyEngineering.net
GATE SOLVED PAPER - EC DIGITAL CIRCUITS

V- = V+ = 5 ...(1)
8
Now applying voltage divider rule
V- = 1k V% = 1 Vo ...(2)
1k + 7k 8
From (1) and (2) we have
Vo = 8 # 5 = 5V
8
Sol. 116 Option (D) is correct.
The truth table is shown below
Z = XQ + YQ
Comparing from the truth table of J - K FF
Y = J,

ww X Y
X =K

w.E 0
0
0
1
Q
0
1
1 asy
0
1
1
Q1

Sol. 117 En
Option (B) is correct.

gin
In the figure the given counter is mod-10 counter, so frequency of output is
10k = 1k

Sol. 118
10
Option (D) is correct. eer
We have y = A + AB
we know from Distributive property ing
Thus
x + yz = (x + y) (x + z)
y = (A + A) (A + B) = A + B .ne ` A+A = 1
Sol. 119 Option (C) is correct.
t
Darligton emitter follower provides a low output impedance in both logical state
(1 or 0). Due to this low output impedance, any stray capacitance is rapidly
charged and discharged, so the output state changes quickly. It improves speed
of operation.
Sol. 120 Option (D) is correct.
Sol. 121 Option (B) is correct.
For ADC we can write
Analog input = (decimal eq of digital output) # resol
6.6 = (decimal eq. of digital output) # 0.5
6.6 = decimal eq of digital. output
0. 5
13.2 = decimal equivalent of digital output so output of ADC is 1101.
Sol. 122 Option (A) is correct.
We use the K -map as below.

Downloaded From : www.EasyEngineering.net


Downloaded From : www.EasyEngineering.net
GATE SOLVED PAPER - EC DIGITAL CIRCUITS

So given expression equal to


= AC + BC + AB
Sol. 123 Option (C) is correct.
For a binary half-subtractor truth table si given below.

ww
w.E from truth table we can find expressions of D & X
D = A 5 B = AB + AB

Sol. 124
asy
X = AB
Option (B) is correct.

En
We have 4 K RAM (12 address lines)

gin
eer
ing
.ne
t
S so here chip select logic CS = A15 A14 A13
address range (111)
A15 A14 A13 A12 A11 A10 A 9 A 8 A7 A6 A5 A 4 A 3 A2 A1 A 0
initial 1 1 1 0 0 0 0 0 0 0 0 0 0 0 0 0
address &7000H
final 1 1 1 1 1 1 1 1 1 1 1 1 1 1 11
address &7FFFH
so address range is (7 0 0 0 H – 7 F F F H)

Downloaded From : www.EasyEngineering.net


Downloaded From : www.EasyEngineering.net
GATE SOLVED PAPER - EC DIGITAL CIRCUITS

Sol. 125 Option (D) is correct.


From the given figure we can write the output

For the state 010 all preset = 1 and output QA QB QC = 111 so here total no. of
states = 5 (down counter)

ww
Sol. 126 Option (C) is correct.
Given boolean function is

w.E Now
Z = ABC
Z = ABC = ACB = AC + B
Thus
we have asy Z = AC + B
Z = X + Y (1 NOR gate)
where
En X = AC (1 NAND gate)
To implement a NOR gate we required 4 NAND gates as shown below in figure.

gin
eer
here total no. of NAND gates required ing
Sol. 127
= 4+1 = 5
Option (B) is correct. .ne
For TTL worst cases low voltages are
VOL (max) = 0.4 V
VIL (max) = 0.8 V
t
Worst case high voltages are
VOH (min) = 2.4 V
VIH (min) = 2 V
The difference between maximum input low voltage and maximum output low
voltage is called noise margin. It is 0.4 V in case of TTL.
Sol. 128 Option (D) is correct.
From the figure we can see
If A =1 B=0
then y =1 x=0
If A =1 B=1
then also y =1 x=0
so for sequence B = 101010....output x and y will be fixed at 0 and 1 respectively.

Downloaded From : www.EasyEngineering.net


Downloaded From : www.EasyEngineering.net
GATE SOLVED PAPER - EC DIGITAL CIRCUITS

Sol. 129 Option (D) is correct.


Given 2’s complement no. 1101; the no. is 0011
for 6 digit output we can write the no. is – 000011
2’s complement representation of above no. is 111101
Sol. 130 Option (A) is correct.
Sol. 131 Option (B) is correct.
An I/O Microprocessor controls data flow between main memory and the I/O
device which wants to communicate.
Sol. 132 Option (D) is correct.
Sol. 133 Option (B) is correct.
Dual slope ADC is more accurate.
Sol. 134 Option (A) is correct.

ww Dual form of any identity can be find by replacing all AND function to OR and
vice-versa. so here dual form will be

w.E
Sol. 135
(A + B) (A + C) (B + C) = (A + B) (A + C)
Option (B) is correct.
Carry flag will be affected by arithmetic instructions only.
Sol. 136
asy
Option (C) is correct.
This is a synchronous counter. we can find output as

EnQA QB
0 0
1 0
0 1 gin
h
0 0
eer
So It counts only three states. It is a mod-3 counter.
K =3 ing
Sol. 137

Sol. 138
Option (B) is correct.
Option (A) is correct. .ne
t
Essential prime implicates for a function is no. of terms that we get by solving K
-map. Here we get 4 terms when solve the K -map.

y = B D + A C D + C AB + CA B
so no of prime implicates is 4

Downloaded From : www.EasyEngineering.net


Downloaded From : www.EasyEngineering.net
GATE SOLVED PAPER - EC DIGITAL CIRCUITS

Sol. 139 Option (A) is correct.


Sol. 140 Option (B) is correct.
For a 2 bit multiplier
B1 B0
# A1 A0
A 0 B1 A0 B0
# A1 B1 A1 B 0
C3 C2 C1 C0
This multiplication is identical to AND operation and then addition.
Sol. 141 Option (C) is correct.
In totem pole stage output resistance will be small so it acts like a output buffer.
Sol. 142 Option (B) is correct.

ww Consider high output state


fan out = IOH max = 400 mA = 20
IIH max 20 mA

w.E Consider low output state


fan out = IOL max = 8 mA = 80
IIL max 0.1 mA

asy
Thus fan out is 20
Sol. 143

En
Option (A) is correct.
The given gate is ex-OR so output

Here input B = 0 so,gin


F = AB + AB

Sol. 144 Option (C) is correct.


F = A1 + A0 = A
eer
enable EI . ing
EI = Enabled Interput flag ,RST will cause an Interrupt only it we

Sol. 145 Option (A) is correct.


.ne
Here only for the range 60 to 63 H chipselect will be 0, so peripheral will correspond

Sol. 146 Option (B) is correct.


By executing instructions one by one
t
in this range only chipselect = 1 for rest of the given address ranges.

LXI H, 8A79 H (Load HL pair by value 8A79)


H = 8AH L = 79 H
MOV A, L (copy contain of L to accumulator)
A = 79 H
ADDH (add contain of H to accumulator)
A = 79 H = 0 1111 0 0 1
H = 8AH = add 1 0 0 0 1 0 1 0 = A = 0 0 0 0 0 0 11
Carry = 1
DAA (Carry Flag is set, so DAA adds 6 to high order four bits)
0 1111 0 0 1
DAA add 1 0 0 0 1 0 1 0

Downloaded From : www.EasyEngineering.net


Downloaded From : www.EasyEngineering.net
GATE SOLVED PAPER - EC DIGITAL CIRCUITS

A = 0 0 0 0 0 0 1 1 = 63 H
MOV H, A (copy contain of A to H)
H = 63 H
PCHL (Load program counter by HL pair)
PC = 6379 H
Sol. 147 Option (C) is correct.
Sol. 148 Option (C) is correct.
NMOS In parallel makes OR Gate & in series makes AND so here we can have
F = A (B + C) + DE
we took complement because there is another NMOS given above (works as an
inverter)
Sol. 149 Option (D) is correct.

ww For a J -K flip flop we have characteristic equation as


Q (t + 1) = JQ (t) + KQ (t)

w.E Q (t) & Q (t + 1) are present & next states.


In given figure J = Q (t), K = 1 so
Q (t + 1) = Q (t) Q (t) + 0Q (t)

asy Q (t + 1) = Q (t)[complement of previous state]


we have initial input Q (t) = 0

En
so for 6 clock pulses sequence at output Q will be 010101
Sol. 150

Sol. 151
Option (C) is correct.
Option (B) is correct. gin
By distributive property in boolean algebra we have
(A + BC) = (A + B) (A + C) eer
(A + B) (A + C) = AA + AC + AB + BC
ing
= A (1 + C) + AB + BC = A + AB + BC
= A (1 + B) + BC = A + BC
.ne
Sol. 152 Option (A) is correct.

t
The current in a p n junction diode is controlled by diffusion of majority carriers
while current in schottky diode dominated by the flow of majority carrier over the
potential barrier at metallurgical junction. So there is no minority carrier storage
in schottky diode, so switching time from forward bias to reverse bias is very
short compared to p n junction diode. Hence the propagation delay will reduces.
Sol. 153 Option (B) is correct.
Sol. 154 Option (D) is correct.
The total conversion time for different type of ADC are given as–
t is clock period
For flash type & 1t
Counter type & (2n - t) = 4095 m sec
n = no.of bits
Integrating type conver time > 4095 m sec
successive approximation type nt = 12 m sec

Downloaded From : www.EasyEngineering.net


Downloaded From : www.EasyEngineering.net
GATE SOLVED PAPER - EC DIGITAL CIRCUITS

here n = 12 so
nt = 12
12t = 12
so this is succ. app. type ADC.
Sol. 155 Option (D) is correct.
LDA 2003 (Load accumulator by a value 2003 H) so here total no. of memory
access will be 4.
1 = Fetching instruction
2 = Read the value from memory
1 = write value to accumulator
Sol. 156 Option (D) is correct.
Storage capacitance

ww C = i = 1 # 10
dv
b dt l
-12

5 - 0.5
b 20 10-3 l

w.E
#
-12 -3
= 1 # 10 # 20 # 10 = 4.4 # 10-15 F
4.5
Sol. 157

asy
Option (A) is correct.
Accuracy ! 1 LSB = Tcoff # DT
2
or En 1 10.24 = T
2 # 210 coff # DT

or Tcoff = gin 10.24


2 # 1024 # (50 - 25) cC
= 200 mV/cC

Sol. 158 Option (D) is correct.


No. of chips = 26 # 210 # 8 = 13eer
Sol. 159 Option (C) is correct.
12
2 #4
ing
Given instruction set
1000 LXI SP 27FF .ne
1003 CALL
1006 POP H
1006

First Instruction will initialize the SP by a value


t
27FF SP ! 27FF
CALL 1006 will “Push PC” and Load PC by value 1006
PUSH PC will store value of PC in stack
PC = 1006

now POP H will be executed

Downloaded From : www.EasyEngineering.net


Downloaded From : www.EasyEngineering.net
GATE SOLVED PAPER - EC DIGITAL CIRCUITS

which load HL pair by stack values


HL = 1006 and
SP = SPl + 2
SP = SPl + 2 = SP - 2 + 2 = SP
SP = 27FF

***********

ww
w.E
asy
En
gin
eer
ing
.ne
t

Downloaded From : www.EasyEngineering.net


Downloaded From : www.EasyEngineering.net

No part of this publication may be reproduced or distributed in any form or any means, electronic, mechanical,
photocopying, or otherwise without the prior permission of the author.

ww
w.E
GATE SOLVED PAPER
Electronics & Communication
Electromagnetics
asy
Copyright © By NODIA & COMPANY
En
gin
eer
ing
Information contained in this book has been obtained by authors, from sources believes to be reliable. However,
neither Nodia nor its authors guarantee the accuracy or completeness of any information herein, and Nodia nor its

.ne
authors shall be responsible for any error, omissions, or damages arising out of use of this information. This book
is published with the understanding that Nodia and its authors are supplying information but are not attempting
to render engineering or other professional services.

t
NODIA AND COMPANY
B-8, Dhanshree Tower Ist, Central Spine, Vidyadhar Nagar, Jaipur 302039
Ph : +91 - 141 - 2101150
www.nodia.co.in
email : enquiry@nodia.co.in

Downloaded From : www.EasyEngineering.net


Downloaded From : www.EasyEngineering.net

GATE SOLVED PAPER - EC


ELECTROMAGNETICS

2013 ONE MARK

Q. 1 Consider a vector field Av ^rvh. The closed loop line integral # Av : dlv can be
expressed as
(A) ## ^d # Avh : dsv over the closed surface bounded by the loop

(B) ### ^d : Avhdv over the closed volume bounded by the loop
ww (C) ### ^d : Avhdv over the open volume bounded by the loop
## ^d # Avh : dsv over the open surface bounded by the loop
w.E
Q. 2
(D)
v = xatx + yaty + zatz is
The divergence of the vector field A
(A) 0
(C) 1 asy (B) 1/3
(D) 3

Q. 3

En
The return loss of a device is found to be 20 dB. The voltage standing wave ratio
(VSWR) and magnitude of reflection coefficient are respectively
(A) 1.22 and 0.1
(C) – 1.22 and 0.1 gin (B) 0.81 and 0.1
(D) 2.44 and 0.2

eer
2013

ing TWO MARKS

Statement for Linked Answer Questions 4 and 5:


.ne
A monochromatic plane wave of wavelength l = 600 mm is propagating in the

t
direction as shown in the figure below. Evi , Evr and Evt denote incident, reflected,
and transmitted electric field vectors associated with the wave.

Q. 4 The angle of incidence qi and the expression for Evi are


p # 10 ^x + 2h
4

(A) 60c and E 0 ^atx - atz h e-j 3 2 V/m


2

Downloaded From : www.EasyEngineering.net


Downloaded From : www.EasyEngineering.net
GATE SOLVED PAPER - EC ELECTROMAGNETICS

(B) 45c and E 0 ^atx + atz h e-j 3 V/m


4
p # 10 z

2
p # 10 ^x + z h
4

(C) 45c and E 0 ^atx - atz h e-j 3 2 V/m


2
(D) 60c and E 0 ^atx - atz h e-j 3 V/m
4
p # 10 z

2
Q. 5 The expression for Evr is
p # 10 ^x - z h
4

(A) 0.23 E 0 ^atx + atz h e-j 3 2 V/m


2
(B) - E 0 ^atx + atz h e j 3 V/m
4
p # 10 z

2
p # 10 ^x - z h
4

(C) 0.44 E 0 ^atx + atz h e-j 3 2 V/m


2

ww (D) E 0 ^atx + atz h e-j


2
p # 10 ^x + z h
4

3 V/m

w.E 2012 ONE MARK

A plane wave propagating in air with E = (8ax + 6ay + 5az ) e j (wt + 3x - 4y) V/m is

asy
Q. 6
incident on a perfectly conducting slab positioned at x # 0 . The E field of the
reflected wave is

En
(A) (- 8ax - 6ay - 5az ) e j (wt + 3x + 4y) V/m
(B) (- 8ax + 6ay - 5az ) e j (wt + 3x + 4y) V/m
-

(C) (- 8ax - 6ay - 5az ) e j (wt - 3x - 4y) V/m


(D) (- 8ax + 6ay - 5az ) e j (wt - 3x - 4y) V/m
gin
Q. 7
eer
The electric field of a uniform plane electromagnetic wave in free space, along

ing
the positive x direction is given by E = 10 (ay + jaz ) e-j 25x . The frequency and
polarization of the wave, respectively, are
(A) 1.2 GHz and left circular
(C) 1.2 GHz and right circular
(B) 4 Hz and left circular

.ne
(D) 4 Hz and right circular

Q. 8

t
A coaxial-cable with an inner diameter of 1 mm and outer diameter of 2.4 mm is
filled with a dielectric of relative permittivity 10.89. Given m0 = 4p # 10-7 H/m,
-9
e0 = 10 F/m , the characteristic impedance of the cable is
36p
(A) 330 W (B) 100 W
(C) 143.3 W (D) 43.4 W

Q. 9 The radiation pattern of an antenna in spherical co-ordinates is given by F (q)


= cos 4 q ; 0 # q # p/2 . The directivity of the antenna is
(A) 10 dB (B) 12.6 dB
(C) 11.5 dB (D) 18 dB

2012 TWO MARKS

Q. 10 A transmission line with a characteristic impedance of 100 W is used to match a


50 W section to a 200 W section. If the matching is to be done both at 429 MHz
and 1 GHz, the length of the transmission line can be approximately

Downloaded From : www.EasyEngineering.net


Downloaded From : www.EasyEngineering.net
GATE SOLVED PAPER - EC ELECTROMAGNETICS

(A) 82.5 cm (b) 1.05 m


(C) 1.58 cm (D) 1.75 m

Q. 11 The magnetic field among the propagation direction inside a rectangular waveguide
with the cross-section shown in the figure is
Hz = 3 cos (2.094 # 102 x) cos (2.618 # 102 y) cos (6.283 # 1010 t - bz)

The phase velocity v p of the wave inside the waveguide satisfies


(A) v p > c (B) v p = c

ww (C) 0 < v p < c (D) v p = 0

w.E Statement for Linked Answer Question 12 and 13 :


An infinitely long uniform solid wire of radius a carries a uniform dc current of

Q. 12
density J
asy
The magnetic field at a distance r from the center of the wire is proportional to

En
(A) r for r < a and 1/r 2 for r > a (B) 0 for r < a and 1/r for r > a

Q. 13
(C) r for r < a and 1/r for r > a

gin (D) 0 for r < a and 1/r 2 for r > a

A hole of radius b (b < a) is now drilled along the length of the wire at a distance
d from the center of the wire as shown below.
eer
ing
.ne
The magnetic field inside the hole is
t
(A) uniform and depends only on d
(B) uniform and depends only on b
(C) uniform and depends on both b and d
(D) non uniform

2011 ONE MARK

Q. 14 Consider the following statements regarding the complex Poynting vector Pv for
the power radiated by a point source in an infinite homogeneous and lossless
medium. Re(Pv ) denotes the real part of Pv, S denotes a spherical surface whose
centre is at the point source, and nt denotes the unit surface normal on S . Which
of the following statements is TRUE?

Downloaded From : www.EasyEngineering.net


Downloaded From : www.EasyEngineering.net
GATE SOLVED PAPER - EC ELECTROMAGNETICS

(A) Re(Pv ) remains constant at any radial distance from the source
(B) Re(Pv ) increases with increasing radial distance from the source
(C) ## Re (Pv) : nt dS remains constant at any radial distance from the source
s

(D) ##s Re (Pv) : nt dS decreases with increasing radial distance from the source

Q. 15 A transmission line of characteristic impedance 50 W is terminated by a 50 W


load. When excited by a sinusoidal voltage source at 10 GHz, the phase difference
between two points spaced 2 mm apart on the line is found to be p/4 radians.
The phase velocity of the wave along the line is
(A) 0.8 # 108 m/s (B) 1.2 # 108 m/s
(C) 1.6 # 108 m/s (D) 3 # 108 m/s

Q. 16 The modes in a rectangular waveguide are denoted by TE mn where

ww TM mn
m and n are the eigen numbers along the larger and smaller dimensions of the
waveguide respectively. Which one of the following statements is TRUE?

w.E (A) The TM 10 mode of the waveguide does not exist


(B) The TE 10 mode of the waveguide does not exist
(C) The TM 10 and the TE 10 modes both exist and have the same cut-off
frequencies
asy
(D) The TM 10 and the TM 01 modes both exist and have the same cut-off
frequencies
En
2011
gin TWO MARKS

Q. 17

eer
A current sheet Jv = 10uty A/m lies on the dielectric interface x = 0 between two
dielectric media with er 1 = 5, mr 1 = 1 in Region-1 (x < 0) and er2 = 2, mr2 = 2 in
Region-2 (x 2 0). If the magnetic field in Region-1 at x = 0- is Hv1 = 3utx + 30uty A/m
the magnetic field in Region-2 at x = 0+ is
ing
.ne
(A) Hv2 = 1.5utx + 30uty - 10utz A/m
t
(B) Hv2 = 3utx + 30uty - 10utz A/m
(C) Hv2 = 1.5utx + 40uty A/m
(D) Hv2 = 3utx + 30uty + 10utz A/m

Q. 18 A transmission line of characteristic impedance 50 W is terminated in a load


impedance ZL . The VSWR of the line is measured as 5 and the first of the voltage
maxima in the line is observed at a distance of l/4 from the load. The value of
ZL is
(A) 10 W (B) 250 W
(C) (19.23 + j 46.15) W (D) (19.23 - j 46.15) W

Q. 19 The electric and magnetic fields for a TEM wave of frequency 14 GHz in a
homogeneous medium of relative permittivity er and relative permeability mr = 1

Downloaded From : www.EasyEngineering.net


Downloaded From : www.EasyEngineering.net
GATE SOLVED PAPER - EC ELECTROMAGNETICS

are given by Ev = E p e j (wt - 280py) utz V/m and Hv = 3e j (wt - 280py) utx A/m . Assuming the
speed of light in free space to be 3 # 108 m/s , the intrinsic impedance of free
space to be 120p , the relative permittivity er of the medium and the electric field
amplitude E p are
(A) er = 3, E p = 120p (B) er = 3, E p = 360p
(C) er = 9, E p = 360p (D) er = 9, E p = 120p

2010 ONE MARK

Q. 20 If the scattering matrix [S ] of a two port network is


0.2+0c 0.9+90c
[S ] = >
0.9+90c 0.1+90cH
, then the network is
(A) lossless and reciprocal (B) lossless but not reciprocal

ww
Q. 21
(C) not lossless but reciprocal (D) neither lossless nor reciprocal

A transmission line has a characteristic impedance of 50 W and a resistance of

w.E 0.1 W/m . If the line is distortion less, the attenuation constant(in Np/m) is
(A) 500
(C) 0.014
(B) 5
(D) 0.002

Q. 22
asy
The electric field component of a time harmonic plane EM wave traveling in a

En
nonmagnetic lossless dielectric medium has an amplitude of 1 V/m. If the relative
permittivity of the medium is 4, the magnitude of the time-average power density
vector (in W/m2 ) is
(A) 1
30p gin
(B) 1
60p
(C) 1
120p
(D) 1
240p
eer
2010
ing TWO MARKS

Q. 23 v = xyatx + x 2 aty , then


If A o
C
.ne
# Av $ dlv over the path shown in the figure is

(A) 0 (B) 2
3
(C) 1 (D) 2 3

Q. 24 A plane wave having the electric field components Evi = 24 cos ^3 # 108 - by h atx
V/m and traveling in free space is incident normally on a lossless medium with
m = m0 and e = 9e0 which occupies the region y $ 0 . The reflected magnetic field
component is given by
(A) 1 cos (3 # 108 t + y) atx A/m
10p

Downloaded From : www.EasyEngineering.net


Downloaded From : www.EasyEngineering.net
GATE SOLVED PAPER - EC ELECTROMAGNETICS

(B) 1 cos (3 108 t + y) at A/m


20p # x

(C) - 1 cos (3 # 108 t + y) atx A/m


20p
(D) - 1 cos (3 # 108 t + y) atx A/m
10p
Q. 25 In the circuit shown, all the transmission line sections are lossless. The Voltage
Standing Wave Ration(VSWR) on the 60 W line is

ww
w.E (A) 1.00
(B) 1.64
(C) 2.50
(D) 3.00 asy
2009 En ONE MARK

Q. 26
gin
Two infinitely long wires carrying current are as shown in the figure below. One
wire is in the y - z plane and parallel to the y - axis. The other wire is in the

eer
x - y plane and parallel to the x - axis. Which components of the resulting
magnetic field are non-zero at the origin ?

ing
.ne
t
(A) x, y, z components (B) x, y components
(C) y, z components (D) x, z components

Q. 27 Which of the following statements is true regarding the fundamental mode of the
metallic waveguides shown ?

(A) Only P has no cutoff-frequency

Downloaded From : www.EasyEngineering.net


Downloaded From : www.EasyEngineering.net
GATE SOLVED PAPER - EC ELECTROMAGNETICS

(B) Only Q has no cutoff-frequency


(C) Only R has no cutoff-frequency
(D) All three have cutoff-frequencies

2009 TWO MARKS

Q. 28 If a vector field V is related to another vector field A through V = 4# A , which


of the following is true? (Note : C and SC refer to any closed contour and any
surface whose boundary is C . )
(A) # V $ dl = # # A $ d S (B) # A $ dl = # # V $ d S
C S C C S C

(C) #C D # V $ dl = #S #C D # A $ d S (D) #C D # V $ dl = #S #CV $ d S


A transmission line terminates in two branches, each of length l , as shown.

ww
Q. 29
4
The branches are terminated by 50W loads. The lines are lossless and have the
characteristic impedances shown. Determine the impedance Zi as seen by the

w.E source.

asy
En
(A) 200W gin
(B) 100W

Q. 30
(C) 50W

A magnetic field in air is measured to be eer


(D) 25W

B = B0 c 2 x 2 yt - 2
x +y
y
x + y2
xt m
ing
What current distribution leads to this field ?
.ne
[Hint : The algebra is trivial in cylindrical coordinates.]
(A) J = B0 z c 2 1 2 m, r ! 0
t
m0 x + y
(C) J = 0, r ! 0
t

t
(B) J =- B0 z c 2 2 2 m, r ! 0
m0 x + y
(D) J = 0 z c 2 1 2 m, r ! 0
B t
m0 x + y

2008 ONE MARK

Q. 31 For a Hertz dipole antenna, the half power beam width (HPBW) in the E -plane
is
(A) 360c (B) 180c
(C) 90c (D) 45c

Q. 32 For static electric and magnetic fields in an inhomogeneous source-free medium,


which of the following represents the correct form of Maxwell’s equations ?
(A) 4$ E = 0 , 4# B = 0 (B) 4$ E = 0 , 4$ B = 0
(C) 4# E = 0 , 4# B = 0 (D) 4# E = 0 , 4$ B = 0

Downloaded From : www.EasyEngineering.net


Downloaded From : www.EasyEngineering.net
GATE SOLVED PAPER - EC ELECTROMAGNETICS

2008 TWO MARKS

Q. 33 A rectangular waveguide of internal dimensions (a = 4 cm and b = 3 cm) is to be


operated in TE11 mode. The minimum operating frequency is
(A) 6.25 GHz (B) 6.0 GHz
(C) 5.0 GHz (D) 3.75 GHz

Q. 34 One end of a loss-less transmission line having the characteristic impedance of


75W and length of 1 cm is short-circuited. At 3 GHz, the input impedance at the
other end of transmission line is
(A) 0 (B) Resistive
(C) Capacitive (D) Inductive

Q. 35 A uniform plane wave in the free space is normally incident on an infinitely


thick dielectric slab (dielectric constant e = 9 ). The magnitude of the reflection

ww coefficient is
(A) 0 (B) 0.3

w.E
Q. 36
(C) 0.5 (D) 0.8

In the design of a single mode step index optical fibre close to upper cut-off, the
single-mode operation is not preserved if

asy
(A) radius as well as operating wavelength are halved
(B) radius as well as operating wavelength are doubled

En
(C) radius is halved and operating wavelength is doubled

Q. 37 gin
(D) radius is doubled and operating wavelength is halved

At 20 GHz, the gain of a parabolic dish antenna of 1 meter and 70% efficiency is
(A) 15 dB
(C) 35 dB
(B) 25 dB
(D) 45 dB eer
2007
ing ONE MARK

Q. 38
.ne
A plane wave of wavelength l is traveling in a direction making an angle 30c with

(A) E = yE
3p p
t 0 e j c wt - l x - l z m
3 p x+ p z
pt
positive x - axis and 90c with positive y - axis. The E field of the plane wave can
be represented as (E0 is constant)
3p
t 0 e jc wt - l x - l z m
(B) E = yE
p 3pz
t 0 e jc wt +
(C) E = yE l l m t 0 e jc wt - l x +
(D) E = yE l m

Q. 39 If C is code curve enclosing a surface S , then magnetic field intensity H , the


current density j and the electric flux density D are related by
(A) ## H $ ds = ## c j + 2D m $ d t
S c 2t
(B) #S H $ d l = ##S c j + 22Dt m $ dS
(C) ##S H $ dS = #C c j + 22Dt m $ d t
(D) #C H $ d l # = ##S c j + 22Dt m $ ds
c

Downloaded From : www.EasyEngineering.net


Downloaded From : www.EasyEngineering.net
GATE SOLVED PAPER - EC ELECTROMAGNETICS

2007 TWO MARKS

Q. 40 The E field in a rectangular waveguide of inner dimension a # b is given by


wm
E = 2 ` l j H0 sin ` 2px j sin (wt - bz) yt
2

h 2 a
Where H0 is a constant, and a and b are the dimensions along the x - axis and
the y - axis respectively. The mode of propagation in the waveguide is
(A) TE20 (B) TM11
(C) TM20 (D) TE10

Q. 41 A load of 50 W is connected in shunt in a 2-wire transmission line of Z0 = 50W


as shown in the figure. The 2-port scattering parameter matrix (s-matrix) of the
shunt element is

ww
w.E -1
(A) > 12
1
1H
2 0 1
(B) = G
2 -2

- 13
(C) > 2
2
1H
asy 1 0
1
(D) > 43
- 43
H
En
3
3 -3 -4 1
4

Q. 42

gin
The parallel branches of a 2-wirw transmission line re terminated in 100W and
200W resistors as shown in the figure. The characteristic impedance of the line is
Z0 = 50W and each section has a length of l . The voltage reflection coefficient G
at the input is
4
eer
ing
.ne
t
(A) - j 7 (B) - 5
5 7
(C) j 5 (D) 5
7 7
Q. 43 The H field (in A/m) of a plane wave propagating in free space is given by
H = xt 5 3 cos (wt - bz) + yt` wt - bz + p j .
h0 2
The time average power flow density in Watts is
h
(A) 0 (B) 100
100 h0
(C) 50h20 (D) 50
h0

Downloaded From : www.EasyEngineering.net


Downloaded From : www.EasyEngineering.net
GATE SOLVED PAPER - EC ELECTROMAGNETICS

Q. 44 An air-filled rectangular waveguide has inner dimensions of 3 cm # 2 cm. The


wave impedance of the TE20 mode of propagation in the waveguide at a frequency
of 30 GHz is (free space impedance h0 = 377 W )
(A) 308 W (B) 355 W
(C) 400 W (D) 461 W

Q. 45 A l2 dipole is kept horizontally at a height of l2 above a perfectly conducting


0

infinite ground plane. The radiation pattern in the lane of the dipole (E plane)
looks approximately as

ww
w.E
Q. 46
asy
A right circularly polarized (RCP) plane wave is incident at an angle 60c to the
normal, on an air-dielectric interface. If the reflected wave is linearly polarized,

En
the relative dielectric constant xr2 is.

gin
eer
(A) 2 (B) 3
ing
(C) 2 (D) 3

.ne
Q. 47
2006
t ONE MARK

The electric field of an electromagnetic wave propagation in the positive direction


is given by E = atx sin (wt - bz) + aty sin (wt - bz + p/2). The wave is
(A) Linearly polarized in the z -direction
(B) Elliptically polarized
(C) Left-hand circularly polarized
(D) Right-hand circularly polarized

Q. 48 A transmission line is feeding 1 watt of power to a horn antenna having a gain of


10 dB. The antenna is matched to the transmission line. The total power radiated
by the horn antenna into the free space is
(A) 10 Watts (B) 1 Watts
(C) 0.1 Watts (D) 0.01 Watt

Downloaded From : www.EasyEngineering.net


Downloaded From : www.EasyEngineering.net
GATE SOLVED PAPER - EC ELECTROMAGNETICS

2006 TWO MARKS

Q. 49 When a planes wave traveling in free-space is incident normally on a medium


having the fraction of power transmitted into the medium is given by
(A) 8 (B) 1
9 2
(C) 1 (D) 5
3 6
Q. 50 A medium of relative permittivity er2 = 2 forms an interface with free - space.
A point source of electromagnetic energy is located in the medium at a depth of
1 meter from the interface. Due to the total internal reflection, the transmitted
beam has a circular cross-section over the interface. The area of the beam cross-
section at the interface is given by
(A) 2p m 2 (B) p2 m 2
(C) p m 2 (D) p m 2

ww
Q. 51
2

A rectangular wave guide having TE10 mode as dominant mode is having a cut

w.E off frequency 18 GHz for the mode TE30 . The inner broad - wall dimension of the
rectangular wave guide is
(A) 5/3 cm (B) 5 cm

(C) 5/2 cm
asy (D) 10 cm

Q. 52
En
A medium is divide into regions I and II about x = 0 plane, as shown in the figure
below.

gin
eer
ing
.ne
An electromagnetic wave with electric field E1 = 4atx + 3aty + 5atz is incident
normally on the interface from region I . The electric file E2 in region II at the
interface is
(A) E2 = E1
(C) 3atx + 3aty + 5atz
t
(B) 4atx + 0.75aty - 1.25atz
(D) - 3atx + 3aty + 5atz

Q. 53 A mast antenna consisting of a 50 meter long vertical conductor operates over a


perfectly conducting ground plane. It is base-fed at a frequency of 600 kHz. The
radiation resistance of the antenna is Ohms is
2 2
(A) 2p (B) p
5 5
2
(C) 4p (D) 20p2
5

2005 ONE MARK

Q. 54 The magnetic field intensity vector of a plane wave is given by


H (x, y, z, t) = 10 sin (50000t + 0.004x + 30) aty
where aty , denotes the unit vector in y direction. The wave is propagating with a

Downloaded From : www.EasyEngineering.net


Downloaded From : www.EasyEngineering.net
GATE SOLVED PAPER - EC ELECTROMAGNETICS

phase velocity.
(A) 5 # 10 4 m/s (B) - 3 # 108 m/s
(C) - 1.25 # 107 m/s (D) 3 # 108 m/s

Q. 55 Refractive index of glass is 1.5. Find the wavelength of a beam of light with
frequency of 1014 Hz in glass. Assume velocity of light is 3 # 108 m/s in vacuum
(A) 3 mm (B) 3 mm
(C) 2 mm (D) 1 mm

2005 TWO MARKS

Q. 56 Which one of the following does represent the electric field lines for the mode in
the cross-section of a hollow rectangular metallic waveguide ?

ww
w.E
asy
Q. 57
En
Characteristic impedance of a transmission line is 50 W. Input impedance of the

input impedance will be.


(A) 50 W
gin
open-circuited line when the transmission line a short circuited, then value of the

(B) 100 + j150W


(C) 7.69 + j11.54W
eer
(D) 7.69 - j11.54W

Q. 58
ing
Two identical and parallel dipole antennas are kept apart by a distance of l in
4
the H - plane. They are fed with equal currents but the right most antenna has a
phase shift of + 90c. The radiation pattern is given as.
.ne
t

Statement of Linked Answer Questions 59 & 60 :


Voltage standing wave pattern in a lossless transmission line with characteristic
impedance 50 and a resistive load is shown in the figure.

Downloaded From : www.EasyEngineering.net


Downloaded From : www.EasyEngineering.net
GATE SOLVED PAPER - EC ELECTROMAGNETICS

Q. 59 The value of the load resistance is


(A) 50 W (B) 200 W
(C) 12.5 W (D) 0

Q. 60 The reflection coefficient is given by


(A) - 0.6 (B) - 1
(C) 0.6 (D) 0

ww
Q. 61 Many circles are drawn in a Smith Chart used for transmission line calculations.
The circles shown in the figure represent

w.E
asy
En
(A) Unit circles
(B) Constant resistance circles gin
(C) Constant reactance circles
(D) Constant reflection coefficient circles. eer
2004 ing ONE MARK

Q. 62
rectangular waveguide in the TE10 mode is .ne
The phase velocity of an electromagnetic wave propagating in a hollow metallic

(A) equal to its group velocity


(B) less than the velocity of light in free space
(C) equal to the velocity of light in free space
t
(D) greater than the velocity of light in free space

Q. 63 Consider a lossless antenna with a directive gain of + 6 dB. If 1 mW of power is


fed to it the total power radiated by the antenna will be
(A) 4 mW (B) 1 mW
(C) 7 mW (D) 1/4 mW

2004 TWO MARKS

Q. 64 A parallel plate air-filled capacitor has plate area of 10 - 4 m 2 and plate separation
of 10 - 3 m. It is connect - ed to a 0.5 V, 3.6 GHz source. The magnitude of the
displacement current is ( e = 361p 10 - 9 F/m)
(A) 10 mA (B) 100 mA

Downloaded From : www.EasyEngineering.net


Downloaded From : www.EasyEngineering.net
GATE SOLVED PAPER - EC ELECTROMAGNETICS

(C) 10 A (D) 1.59 mA

Q. 65 Consider a 300 W, quarter - wave long (at 1 GHz) transmission line as shown in
Fig. It is connected to a 10 V, 50 W source at one end and is left open circuited at
the other end. The magnitude of the voltage at the open circuit end of the line is

(A) 10 V (B) 5 V
(C) 60 V (D) 60/7 V

ww
Q. 66 In a microwave test bench, why is the microwave signal amplitude modulated at
1 kHz
(A) To increase the sensitivity of measurement

w.E (B) To transmit the signal to a far-off place


(C) To study amplitude modulations

Q. 67 asy
(D) Because crystal detector fails at microwave frequencies

If E = (atx + jaty) e jkz - kwt and H = (k/wm) (aty + katx ) e jkz - jwt , the time-averaged
Poynting vector is
(A) null vector En (B) (k/wm) atz
(C) (2k/wm) atz
gin(D) (k/2wm) atz

Q. 68

eer
Consider an impedance Z = R + jX marked with point P in an impedance Smith
chart as shown in Fig. The movement from point P along a constant resistance
circle in the clockwise direction by an angle 45c is equivalent to

ing
.ne
t
(A) adding an inductance in series with Z
(B) adding a capacitance in series with Z
(C) adding an inductance in shunt across Z
(D) adding a capacitance in shunt across Z

Q. 69 A plane electromagnetic wave propagating in free space is incident normally on a


large slab of loss-less, non-magnetic, dielectric material with e > e0 . Maxima and
minima are observed when the electric field is measured in front of the slab. The
maximum electric field is found to be 5 times the minimum field. The intrinsic
impedance of the medium should be
(A) 120p W (B) 60p W
(C) 600p W (D) 24p W

Downloaded From : www.EasyEngineering.net


Downloaded From : www.EasyEngineering.net
GATE SOLVED PAPER - EC ELECTROMAGNETICS

Q. 70 A lossless transmission line is terminated in a load which reflects a part of the


incident power. The measured VSWR is 2. The percentage of the power that is
reflected back is
(A) 57.73 (B) 33.33
(C) 0.11 (D) 11.11

2003 ONE MARK

Q. 71 The unit of 4# H is
(A) Ampere (B) Ampere/meter
(C) Ampere/meter 2 (D) Ampere-meter

Q. 72 The depth of penetration of electromagnetic wave in a medium having conductivity


s at a frequency of 1 MHz is 25 cm. The depth of penetration at a frequency of

ww 4 MHz will be
(A) 6.25 dm (B) 12.50 cm

w.E (C) 50.00 cm (D) 100.00 cm

Q. 73
2003

asy TWO MARKS

Medium 1 has the electrical permittivity e1 = 1.5e0 farad/m and occupies the

En
region to the left of x = 0 plane. Medium 2 has the electrical permittivity e2 = 2.5e0
farad/m and occupies the region to the right of x = 0 plane. If E1 in medium 1 is

(A) (2.0ux - 7.5uy + 2.5uz ) volt/m


(B) (2.0ux - 2.0uy + 0.6uz ) volt/m
gin
E1 = (2ux - 3uy + 1uz ) volt/m, then E2 in medium 2 is

(C) (2.0ux - 3.0uy + 1.0uz ) volt/m


eer
Q. 74
(D) (2.0ux - 2.0uy + 0.6uz ) volt/m
ing
If the electric field intensity is given by E = (xux + yuy + zuz ) volt/m, the potential
difference between X (2, 0, 0) and Y (1, 2, 3) is
(A) + 1 volt .ne
(B) - 1 volt

Q. 75
(C) + 5 volt (D) + 6 volt
t
A uniform plane wave traveling in air is incident on the plane boundary between
air and another dielectric medium with er = 4 . The reflection coefficient for the
normal incidence, is
(A) zero (B) 0.5+180c
(B) 0.333+0c (D) 0.333+180c

Q. 76 If the electric field intensity associated with a uniform plane electromagnetic wave
traveling in a perfect dielectric medium is given by E (z, t) = 10 cos (2p107 t - 0.1pz)
V/m, then the velocity of the traveling wave is
(A) 3.00 # 108 m/sec (B) 2.00 # 108 m/sec
(C) 6.28 # 107 m/sec (D) 2.00 # 107 m/sec

Q. 77 Two identical antennas are placed in the q = p/2 plane as shown in Fig. The
elements have equal amplitude excitation with 180c polarity difference, operating
at wavelength l. The correct value of the magnitude of the far-zone resultant
electric field strength normalized with that of a single element, both computed

Downloaded From : www.EasyEngineering.net


Downloaded From : www.EasyEngineering.net
GATE SOLVED PAPER - EC ELECTROMAGNETICS

for f = 0 , is

(A) 2 cos b 2ps l (B) 2 sin b 2ps l


l l
p
(C) 2 cos a s k p
(D) 2 sin a s k
l l

Q. 78 A short - circuited stub is shunt connected to a transmission line as shown in


fig. If Z0 = 50 ohm, the admittance Y seen at the junction of the stub and the

ww transmission line is

w.E
asy
En
gin
eer
(A) (0.01 - j0.02) mho
(C) (0.04 - j0.02) mho
ing
(B) (0.02 - j0.01) mho
(D) (0.02 + j0) mho

Q. 79

.ne
A rectangular metal wave guide filled with a dielectric material of relative
permittivity er = 4 has the inside dimensions 3.0 cm # 1.2 cm. The cut-off
frequency for the dominant mode is
(A) 2.5 GHz
(C) 10.0 GHz
(B) 5.0 GHz
(D) 12.5 GHz
t
2002 ONE MARK

Q. 80 The VSWR can have any value between


(A) 0 and 1 (B) - 1 and + 1
(C) 0 and 3 (D) 1 and 3

Q. 81 In in impedance Smith movement along a constant resistance circle gives rise to


(A) a decrease in the value of reactance
(B) an increase in the value of reactance
(C) no change in the reactance value
(D) no change in the impedance

Downloaded From : www.EasyEngineering.net


Downloaded From : www.EasyEngineering.net
GATE SOLVED PAPER - EC ELECTROMAGNETICS

Q. 82 The phase velocity for the TE10 -mode in an air-filled rectangular waveguide is (
c is the velocity of plane waves in free space)
(A) less than c (B) equal to c
(C) greater than c (D) none of these

2002 TWO MARKS

Q. 83 t jp/2) e jwt - jkz . This wave is


A plane wave is characterized by E = (0.5xt + ye
(A) linearly polarized (B) circularly polarized
(C) elliptically polarized (D) unpolarized

Q. 84 Distilled water at 25c C is characterized by s = 1.7 # 10 - 4 mho/m and e = 78eo


at a frequency of 3 GHz. Its loss tangent tan d is
( e = 10
36p F/m)
-9

ww (A) 1.3 # 10-5


(C) 1.3 # 10-4 /78
(B) 1.3 # 10-3
(D) 1.3 # 10-5 /78e0

w.E
Q. 85 The electric field on the surface of a perfect conductor is 2 V/m. The conductor
is immersed in water with e = 80eo . The surface charge density on the conductor
is ( e = 10
36p F/m)
(A) 0 C/m 2
-9

asy (B) 2 C/m 2


(C) 1.8 # 10
En
- 11
C/m 2 (D) 1.41 # 10 - 9 C/m 2

Q. 86

gin
A person with receiver is 5 Km away from the transmitter. What is the distance
that this person must move further to detect a 3-dB decrease in signal strength
(A) 942 m (B) 2070 m
(C) 4978 m
eer
(D) 5320 m

2001 ing ONE MARK

Q. 87 A transmission line is distortonless if


(A) RL = 1 (B) RL = GC .ne
Q. 88
GC
(C) LG = RC (D) RG = LC
2
t2
If a plane electromagnetic wave satisfies the equal d E2x = c2 d E2x , the wave
propagates in the dZ dt
(A) x - direction
(B) z - direction
(C) y - direction
(D) xy plane at an angle of 45c between the x and z direction

Q. 89 The plane velocity of wave propagating in a hollow metal waveguide is


(A) grater than the velocity of light in free space
(B) less than the velocity of light in free space
(C) equal to the velocity of light free space
(D) equal to the velocity of light in free

Q. 90 The dominant mode in a rectangular waveguide is TE10 , because this mode has

Downloaded From : www.EasyEngineering.net


Downloaded From : www.EasyEngineering.net
GATE SOLVED PAPER - EC ELECTROMAGNETICS

(A) the highest cut-off wavelength


(B) no cut-off
(C) no magnetic field component
(D) no attenuation

2001 TWO MARKS

Q. 91 A material has conductivity of 10 - 2 mho/m and a relative permittivity of 4.


The frequency at which the conduction current in the medium is equal to the
displacement current is
(A) 45 MHz (B) 90 MHz
(C) 450 MHz (D) 900 MHz

ww
Q. 92 A uniform plane electromagnetic wave incident on a plane surface of a dielectric
material is reflected with a VSWR of 3. What is the percentage of incident power
that is reflected ?

w.E (A) 10%


(C) 50%
(B) 25%
(D) 75%

Q. 93

asy
A medium wave radio transmitter operating at a wavelength of 492 m has a tower
antenna of height 124. What is the radiation resistance of the antenna?
(A) 25 W
(C) 50 W
En (B) 36.5 W
(D) 73 W

Q. 94
gin
In uniform linear array, four isotropic radiating elements are spaced l apart. The
4
progressive phase shift between required for forming the main beam at 60c off the
end - fire is :
(A) - p eer
(B) - p2 radians
(C) - p
4 radians (D) - p
8
ing
radians

2000
.ne ONE MARK

Q. 95

t
The magnitudes of the open-circuit and short-circuit input impedances of a
transmission line are 100 W and 25 W respectively. The characteristic impedance
of the line is,
(A) 25 W (B) 50 W
(C) 75 W (D) 100 W

Q. 96 A TEM wave is incident normally upon a perfect conductor. The E and H field
at the boundary will be respectively,
(A) minimum and minimum (B) maximum and maximum
(C) minimum and maximum (D) maximum and minimum

Q. 97 If the diameter of a l dipole antenna is increased from l to l , then its


2 100 50
(A) bandwidth increases (B) bandwidth decrease
(C) gain increases (D) gain decreases

Downloaded From : www.EasyEngineering.net


Downloaded From : www.EasyEngineering.net
GATE SOLVED PAPER - EC ELECTROMAGNETICS

2000 TWO MARKS

Q. 98 A uniform plane wave in air impings at 45c angle on a lossless dielectric material
with dielectric constant dr . The transmitted wave propagates is a 30c direction
with respect to the normal. The value of dr is
(A) 1.5 (B) 1.5
(C) 2 (D) 2

Q. 99 A rectangular waveguide has dimensions 1 cm # 0.5 cm. Its cut-off frequency is


(A) 5 GHz (B) 10 GHz
(C) 15 GHz (D) 12 GHz

Q. 100 Two coaxial cable 1 and 2 are filled with different dielectric constants er1 and er2
respectively. The ratio of the wavelength in the cables (l1 /l2) is
(A) er1 /er2 (B) er2 /er1

ww
Q. 101
(C) er1 /er2 (D) er2 /er1

For an 8 feet (2.4m) parabolic dish antenna operating at 4 GHz, the minimum

w.E distance required for far field measurement is closest to


(A) 7.5 cm (B) 15 cm
(C) 15 m
asy (D) 150 m

Q. 102
1999

En
An electric field on a place is described by its potential
ONE MARK

gin
V = 20 (r-1 + r-2)
where r is the distance from the source. The field is due to
(A) a monopole
(C) both a monopole and a dipole eer
(B) a dipole
(D) a quadruple

Q. 103
ing
Assuming perfect conductors of a transmission line, pure TEM propagation is
NOT possible in
(A) coaxial cable
.ne
(B) air-filled cylindrical waveguide

Q. 104
(C) parallel twin-wire line in air
(D) semi-infinite parallel plate wave guide
t
Indicate which one of the following will NOT exist in a rectangular resonant
cavity.
(A) TE110 (B) TE 011
(C) TM110 (D) TM111

Q. 105 Identify which one of the following will NOT satisfy the wave equation.
(A) 50e j (wt - 3z) (B) sin [w (10z + 5t)]
2
(C) cos (y + 5t) (D) sin (x) cos (t)

1999 TWO MARKS

Q. 106 In a twin-wire transmission line in air, the adjacent voltage maxima are at
12.5 cm and 27.5 cm . The operating frequency is
(A) 300 MHz (B) 1 GHz

Downloaded From : www.EasyEngineering.net


Downloaded From : www.EasyEngineering.net
GATE SOLVED PAPER - EC ELECTROMAGNETICS

(C) 2 GHz (D) 6.28 GHz

Q. 107 A transmitting antenna radiates 251 W isotropically. A receiving antenna, located


100 m away from the transmitting antenna, has an effective aperture of 500 cm2 .
The total received by the antenna is
(A) 10 mW (B) 1 mW
(C) 20 mW (D) 100 mW

Q. 108 In air, a lossless transmission line of length 50 cm with L = 10 mH/m , C = 40 pF/m


is operated at 25 MHz . Its electrical path length is
(A) 0.5 meters (B) l meters
(C) p/2 radians (D) 180 deg rees

Q. 109 A plane wave propagating through a medium [er = 8, mr = 2, and s = 0] has its
t - (z/3) sin (108 t - bz) V/m . The wave impedance, in
electric field given by Ev = 0.5Xe

ww ohms is
(A) 377 (B) 198.5+180c

w.E (C) 182.9+14c (D) 133.3

Q. 110
1998

asy
The intrinsic impedance of copper at high frequencies is
ONE MARK

(A) purely resistive


(B) purely inductiveEn
(C) complex with a capacitive component
(D) complex with an inductive component gin
Q. 111 The Maxwell equation V # H = J + 2D is based on
2t eer
(A) Ampere’s law
(C) Faraday’s law
(B) Gauss’ law
ing
(D) Coulomb’s law

Q. 112
.ne
All transmission line sections shown in the figure is have a characteristic impedance
R 0 + j 0 . The input impedance Zin equals
t

(A) 2 R 0 (B) R 0
3
(C) 3 R 0 (D) 2R 0
2

Downloaded From : www.EasyEngineering.net


Downloaded From : www.EasyEngineering.net
GATE SOLVED PAPER - EC ELECTROMAGNETICS

1998 TWO MARKS

Q. 113 The time averages Poynting vector, in W/m2 , for a wave with Ev = 24e j (wt + bz) avy V/m
in free space is
(A) - 2.4 avz (B) 2.4 avz
p p
(C) 4.8 avz (D) - 4.8 avz
p p
Q. 114 The wavelength of a wave with propagation constant (0.1p + j0.2p) m-1 is
(A) 2 m (B) 10 m
0.05
(C) 20 m (D) 30 m

Q. 115 The depth of penetration of wave in a lossy dielectric increases with increasing
(A) conductivity

ww (B) permeability
(C) wavelength

w.E
Q. 116
(D) permittivity

The polarization of wave with electric field vector Ev = E 0 e j^wt + bz h ^avx + avy h is
(A) linear

asy
(C) left hand circular
(B) elliptical
(D) right hand circular

Q. 117

En
The vector H in the far field of an antenna satisfies
(A) d $ Hv = 0 and d # Hv = 0
(B) d $ Hv ! 0 and d # Hv ! 0
(C) d $ Hv = 0 and d # Hv ! 0 gin
(D) d $ Hv ! 0 and d # Hv = 0
eer
Q. 118
resistance of five turns of such a loop will be ing
The radiation resistance of a circular loop of one turn is 0.01 W. The radiation

(A) 0.002 W
(C) 0.05 W
(B) 0.01 W
(D) 0.25 W
.ne
Q. 119 An antenna in free space receives 2 mW
is 20 mV/m rms. The effective aperture
(A) 0.005 m2
of the antenna is
(B) 0.05 m2
t
of power when the incident electric field

(C) 1.885 m2 (D) 3.77 m2

Q. 120 The maximum usable frequency of an ionospheric layer at 60c incidence and with
8 MHz critical frequency is
(A) 16 MHz (B) 16 MHz
3
(C) 8 MHz (D) 6.93 MHz

Q. 121 A loop is rotating about they y -axis in a magnetic field Bv = B 0 cos (wt + f) avx T.
The voltage in the loop is
(A) zero
(B) due to rotation only
(C) due to transformer action only
(D) due to both rotation and transformer action

Downloaded From : www.EasyEngineering.net


Downloaded From : www.EasyEngineering.net
GATE SOLVED PAPER - EC ELECTROMAGNETICS

Q. 122 The far field of an antenna varies with distance r as


(A) 1 (B) 12
r r
(C) 13 (D) 1
r r

1997 ONE MARK

Q. 123 A transmission line of 50 W characteristic impedance is terminated with a 100 W


resistance. The minimum impedance measured on the line is equal to
(A) 0 W (B) 25 W
(C) 50 W (D) 100 W

Q. 124 A rectangular air filled waveguide has cross section of 4 cm #10 cm . The minimum
frequency which can propagate in the waveguide is

ww (A) 0.75 GHz


(C) 2.5 GHz
(B) 2.0 GHz
(D) 3.0 GHz

w.E 1997 TWO MARKS

Q. 125

asy
A very lossy, l/4 long, 50 W transmission line is open circuited at the load end.
The input impedance measured at the other end of the line is approximately
(A) 0
(C) 3
En (B) 50 W
(D) None of the above

Q. 126
gin
The skin depth at 10 MHz for a conductor is 1 cm. The phase velocity of an
electromagnetic wave in the conductor at 1, 000 MHz is about
(A) 6 # 106 m/ sec
(B) 6 # 107 m/ sec eer
(C) 3 # 108 m/ sec
(D) 6 # 108 m/ sec ing
.ne
Q. 127
1996

t ONE MARK

A lossless transmission line having 50 W characteristic impedance and length l/4


is short circuited at one end and connected to an ideal voltage source of 1 V at
the other end. The current drawn from the voltage source is
(A) 0 (B) 0.02 A
(C) 3 (D) none of these

Q. 128 The capacitance per unit length and the characteristic impedance of a lossless
transmission line are C and Z 0 respectively. The velocity of a travelling wave on
the transmission line is
(A) Z 0 C (B) 1
Z0 C
(C) Z 0 (D) C
C Z0

Downloaded From : www.EasyEngineering.net


Downloaded From : www.EasyEngineering.net
GATE SOLVED PAPER - EC ELECTROMAGNETICS

1996 TWO MARKS

Q. 129 A uniform plane wave in air is normally incident on infinitely thick slab. If the
refractive index of the glass slab is 1.5, then the percentage of incident power that
is reflected from the air-glass interface is
(A) 0% (B) 4%
(C) 20% (D) 100%

Q. 130 The critical frequency of an ionospheric layer is 10 MHz. What is the maximum
launching angle from the horizon for which 20 MHz wave will be reflected by the
layer ?
(A) 0c (B) 30c
(C) 45c (D) 90c

Q. 131 Some unknown material has a conductivity of 106 mho/m and a permeability of

ww 4p # 10-7 H/m . The skin depth for the material at 1 GHz is


(A) 15.9 mm (B) 20.9 mm

w.E (C) 25.9 mm

***********
(D) 30.9 mm

asy
En
gin
eer
ing
.ne
t

Downloaded From : www.EasyEngineering.net


Downloaded From : www.EasyEngineering.net
GATE SOLVED PAPER - EC ELECTROMAGNETICS

SOLUTION

Sol. 1 Option (D) is correct.


Stoke’s theorem states that the circulation a vector field A v around a closed path
v over the open surface S bounded
l is equal to the surface integral of the curl of A
by l .
i.e., # Av : dlv = ## ^d # Avh : dsv
Here, line integral is taken across a closed path which is denoted by a small
circle on the integral notation where as, the surface integral of ^d # A
vh is taken

ww
Sol. 2
over open surface bounded by the loop.
Option (D) is correct.

w.E Given, the vector field

so,
v = xavx + yavy + zavz
A

d$A asy v ) = 2Ax + 2Ay + 2Az


v (Divergence of A
2x 2y 2z

En = 1+1+1 = 3
Sol. 3 Option (A) is correct.
Given, the return loss of device as 20 dB
i.e., G in dB =- 20 dB (loss) gin
or,
^ h

20 log G =- 20
eer
& G = 10-1 = 0.1
Therefore, the standing wave ration is given by ing
VSWR =
1+ G
1- G
.ne
Sol. 4 Option (C) is correct.
= 1 + 0.1 = 1.1 = 1.22
1 - 0.1 0.9
t
For the given incidence of plane wave, we have the transmitting angle
qt = 19.2c
From Snell’s law, we know
n1 sin qi = n2 sin qt
c m1 e1 sin qi = c m2 e2 sin qt ...(1)
For the given interfaces, we have
m1 = m2 = 1
e1 = 1, e2 = 4.5
So, from Eq. (1)
sin qi = 4.5 sin 19.2
or, qi . 45c
Now, the component of Evi can be obtained as
Evi = _Eox avx - Eoz avz i e-jbk (observed from the shown figure)
Since, the angle qi = 45c so,

Downloaded From : www.EasyEngineering.net


Downloaded From : www.EasyEngineering.net
GATE SOLVED PAPER - EC ELECTROMAGNETICS

Eox = Eoz = Eo
2
_ax - avz i e-jbk
Therefore, v
Ei = Eo v ...(1)
2
Now, the wavelength of EM wave is
l = 600 mm

So, b = 2p = p # 10 4
l 3
Also, direction of propagation is
v v
avk = ax + az
2
So, k = x+z
2
Substituting it in equation (1), we get
p # 10 ^x + z h

ww
4

Evi = Eo _avx - avz i e-j 3 2


2
Sol. 5 Option (A) is correct.

w.E We obtain the reflection coefficient for parallel polarized wave (since, electric field
is in the plane of wave propagation) as
h cos qt - h1 cos qi

asy Gz = 2
h2 cos qt + h1 cos qi
As we have already obtained
...(1)

En qi = 45c, qt = 19.2c
m 1 = h0
Also,

and
h2 =

h1 =
e
m
e
= h0

= h0 1
1
gin
4. 5
= h0
4. 5

Substituting these in eq. (1) we get


G z = cos 19.2c - 4.5 cos 45c
eer
cos 19.2c + 4.5 cos 45c
=- 0.227 ing
.- 0.23
Therefore, the reflected field has the magnitude given by .ne
or
Ero
Eio
= T 11'
Ero = G z Eio =- 0.23 Eio
Hence, the expression of reflected electric field is
t
Evr =- 0.23 Eo _- avx - avz i e-j 3 k
4
p # 10
(2)
2
Again, we have the propagation vector of reflected wave as
v v
avk = ax - az
2
or, k = x - z
2
Substituting it in Eq. (2), we get
Evr =- 0.23 Eo _- avx - avz i e-j 3 b 2 l
4
p # 10 x - z

2
jp # 10 ^x - z h V
4

Evr = 0.23 o _avx + avz i e- 3 2


E m
2
Sol. 6 Option (C) is correct.
Electric field of the propagating wave in free space is given as

Downloaded From : www.EasyEngineering.net


Downloaded From : www.EasyEngineering.net
GATE SOLVED PAPER - EC ELECTROMAGNETICS

Ei = (8ax + 6ay + 5az ) e j (wt + 3x - 4y) V/m


So, it is clear that wave is propagating in the direction (- 3ax + 4ay).
Since, the wave is incident on a perfectly conducting slab at x = 0 . So, the
reflection coefficient will be equal to - 1.
i.e. Er = (- 1) Ei =- 8ax - 6ay - 5az
0 0

Again, the reflected wave will be as shown in figure.

ww i.e. the reflected wave will be in direction 3ax + 4ay . Thus, the electric field of the
reflected wave will be.

w.E
Sol. 7
Ex = (- 8ax - 6ay - 5az ) e j (wt - 3x - 4y) V/m
Option (A) is correct.
The field in circular polarization is found to be

asy
Es = E 0 (ay ! jaz ) e-jbx propagating in + ve x -direction.
where, plus sign is used for left circular polarization and minus sign for right

En
circular polarization. So, the given problem has left circular polarization.
b = 25 = w

2pf
c
gin 8
& f = 25 # c = 25 # 3 # 10 = 1.2 GHz

Sol. 8
25 =

Option (B) is correct.


c 2p 2 # 3.14
eer
Let b " outer diameter
a " inner diameter
ing
Characteristic impedance,
.ne
Sol. 9
Z0 =
m0
e0 er
ln b b l =

Option (A) is correct.


a

The directivity is defined as


4p # 10-7 # 36p ln 2.4 = 100 W
10-9 # 10.89 b 1 l
t
D = Fmax
Favg
Fmax = 1
2p 2p
Favg = 1 # F (q, f) dW = 1 ;# # F (q, f) sin qdq dfE
4p 4p 0 0
2p p/2
= 1 ;# # cos q sin qdqdfE
4
4p 0 0
5 p/2
= 1 ;2p b- cos q lE = 1 # 2p :- 0 + 1 D
4p 5 0
4p 5
= 1 # 2p = 1
4p 5 10
D = 1 = 10
10

Downloaded From : www.EasyEngineering.net


Downloaded From : www.EasyEngineering.net
GATE SOLVED PAPER - EC ELECTROMAGNETICS

or, D (in dB) = 10 log 10 = 10 dB


Sol. 10 Option (C) is correct.
Since Z0 = Z1 Z 2
100 = 50 # 200
This is quarter wave matching. The length would be odd multiple of l/4 .
l = (2m + 1) l
4

f1 = 429 MHz, l1 =c = 3 # 108 = 0.174 m


f1 # 4 429 # 106 # 4
8
f2 = 1 GHz , l2 = c = 3 # 10 = 0.075 m
f2 # 4 1 # 10 # 49

Only option (C) is odd multiple of both l1 and l2 .


(2m + 1) = 1.58 = 9

ww 1
l1
(2m + 1) = 58 - 21
.
l2

w.E
Sol. 11 Option (D) is correct.
Hz = 3 cos (2.094 # 102 x) cos (2.618 # 102 y) cos (6.283 # 1010 t - bz)

asy bx = 2.094 # 102


by = 2.618 # 102

En
w = 6.283 # 1010 rad/s
For the wave propagation,

b =
c2
x
gin
w 2 - (b 2 + b 2)
y

Substituting above values,

b = c
3 # 108 m
eer
6.283 # 1010 2 - (2.0942 + 2.6182) 10 4 - j261

ing
#
b is imaginary so mode of operation is non-propagating.

Sol. 12
vp = 0
Option (C ) is correct. .ne
For r > a , Ienclosed = (pa2) J
# H : dl = Ienclosed t
H # 2pr = (pa2) J
H = Io Io = (pa2) J
2pr
H \ 1 , for r > a
r
J (pr 2) Jr 2
For r < a , Ienclosed = = 2
pa 2 a
So, # H : dl= Ienclosed
2
H # 2pr = Jr2
a
H = Jr 2
2pa
H \ r , for r < a

Downloaded From : www.EasyEngineering.net


Downloaded From : www.EasyEngineering.net
GATE SOLVED PAPER - EC ELECTROMAGNETICS

Sol. 13 Option (A) is correct.


Assuming the cross section of the wire on x -y plane as shown in figure.

ww
w.E Since, the hole is drilled along the length of wire. So, it can be assumed that the

asy
drilled portion carriers current density of - J .
Now, for the wire without hole, magnetic field intensity at point P will be given
as

En
Hf1 (2pR) = J (pR2)
Hf1 (2pR) = JR
2
gin
Since, point o is at origin. So, in vector form
H1 = J (xax + yay)
2
eer
(Hf2) (2pr) =- J (pr 2)
Hf2 = - Jr
ing
Again only due to the hole magnetic field intensity will be given as.

2
Again, if we take Ol at origin then in vector form .ne
H2 = - J (xlax + ylay)
2 t
where xl and yl denotes point ‘P ’ in the new co-ordinate system.
Now the relation between two co-ordinate system will be.
x = xl + d
y = yl
So, H2 = - J [(x - d) ax + yay]
2
So, total magnetic field intensity = H1 + H2 = J dax
2
So, magnetic field inside the hole will depend only on ‘d ’.
Sol. 14 Option (C) is correct.
Power radiated from any source is constant.
Sol. 15 Option (C) is correct.
We have d = 2 mm and f = 10 GHz

Downloaded From : www.EasyEngineering.net


Downloaded From : www.EasyEngineering.net
GATE SOLVED PAPER - EC ELECTROMAGNETICS

Phase difference = 2p d = p ;
l 4
or = l = 8d = 8 # 2 mm = 16 mm
v = fl = 10 # 109 # 16 # 10-3
= 1.6 # 108 m/ sec
Sol. 16 Option (A) is correct.
TM11 is the lowest order mode of all the TMmn modes.
Sol. 17 Option (A) is correct.
From boundary condition
Bn1 = Bn2
m1 Hx1 = m2 Hx2
or Hx2 = Hx1 = 1.5
2

ww or
Further if
Hx2 = 1.5utx
H z = 1.5utx + Auty + Buz

w.E Then from Boundary condition


10ut
(3utx + 30uty) utx = (1.5utx + Auty + Butz ) xt + v y

asy =- 30utz =- Autz + Buty + 10uty


Comparing we get A = 30 and B =- 10
J

So
En H 2 = 1.5utx + 30uty - 10utz A/m
Sol. 18 Option (A) is correct.

gin
Since voltage maxima is observed at a distance of l/4 from the load and we know
that the separation between one maxima and minima equals to l/4 so voltage

eer
minima will be observed at the load, Therefore load can not be complex it must
be pure resistive.
Now G = s-1
s+1 ing
s
RL = 50 = 10 W
.ne
also RL = R 0 (since voltage maxima is formed at the load)

Sol. 19 Option (D) is correct.


5

From the expressions of Ev & Hv , we can write,


t
b = 280 p
or 2 p = 280 p & l = 1
l 140
v E
Wave impedance, Zw = E = p = 120 p
Hv 3 er
again, f = 14 GHz
8
Now l = C = 3 # 10 9 = 3
er f er 14 # 10 140 er
or 3 = 1
140 er 140
or er = 9
Ep
Now = 120p = E p = 120p
3 9

Downloaded From : www.EasyEngineering.net


Downloaded From : www.EasyEngineering.net
GATE SOLVED PAPER - EC ELECTROMAGNETICS

Sol. 20 Option (C) is correct.


For a lossless network
S11 2 + S21 2 = 1
For the given scattering matrix
S11 = 0.2 0c , S12 = 0.9 90c
S21 = 0.9 90c , S22 = 0.1 90c
Here, (0.2) 2 + (0.9) 2 ! 1 (not lossless)
Reciprocity :
S12 = S21 = 0.9 90c (Reciprocal)
Sol. 21 Option (D) is correct.
For distortion less transmission line characteristics impedance

ww Attenuation constant
Z0 = R
G

w.E So,
a = RG
a = R = 0.1 = 0.002
Z0 50
Sol. 22

asy
Option (C) is correct.
Intrinsic impedance of EM wave
m m0
= 120p = 60p
En
h =
Time average power density
e
=
4e0 2

2 2 h
2
Pav = 1 EH = 1 E = 1
gin
= 1
2 # 60p 120p
Sol. 23 Option (C) is correct.
Av = xyatx + x 2 aty eer
v = dxatx + dyaty
dl
# Av : dl
ing
v = # (xyatx + x 2 aty) : (dxatx + dyaty) = # (xydx + x 2 dy)
C

=
C

#1/
2/ 3
xdx +
1/ 3
#2/ 3xdx + .ne
#1
3 4 dy +
C

#3
1 1 dy
3 3

t 3
= 1 : 4 - 1 D + 3 :1 - 4 D + 4 [3 - 1] + 1 [1 - 3] = 1
2 3 3 2 3 3 3 3
3

Sol. 24 Option (A) is correct.


In the given problem

Reflection coefficient
h2 - h 1
t = = 400p - 120p =- 1
h2 + h 1 40p + 120p 2
t is negative So magnetic field component does not change its direction Direction

Downloaded From : www.EasyEngineering.net


Downloaded From : www.EasyEngineering.net
GATE SOLVED PAPER - EC ELECTROMAGNETICS

of incident magnetic field


atE # atH = atK
atZ # atH = aty
atH = atx ( + x direction)
So, reflection magnetic field component
Hr = t # 24 cos (3 # 108 + by) atx , y $ 0
h
= 1 # 24 cos (3 # 108 + by) atx , y $ 0
2 # 120p
8
b = w = 3 # 108 = 1
vC 3 # 10
So, Hr = 1 cos (3 # 108 t + y) atx , y $ 0
10p

ww
Sol. 25 Option (B) is correct.
For length of l/4 transmission line
Z + jZo tan bl

w.E ZL = 30 W ,
Z in = Zo ; L
Zo + jZL tan bl E
Zo = 30 W, b = 2p , l = l
l 4
So,
asy tan bl = tan b 2
l
p l
: l=3
4

En R ZL V
S tan bl + jZo W
Z in = Zo S
2
W = Z 0 = 60 W

T
For length of l/8 transmission line
gin
S Zo + jZL W ZL
S tan bl W
X

Z + jZo tan bl
Z in = Zo ; L
Zo + jZL tan bl E eer
Zo = 30 W, ZL = 0 (short)
tan bl = tan b 2p : l l = 1
ing
l 8
Z in = jZo tan bl = 30j .ne
Circuit is shown below.
t

Reflection coefficient
60 + 3j - 60
t = ZL - Zo = = 1
ZL + Zo 60 + 3j + 60 17
1+ t
VSWR = = 1 + 17 = 1.64
1- t 1 - 17
Sol. 26 Option (D) is correct.

Downloaded From : www.EasyEngineering.net


Downloaded From : www.EasyEngineering.net
GATE SOLVED PAPER - EC ELECTROMAGNETICS

Due to 1 A current wire in x - y plane, magnetic field be at origin will be in x


direction.
Due to 1 A current wire in y - z plane, magnetic field be at origin will be in z
direction.
Thus x and z component is non-zero at origin.
Sol. 27 Option (A) is correct.
Rectangular and cylindrical waveguide doesn’t support TEM modes and have cut
off frequency.
Coaxial cable support TEM wave and doesn’t have cut off frequency.
Sol. 28 Option (B) is correct.
We have V = 4# A ...(1)
By Stokes theorem
# A $ dl
= ## (4 # A) $ ds
ww From (1) and (2) we get
# A $ dl ## V $ ds
...(2)

w.E
Sol. 29 Option (D) is correct.
=

The transmission line are as shown below. Length of all line is l

asy 4

En
gin
2 2
Zi1 = Z01 = 100 = 200W eer
ZL1
2
50
2
Zi2 = Z02 = 100 = 200W ing
ZL2 50
ZL3 = Zi1 Zi2 = 200W 200W = 100W
.ne
Sol. 30
2 2
Zi = Z0 = 50 = 25W

Option (C) is correct.


ZL3 100 t
x a - y a
We have Bv = B0 c y xm ...(1)
x2 + y2 x2 + y2
To convert in cylindrical substituting
x = r cos f and y = r sin f
ax = cos far - sin faf
and ay = sin far + cos faf
In (1) we have
Bv = Bv0 af
v Bv a
Now Hv = B = 0 f constant
m0 m0
v v
J = 4# H = 0 since H is constant

Downloaded From : www.EasyEngineering.net


Downloaded From : www.EasyEngineering.net
GATE SOLVED PAPER - EC ELECTROMAGNETICS

Sol. 31 Option (C) is correct.


The beam-width of Hertizian dipole is 180c and its half power beam-width is 90c.
Sol. 32 Option (D) is correct.
Maxwell equations
4- B = 0
4$ E = r/E
4# E =- B
4# Ht = D + J
For static electric magnetic fields
4$ B = 0
4$ E = r/E
4# E = 0

ww
Sol. 33
4# Ht = J
Option (A) is correct.

w.E Cut-off Frequency is


fc = c
2
m 2 n 2
` a j +`b j

asy
For TE11 mode,
fc =
3 # 1010 1 2 1 2
` 4 j + ` 3 j = 6.25 GHz
Sol. 34 En
Option (D) is correct.
2

Zin = Zo gin
ZL + iZo tan (bl)
Zo + iZL tan (bl)
For ZL = 0 ,
The wavelength is
Zin = iZo tan (bl)
eer
8
l = c = 3 # 109 = 0.1 m or 10 cm
f 3 # 10 ing
bl = 2
l
p l = 2p # 1 = p
10 5 .ne
Thus Zin = iZo tan p
5
Thus Zin is inductive because Zo tan p is positive
5
t
Sol. 35 Option (C) is correct.
m
We have h =
e
Reflection coefficient
h - h1
G= 2
h2 + h1
Substituting values for h1 and h2 we have
mo mo
-
t = eo er
mo
eo
mo
= 1- er = 1 - 9 since er = 9
eo er + eo 1+ er 1+ 9
=- 0.5
Sol. 36 Option (B) is correct.
In single mode optical fibre, the frequency of limiting mode increases as radius
decreases

Downloaded From : www.EasyEngineering.net


Downloaded From : www.EasyEngineering.net
GATE SOLVED PAPER - EC ELECTROMAGNETICS

Hence r \ 1
f
So. if radius is doubled, the frequency of propagating mode gets halved, and
wavelength is doubled.
Sol. 37 Option (D) is correct.
8
l = c = 3 # 10 9 = 3
f 20 # 10 200
2
Gp = hp2 ` D j = 0.7 # p2 c 13 m = 30705.4
2
Gain
l 100
= 44.87 dB
Sol. 38 Option (A) is correct.
g = b cos 30cx ! b sin 30cy
= 2p 3 x ! 2p 1 y
ww l 2
= p 3 x! py
l 2

w.E l l
E = ay E0 e j (wt - g) = ay E0 e j;wt - c
p 3 x! p y
l l mE

Sol. 39

asy
Option (D) is correct.
4# H = J + 2D
2t
Maxwell Equations

En
## 4# H $ ds = ## `J + 22Dt j .ds Integral form

gin
s s

# H $ dl = ## `J + 22Dt j .ds Stokes Theorem

Sol. 40 Option (A) is correct.


wm p
s

H sin ` 2px j sin (wt - bz) yt


2 `2j 0
2 eer
E =
h
This is TE mode and we know that
a
ing
Ey \ sin ` mpx j cos `
a
mpy
b j
.ne
Sol. 41
Thus m = 2 and n = 0 and mode is TE20
Option (C) is correct.
The 2-port scattering parameter matrix is
t
S11 S12
S ==
S21 S22 G
(Z Z ) - Zo (50 50) - 50
S11 = L 0 = =- 1
(ZL Z0) + Zo (50 50) + 50 3
2 (ZL Zo) 2 (50 50)
S12 = S21 = = =2
(ZL Zo) + Zo (50 50) + 50 3
(Z Z ) - Zo (50 50) - 50
S22 = L o = =- 1
(ZL Zo) + Zo (50 50) + 50 3
Sol. 42 Option (D) is correct.
The input impedance is
2
Zin = Zo ; if l = l
ZL 4
2 2
Zin1 = Zo1 = 50 = 25
ZL1 100

Downloaded From : www.EasyEngineering.net


Downloaded From : www.EasyEngineering.net
GATE SOLVED PAPER - EC ELECTROMAGNETICS

2 2
Zin2 = Zo2 = 50 = 12.5
ZL2 200
Now ZL = Zin1 Zin2
25 12.5 = 25
3
(50) 2
Zs = = 300
25/3
G = ZS - Zo = 300 - 50 = 5
ZS + Zo 300 + 50 7
Sol. 43 Option (D) is correct.
2
= Hx2 + Hy2 = c 5 3 m + c 5 m = c 10 m
2 2
We have H 2
ho ho ho
E2 ho H 2 h

ww = o c 10 m = 50 watts
2
For free space P = =
2ho 2 2 ho ho
Sol. 44 Option (C) is correct.

w.E The cut-off frequency is


fc = c
2
m 2 n 2
` a j +`b j

asy
Since the mode is TE20, m = 2 and n = 0
8
fc = c m = 3 # 10 # 2 = 10 GHz

En h' =
2 2
ho
2 # 0.03
= 377
1-c m
fc 2
f gin 10
1 - c 10 10 m
3 # 10
2
= 400W

Sol. 45 Option (B) is correct.

eer
Using the method of images, the configuration is as shown below

ing
.ne
Here d = l, a = p, thus bd = 2p

= cos ;
bd cos y + a
t
Array factor is
2 E
2p cos y + p
= cos ; E = sin (p cos y)
2
Sol. 46 Option (D) is correct.
The Brewster angle is
tan qn = er2
er1
tan 60c = er2
1
or er2 = 3
Sol. 47 Option (C) is correct.
We have E = atxx sin (wt - bz) + aty sin (wt - bz + p/2)
Here Ex = Ey and fx = 0, fy = p2

Downloaded From : www.EasyEngineering.net


Downloaded From : www.EasyEngineering.net
GATE SOLVED PAPER - EC ELECTROMAGNETICS

p
Phase difference is 2 , thus wave is left hand circularly polarized.
Sol. 48 Option (A) is correct.
We have 10 log G = 10 dB
or G = 10
Now gain G = Prad
Pin
or 10 = Prad
1W
or Prad = 10 Watts
Sol. 49 Option (A) is correct.
mo mo
h2 - h1 eo er - eo
G= =
h2 + h1 mo
eo er + mo
eo
= 1 + er = 1 - 4 =- 1

ww 1 + er 1+ 4
The transmitted power is
3

w.E or
Pt = (1 - G2) Pi = 1 - 1 = 8
Pt = 8
9 9

Sol. 50
asy
Option (D) is correct.
Pi 9

En
sin q = 1 = 1
er
p
2
or q = 45c =
4
gin
The configuration is shown below. Here A is point source.

eer
ing
.ne
Now
From geometry
Thus area
AO = 1 m
BO = 1 m
= pr2 = p # OB = p m 2
t
Sol. 51 Option (C) is correct.
The cut-off frequency is
fc = c m 2 m 2
`a j +`b j
2
Since the mode is TE30 , m = 3 and n = 0
fc = c m
2 a
8
or 18 # 109 = 3 # 10 3
2 a
or a = 1 m = 5 cm
40 2
Sol. 52 Option (C) is correct.
We have E1 = 4ux + 3uy + 5uz

Downloaded From : www.EasyEngineering.net


Downloaded From : www.EasyEngineering.net
GATE SOLVED PAPER - EC ELECTROMAGNETICS

Since for dielectric material at the boundary, tangential component of electric


field are equal
E21 = E1t = 3aty + 5atz
at the boundary, normal component of displacement vector are equal
i.e. Dn2 = Dn1
or e2 E2n = e1 E1n
or 4eo E2n = 3eo 4atz
or E2n = 3atx
Thus E2 = E2t + E2a = 3atx + 3aty + 5atz
Sol. 53 Option (C) is correct.
Since antenna is installed at conducting ground,
2
= 4p W
2
Rrad = 80p2 ` dl j = 80p2 c 50 2
3m

ww
Sol. 54 Option (C) is correct.
l 0.5 # 10 5

w.E Phase Velocity is


w = 50, 000 and b =- 0.004
4
vP = w = 5 # 10 - 3 = 1.25 # 107 m/s
b - 4 # 10
Sol. 55
asy
Option (C) is correct.
Refractive index of glass m = 1.5
Frequency
En f = 1014 Hz
c = 3 # 108 m/sec
l = c = 3 # 10
f 1014
8
gin
= 3 # 10 - 6

wavelength in glass is
-6
lg = a = 3 # 10 = 2 # 10 - 6 m
eer
Sol. 56 Option (D) is correct.
m 1.5
ing
Sol. 57 Option (D) is correct.
.ne
Zo2 = ZOC .ZSC
2
ZZC = Zo = 50 # 50 = 50
ZOC 100 + j150
50 (2 - 3j)
2 + 3j
t
= = 7.69 - 11.54j
13
Sol. 58 Option (A) is correct.
The array factor is
bd sin q + a
A = cos b l
2
Here b = 2p , d = l and a = 90c
l 4
2p l p
sin q + p p
Thus A = cos c l 4 2
m = cos ` sin q + j
2 4 2
The option (A) satisfy this equation.
Sol. 59 Option (C) is correct.
From the diagram, VSWR is

Downloaded From : www.EasyEngineering.net


Downloaded From : www.EasyEngineering.net
GATE SOLVED PAPER - EC ELECTROMAGNETICS

s = Vmax = 4 = 4
Vmin 1
When minima is at load ZO = s.ZL
or ZL = Zo = 50 = 12.5W
s 4
Sol. 60 Option (A) is correct.
The reflection coefficient is
G = ZL - ZO = 12.5 - 50 =- 0.6
ZL + ZO 125. + 50
Sol. 61 Option (C) is correct.
The given figure represent constant reactance circle.
Sol. 62 Option (D) is correct.
We know that vp > c > vg .

ww
Sol. 63 Option (A) is correct.
We have GD (q, f) =
4pU (q, f)

w.E For lossless antenna


Prad = Pin
Prad

Here we have
and
asy Prad = Pin = 1 mW
10 log GD (q, f) = 6 dB
or
En
GD (q, f) = 3.98
Thus the total power radiated by antenna is
gin
4pU (q, f) = Prad GD (q, f) = 1 m # 3.98 = 3.98 mW
Sol. 64 Option (D) is correct.
The capacitance is eer
- 12 -4

ing
C = eo A = 8.85 # 10 - 3 # 10 = 8.85 # 10 - 13
d 10
The charge on capacitor is
Q = CV = 8.85 # 10 - 13 = 4.427 # 10 - 13 .ne
Displacement current in one cycle
Q
I = = fQ = 4.427 # 10 - 13 # 3.6 # 109 = 1.59 mA
T
t
Sol. 65 Option (C) is correct.
VL = ZO
Vin Zin
or VL = ZO Vin = 10 # 300 = 60 V
Zin 50
Sol. 66 Option (D) is correct.
Sol. 67 Option (A) is correct.
Ravg = 1 Re [E # H*]
2
E # H* = (atx + jaty) e jkz - jwt # k (- jatx + aty) e-jkz + jwt
wm
= atz ; k - (- j) (j) k E = 0
wm wm

Downloaded From : www.EasyEngineering.net


Downloaded From : www.EasyEngineering.net
GATE SOLVED PAPER - EC ELECTROMAGNETICS

Thus Ravg = 1 Re [E # H*] = 0


2
Sol. 68 Option (A) is correct.
Suppose at point P impedance is
Z = r + j (- 1)
If we move in constant resistance circle from point P in clockwise direction by
an angle 45c, the reactance magnitude increase. Let us consider a point Q at
45c from point P in clockwise direction. It’s impedance is
Z1 = r - 0.5j
or Z1 = Z + 0.5j
Thus movement on constant r - circle by an +45c in CW direction is the
addition of inductance in series with Z .

ww
Sol. 69 Option (D) is correct.

We have VSWR = Emax = 5 =


1- G

w.E or G =2
Emin

3
1+ G

Thus
asy G =- 2
3
h2 - h1
Now
En G=
h2 + h1
h - 120p
-2 = 2
or

or
3
h2 = 24p gin
h2 + 120p

Sol. 70 Option (D) is correct.


1- G eer
The VSWR

or
2=
1+ G
G =1
ing
Thus
Pref
3
= G2= 1 .ne
or
Pinc
Pref = Pinc
9
9

i.e. 11.11% of incident power is reflected.


t
Sol. 71 Option (C) is correct.
By Maxwells equations
4# H = 2D + J
2t
Thus 4# H has unit of current density J that is A/m2
Sol. 72 Option (B) is correct.
We know that d \ 1
f
d2 = f1
Thus
d1 f2
d2 = 1
25 4

Downloaded From : www.EasyEngineering.net


Downloaded From : www.EasyEngineering.net
GATE SOLVED PAPER - EC ELECTROMAGNETICS

or d2 = 1 # 25 = 12.5 cm
4
Sol. 73 Option (C) is correct.
We have E1 = 2ux - 3uy + 1uz
E1t =- 3uy + uz and E1n = 2ux
Since for dielectric material at the boundary, tangential component of electric
field are equal
E1t =- 3uy + uz = E2t (x = 0 plane)
E1n = 2ux
At the boundary the for normal component of electric field are
D1n = D2n
or e1 E1n = e2 E2n

ww or
or
1.5eo 2ux = 2.5eo E2n
E2n = 3 ux = 1.2ux
2.5

w.E
Sol. 74
Thus E2 = E2t + E2n =- 3uy + uz + 1.2ux
Option (C) is correct.
We have
asyE = xux + yuy + zuz
dl = utx dx + uty dy + utz dz

En Y
VXY =- # E.dl =
X
#1
2
xdxutx +
0 0
#2 ydyutz + #3 zdzuzt
2 2
=-= x +
y2
2 1 2 2 2 3
0
+z G
2 0

gin
=- 1 [22 - 12 + 02 - 22 + 02 - 32] = 5
2 eer
Sol. 75 Option (D) is correct.
h =
m ing
Reflection coefficient
e
.ne
t =
h2 - h1
h2 + h1
Substituting values for h1 and h2 we have
mo m0
t
-
t = eo er
mo
eo
mo
= 1- er = 1 - 4 since er = 4
eo er + eo 1+ er 1+ 4

= - 1 = 0.333+180c
3
Sol. 76 Option (B) is correct.
We have E (z, t) = 10 cos (2p # 107 t - 0.1pz)
where w = 2p # 107 t
b = 0.1p
7
Phase Velocity u = w = 2p # 10 = 2 # 108 m/s
b 0.1p
Sol. 77 Option (D) is correct.
y
Normalized array factor = 2 cos
2

Downloaded From : www.EasyEngineering.net


Downloaded From : www.EasyEngineering.net
GATE SOLVED PAPER - EC ELECTROMAGNETICS

y = bd sin q cos f + d
q = 90c,
d = 2 s,
f = 45c,
d = 180c
y bd sin q cos f + d
Now 2 cos = 2 cos ; E
2 2
= 2 cos 8 2p 2 s cos 45c + 180 B
l. 2 2
p p
= 2 cos 8 + 90cB = 2 sin ` s j
s
l l
Sol. 78 Option (A) is correct.
The fig of transmission line is as shown below.

ww We know that
l
Zin = Zo L
2 p
[Z + jZo tan bl]
[Zo + jZL tan bl]

w.E For line 1, l = and b =

Thus
2
, Z = 100W
l L1
[Z + jZo tan p]
Zin1 = Zo L
[Zo + jZL tan p]
= ZL = 100W

asy
For line 2, l = l and b = 2p , ZL2 = 0 (short circuit)
8 l

Thus
En Zin2 = Zo
[0 + jZo tan p4 ]
[Zo + 0]
= jZo = j50W

Zin1 Zin2 gin


Y = 1 + 1 = 1 + 1 = 0.01 - j0.02
100 j50

eer
ing
.ne
t
Sol. 79 Option (A) is correct.
8
u = c = 3 # 10 = 1.5 # 108
e0 2
In rectangular waveguide the dominant mode is TE10 and
fC = v ` m j2 + ` n j2
2 a b
8
= 1.5 # 10 `
1 2 + 0 2 = 1.5 # 108 = 2.5 GHz
2 0.03 j ` b j 0.06

Sol. 80 Option (D) is correct.

Downloaded From : www.EasyEngineering.net


Downloaded From : www.EasyEngineering.net
GATE SOLVED PAPER - EC ELECTROMAGNETICS

VSWR s = 1+G where G varies from 0 to 1


1-G
Thus s varies from 1 to 3.
Sol. 81 Option (B) is correct.
Reactance increases if we move along clockwise direction in the constant resistance
circle.
Sol. 82 Option (C) is correct.
Phase velocity
VC
VP =
f 2
1-c c m
f
When wave propagate in waveguide fc < f $ VP > VC
Sol. 83 Option (C) is correct.

ww We have E = (0.5xt + ye
p
t j ) e j (wt - kz)
2

w.E Ex = 0.5e j (wt - kz)


p
Ey = e j 2 e j (wt - kz)

asy
Ex
Ey
p
= 0.5e- 2

Since
Ex
Ey En
! 1, it is elliptically polarized.

Sol. 84 Option (A) is correct.


tan a = s =
gin
1.7 # 10 - 4
Loss tangent
we 2p # 3 # 109 # 78eo
eer
ing
-4 9
= 1.7 # 10 9# 9 # 10 = 1.3 # 10 - 5
3 # 10 # 39
Option (D) is correct.

.ne
Sol. 85

The flux density is


s = eE = e0 er E = 80 # 8.854 # 10 - 12 # 2

Sol. 86
or s = 1.41 # 10 - 9 C/m 2
Option (B) is correct.
t
P \ 12
r
Thus P1 = r22
P2 r12
3 dB decrease $ Strength is halved
Thus P1 = 2
P2
Substituting values we have
2
2 = r22
5
or r2 = 5 2 kM = 7071 m
Distance to move = 7071 - 5000 = 2071 m

Downloaded From : www.EasyEngineering.net


Downloaded From : www.EasyEngineering.net
GATE SOLVED PAPER - EC ELECTROMAGNETICS

Sol. 87 Option (C) is correct.


A transmission line is distortion less if LG = RC
Sol. 88 Option (B) is correct.
We have d2 Ex = c2 d2 Ex
dz2 dt2
This equation shows that x component of electric fields Ex is traveling in z
direction because there is change in z direction.
Sol. 89 Option (A) is correct.
In wave guide vp > c > vg and in vacuum vp = c = vg
where vp $ Phase velocity
c $ Velocity of light
vg $ Group velocity

ww
Sol. 90 Option (A) is correct.
In a wave guide dominant gives lowest cut-off frequency and hence the highest

w.E
Sol. 91
cut-off wavelength.
Option (A) is correct.
Ic = Id
or
or
asy sE = jw d E
s = 2pfeo er w = 2pf and e = er e0
or En f = s
2p # eo er 4peo er 4
9
= 2s = 9 # 10 # 2 # 10
-2

or
gin
f = 45 # 106 = 45 MHz
Sol. 92 Option (B) is correct.
VSWR = 1 + G
1-G eer
or 3 = 1+G
1-G ing
or
Now
G = 0.5
Pr = G2 = 0.25 .ne
Sol. 93
Pi
Thus 25% of incident power is reflected.
Option (A) is correct.
t
We have l = 492 m
and height of antenna = 124 m . l
4
It is a quarter wave monopole antenna and radiation resistance is 25 W.
Sol. 94 Option (C) is correct.
The array factor is
y = bd cos q + d
where d =l Distance between elements
4
y =0 Because of end fire
q = 60c
Thus 0 = 2p # l cos 60c + d = p # 1 + d
l 4 2 2

Downloaded From : www.EasyEngineering.net


Downloaded From : www.EasyEngineering.net
GATE SOLVED PAPER - EC ELECTROMAGNETICS

or d =- p
4

Sol. 95 Option (B) is correct.


Zo = ZOC .ZSC = 100 # 25 = 10 # 5 = 50W
Sol. 96 Option (C) is correct.
As the impedance of perfect conductor is zero, electric field is minimum and
magnetic field is maximum at the boundary.
Sol. 97 Option (B) is correct.
BW \ 1
(Diameter)
As diameter increases Bandwidth decreases.
Sol. 98 Option (C) is correct.

ww The fig is as shown below :

w.E
asy
As per snell law
En
sin qt = 1

or
sin qi
sin 30c = 1
er
gin
sin 45c
1
2
= 1
er
eer
ing
1
2
er
or er = 2
Sol. 99 Option (C) is correct.
Cutoff frequency fc =
vp m 2 n 2
` a j +`b j .ne
Thus
2
For rectangular waveguide dominant mode is TE01
v 8
fc = p = 3 # 10- 2 = 15 # 109
2a 2 # 10
t
For air vp = 3 # 108

= 15 GHz

Sol. 100 Option (B) is correct.


Phase Velocity b = 2p = w me
l
or l = 2p
w me

Thus l \ 1
e

we get l1 = e2
l2 e1

Downloaded From : www.EasyEngineering.net


Downloaded From : www.EasyEngineering.net
GATE SOLVED PAPER - EC ELECTROMAGNETICS

Sol. 101 Option (D) is correct.


l
` 2 jd = l
2

8
l = c = 3 # 10 9 = 3 m
f 4 # 10 40
3
` 40 # 2 j d = (2.4)
2

80 # (2.4) 2
or d = . 150 m
3
Sol. 102 Option (C) is correct.
We know that for a monopole its electric field varies inversely with r 2 while its
potential varies inversely with r . Similarly for a dipole its electric field varies
inversely as r 3 and potential varies inversely as r 2 .
In the given expression both the terms a _ r1 + r1 i are present, so this potential is

ww
Sol. 103
due to both monopole & dipole.
Option (D) is correct.
-1 -2

w.E In TE mode Ez = 0 , at all points within the wave guide. It implies that electric
field vector is always perpendicular to the waveguide axis. This is not possible in

Sol. 104
asy
semi-infine parallel plate wave guide.
Option (A) is correct.
Sol. 105

En
Option (C) is correct.
A scalar wave equation must satisfy following relation
2 2 E - m 22 2 E = 0
2t 2 2z 2 gin ...(1)

Where m = w (Velocity)
b
eer
Basically w is the multiply factor of t and b is multiply factor of z or x or y .
In option (A) E = 50e j (wt - 3z)
m =w=w
ing
b 3
.ne
We can see that equations in option (C) does not satisfy equation (1)
Sol. 106 Option (B) is correct.
t
We know that distance between two adjacent voltage maxima is equal to l/2 ,
where l is wavelength.
l = 27.5 - 12.5
2
l = 2 # 15 = 30 cm
10
Frequency u = C = 3 # 10 = 1 GHz
l 30
Sol. 107 Option (D) is correct.
Power received by antenna
-4
PR = PT 2 # (apeture) = 251 # 500 # 102 = 100 mW
4p r 4 # p # (100)
Sol. 108 Option (C) is correct.
Electrical path length = bl
Where b = 2p , l = 50 cm
l

Downloaded From : www.EasyEngineering.net


Downloaded From : www.EasyEngineering.net
GATE SOLVED PAPER - EC ELECTROMAGNETICS

We know that
l =u =1# 1 au= 1
f f LC LC
= 1 1
#
25 # 106 10 # 10-6 # 40 # 10-12
7
= 5 # 10 6 = 2 m
25 # 10
Electric path length = 2p # 50 # 10-2
5
= p radian
2
Sol. 109 Option (D) is correct.
In a lossless dielectric (s = 0) median, impedance is given by

ww h =
m
e
0c

w.E =
m0 mr
e0 er

= 120p #
mr

asy er
= 120p # 2 = 188.4 W
8
Sol. 110
En
Option (D) is correct.
Impedance is written as
h =
jwm gin
s + jwe
Copper is good conductor i.e. s >> we
eer
So h =
jwm
s
=
wm
s
45c
Impedance will be complex with an inductive component. ing
Sol. 111 Option (A) is correct.
This equation is based on ampere’s law as we can see .ne
or
#l H $ dl
#l H $ dl
= I enclosed

= #s Jds
(ampere's law)
t
Applying curl theorem
#s (4 # H) $ ds
= # Jds
s
4# H = J
then it is modified to
4# H = J + 2D Based on continuity equation
2t
Sol. 112 Option (A) is correct.
Sol. 113 Option (B) is correct.
Sol. 114 Option (B) is correct.
Propagation constant
r = a + ib = 0.1p + j0.2p

Downloaded From : www.EasyEngineering.net


Downloaded From : www.EasyEngineering.net
GATE SOLVED PAPER - EC ELECTROMAGNETICS

here b = 2p = 0.2p
l
l = 2 = 10 m
0.2
Sol. 115 Option (C) is correct.
The depth of penetration or skin depth is defined as –
d= 1
pfms

d\ 1 \ l
f
so depth increases with increasing in wavelength.
Sol. 116 Option (A) is correct.
Given

ww Generalizing
E (z, t) = Eo e
j (wt + bz) v
a x + e0 e j (wt + bz) avy ...(1)

w.E E (z) = avx E1 (z) + avy E2 (z) ...(2)


Comparing (1) and (2) we can see that E1 (z) and E2 (z) are in space quadrature
but in time phase, their sum E will be linearly polarized along a line that makes

asy
an angle f with x -axis as shown below.
Option (C) is correct.

En
Sol. 117

Hv = 1 4v Av
m #
v is auxiliary potential function.
where A
So
gin
4: H = 4: (4 # A) = 0
4# H = 4# (4 # A) =
Y 0
eer
Sol. 118 Option (D) is correct.
Radiation resistance of a circular loop is given as
Rr = 8 hp3 :ND2 S D
ing
3
2
l
Rx \ N N " no. of turns .ne
So, Rr 2 = N 2 # Rr 1
= (5) 2 # 0.01 = 0.25 W
t
Sol. 119 Option (C) is correct.
We have
Aperture Area = Power Re ceived
Polynting vector of incident wave
A =W
P
2
P = E h0 = 120p is intrinsic impedance of space
h0
-6 -6
So A = 2 # 102 = 2 # 10 -3 2 # 120 # 3.14
E (20 # 10 )
c h0 m
-6
= 2 # 10 # 12 -# 3.14 = 1.884 m2
400 # 10 6

Downloaded From : www.EasyEngineering.net


Downloaded From : www.EasyEngineering.net
GATE SOLVED PAPER - EC ELECTROMAGNETICS

Sol. 120 Option (B) is correct.


Maximum usable frequency
fo
fm =
sin Ae
fm = 8MHz = 8 = 16 MHz
sin 60c 3 3
c 2 m
Sol. 121 Option (D) is correct.
When a moving circuit is put in a time varying magnetic field educed emf have
two components. One for time variation of B and other turn motion of circuit in
B.
Sol. 122 Option (A) is correct.
Far field \ 1
r

ww
Sol. 123 Option (B) is correct.
Z in = Z0

w.E S
min

where S = standing wave ratio


1 + GL

asy
S =
1 - GL
GL = reflection coefficient

EnGL = ZL - Z 0 = 100 - 50 = 50 = 1
ZL + Z 0 100 + 50 150 3

S =
1+1
3 =2 gin
1-
50
1
3
eer
Sol. 124
Z in min

Option (A) is correct.


=
2
= 25 W

ing
The cutoff frequency is given by
fc =
ml m 2+ n 2
2 a a k a2k
.ne
Here a < b , so minimum cut off frequency will be for mode TE 01
m = 0, n = 1
t
8 a ml = c
fc = 3 # 10 1
* 2
2#2 (10 # 10-12)
c = 3 # 108
= 3 # 108 = 0.75 GHz
2 # 2 # 10 # 10-2
Sol. 125 Option (A) is correct.
For any transmission line we can write input impedance
Z + jZ 0 tanh lg
Zin = Z 0 ; L
Z 0 + jZL tanh lg E
Here given ZL = 3 (open circuited at load end)
R jZ tanh lg V
S1 + 0 W
so Zin = Z 0 lim S ZL W= Z0
Z " 3S Z 0 W j tanh lg
S ZL + j tanh lg W
L

T X

Downloaded From : www.EasyEngineering.net


Downloaded From : www.EasyEngineering.net
GATE SOLVED PAPER - EC ELECTROMAGNETICS

Sol. 126 Option (A) is correct.


We know that skin depth is given by
s = 1 = 1 # 10-2 m
pf1 ms
or 1 = 10-2 f 1 = 10 MHz
6
p # 10 # 10 # m s
-3
or ms = 10
p
Now phase velocity at another frequency
f 2 = 1000 MHz is
4p f 2
V =
ms

ww
-3
Put ms = 10 in above equation
p

V = 4 # p # 1000 # 106 # p - 6 106 m/ sec

w.E
Sol. 127 Option (A) is correct.
10-3
#

asy
Input impedance of a lossless transmission line is given by
Z + jZ 0 tan bl
Zin = Z 0 ; L
Z 0 + jZL tan bl E
where
En Z 0 = Charateristic impedance of line
ZL = Load impedance
b = 2p
l gin l = length

so here bl = 2p l = p
l 4 2 eer
and
ZL = 0
Z 0 = 50 W ing
(Short circuited)

so Zin = 50 =
0 + j50 tan p/2
50 + j0 tan p/2G
=3
.ne
Sol. 128
Thus infinite impedance, and current will be zero.
Option (B) is correct.
For lossless transmission line, we have
t
Velocity V =w= 1 ...(1)
b LC
Characteristics impedance for a lossless transmission line
Z0 = L ...(2)
C
From eqn. (1) and (2)
V = 1 = 1
C (Z 0 C ) Z 0 C
Sol. 129 Option (C) is correct.
Reflected power
Er = GEi Ei " Incident power
G = Reflection coefficient

Downloaded From : www.EasyEngineering.net


Downloaded From : www.EasyEngineering.net
GATE SOLVED PAPER - EC ELECTROMAGNETICS

h2 - h1
G = = 1.5 - 1 = 1
h 2 + h1 1.5 + 1 5
So Er = 1 # Ei
5
Er = 20%
Ei
Sol. 130 Option (B) is correct.
We have maximum usable frequency formulae as
f0
fm =
sin Ae
6
20 # 106 = 10 # 10
sin Ae
sin Ae = 1
2

ww
Sol. 131 Option (A) is correct.
Ae = 30c

w.E Skin depth d=


Putting the given value
1
pfms

asy d= 1
3.14 # 1 # 109 # 4p # 10-7 # 106
= 15.9 mm

En
gin
***********

eer
ing
.ne
t

Downloaded From : www.EasyEngineering.net


Downloaded From : www.EasyEngineering.net

No part of this publication may be reproduced or distributed in any form or any means, electronic, mechanical,
photocopying, or otherwise without the prior permission of the author.

ww
w.E
GATE SOLVED PAPER
Electronics & Communication
Electronic Devices
asy
Copyright © By NODIA & COMPANY
En
gin
eer
ing
Information contained in this book has been obtained by authors, from sources believes to be reliable. However,
neither Nodia nor its authors guarantee the accuracy or completeness of any information herein, and Nodia nor its

.ne
authors shall be responsible for any error, omissions, or damages arising out of use of this information. This book
is published with the understanding that Nodia and its authors are supplying information but are not attempting
to render engineering or other professional services.

t
NODIA AND COMPANY
B-8, Dhanshree Tower Ist, Central Spine, Vidyadhar Nagar, Jaipur 302039
Ph : +91 - 141 - 2101150
www.nodia.co.in
email : enquiry@nodia.co.in

Downloaded From : www.EasyEngineering.net


Downloaded From : www.EasyEngineering.net

GATE SOLVED PAPER - EC


ELECTRONIC DEVICES

2013 ONE MARK

Q. 1 In a forward biased pn junction diode, the sequence of events that best describes
the mechanism of current flow is
(A) injection, and subsequent diffusion and recombination of minority carriers
(B) injection, and subsequent drift and generation of minority carriers
(C) extraction, and subsequent diffusion and generation of minority carriers

ww
Q. 2
(D) extraction, and subsequent drift and recombination of minority carriers

In IC technology, dry oxidation (using dry oxygen) as compared to wet oxidation

w.E (using steam or water vapor) produces


(A) superior quality oxide with a higher growth rate
(B) inferior quality oxide with a higher growth rate

asy
(C) inferior quality oxide with a lower growth rate
(D) superior quality oxide with a lower growth rate

Q. 3 En
In a MOSFET operating in the saturation region, the channel length modulation
effect causes
gin
(A) an increase in the gate-source capacitance
(B) a decrease in the transconductance
eer
(C) a decrease in the unity-gain cutoff frequency
(D) a decrease in the output resistance
ing
2013
.ne TWO MARKS

Q. 4

t
The small-signal resistance (i.e., dVB /dID ) in kW offered by the n-channel MOSFET
M shown in the figure below, at a bias point of VB = 2 V is (device data for
M: device transconductance parameter kN = mn C 0' x ^W/L h = 40 mA/V2 , threshold
voltage VTN = 1 V , and neglect body effect and channel length modulation effects)

(A) 12.5
(B) 25
(C) 50
(D) 100

Downloaded From : www.EasyEngineering.net


Downloaded From : www.EasyEngineering.net
GATE SOLVED PAPER - EC ELECTRONIC DEVICES

2012 TWO MARKS

Q. 5 The source of a silicon (ni = 1010 per cm3) n -channel MOS transistor has an area
of 1 sq mm and a depth of 1 mm . If the dopant density in the source is 1019 /cm3 ,
the number of holes in the source region with the above volume is approximately
(A) 107 (B) 100
(C) 10 (D) 0

Q. 6 In the CMOS circuit shown, electron and hole mobilities are equal, and M1 and
M2 are equally sized. The device M1 is in the linear region if

ww
w.E (A) Vin < 1.875 V
(B) 1.875 V < Vin < 3.125 V
(C) Vin > 3.125 V

asy
(D) 0 < Vin < 5 V

En
Common Data For Q. 7 and 8

gin
In the three dimensional view of a silicon n -channel MOS transistor shown below,
d = 20 nm . The transistor is of width 1 mm . The depletion width formed at every

eer
p-n junction is 10 nm. The relative permittivity of Si and SiO 2 , respectively, are
11.7 and 3.9, and e0 = 8.9 # 10-12 F/m .

ing
.ne
t

Q. 7 The gate source overlap capacitance is approximately


(A) 0.7 fF (B) 0.7 pF
(C) 0.35 fF (D) 0.24 pF

Q. 8 The source-body junction capacitance is approximately


(A) 2 fF (B) 7 fF
(C) 2 pF (D) 7 pF

Downloaded From : www.EasyEngineering.net


Downloaded From : www.EasyEngineering.net
GATE SOLVED PAPER - EC ELECTRONIC DEVICES

2011 ONE MARK

Q. 9 Drift current in the semiconductors depends upon


(A) only the electric field
(B) only the carrier concentration gradient
(C) both the electric field and the carrier concentration
(D) both the electric field and the carrier concentration gradient

Q. 10 A Zener diode, when used in voltage stabilization circuits, is biased in


(A) reverse bias region below the breakdown voltage
(B) reverse breakdown region
(C) forward bias region
(D) forward bias constant current mode

ww
Q. 11 A silicon PN junction is forward biased with a constant current at room
temperature. When the temperature is increased by 10ºC, the forward bias
voltage across the PN junction

w.E (A) increases by 60 mV


(B) decreases by 60 mV

asy
(C) increases by 25 mV
(D) decreases by 25 mV

2011 En TWO MARKS

Common Data For Q. 12 and 13


gin
eer
The channel resistance of an N-channel JFET shown in the figure below is 600 W

ing
when the full channel thickness (tch ) of 10 μm is available for conduction. The
built-in voltage of the gate P+ N junction (Vbi ) is - 1 V . When the gate to source
voltage (VGS ) is 0 V, the channel is depleted by 1 μm on each side due to the built

.ne
in voltage and hence the thickness available for conduction is only 8 μm

Q. 12 The channel resistance when VGS =- 3 V is


(A) 360 W (B) 917 W
(C) 1000 W (D) 3000 W

Q. 13 The channel resistance when VGS = 0 V is


(A) 480 W (B) 600 W
(C) 750 W (D) 1000 W

Downloaded From : www.EasyEngineering.net


Downloaded From : www.EasyEngineering.net
GATE SOLVED PAPER - EC ELECTRONIC DEVICES

2010 ONE MARK

Q. 14 At room temperature, a possible value for the mobility of electrons in the inversion
layer of a silicon n -channel MOSFET is
(A) 450 cm2 / V-s (B) 1350 cm2 / V-s
(C) 1800 cm2 / V-s (D) 3600 cm2 / V-s

Q. 15 Thin gate oxide in a CMOS process in preferably grown using


(A) wet oxidation (B) dry oxidation
(C) epitaxial oxidation (D) ion implantation

2010 TWO MARKS

Q. 16 In a uniformly doped BJT, assume that NE , NB and NC are the emitter, base and
collector doping in atoms/cm3 , respectively. If the emitter injection efficiency of

ww the BJT is close unity, which one of the following condition is TRUE
(A) NE = NB = NC

w.E (B) NE >> NB and NB > NC


(C) NE = NB and NB < NC

Q. 17
(D) NE < NB < NC

asy
Compared to a p-n junction with NA = ND = 1014 /cm3 , which one of the following

En
statements is TRUE for a p-n junction with NA = ND = 1020 /cm3 ?
(A) Reverse breakdown voltage is lower and depletion capacitance is lower

gin
(B) Reverse breakdown voltage is higher and depletion capacitance is lower
(C) Reverse breakdown voltage is lower and depletion capacitance is higher

eer
(D) Reverse breakdown voltage is higher and depletion capacitance is higher

Statements for Linked Answer Question : 18 and 19


ing
.ne
The silicon sample with unit cross-sectional area shown below is in thermal
equilibrium. The following information is given: T = 300 K electronic charge

t
= 1.6 # 10-19 C , thermal voltage = 26 mV and electron mobility = 1350 cm2 / V-s

Q. 18 The magnitude of the electric field at x = 0.5 mm is


(A) 1 kV/cm (B) 5 kV/cm
(C) 10 kV/cm (D) 26 kV/cm

Q. 19 The magnitude of the electron of the electron drift current density at x = 0.5 mm
is
(A) 2.16 # 10 4 A/cm2 (B) 1.08 # 10 4 A/m2
(C) 4.32 # 103 A/cm2 (D) 6.48 # 102 A/cm2

Downloaded From : www.EasyEngineering.net


Downloaded From : www.EasyEngineering.net
GATE SOLVED PAPER - EC ELECTRONIC DEVICES

2009 ONE MARK

Q. 20 In an n-type silicon crystal at room temperature, which of the following can have
a concentration of 4 # 1019 cm - 3 ?
(A) Silicon atoms (B) Holes
(C) Dopant atoms (D) Valence electrons

Q. 21 The ratio of the mobility to the diffusion coefficient in a semiconductor has the
units
(A) V - 1 (B) cm.V1
(C) V.cm - 1 (D) V.s

2009 TWO MARKS

Consider the following two statements about the internal conditions in a n -

ww
Q. 22
channel MOSFET operating in the active region.
S1 : The inversion charge decreases from source to drain

w.E S2 : The channel potential increases from source to drain.


Which of the following is correct?
(A) Only S2 is true

asy
(B) Both S1 and S2 are false
(C) Both S1 and S2 are true, but S2 is not a reason for S1

En
(D) Both S1 and S2 are true, and S2 is a reason for S1

Common Data For Q. 23 and 24 gin


eer
Consider a silicon p - n junction at room temperature having the following
parameters:
Doping on the n -side = 1 # 1017 cm - 3
Depletion width on the n -side = 0.1mm ing
Depletion width on the p -side = 1.0mm
Intrinsic carrier concentration = 1.4 # 1010 cm - 3 .ne
Thermal voltage = 26 mV
Permittivity of free space = 8.85 # 10 - 14 F.cm - 1
Dielectric constant of silicon = 12
t
Q. 23 The built-in potential of the junction
(A) is 0.70 V
(B) is 0.76 V
(C) is 0.82 V
(D) Cannot be estimated from the data given

Q. 24 The peak electric field in the device is


(A) 0.15 MV . cm - 1, directed from p -region to n -region
(B) 0.15 MV . cm - 1, directed from n -region to p -region
(C) 1.80 MV . cm - 1, directed from p-region to n -region
(D) 1.80 MV . cm - 1, directed from n -region to p -region

Downloaded From : www.EasyEngineering.net


Downloaded From : www.EasyEngineering.net
GATE SOLVED PAPER - EC ELECTRONIC DEVICES

2008 ONE MARK

Q. 25 Which of the following is NOT associated with a p - n junction ?


(A) Junction Capacitance (B) Charge Storage Capacitance
(C) Depletion Capacitance (D) Channel Length Modulations

Q. 26 Which of the following is true?


(A) A silicon wafer heavily doped with boron is a p+ substrate
(B) A silicon wafer lightly doped with boron is a p+ substrate
(C) A silicon wafer heavily doped with arsenic is a p+ substrate
(D) A silicon wafer lightly doped with arsenic is a p+ substrate

Q. 27 A silicon wafer has 100 nm of oxide on it and is furnace at a temperature above


1000c C for further oxidation in dry oxygen. The oxidation rate
(A) is independent of current oxide thickness and temperature

ww (B) is independent of current oxide thickness but depends on temperature


(C) slows down as the oxide grows

w.E
Q. 28
(D) is zero as the existing oxide prevents further oxidation

The drain current of MOSFET in saturation is given by ID = K (VGS - VT ) 2 where


K is a constant.
asy
The magnitude of the transconductance gm is

(A)
K (VGS - VT ) 2
VDS En (B) 2K (VGS - VT )

(C) Id
VGS - VDS gin (D)
K (VGS - VT ) 2
VGS

2008 eer TWO MARKS

Q. 29
ing
The measured trans conductance gm of an NMOS transistor operating in the
linear region is plotted against the gate voltage VG at a constant drain voltage VD

.ne
. Which of the following figures represents the expected dependence of gm on VG ?

Q. 30 Silicon is doped with boron to a concentration of 4 # 1017 atoms cm3 . Assume


the intrinsic carrier concentration of silicon to be 1.5 # 1010 / cm 3 and the value
of kT/q to be 25 mV at 300 K. Compared to undopped silicon, the fermi level of
doped silicon
(A) goes down by 0.31 eV (B) goes up by 0.13 eV
(C) goes down by 0.427 eV (D) goes up by 0.427 eV

Downloaded From : www.EasyEngineering.net


Downloaded From : www.EasyEngineering.net
GATE SOLVED PAPER - EC ELECTRONIC DEVICES

Q. 31 The cross section of a JFET is shown in the following figure. Let Vc be - 2 V


and let VP be the initial pinch -off voltage. If the width W is doubled (with other
geometrical parameters and doping levels remaining the same), then the ratio
between the mutual trans conductances of the initial and the modified JFET is

1 - 2/Vp
(B) 1 e
2 1 - 1/2Vp o
(A) 4

ww (C) e
1 - 2/Vp
1 - 1/2Vp
o (D)
1 - (2 - Vp )
1 - [1 (2 Vp )]

w.E
Q. 32 Consider the following assertions.
S1 : For Zener effect to occur, a very abrupt junction is required.
S2 : For quantum tunneling to occur, a very narrow energy barrier is required.

asy
Which of the following is correct ?
(A) Only S2 is true

En
(B) S1 and S2 are both true but S2 is not a reason for S1
(C) S1 and S2 and are both true but S2 is not a reason for S1
(D) Both S1 and S2 are false
gin
2007 eer ONE MARK

Q. 33
ing
The electron and hole concentrations in an intrinsic semiconductor are ni per cm3
at 300 K. Now, if acceptor impurities are introduced with a concentration of NA

(A) ni (B) ni + NA .ne


per cm3 (where NA >> ni , the electron concentration per cm3 at 300 K will be)

Q. 34
(C) NA - ni
+
(D) ni
NA
2

t
In a p n junction diode under reverse biased the magnitude of electric field is
maximum at
(A) the edge of the depletion region on the p-side
(B) the edge of the depletion region on the n -side
(C) the p+ n junction
(D) the centre of the depletion region on the n -side

2007 TWO MARKS

Q. 35 Group I lists four types of p - n junction diodes. Match each device in Group I
with one of the option in Group II to indicate the bias condition of the device in
its normal mode of operation.
Group - I Group-II
(P) Zener Diode (1) Forward bias

Downloaded From : www.EasyEngineering.net


Downloaded From : www.EasyEngineering.net
GATE SOLVED PAPER - EC ELECTRONIC DEVICES

(Q) Solar cell (2) Reverse bias


(R) LASER diode
(S) Avalanche Photodiode
(A) P - 1, Q - 2, R - 1, S - 2
(B) P - 2, Q - 1, R - 1, S - 2
(C) P - 2, Q - 2, R - 1, S- -2
(D) P - 2, Q - 1, R - 2, S - 2

Q. 36 Group I lists four different semiconductor devices. match each device in Group I
with its charactecteristic property in Group II
Group-I Group-II
(P) BJT (1) Population iniversion
(Q) MOS capacitor (2) Pinch-off voltage

ww (R) LASER diode


(S) JFET
(3) Early effect
(4) Flat-band voltage

w.E (A) P - 3, Q - 1, R - 4, S - 2
(B) P - 1, Q - 4, R - 3, S - 2
(C) P - 3, Q - 4, R - 1, S - 2

asy
(D) P - 3, Q - 2, R - 1, S - 4

Q. 37

En
A p+ n junction has a built-in potential of 0.8 V. The depletion layer width a
reverse bias of 1.2 V is 2 mm. For a reverse bias of 7.2 V, the depletion layer width
will be
(A) 4 mm
gin(B) 4.9 mm

Q. 38
(C) 8 mm (D) 12 mm

eer
The DC current gain (b) of a BJT is 50. Assuming that the emitter injection
efficiency is 0.995, the base transport factor is
(A) 0.980 (B) 0.985 ing
(C) 0.990 (D) 0.995
.ne
Common Data For Q. 39 to 41
t
The figure shows the high-frequency capacitance - voltage characteristics of
Metal/Sio 2 /silicon (MOS) capacitor having an area of 1 # 10 - 4 cm 2 . Assume that
the permittivities (e0 er ) of silicon and Sio2 are 1 # 10 - 12 F/cm and 3.5 # 10 - 13 F/
cm respectively.

Q. 39 The gate oxide thickness in the MOS capacitor is


(A) 50 nm (B) 143 nm
(C) 350 nm (D) 1 mm

Downloaded From : www.EasyEngineering.net


Downloaded From : www.EasyEngineering.net
GATE SOLVED PAPER - EC ELECTRONIC DEVICES

Q. 40 The maximum depletion layer width in silicon is


(A) 0.143 mm (B) 0.857 mm
(C) 1 mm (D) 1.143 mm

Q. 41 Consider the following statements about the C - V characteristics plot :


S1 : The MOS capacitor has as n -type substrate
S2 : If positive charges are introduced in the oxide, the C - V polt will shift to
the left.
Then which of the following is true?
(A) Both S1 and S2 are true
(B) S1 is true and S2 is false
(C) S1 is false and S2 is true
(D) Both S1 and S2 are false

ww 2006 ONE MARK

w.E
Q. 42 The values of voltage (VD) across a tunnel-diode corresponding to peak and valley
currents are Vp, VD respectively. The range of tunnel-diode voltage for VD which

asy
the slope of its I - VD characteristics is negative would be
(A) VD < 0
(C) Vp # VD < Vv
(B) 0 # VD < Vp
(D) VD $ Vv

Q. 43
En
The concentration of minority carriers in an extrinsic semiconductor under
equilibrium is
gin
(A) Directly proportional to doping concentration

eer
(B) Inversely proportional to the doping concentration
(C) Directly proportional to the intrinsic concentration

ing
(D) Inversely proportional to the intrinsic concentration

Under low level injection assumption, the injected minority carrier current for an

.ne
Q. 44
extrinsic semiconductor is essentially the
(A) Diffusion current (B) Drift current

Q. 45
(C) Recombination current
t
(D) Induced current

The phenomenon known as “Early Effect” in a bipolar transistor refers to a


reduction of the effective base-width caused by
(A) Electron - hole recombination at the base
(B) The reverse biasing of the base - collector junction
(C) The forward biasing of emitter-base junction
(D) The early removal of stored base charge during saturation-to-cut off
switching

2006 TWO MARKS

Q. 46 In the circuit shown below, the switch was connected to position 1 at t < 0 and
at t = 0 , it is changed to position 2. Assume that the diode has zero voltage drop
and a storage time ts . For 0 < t # ts, vR is given by (all in Volts)

Downloaded From : www.EasyEngineering.net


Downloaded From : www.EasyEngineering.net
GATE SOLVED PAPER - EC ELECTRONIC DEVICES

(A) vR =- 5 (B) vR =+ 5
(C) 0 # vR < 5 (D) - 5 # vR < 0

Q. 47 The majority carriers in an n-type semiconductor have an average drift velocity


v in a direction perpendicular to a uniform magnetic field B . The electric field E
induced due to Hall effect acts in the direction
(A) v # B (B) B # v
(C) along v (D) opposite to v

ww
Q. 48 Find the correct match between Group 1 and Group 2
Group 1 Group 2

w.E E - Varactor diode


F - PIN diode
G - Zener diode
1. Voltage reference
2. High frequency switch
3. Tuned circuits

asy
H - Schottky diode 4. Current controlled attenuator
(A) E - 4, F - 2, G - 1, H - 3

En
(B) E - 3, F - 4, G - 1, H - 3
(C) E - 2, F - 4, G - 1, H - 2
(D) E - 1, F - 3, G - 2, H - 4
gin
Q. 49

Hole-electron ratio : 0.4 eer


A heavily doped n - type semiconductor has the following data:

Doping concentration
Intrinsic concentration
: 4.2 # 108 atoms/m3
: 1.5 # 10 4 atoms/m 3
ing
The ratio of conductance of the n -type semiconductor to that of the intrinsic

(A) 0.00005 (B) 2000 .ne


semiconductor of same material and ate same temperature is given by

(C) 10000 (D) 20000


t
2005 ONE MARK

Q. 50 The bandgap of Silicon at room temperature is


(A) 1.3 eV (B) 0.7 eV
(C) 1.1 eV (D) 1.4 eV

Q. 51 A Silicon PN junction at a temperature of 20c C has a reverse saturation current


of 10 pico - Ameres (pA). The reserve saturation current at 40cC for the same
bias is approximately
(A) 30 pA (B) 40 pA
(C) 50 pA (D) 60 pA

Q. 52 The primary reason for the widespread use of Silicon in semiconductor device
technology is
(A) abundance of Silicon on the surface of the Earth.

Downloaded From : www.EasyEngineering.net


Downloaded From : www.EasyEngineering.net
GATE SOLVED PAPER - EC ELECTRONIC DEVICES

(B) larger bandgap of Silicon in comparison to Germanium.


(C) favorable properties of Silicon - dioxide (SiO2)
(D) lower melting point

2005 TWO MARKS

Q. 53 A Silicon sample A is doped with 1018 atoms/cm 3 of boron. Another sample B


of identical dimension is doped with 1018 atoms/cm 3 phosphorus. The ratio of
electron to hole mobility is 3. The ratio of conductivity of the sample A to B is
(A) 3 (B) 1
3
(C) 2 (D) 3
3 2
Q. 54 A Silicon PN junction diode under reverse bias has depletion region of width 10

ww mm. The relative permittivity of Silicon, er = 11.7 and the permittivity of free
space e0 = 8.85 # 10 - 12 F/m. The depletion capacitance of the diode per square

w.E meter is
(A) 100 mF
(C) 1 mF
(B) 10 mF
(D) 20 mF

Q. 55
asy
A MOS capacitor made using p type substrate is in the accumulation mode. The
dominant charge in the channel is due to the presence of
(A) holes
En (B) electrons

Q. 56
(C) positively charged icons

gin (D) negatively charged ions

For an n -channel MOSFET and its transfer curve shown in the figure, the
threshold voltage is
eer
ing
.ne
(A) 1 V and the device is in active region
(B) - 1 V and the device is in saturation region
t
(C) 1 V and the device is in saturation region
(D) - 1 V and the device is an active region

2004 ONE MARK

Q. 57 The impurity commonly used for realizing the base region of a silicon n - p - n
transistor is
(A) Gallium (B) Indium
(C) Boron (D) Phosphorus

Q. 58 If for a silicon npn transistor, the base-to-emitter voltage (VBE ) is 0.7 V and the
collector-to-base voltage (VCB) is 0.2 V, then the transistor is operating in the
(A) normal active mode (B) saturation mode
(C) inverse active mode (D) cutoff mode

Downloaded From : www.EasyEngineering.net


Downloaded From : www.EasyEngineering.net
GATE SOLVED PAPER - EC ELECTRONIC DEVICES

Q. 59 Consider the following statements S1 and S2.


S1 : The b of a bipolar transistor reduces if the base width is increased.
S2 : The b of a bipolar transistor increases if the dopoing concentration in the
base is increased.
Which remarks of the following is correct ?
(A) S1 is FALSE and S2 is TRUE
(B) Both S1 and S2 are TRUE
(C) Both S1 and S2 are FALSE
(D) S1 is TRUE and S2 is FALSE

Q. 60 Given figure is the voltage transfer characteristic of

ww
w.E (A) an NOMS inverter with enhancement mode transistor as load

asy
(B) an NMOS inverter with depletion mode transistor as load
(C) a CMOS inverter
(D) a BJT inverter
En
Q. 61

gin
Assuming VCEsat = 0.2 V and b = 50 , the minimum base current (IB) required to
drive the transistor in the figure to saturation is

eer
ing
.ne
(A) 56 mA
(C) 60 mA
(B) 140 mA
(D) 3 mA
t
2004 TWO MARKS

Q. 62 In an abrupt p - n junction, the doping concentrations on the p -side and n -side


are NA = 9 # 1016 /cm 3 respectively. The p - n junction is reverse biased and the
total depletion width is 3 mm. The depletion width on the p -side is
(A) 2.7 mm (B) 0.3 mm
(C) 2.25 mm (D) 0.75 mm

Q. 63 The resistivity of a uniformly doped n -type silicon sample is 0.5W - mc. If


the electron mobility (mn) is 1250 cm 2 /V-sec and the charge of an electron is
1.6 # 10 - 19 Coulomb, the donor impurity concentration (ND) in the sample is
(A) 2 # 1016 /cm 3 (B) 1 # 1016 /cm 3
(C) 2.5 # 1015 /cm 3 (D) 5 # 1015 /cm 3

Downloaded From : www.EasyEngineering.net


Downloaded From : www.EasyEngineering.net
GATE SOLVED PAPER - EC ELECTRONIC DEVICES

Q. 64 Consider an abrupt p - n junction. Let Vbi be the built-in potential of this junction
and VR be the applied reverse bias. If the junction capacitance (Cj ) is 1 pF for
Vbi + VR = 1 V, then for Vbi + VR = 4 V, Cj will be
(A) 4 pF (B) 2 pF
(C) 0.25 pF (D) 0.5 pF

Q. 65 Consider the following statements Sq and S2.


S1 : The threshold voltage (VT ) of MOS capacitor decreases with increase in
gate oxide thickness.
S2 : The threshold voltage (VT ) of a MOS capacitor decreases with increase in
substrate doping concentration.
Which Marks of the following is correct ?
(A) S1 is FALSE and S2 is TRUE

ww (B) Both S1 and S2 are TRUE


(C) Both S1 and S2 are FALSE
(D) S1 is TRUE and S2 is FALSE

w.E
Q. 66 The drain of an n-channel MOSFET is shorted to the gate so that VGS = VDS . The
threshold voltage (VT ) of the MOSFET is 1 V. If the drain current (ID) is 1 mA

(A) 2 mA asy
for VGS = 2 V, then for VGS = 3 V, ID is
(B) 3 mA
(C) 9 mA
En (D) 4 mA

Q. 67

gin
The longest wavelength that can be absorbed by silicon, which has the bandgap
of 1.12 eV, is 1.1 mm. If the longest wavelength that can be absorbed by another
material is 0.87 mm, then bandgap of this material is
(A) 1.416 A/cm 2
(B) 0.886 eV eer
(C) 0.854 eV
(D) 0.706 eV
ing
Q. 68
.ne
The neutral base width of a bipolar transistor, biased in the active region, is

t
0.5 mm. The maximum electron concentration and the diffusion constant in
the base are 1014 / cm 3 and Dn = 25 cm 2 /sec respectively. Assuming negligible
recombination in the base, the collector current density is (the electron charge is
1.6 # 10 - 19 Coulomb)
(A) 800 A/cm 2 (B) 8 A/cm 2
(C) 200 A/cm 2 (D) 2 A/cm 2

2003 ONE MARK

Q. 69 n -type silicon is obtained by doping silicon with


(A) Germanium (B) Aluminium
(C) Boron (D) Phosphorus

Q. 70 The Bandgap of silicon at 300 K is


(A) 1.36 eV (B) 1.10 eV
(C) 0.80 eV (D) 0.67 eV

Downloaded From : www.EasyEngineering.net


Downloaded From : www.EasyEngineering.net
GATE SOLVED PAPER - EC ELECTRONIC DEVICES

Q. 71 The intrinsic carrier concentration of silicon sample at 300 K is 1.5 # 1016 /m 3 . If


after doping, the number of majority carriers is 5 # 1020 /m 3 , the minority carrier
density is
(A) 4.50 # 1011/m 3 (B) 3.333 # 10 4 /m 3
(C) 5.00 # 1020 /m 3 (D) 3.00 # 10 - 5 /m 3

Q. 72 Choose proper substitutes for X and Y to make the following statement correct
Tunnel diode and Avalanche photo diode are operated in X bias ad Y bias
respectively
(A) X: reverse, Y: reverse
(B) X: reverse, Y: forward
(C) X: forward, Y: reverse
(D) X: forward, Y: forward

ww
Q. 73 For an n - channel enhancement type MOSFET, if the source is connected at a
higher potential than that of the bulk (i.e. VSB > 0 ), the threshold voltage VT of

w.E the MOSFET will


(A) remain unchanged
(C) change polarity
(B) decrease
(D) increase

2003
asy TWO MARKS

Q. 74
En
An n -type silicon bar 0.1 cm long and 100 mm2 i cross-sectional area has a

gin
majority carrier concentration of 5 # 1020 /m 2 and the carrier mobility is 0.13
m2 /V-s at 300 K. If the charge of an electron is 1.5 # 10 - 19 coulomb, then the
resistance of the bar is
(A) 106 Ohm
(C) 10 - 1 Ohm
(B) 10 4 Ohm
eer
(D) 10 - 4 Ohm

Q. 75 ing
The electron concentration in a sample of uniformly doped n -type silicon at 300

.ne
K varies linearly from 1017 /cm 3 at x = 0 to 6 # 1016 /cm 3 at x = 2mm . Assume a
situation that electrons are supplied to keep this concentration gradient constant

t
with time. If electronic charge is 1.6 # 10 - 19 coulomb and the diffusion constant
Dn = 35 cm 2 /s, the current density in the silicon, if no electric field is present, is
(A) zero
(B) -112 A/cm 2
(C) +1120 A/cm 2
(D) -1120 A/cm 2

Q. 76 Match items in Group 1 with items in Group 2, most suitably.


Group 1 Group 2
P. LED 1. Heavy doping
Q. Avalanche photo diode 2. Coherent radiation
R. Tunnel diode 3. Spontaneous emission
S. LASER 4. Current gain
(A) P - 1, Q - 2, R - 4, S - 3
(B) P - 2, Q - 3, R - 1, S - 4
(C) P - 3 Q - 4, R - 1, S - 2
(D) P - 2, Q - 1, R - 4, S - 3

Downloaded From : www.EasyEngineering.net


Downloaded From : www.EasyEngineering.net
GATE SOLVED PAPER - EC ELECTRONIC DEVICES

Q. 77 At 300 K, for a diode current of 1 mA, a certain germanium diode requires a


forward bias of 0.1435 V, whereas a certain silicon diode requires a forward bias
of 0.718 V. Under the conditions state above, the closest approximation of the
ratio of reverse saturation current in germanium diode to that in silicon diode is
(A) 1 (B) 5
(C) 4 # 103 (D) 8 # 103

Q. 78 A particular green LED emits light of wavelength 5490 Ac. The energy bandgap
of the semiconductor material used there is
(Plank’s constant = 6.626 # 10 - 34 J - s )
(A) 2.26 eV (B) 1.98 eV
(C) 1.17 eV (D) 0.74 eV

Q. 79 When the gate-to-source voltage (VGs) of a MOSFET with threshold voltage of

ww 400 mV, working in saturation is 900 mV, the drain current is observed to be
1 mA. Neglecting the channel width modulation effect and assuming that the
MOSFET is operating at saturation, the drain current for an applied VGS of 1400

w.E mV is
(A) 0.5 mA (B) 2.0 mA

Q. 80
(C) 3.5 mA

asy (D) 4.0 mA

If P is Passivation, Q is n -well implant, R is metallization and S is source/

En
drain diffusion, then the order in which they are carried out in a standard n -well
CMOS fabrication process, is
(A) P - Q - R - S
(B) Q - S - R - P gin
(C) R - P - S - Q
(D) S - R - Q - P eer
Q. 81
(A) Current controlled current source ing
The action of JFET in its equivalent circuit can best be represented as a

(B) Current controlled voltage source


(C) Voltage controlled voltage source .ne
(D) Voltage controlled current source
t
2002 ONE MARK

Q. 82 In the figure, silicon diode is carrying a constant current of 1 mA. When the
temperature of the diode is 20cC, VD is found to be 700 mV. If the temperature
rises to 40cC, VD becomes approximately equal to

(A) 740 mV (B) 660 mV


(C) 680 mV (D) 700 mV

Downloaded From : www.EasyEngineering.net


Downloaded From : www.EasyEngineering.net
GATE SOLVED PAPER - EC ELECTRONIC DEVICES

Q. 83 If the transistor in the figure is in saturation, then

(A) IC is always equal to bdc IB


(B) IC is always equal to - bde IB
(C) IC is greater than or equal to bdc IB
(D) IC is less than or equal to bdc IB

ww
Q. 84
2001

MOSFET can be used as a


ONE MARK

w.E (A) current controlled capacitor


(B) voltage controlled capacitor

asy
(C) current controlled inductor
(D) voltage controlled inductor

Q. 85

(A) gate voltage En


The effective channel length of MOSFET in saturation decreases with increase in

(B) drain voltage


(C) source voltage
gin
(D) body voltage
eer
1999 ing ONE MARK

Q. 86
(A) fast turn-on .ne
The early effect in a bipolar junction transistor is caused by

(B) fast turn-off


(C) large collector-base reverse bias
(D) large emitter-base forward bias
t
1999 TWO MARKS

Q. 87 An n -channel JEFT has IDSS = 2 mA and Vp =- 4 V . Its transconductance gm (in


milliohm) for an applied gate-to-source voltage VGS of - 2 V is
(A) 0.25 (B) 0.5
(C) 0.75 (D) 1.0

Q. 88 An npn transistor (with C = 0.3 pF ) has a unity-gain cutoff frequency fT of


400 MHz at a dc bias current Ic = 1 mA . The value of its Cm (in pF) is approximately
(VT = 26 mV)
(A) 15 (B) 30
(C) 50 (D) 96

Downloaded From : www.EasyEngineering.net


Downloaded From : www.EasyEngineering.net
GATE SOLVED PAPER - EC ELECTRONIC DEVICES

1998 ONE MARK

Q. 89 The electron and hole concentrations in a intrinsic semiconductor are ni and


pi respectively. When doped with a p-type material, these change to n and p,
respectively, Then
(A) n + p = ni + pi (B) n + ni = p + pi
(C) npi = ni p (D) np = ni pi

Q. 90 The fT of a BJT is related to its gm, C p and C m as follows


Cp + Cm 2p (C p + C m)
(A) fT = (B) fT =
gm gm
gm gm
(C) fT = (D) fT =
Cp + Cm 2p (C p + C m)
Q. 91 The static characteristic of an adequately forward biased p-n junction is a straight

ww line, if the plot is of


(A) log I vs log V
(C) I vs log V
(B) log I vs V
(D) I vs V

w.E
Q. 92 A long specimen of p-type semiconductor material
(A) is positively charged

asy
(B) is electrically neutral
(C) has an electric field directed along its length

En
(D) acts as a dipole

Q. 93

gin
Two identical FETs, each characterized by the parameters gm and rd are connected
in parallel. The composite FET is then characterized by the parameters
g
(A) m and 2rd
2
(C) 2gm and rd
g
2
eer
(B) m and rd
2

Q. 94 The units of
2
q
are
ing
(D) 2gm and 2rd

(A) V
(C) J
kT
(B) V-1
(D) J/K .ne
1997
t ONE MARK

Q. 95 For a MOS capacitor fabricated on a p-type semiconductor, strong inversion


occurs when
(A) surface potential is equal to Fermi potential
(B) surface potential is zero
(C) surface potential is negative and equal to Fermi potential in magnitude
(D) surface potential is positive and equal to twice the Fermi potential

Q. 96 The intrinsic carrier density at 300 K is 1.5 # 1010 /cm3 , in silicon. For n -type
silicon doped to 2.25 # 1015 atoms/cm3 , the equilibrium electron and hole densities
are
(A) n = 1.5 # 1015 /cm3, p = 1.5 # 1010 /cm3
(B) n = 1.5 # 1010 /cm3, p = 2.25 # 1015 /cm3
(C) n = 2.25 # 1015 /cm3, p = 1.0 # 1015 /cm3

Downloaded From : www.EasyEngineering.net


Downloaded From : www.EasyEngineering.net
GATE SOLVED PAPER - EC ELECTRONIC DEVICES

(D) n = 1.5 # 1010 /cm3, p = 1.5 # 1010 /cm3

1996 ONE MARK

Q. 97 The p-type substrate in a conventional pn -junction isolated integrated circuit


should be connected to
(A) nowhere, i.e. left floating
(B) a DC ground potential
(C) the most positive potential available in the circuit
(D) the most negative potential available in the circuit

Q. 98 If a transistor is operating with both of its junctions forward biased, but with the
collector base forward bias greater than the emitter base forward bias, then it is
operating in the

ww (A) forward active mode


(B) reverse saturation mode

w.E (C) reverse active mode


(D) forward saturation mode

Q. 99

asy
The common-emitter short-circuit current gain b of a transistor
(A) is a monotonically increasing function of the collector current IC

En
(B) is a monotonically decreasing function of IC
(C) increase with IC , for low IC , reaches a maximum and then decreases with
further increase in IC
(D) is not a function of IC gin
Q. 100
eer
A n -channel silicon (Eg = 1.1 eV) MOSFET was fabricated using n +poly-silicon

ing
gate and the threshold voltage was found to be 1 V. Now, if the gate is changed
to p+ poly-silicon, other things remaining the same, the new threshold voltage
should be
(A) - 0.1 V
(C) 1.0 V
(B) 0 V
(D) 2.1 V .ne
1996
t TWO MARKS

Q. 101 In a bipolar transistor at room temperature, if the emitter current is doubled the
voltage across its base-emitter junction
(A) doubles (B) halves
(C) increases by about 20 mV (D) decreases by about 20 mV

Q. 102 An npn transistor has a beta cut-off frequency fb of 1 MHz and common emitter
short circuit low-frequency current gain bo of 200 it unity gain frequency fT and
the alpha cut-off frequency fa respectively are
(A) 200 MHz, 201 MHz
(B) 200 MHz, 199 MHz
(C) 199 MHz, 200 MHz
(D) 201 MHz, 200 MHz

Downloaded From : www.EasyEngineering.net


Downloaded From : www.EasyEngineering.net
GATE SOLVED PAPER - EC ELECTRONIC DEVICES

Q. 103 A silicon n MOSFET has a threshold voltage of 1 V and oxide thickness of Ao.
[er (SiO 2) = 3.9, e0 = 8.854 # 10-14 F/cm, q = 1.6 # 10-19 C]
The region under the gate is ion implanted for threshold voltage tailoring. The
dose and type of the implant (assumed to be a sheet charge at the interface)
required to shift the threshold voltage to - 1 V are
(A) 1.08 # 1012 /cm2 , p-type
(B) 1.08 # 1012 /cm2 , n-type
(C) 5.4 # 1011 /cm2 , p-type
(D) 5.4 # 1011 /cm2 , n-type

***********

ww
w.E
asy
En
gin
eer
ing
.ne
t

Downloaded From : www.EasyEngineering.net


Downloaded From : www.EasyEngineering.net
GATE SOLVED PAPER - EC ELECTRONIC DEVICES

SOLUTIONS

Sol. 1 Option (A) is correct.


The potential barrier of the pn junction is lowered when a forward bias voltage
is applied, allowing electrons and holes to flow across the space charge region
(Injection) when holes flow from the p region across the space charge region into
the n region, they become excess minority carrier holes and are subject to diffuse,
drift and recombination processes.
Sol. 2 Option (D) is correct.

ww
Sol. 3
In IC technology, dry oxidation as compared to wet oxidation produces superior
quality oxide with a lower growth rate
Option (D) is correct.

w.E In a MOSFET operating in the saturation region, the channel length modulation
effect causes a decrease in output resistance.
Sol. 4

Given, asy
Option (A) is correct.

En
VB = 2V
VTN = 1V
So, we have
Drain voltage VD = 2 volt gin
VG = 2 volt
VS = 0 (Ground) eer
Therefore,
and
VGS = 2 > VTN
VDS = 2 > VGS - VTN
ing
So, the MOSFET is in the saturation region. Therefore, drain current is
ID = kN ^VGS - VTN h2
ID = kN ^VB - 1h
.ne
t
2
or,
Differentiating both side with respect to ID
1 = kN 2 ^VB - 1hdVB
dID
Since, VBQ = 2 volt (at D.C. Voltage)
Hence, we obtain
dVB = 1 = 1
dID 2kN ^VB - 1h 2 # 40 # 10-6 # ^2 - 1h
= 12.5 # 103 W = 12.5 kW
Sol. 5 Option (D) is correct.
For the semiconductor, n 0 p 0 = n i2
2 20
p 0 = n i = 1019 = 10 per cm3
n 0 10
Volume of given device, V = Area # depth = 1 mm2 # 1 mm
= 10-8 cm2 # 10-4 cm = 10-12 cm3
So total no. of holes is,

Downloaded From : www.EasyEngineering.net


Downloaded From : www.EasyEngineering.net
GATE SOLVED PAPER - EC ELECTRONIC DEVICES

p = p 0 # V = 10 # 10-12 = 10-11
Which is approximately equal to zero.
Sol. 6 Option (A) is correct.
Given the circuit as below :

ww Since all the parameters of PMOS and NMOS are equal.


So, mn = mp

w.E COX bW l = COX bW l = COX bW l


L M1 L M2 L
Given that M1 is in linear region. So, we assume that M2 is either in cutoff or
saturation.

asy
Case 1 : M2 is in cut off
So, I 2 = I1 = 0

En
Where I1 is drain current in M1 and I2 is drain current in M2 .
Since,
m C

m C
2 L
gin
I1 = p OX bW l82VSD ^VSG - VTp h - V SD
2
B

&
Solving it we get,
2 L
eer
0 = p OX bW l [2VSD ^VSG - VTp h - V SD
2
]

2 ^VSG - VTp h = VSD


2 ^5 - Vin - 1h = 5 - VD
ing
.ne
&
& Vin = VD + 3
2
For
So,
So for the NMOS
I1 = 0 , VD = 5 V
Vin = 5 + 3 = 4 V
2
t
VGS = Vin - 0 = 4 - 0 = 4 V and VGS > VTn
So it can’t be in cutoff region.
Case 2 : M2 must be in saturation region.
So, I1 = I 2
mp COX W mn COX W
2 L6 @ = 2 L (VGS - VTn)
2 2
2 (VSG - VTp) VSD - V SD
2
& 2 (VSG - VTp) VSD - V SD = (VGS - VTn) 2
& 2 (5 - Vin - 1) (5 - VD) - (5 - VD) 2 = (Vin - 0 - 1) 2
& 2 (4 - Vin) (5 - VD) - (5 - VD) 2 = (Vin - 1) 2
Substituting VD = VDS = VGS - VTn and for N -MOS & VD = Vin - 1
& 2 (4 - Vin) (6 - Vin) - (6 - Vin) 2 = (Vin - 1) 2

Downloaded From : www.EasyEngineering.net


Downloaded From : www.EasyEngineering.net
GATE SOLVED PAPER - EC ELECTRONIC DEVICES

& 48 - 36 - 8Vin =- 2Vin + 1


& 6Vin = 11
& Vin = 11 = 1.833 V
6
So for M2 to be in saturation Vin < 1.833 V or Vin < 1.875 V
Sol. 7 Option (B) is correct.
Gate source overlap capacitance.
Co = dWeox e0 (medium Sio 2 )
tox
-9 -6 -12
= 20 # 10 # 1 # 10 #-93.9 # 8.9 # 10 = 0.69 # 10-15 F
1 # 10
Sol. 8 Option (B) is correct.
Source body junction capacitance.

ww Cs = Aer e0
d
A = (0.2 mm + 0.2 mm + 0.2 mm) # 1 mm + 2 (0.2 mm # 0.2 mm)

w.E = 0.68 mm2


d = 10 nm (depletion width of all junction)

asy -12
Cs = 0.68 # 10 # 11.7 -#
10 # 10 9
8.9 # 10-12 = 7 10-15 F
#

Sol. 9

Drift current En
Option (C) is correct.
Id = qnmn E

Sol. 10 Option (B) is correct. gin


It depends upon Electric field E and carrier concentration n

Zener diode operates in reverse breakdown region.


eer
ing
.ne
t
Sol. 11 Option (D) is correct.
For every 1c C increase in temperature, forward bias voltage across diode decreases
by 2.5 mV. Thus for 10c C increase, there us 25 mV decreases.
Sol. 12 Option (B) is correct.
Full channel resistance is
r L
r # = 600 W ...(1)
W#a
If VGS is applied, Channel resistance is
r L VGS
where b = a c1 -
Vp m
rl = #
W#b
Pinch off voltage,
qN
Vp = D a2 ...(2)
2e

Downloaded From : www.EasyEngineering.net


Downloaded From : www.EasyEngineering.net
GATE SOLVED PAPER - EC ELECTRONIC DEVICES

If depletion on each side is d = 1 μm at VGS = 0 .


qN
Vj = D d2
2e
qN qND
or 1 = D (1 # 10-6) 2 & = 1012
2e 2e
Now from equation (2), we have
Vp = 1012 # (5 # 10-6) 2
or Vp =- 25 V
At VGS =- 3 V ;
b = 5 b1 - - 3 mm = 3.26 mm
- 25 l
rL rL a = 600 5
rl = = # 3.26 = 917 W
W # b Wa # b

ww
Sol. 13 Option (C) is correct.
At VGS = 0 V , b = 4 mm since 2b = 8 mm

w.E
Sol. 14
Thus

Option (A) is correct.


rl =
rL a = 600 5 = 750 W
Wa # b #4

asy
At room temperature mobility of electrons for Si sample is given mn = 1350 cm2 /Vs
. For an n -channel MOSFET to create an inversion layer of electrons, a large

En
positive gate voltage is to be applied. Therefore, induced electric field increases
and mobility decreases.

Sol. 15 Option (B) is correct. gin


So, Mobility mn < 1350 cm2 /Vs for n -channel MOSFET

Sol. 16 Option (B) is correct. eer


Dry oxidation is used to achieve high quality oxide growth.

Emitter injection efficiency is given as


g = 1 ing
To achieve
1 + NB
NE
g = 1, NE >> NB .ne
Sol. 17 Option (C) is correct.
t
Reverse bias breakdown or Zener effect occurs in highly doped PN junction
through tunneling mechanism. In a highly doped PN junction, the conduction
and valence bands on opposite sides of the junction are sufficiently close during
reverse bias that electron may tunnel directly from the valence band on the p-side
into the conduction band on n -side.
Breakdown voltage VB \ 1
NA ND
So, breakdown voltage decreases as concentration increases
Depletion capacitance
C =' ees NA ND 1/2

2 (Vbi + VR) (NA + ND) 1


Thus C \ NA ND
Depletion capacitance increases as concentration increases
Sol. 18 Option (C) is correct.
Sample is in thermal equilibrium so, electric field

Downloaded From : www.EasyEngineering.net


Downloaded From : www.EasyEngineering.net
GATE SOLVED PAPER - EC ELECTRONIC DEVICES

E = 1 = 10 kV/cm
1 mm
Sol. 19 Option (A) is correct.
Electron drift current density
Jd = ND mn eE = 1016 # 1350 # 1.6 # 10-19 # 10 # 1013
= 2.16 # 10 4 A/cm2
Sol. 20 Option (C) is correct.
Only dopant atoms can have concentration of 4 # 1019 cm - 3 in n -type silicon at
room temperature.
Sol. 21 Option (A) is correct.
2
Unit of mobility mn is = cm
V. sec

ww
2
Unit of diffusion current Dn is = cm
sec
m 2 2
Thus unit of n is = cm / cm = 1 = V-1

w.E
Sol. 22
Dn
Option (D) is correct.
V $ sec sec V

Sol. 23
asy
Both S1 and S2 are true and S2 is a reason for S1.
Option (B) is correct.
We know that

En
NA WP = ND WN
or
17

gin -6
NA = ND WN = 1 # 10 # 0.1-6# 10 = 1 # 1016
WP 1 # 10
The built-in potential is
Vbi = VT 1nc NA N
n i2 m
D
eer
17
= 26 # 10-3 ln e 1 # 10 # 1 10 ing16
# 10 = 0.760
(1.4 # 10 ) 2 o

Sol. 24 Option (B) is correct.


.ne
The peak electric field in device is directed from p to n and is
E =- eND xn
es
= eND xn
t from p to n

from n to p
es
-19 17 -5
= 1.6 # 10 # 1 #-10 14
# 1 # 10 = 0.15 MV/cm
8.85 # 10 # 12
Sol. 25 Option (D) is correct.
Channel length modulation is not associated with a p - n junction. It is being
associated with MOSFET in which effective channel length decreases, producing
the phenomenon called channel length modulation.
Sol. 26 Option (A) is correct.
Trivalent impurities are used for making p - type semiconductors. So, Silicon
wafer heavily doped with boron is a p+ substrate.
Sol. 27 Option (D) is correct.
Oxidation rate is zero because the existing oxide prevent the further oxidation.

Downloaded From : www.EasyEngineering.net


Downloaded From : www.EasyEngineering.net
GATE SOLVED PAPER - EC ELECTRONIC DEVICES

Sol. 28 Option (B) is correct.


gm = 2ID = 2 K (VGS - VT ) 2 = 2K (VGS - VT )
2VGS 2VGS
Sol. 29 Option (C) is correct.
As VD = constant
Thus gm \ (VGS - VT ) Which is straight line.
Sol. 30 Option (C) is correct.
E2 - E1 = kT ln NA
ni
NA = 4 # 1017
ni = 1.5 # 1010
17
E2 - E1 = 25 # 10-3 e ln 4 # 10 10 = 0.427 eV
1.5 # 10

ww
Sol. 31
Hence fermi level goes down by 0.427 eV as silicon is doped with boron.
Option (C) is correct.

w.E Pinch off voltage


2
VP = eW ND
es
Let
Now asy VP = VP1
VP1 = W12 = W2
VP2 W22 (2W) 2
or
En 4VP1 = VP2
Initial transconductance
gin
gm = Kn ;1 - Vbi - VGS E
Vp

For first condition gm1 = Kn =1 - eer


0 - (- 2)
VP1 G
= Kn ;1 - 2
VP1 E
For second condition
ing
gm2 = Kn =1 -
0 - (- 2)
VP2 G
.ne
= K2 ;1 - 2
4VP1 E

Dividing

Hence
gm1
gm2
=f
1 - 2/VP1
1 - 1/ (2VP1)
VP = VP1
p
t
Sol. 32 Option (A) is correct.
Sol. 33 Option (D) is correct.
As per mass action law
np = ni2
If acceptor impurities are introduces
p = NA
Thus nNA = ni2
2
or n = ni
NA
Sol. 34 Option (C) is correct.
The electric field has the maximum value at the junction of p+ n .
Sol. 35 Option (B) is correct.

Downloaded From : www.EasyEngineering.net


Downloaded From : www.EasyEngineering.net
GATE SOLVED PAPER - EC ELECTRONIC DEVICES

Zener diode and Avalanche diode works in the reverse bias and laser diode works
in forward bias.
In solar cell diode works in forward bias but photo current is in reverse direction.
Thus
Zener diode : Reverse Bias
Solar Cell : Forward Bias
Laser Diode : Forward Bias
Avalanche Photo diode : Reverse Bias

Sol. 36 Option (C) is correct.


In BJT as the B-C reverse bias voltage increases, the B-C space charge region
width increases which xB (i.e. neutral base width) > A change in neutral base

ww width will change the collector current. A reduction in base width will causes
the gradient in minority carrier concentration to increase, which in turn causes
an increased in the diffusion current. This effect si known as base modulation as

w.E early effect.


In JFET the gate to source voltage that must be applied to achieve pinch off
voltage is described as pinch off voltage and is also called as turn voltage or

asy
threshold voltage.
In LASER population inversion occurs on the condition when concentration of

En
electrons in one energy state is greater than that in lower energy state, i.e. a non
equilibrium condition.

gin
In MOS capacitor, flat band voltage is the gate voltage that must be applied to
create flat ban condition in which there is no space charge region in semiconductor
under oxide.
Therefore eer
BJT : Early effect
MOS capacitor : Flat-band voltage ing
LASER diode : Population inversion
JFET : Pinch-off voltage .ne
Sol. 37 Option (A) is correct.

Now
W = K V + VR
2m = K 0.8 + 1.2
t
From above two equation we get
W = 0.8 + 7.2 = 8 =2
2m 0.8 + 1.2 2
or W2 = 4 m m
Sol. 38 Option (B) is correct.
b
a= = 50 = 50
b + 1 50 + 1 51
Current Gain = Base Transport Factor # Emitter injection Efficiency
a = b1 # b2
or b1 = a = 50 = 0.985
b2 51 # 0.995

Downloaded From : www.EasyEngineering.net


Downloaded From : www.EasyEngineering.net
GATE SOLVED PAPER - EC ELECTRONIC DEVICES

Sol. 39 Option (A) is correct.


At low voltage when there is no depletion region and capacitance is decide by
SiO2 thickness only,
C = e0 er1 A
D
-13
or D = e0 er1 A = 3.5 # 10 -# 10-4 = 50 nm
C 7 # 10 12

Sol. 40 Option (B) is correct.


The construction of given capacitor is shown in fig below

ww
w.E When applied voltage is 0 volts, there will be no depletion region and we get

asy C1 = 7 pF
When applied voltage is V , a depletion region will be formed as shown in fig an

En
total capacitance is 1 pF. Thus
CT = 1 pF
or CT = C1 C2 = 1 pF
C1 + C2
gin
or 1
CT
= 1 + 1
C1 C2
Substituting values of CT and C1 we get eer
C2 = 7 pF
6 ing
Now D2 = e0 er2 A
C2 .ne
- 12 -4
= 1 # 710 #- 1210 = 6 # 10 - 4 cm
6 # 10
= 0.857 mm
7 t
Sol. 41 Option (C) is correct.
Depletion region will not be formed if the MOS capacitor has n type substrate
but from C-V characteristics, C reduces if V is increased. Thus depletion region
must be formed. Hence S1 is false
If positive charges is introduced in the oxide layer, then to equalize the effect the
applied voltage V must be reduced. Thus the C - V plot moves to the left. Hence
S2 is true.
Sol. 42 Option (C) is correct.
For the case of negative slope it is the negative resistance region

Downloaded From : www.EasyEngineering.net


Downloaded From : www.EasyEngineering.net
GATE SOLVED PAPER - EC ELECTRONIC DEVICES

Sol. 43 Option (A) is correct.


For n -type p is minority carrier concentration
np = ni2
np = Constant Since ni is constant
p \ 1
n
Thus p is inversely proportional to n .

ww
Sol. 44 Option (A) is correct.
Diffusion current, since the drift current is negligible for minority carrier.

w.E
Sol. 45 Option (B) is correct.
In BJT as the B-C reverse bias voltage increases, the B-C space charge region
width increases which xB (i.e. neutral base width) > A change in neutral base

asy
width will change the collector current. A reduction in base width will causes
the gradient in minority carrier concentration to increases, which in turn causes

En
an increases in the diffusion current. This effect si known as base modulation as
early effect.
Sol. 46 Option (A) is correct.
gin
For t < 0 diode forward biased and VR = 5 . At t = 0 diode abruptly changes to

eer
reverse biased and current across resistor must be 0. But in storage time 0 < t < ts
diode retain its resistance of forward biased. Thus for 0 < t < ts it will be ON and

Sol. 47
VR =- 5 V
Option (B) is correct. ing
.ne
According to Hall effect the direction of electric field is same as that of direction
of force exerted.

Sol. 48
or
E =- v # B
E = B#v
Option (B) is correct.
t
The varacter diode is used in tuned circuit as it can provide frequently stability.
PIN diode is used as a current controlled attenuator.
Zener diode is used in regulated voltage supply or fixed voltage reference.
Schottkey diode has metal-semiconductor function so it has fast switching action
so it is used as high frequency switch
Varactor diode : Tuned circuits
PIN Diode : Current controlled attenuator
Zener diode : Voltage reference
Schottky diode : High frequency switch
Sol. 49 Option (D) is correct.
mP
We have = 0.4
mn

Downloaded From : www.EasyEngineering.net


Downloaded From : www.EasyEngineering.net
GATE SOLVED PAPER - EC ELECTRONIC DEVICES

Conductance of n type semiconductor


sn = nqmn
Conductance of intrinsic semiconductor
si = ni q (mn + mp)
sn = nmn n
Ratio is =
si ni (mn + mp) ni ^1 + mn h
mp

= 4.2 # 108 = 2 # 10 4
1.5 # 10 4 (1 + 0.4)
Sol. 50 Option (C) is correct.
For silicon at 0 K,
Eg0 = 1.21 eV
At any temperature

ww At T = 300 K,
EgT = Eg0 - 3.6 # 10 - 4 T

Eg300 = 1.21 - 3.6 # 10 - 4 # 300 = 1.1 eV

w.E
Sol. 51
This is standard value, that must be remembered.
Option (B) is correct.

asy
The reverse saturation current doubles for every 10cC rise in temperature as
follows :

En
I0 (T) = I 01 # 2(T - T )/10
Thus at 40c C, I0 = 40 pA
1

Sol. 52 Option (A) is correct.


gin
Silicon is abundant on the surface of earth in the from of SiO2 .
Sol. 53 Option (B) is correct.
eer
sn = nqmn
sp = pqmp
ing (n = p)
sp
sn
m
= p =1
mn 3
.ne
Sol. 54 Option (B) is correct.
C = e0 er A
d
C = e0 er = 8.85 # 10-12 # 11.7 = 10.35 m F
t
or
A d 10 # 10-6
Sol. 55 Option (B) is correct.
In accumulation mode for NMOS having p -substrate, when positive voltage
is applied at the gate, this will induce negative charge near p - type surface
beneath the gate. When VGS is made sufficiently large, an inversion of electrons is
formed and this in effect forms and n - channel.
Sol. 56 Option (C) is correct.
From the graph it can be easily seen that Vth = 1 V
Now VGS = 3 - 1 = 2 V
and VDS = 5 - 1 = 4 V
Since VDS > VGS $ VDS > VGS - Vth
Thus MOSFET is in saturation region.

Downloaded From : www.EasyEngineering.net


Downloaded From : www.EasyEngineering.net
GATE SOLVED PAPER - EC ELECTRONIC DEVICES

Sol. 57 Option (C) is correct.


Trivalent impurities are used for making p type semiconductor. Boron is trivalent.
Sol. 58 Option (A) is correct.
Here emitter base junction is forward biased and base collector junction is reversed
biased. Thus transistor is operating in normal active region.
Sol. 59 Option (D) is correct.
We have b = a
1-a
Thus a -" b -
a ." b .
If the base width increases, recombination of carrier in base region increases
and a decreases & hence b decreases. If doping in base region increases,

ww recombination of carrier in base increases and a decreases thereby decreasing b .


Thus S1 is true and S2 is false.

w.E
Sol. 60

Sol. 61
Option (C) is correct.
Option (A) is correct.
Applying KVL we get

or
asy
VCC - IC RC - VCE = 0
IC = VCC - VCE = 3 - 0.2 = 2.8 mA

Now En I
RC 1k
IB = C = 2.8m = 56 mA

Sol. 62 Option (B) is correct.


b 50
gin
We know that
Wp NA = Wn ND eer
or Wp = Wn # ND
NA ing
=
3 m # 1016
9 # 1016
= 0.3 m m
.ne
Sol. 63 Option (B) is correct.
Conductivity
or resistivity
s = nqun
r = 1 = 1
s nqmn
t
Thus n = 1 = 1 = 1016 /cm 3
qrmn 1.6 # 10 - 19 # 0.5 # 1250
For n type semiconductor n = ND
Sol. 64 Option (D) is correct.
We know that
eeS NA ND
1

Cj = ;
2 (Vbi + VR)( NA + ND) E
2

Thus Cj \ 1
(Vbi + VR)
C j2 (Vbi + VR) 1 1 =1
Now = =
C j1 (Vbi + VR) 2 4 2
C
or Cj2 = j1 = 1 = 0.5 pF
2 2

Downloaded From : www.EasyEngineering.net


Downloaded From : www.EasyEngineering.net
GATE SOLVED PAPER - EC ELECTRONIC DEVICES

Sol. 65 Option (C) is correct.


Increase in gate oxide thickness makes difficult to induce charges in channel. Thus
VT increases if we increases gate oxide thickness. Hence S1 is false.
Increase in substrate doping concentration require more gate voltage because
initially induce charges will get combine in substrate. Thus VT increases if we
increase substrate doping concentration. Hence S2 is false.
Sol. 66 Option (D) is correct.
We know that
ID = K (VGS - VT ) 2
2
Thus IDS = (VGS2 - VT )
IDI (VGS1 - VT ) 2

Substituting the values we have


2
ID2 = (3 - 1) = 4

ww or
ID1 (2 - 1) 2
ID2 = 4IDI = 4 mA

w.E
Sol. 67 Option (A) is correct.
Eg \ 1
l
Thus asy Eg2
Eg1
= l1 = 1.1
l2 0.87
or
En Eg2 = 1.1 # 1.12 = 1.416 eV
0.87
Sol. 68 Option (B) is correct.
Concentration gradient gin
dn =
dx
10 14

0.5 # 10 - 4
= 2 # 1018
eer
q = 1.6 # 10 - 19 C
Dn = 25 ing
dn =
dx
1014
0.5 # 10 - 4 .ne
JC = qDn dn
dx
= 1.6 # 10 - 19 # 25 # 2 # 1018 = 8 A/cm 2
t
Sol. 69 Option (D) is correct.
Pentavalent make n -type semiconductor and phosphorous is pentavalent.
Sol. 70 Option (C) is correct.
For silicon at 0 K Eg0 = 1.21 eV
At any temperature
EgT = Eg0 - 3.6 # 10 - 4 T
At T = 300 K,
Eg300 = 1.21 - 3.6 # 10 - 4 # 300 = 1.1 eV
This is standard value, that must be remembered.
Sol. 71 Option (A) is correct.
By Mass action law
np = ni2

Downloaded From : www.EasyEngineering.net


Downloaded From : www.EasyEngineering.net
GATE SOLVED PAPER - EC ELECTRONIC DEVICES

2 16
p = ni = 1.5 # 10 # 120.5 # 1016 = 4.5 # 1011
n 5 # 10
Sol. 72 Option (C) is correct.
Tunnel diode shows the negative characteristics in forward bias. It is used in
forward bias.
Avalanche photo diode is used in reverse bias.
Sol. 73 Option (D) is correct.
Sol. 74 Option (A) is correct.
rl
We that R = , r = 1 and a = nqun
A s
From above relation we have
R = 1
nqmn A

ww = 0.1 # 10 - 2
5 # 10 # 1.6 # 10 - 19 # 0.13 # 100 # 10 - 12
20
= 106 W

w.E
Sol. 75 Option (D) is correct.
dn = 6 # 1016 - 1017 =- 2 # 1020
dx 2 # 10 - 4 - 0
Now
asy
Jn = nqme E + Dn q dn
dx

So, Jn = qDn dn En
Since no electric field is present, E = 0 and we get

dx
gin
= 1.6 # 10 - 19 # 35 # (- 2 # 1020) =- 1120 A/cm 2

Sol. 76 Option (C) is correct.


LED works on the principal of spontaneous emission. eer
ing
In the avalanche photo diode due to the avalanche effect there is large current
gain.
Tunnel diode has very large doping.
LASER diode are used for coherent radiation. .ne
Sol. 77 Option (C) is correct.
We know that I = Io `e h V - 1j si
VD1
T

where h = 1 for germanium and h = 2 silicon. As per question


t
Io `e e - 1j = Io `e hV - 1j
VDsi VDGe
hVT
n Ge T

VDsi 0.718
Io e hVT - 1 = e 2 # 26 # 10 - 1 = 4 103
=
-3
or si
#
Io VDGe 0.1435
e 26 # 10 - 1
si
e hVT -1 -3

Sol. 78 Option (A) is correct.


Eg = hc
l
-34 8
= 6.626 # 10 # -310# 10 = 3.62 J
54900 # 10
E (J) -19
In eV Eg (eV) = g = 3.62 # 10-19 = 2.26 eV
e 1.6 # 10
Alternatively

Downloaded From : www.EasyEngineering.net


Downloaded From : www.EasyEngineering.net
GATE SOLVED PAPER - EC ELECTRONIC DEVICES

Eg = 1.24 eV = 1.24 = 2.26 eV


l (mm) 5490 # 10-4 mm
Sol. 79 Option (D) is correct.
We know that
ID = K (VGS - VT ) 2
2
Thus ID2 = (VGS2 - VT )
ID1 (VGS1 - VT ) 2
Substituting the values we have
2
ID2 = (1.4 - 0.4) = 4
ID1 (0.9 - 0.4) 2
or ID2 = 4IDI = 4 mA
Sol. 80 Option (B) is correct.

ww In n -well CMOS fabrication following are the steps :


(A) n - well implant
(B) Source drain diffusion

w.E (C) Metalization


(D) Passivation
Sol. 81

asy
Option (D) is correct.
For a JFET in active region we have
IDS = IDSS c1 - VGS m
2

En VP
From above equation it is clear that the action of a JFET is voltage controlled
current source.
gin
Sol. 82 Option (B) is correct.

eer
At constant current the rate of change of voltage with respect to temperature is
dV =- 2.5 mV per degree centigrade

Here
dT
3 T = T2 - T1 = 40 - 20 = 20cC ing
Thus
Therefore,
3 VD =- 2.5 # 20 = 50 mV
VD = 700 - 50 = 650 mV .ne
Sol. 83 Option (D) is correct.
Condition for saturation is IC < bIB
t
Sol. 84 Option (B) is correct.
The metal area of the gate in conjunction with the insulating dielectric oxide
layer and semiconductor channel, form a parallel plate capacitor. It is voltage
controlled capacitor because in active region the current voltage relationship is
given by
IDS = K (VGS - VT ) 2
Sol. 85 Option (D) is correct.
In MOSFET the body (substrate) is connected to power supply in such a way
to maintain the body (substrate) to channel junction in cutoff condition. The
resulting reverse bias voltage between source and body will have an effect on
device function. The reverse bias will widen the depletion region resulting the
reduction in channel length.
Sol. 86 Option (C) is correct.

Downloaded From : www.EasyEngineering.net


Downloaded From : www.EasyEngineering.net
GATE SOLVED PAPER - EC ELECTRONIC DEVICES

At a given value of vBE , increasing the reverse-bias voltage on the collector-base


junction and thus increases the width of the depletion region of this junction.
This in turn results in a decrease in the effective base width W . Since IS is
inversely proportional to W , IS increases and that iC increases proportionally.
This is early effect.
Sol. 87 Option (B) is correct.
For an n -channel JEFT trans-conductance is
-3 (- 2)
gm = - 2IDSS b1 - VGS l = - 2 # 2 # 10 =1 -
VP VP -4 (- 4)G
= 10-3 # 1 = 0.5 mho
2
Sol. 88 Option (A) is correct.
We have gm = IC = 1

ww Now fT =
VT 26
gm
2p (C p + C m)

w.E or 400 =
1/26
2p (0.3 # 10-12 + C m)
or

or asy
(0.3 # 10-12 + C m) = 1
2p # 26 # 400
= 15.3 # 10-12

C m 15.3 # 10-12 - 0.3 # 10-12 = 15 # 10-12 15 pF


Sol. 89
En
Option (D) is correct.

at a given temperature so here


np = ni pi
gin
For any semiconductor (Intrinsic or extrinsic) the product n p remains constant

Sol. 90 Option (D) is correct.


eer
fT =
gm
2p (C p + C m)
ing
Sol. 91 Option (B) is correct.
For a Forward Bias p-n junction, current equation
.ne
or
I = I 0 (eV/kT - 1)
I + 1 = eV/kT
I0 t
or kT log b I + 1l = V
I0
So if we plot log I vs V we get a straight line.
Sol. 92 Option (B) is correct.
A specimen of p - type or n - type is always electrical neutral.
Sol. 93 Option (C) is correct.
Sol. 94 Option (B) is correct.
The unit of q is e and unit of kT is eV. Thus unit of e/kT is e/eV = V-1 .
Sol. 95 Option (D) is correct.
Sol. 96 Option (C) is correct.
We have ni = 1.5 # 1010 /cm3
Nd = 2.25 # 1015 atoms/cm3

Downloaded From : www.EasyEngineering.net


Downloaded From : www.EasyEngineering.net
GATE SOLVED PAPER - EC ELECTRONIC DEVICES

For n type doping we have electron concentration


n - Nd = 2.25 # 1015 atom/cm3
For a given temperature
np = n i2
2
(1.5 # 1010) 2
Hole concentration p = ni = = 1.0 # 105 /cm3
n 2.25 # 1015
Sol. 97 Option (D) is correct.
In p n -junction isolated circuit we should have high impedance, so that p n
junction should be kept in reverse bias. (So connect p to negative potential in
the circuit)
Sol. 98 Option (B) is correct.

ww
w.E If both junction are forward biased and collector base junction is more forward
biased then IC will be flowing out wards (opposite direction to normal mode) the

Sol. 99 asy
collector and it will be in reverse saturation mode.
Option (C) is correct.

En
For normal active mode we have
b = IC
IB
gin
For small values of IC , if we increases IC , b also increases until we reach (IC )

eer
saturation. Further increases in IC (since transistor is in saturation mode know)
will increases IB and b decreases.
Sol. 100 Option (C) is correct.
For a n -channel mosfet thresholds voltage is given by ing
VTN = VGS - VDS (sat)
for p-channel [p+ polysilicon used in gate] .ne
so
VTP = VSD (sat) - VGS
VTP =- VDS (sat) + VGS
so threshold voltage will be same.
t
Sol. 101 Option (C) is correct.
Emitter current is given by
IE = I 0 (eV /kT - 1)
BE

or IE = I 0 eV /kT
BE
eVBE /kT
>> 1
or VBE = kT ln b IE l
I0
Now (VBE ) 1 = kT ln b IE 1 l
I0
(VBE ) 2 = kT ln b IE 2 l
I0
or (VBE ) 2 - (VBE ) 1 = kT ;ln b IE 2 lE = kT ln b 2IE 1 l
IE 1 IE 1
Now if emitter current is double i.e. IE 2 = 2IE1

Downloaded From : www.EasyEngineering.net


Downloaded From : www.EasyEngineering.net
GATE SOLVED PAPER - EC ELECTRONIC DEVICES

(VBE ) 2 = (VBE ) 1 + (25 # 0.60) m volt


= (VBE ) 1 + 15 m volt
Thus if emitter current is doubled the base emitter junction voltage is increased
by 15 mV.
Sol. 102 Option (A) is correct.
Unity gain frequency is given by
fT = fB # b = 106 # 200 = 200 MHz
a-cutoff frequency is given by
f fb
fa = b = = fb (b + 1)
1-a 1 - b +b 1
= 106 # (200 + 1) = 201 MHz
Sol. 103 Option (A) is correct.

ww ***********

w.E
asy
En
gin
eer
ing
.ne
t

Downloaded From : www.EasyEngineering.net


Downloaded From : www.EasyEngineering.net

No part of this publication may be reproduced or distributed in any form or any means, electronic, mechanical,
photocopying, or otherwise without the prior permission of the author.

ww
w.E
GATE SOLVED PAPER
Electronics & Communication
Engineering Mathematics
asy
Copyright © By NODIA & COMPANY
En
gin
eer
ing
Information contained in this book has been obtained by authors, from sources believes to be reliable. However,
neither Nodia nor its authors guarantee the accuracy or completeness of any information herein, and Nodia nor its

.ne
authors shall be responsible for any error, omissions, or damages arising out of use of this information. This book
is published with the understanding that Nodia and its authors are supplying information but are not attempting
to render engineering or other professional services.

t
NODIA AND COMPANY
B-8, Dhanshree Tower Ist, Central Spine, Vidyadhar Nagar, Jaipur 302039
Ph : +91 - 141 - 2101150
www.nodia.co.in
email : enquiry@nodia.co.in

Downloaded From : www.EasyEngineering.net


Downloaded From : www.EasyEngineering.net

GATE SOLVED PAPER - EC


ENGINEERING MATHEMATICS

2013 ONE MARK

Q. 1 The maximum value of q until which the approximation sin q . q holds to within
10% error is
(A) 10c (B) 18c
(C) 50c (D) 90c

Q. 2 The minimum eigen value of the following matrix is

ww R3 5 2V
S W
S5 12 7W
S2 7 5W

w.E T
(A) 0
(C) 2
X
(B) 1
(D) 3

Q. 3
asy
A polynomial f (x) = a 4 x 4 + a 3 x3 + a2 x2 + a1 x - a 0 with all coefficients positive has
(A) no real roots

En
(B) no negative real root
(C) odd number of real roots
gin
(D) at least one positive and one negative real root

2013 eer TWO MARKS

Q. 4
ing
Let A be an m # n matrix and B an n # m matrix. It is given that determinant
^Im + AB h = determinant ^In + BAh, where Ik is the k # k identity matrix. Using
R V
S2 1 1 1W .ne
the above property, the determinant of the matrix given below is

S1 2 1 1W
S1 1 2 1W
S W
S1 1 1 2W
t
T X
(A) 2 (B) 5
(C) 8 (D) 16

2012 ONE MARK

Q. 5 With initial condition x (1) = 0.5 , the solution of the differential equation
t dx + x = t , is
dt
(A) x = t - 1 (B) x = t 2 - 1
2 2
2
t
(C) x = t (D) x = 2
2

Downloaded From : www.EasyEngineering.net


Downloaded From : www.EasyEngineering.net
GATE SOLVED PAPER - EC ENGINEERING MATHEMATICS

Q. 6 Given f (z) = 1 - 2 .
z+1 z+3
If C is a counter clockwise path in the z -plane such that z + 1 = 1, the value of
1
2pj #C
f (z) dz is
(A) - 2 (B) - 1
(C) 1 (D) 2

Q. 7 If x = - 1, then the value of xx is


(A) e- p/2 (B) e p/2
(C) x (D) 1

2012 TWO MARKS

ww
Q. 8 Consider the differential equation
d 2 y (t)
dt 2
+2
dy (t)
dt
+ y (t)= d (t) with y (t) t = 0 =- 2 and
-
dy
dt
=0

w.E The numerical value of


(A) - 2
dy
dt t = 0
+
is
(B) - 1
t = 0-

Q. 9
(C) 0
asy (D) 1

The direction of vector A is radially outward from the origin, with A = krn .

(A) - 2 En
where r2 = x2 + y2 + z2 and k is a constant. The value of n for which d:A = 0 is
(B) 2
(C) 1
gin
(D) 0

Q. 10
number of required tosses is odd, is
eer
A fair coin is tossed till a head appears for the first time. The probability that the

(A) 1/3
(C) 2/3
(B) 1/2
(D) 3/4
ing
Q. 11
(A) 21 (B) 25 .ne
The maximum value of f (x) = x3 - 9x2 + 24x + 5 in the interval [1, 6] is

Q. 12
(C) 41

Given that
-5 -3 1 0
(D) 46
t
A=>
2 0H
and I = >
0 1H
, the value of A3 is
(A) 15A + 12I (B) 19A + 30I
(C) 17A + 15I (D) 17A + 21I

2011 ONE MARK

Q. 13 Consider a closed surface S surrounding volume V . If rv is the position vector of a


point inside S , with nt the unit normal on S , the value of the integral ## 5rv $ nt dS
S
is
(A) 3V (B) 5V
(C) 10V (D) 15V

Downloaded From : www.EasyEngineering.net


Downloaded From : www.EasyEngineering.net
GATE SOLVED PAPER - EC ENGINEERING MATHEMATICS

dy
Q. 14 The solution of the differential equation = ky, y (0) = c is
dx
(A) x = ce-ky (B) x = kecy
(C) y = cekx (D) y = ce-kx

Q. 15 The value of the integral # - 3z + 4 dz where c is the circle z = 1 is given


(z 2 + 4z + 5)
by c

(A) 0 (B) 1/10


(C) 4/5 (D) 1

2011 TWO MARKS

Q. 16 A numerical solution of the equation f (x) + x - 3 = 0 can be obtained using


Newton- Raphson method. If the starting value is x = 2 for the iteration, the

ww value of x that is to be used in the next step is


(A) 0.306 (B) 0.739

w.E
Q. 17
(C) 1.694

The system of equations


(D) 2.306

asy
x+y+z = 6
x + 4y + 6y = 20

En
x + 4y + l z = m
has NO solution for values of l and μ given by
(A) l = 6, m = 20
(C) l =
Y 6, m = 20 gin (B) l = 6, m =
(D) l =
Y 20
Y 6, m = 20
Q. 18
value that is higher than the first toss is eer
A fair dice is tossed two times. The probability that the second toss results in a

(A) 2/36
(C) 5/12
(B) 2/6
(D) 1/2 ing
.ne
Q. 19
2010

The eigen values of a skew-symmetric matrix are


(A) always zero
t
(B) always pure imaginary
ONE MARKS

(C) either zero or pure imaginary (D) always real

Q. 20 The trigonometric Fourier series for the waveform f (t) shown below contains

(A) only cosine terms and zero values for the dc components

Downloaded From : www.EasyEngineering.net


Downloaded From : www.EasyEngineering.net
GATE SOLVED PAPER - EC ENGINEERING MATHEMATICS

(B) only cosine terms and a positive value for the dc components
(C) only cosine terms and a negative value for the dc components
(D) only sine terms and a negative value for the dc components

Q. 21 A function n (x) satisfied the differential equation


d 2 n (x) n (x)
- 2 =0
dx 2 L
where L is a constant. The boundary conditions are : n (0) = K and n (3) = 0 .
The solution to this equation is
(A) n (x) = K exp (x/L) (B) n (x) = K exp (- x/ L )
2
(C) n (x) = K exp (- x/L) (D) n (x) = K exp (- x/L)

2010 TWO MARKS

ww
Q. 22 If ey = x1/x , then y has a
(A) maximum at x = e
(C) maximum at x = e-1
(B) minimum at x = e
(D) minimum at x = e-1

w.E
Q. 23 A fair coin is tossed independently four times. The probability of the event “the
number of time heads shown up is more than the number of times tail shown up”
(A) 1/16
(C) 1/4 asy (B) 1/3
(D) 5/16

Q. 24
En
v = xyatx + x 2 aty , then
If A # Av $ dlv over the path shown in the figure is
C

gin
eer
ing
.ne
(A) 0

(C) 1
(B) 2
3
(D) 2 3
t
Q. 25 The residues of a complex function
x (z) = 1 - 2z
z (z - 1) (z - 2)
at its poles are
(A) 1 , - 1 and 1 (B) 1 , - 1 and - 1
2 2 2 2
(C) 1 , 1 and - 3 (D) 1 , - 1 and 3
2 2 2 2
dy (x)
Q. 26 Consider differential equation - y (x) = x , with the initial
dx
condition y (0) = 0 . Using Euler’s first order method with a step size of 0.1, the
value of y (0.3) is
(A) 0.01 (B) 0.031
(C) 0.0631 (D) 0.1

Downloaded From : www.EasyEngineering.net


Downloaded From : www.EasyEngineering.net
GATE SOLVED PAPER - EC ENGINEERING MATHEMATICS

Given f (t) = L-1 ; 3 3s + 1


s + 4s2 + (k - 3) s E
Q. 27 . If lim f (t) = 1, then the value
t"3
of k is
(A) 1 (B) 2
(C) 3 (D) 4

2009 ONE MARK

d2y dy 3
2 + c dt m + y = e
4 -t
Q. 28 The order of the differential equation is
dt
(A) 1 (B) 2
(C) 3 (D) 4

Q. 29 A fair coin is tossed 10 times. What is the probability that only the first two
tosses will yield heads?

ww (A) c 1 m
2
2
(B) 10C2 b 1 l
2
2

w.E (C) c 1 m (D) 10C2 b 1 l


10 10

2 2

# 1 +zf (z) dz is given by


Q. 30

(A) 2pc1 asy


If f (z) = c 0 + c1 z-1 , then
unit circle
(B) 2p (1 + c0)
(C) 2pjc1
En (D) 2p (1 + c0)

2009
gin TWO MARKS

Q. 31 The Taylor series expansion of sin x at x = p is given by


(x - p) 2
x-p
(x - p) 2 eer
(A) 1 +
3!
+ ...

(x - p) 2
(B) - 1 -
3!
+ ...
ing
(x - p) 2
(C) 1 -
3!
+ ... (D) - 1 +

.ne
3!
+ ...

Q. 32 Match each differential equation


Group II
Group I
A.
dy
=
y
Group II
1. Circles
t
in Group I to its family of solution curves from

dx x
dy y
B. =- 2. Straight lines
dx x
dy x
C. = 3. Hyperbolas
dx y
dy
D. =- x
dx y
(A) A - 2, B - 3, C - 3, D - 1
(B) A - 1, B - 3, C - 2, D - 1
(C) A - 2, B - 1, C - 3, D - 3
(D) A - 3, B - 2, C - 1, D - 2

Downloaded From : www.EasyEngineering.net


Downloaded From : www.EasyEngineering.net
GATE SOLVED PAPER - EC ENGINEERING MATHEMATICS

Q. 33 The Eigen values of following matrix are


R V
S- 1 3 5 W
S- 3 - 1 6 W
SS 0 0 3 WW
(A) 3, 3 + 5j, 6 - j T X (B) - 6 + 5j, 3 + j, 3 - j
(C) 3 + j, 3 - j, 5 + j (D) 3, - 1 + 3j, - 1 - 3j

2008 ONE MARKS


p11 p12
All the four entries of the 2 # 2 matrix P = =
p21 p22 G
Q. 34 are nonzero,
and one of its eigenvalue is zero. Which of the following statements is true?
(A) p11 p12 - p12 p21 = 1 (B) p11 p22 - p12 p21 =- 1
(C) p11 p22 - p12 p21 = 0 (D) p11 p22 + p12 p21 = 0

ww
Q. 35 The system of linear equations

w.E 4 x + 2y = 7
(A) a unique solution
(B) no solution
2x + y = 6 has

asy
(C) an infinite number of solutions
(D) exactly two distinct solutions
Q. 36
En
The equation sin (z) = 10 has
(A) no real or complex solution
(B) exactly two distinct complex solutions
(C) a unique solution
gin
(D) an infinite number of complex solutions
eer
Q. 37 For real values of x , the minimum value of the function
f (x) = exp (x) + exp (- x) is
(A) 2 (B) 1
ing
(C) 0.5 (D) 0
.ne
Q. 38
series expansion about the point x = 0 ?
(A) sin (x3) (B) sin (x2)
t
Which of the following functions would have only odd powers of x in its Taylor

(C) cos (x3) (D) cos (x2)

Q. 39 Which of the following is a solution to the differential equation


dx (t)
+ 3x (t) = 0 ?
dt
(A) x (t) = 3e - t (B) x (t) = 2e - 3t
(C) x (t) =- 23 t2 (D) x (t) = 3t2

2008 TWO MARKS


-x
Q. 40 The recursion relation to solve x = e using Newton - Raphson method is
(A) xn + 1 = e-xn
(B) xn + 1 = xn - e-x
n

-x x 2 - e-x (1 - xn) - 1
(C) xn + 1 = (1 + xn) e -x
n n

(D) xn + 1 = n
1+e n
xn - e-x
n

Downloaded From : www.EasyEngineering.net


Downloaded From : www.EasyEngineering.net
GATE SOLVED PAPER - EC ENGINEERING MATHEMATICS

Q. 41 The residue of the function f (z) = 1 at z = 2 is


(z + 2) 2 (z - 2) 2
(A) - 1 (B) - 1
32 16
(C) 1 (D) 1
16 32
0 1
Consider the matrix P = =
- 2 - 3G
Q. 42 . The value of e p is
2e-2 - 3e-1 e-1 - e-2
(A) > -2 H
2e - 2e-1 5e-2 - e-1
e-1 + e-1 2e-2 - e-1
(B) > -1 H
2e - 4e2 3e-1 + 2e-2
5e-2 - e-1 3e-1 - e-2
(C) > -2 H
ww (D) >
2e - 6e-1 4e-2 + 6-1
2e-1 - e-2 e-1 - e-2
H
w.E
Q. 43
- 2e-1 + 2e-2 - e-1 + 2e-2

In the Taylor series expansion of exp (x) + sin (x) about the point x = p , the

(A) exp (p)


asy
coefficient of (x - p) 2 is
(B) 0.5 exp (p)

Q. 44
(C) exp (p) + 1

En (D) exp (p) - 1

The value of the integral of the function g (x, y) = 4x3 + 10y 4 along the straight

gin
line segment from the point (0, 0) to the point (1, 2) in the x - y plane is
(A) 33
(B) 35
(C) 40 eer
(D) 56
ing
Q. 45
Q
.ne
Consider points P and Q in the x - y plane, with P = (1, 0) and Q = (0, 1). The
line integral 2 # (xdx + ydy) along the semicircle with the line segment PQ as
its diameter
(A) is - 1
(B) is 0
P

t
(C) is 1
(D) depends on the direction (clockwise or anit-clockwise) of the semicircle

2007 ONE MARK

Q. 46 The following plot shows a function which varies linearly with x . The value of the
2
integral I = #1 ydx is

Downloaded From : www.EasyEngineering.net


Downloaded From : www.EasyEngineering.net
GATE SOLVED PAPER - EC ENGINEERING MATHEMATICS

(A) 1.0 (B) 2.5


(C) 4.0 (D) 5.0

Q. 47 For x << 1, coth (x) can be approximated as


(A) x (B) x2
(C) 1 (D) 12
x x

sin b q l
2
Q. 48 lim is
q"0 q
(A) 0.5 (B) 1
(C) 2 (D) not defined

Q. 49 Which one of following functions is strictly bounded?

ww (A) 1/x2
(C) x2
(B) ex
(D) e - x
2

w.E
Q. 50 For the function e - x , the linear approximation around x = 2 is
(A) (3 - x) e - 2
(B) 1 - x
(C) 63 + 3 2 - (1 -asy 2 ) x @e - 2
(D) e - 2
En
2007
gin d2 y
TWO MARKS

Q. 51

conditions dx
eer
The solution of the differential equation k2 2 = y - y2 under the boundary

(i) y = y1 at x = 0 and
(ii) y = y2 at x = 3 , where k, y1 and y2 are constants, is ing
(A) y = (y1 - y2) exp a- x2 k + y2
k .ne
x
(B) y = (y2 - y1) exp a- k + y1
k
(C) y = ^y1 - y2h sinh a x k + y1
k
t
(D) y = ^y1 - y2h exp a- x k + y2
k

Q. 52 The equation x3 - x2 + 4x - 4 = 0 is to be solved using the Newton - Raphson


method. If x = 2 is taken as the initial approximation of the solution, then next
approximation using this method will be
(A) 2/3 (B) 4/3
(C) 1 (D) 3/2

Q. 53 Three functions f1 (t), f2 (t) and f3 (t) which are zero outside the interval [0, T] are
shown in the figure. Which of the following statements is correct?

Downloaded From : www.EasyEngineering.net


Downloaded From : www.EasyEngineering.net
GATE SOLVED PAPER - EC ENGINEERING MATHEMATICS

ww
w.E (A) f1 (t) and f2 (t) are orthogonal
(C) f2 (t) and f3 (t) are orthogonal
(B) f1 (t) and f3 (t) are orthogonal
D) f1 (t) and f2 (t) are orthonormal

Q. 54
asy
If the semi-circular control D of radius 2 is as shown in the figure, then the value
of the integral # 2 1 ds is

En
D
(s - 1)

gin
eer
(A) jp (B) - jping
(C) - p (D) p
.ne
Q. 55

t
It is given that X1, X2 ...XM at M non-zero, orthogonal vectors. The dimension of
the vector space spanned by the 2M vectors X1, X2,... XM , - X1, - X2,... - XM is
(A) 2M
(C) M
(B) M + 1

(D) dependent on the choice of X1, X2,... XM

Q. 56 Consider the function f (x) = x2 - x - 2 . The maximum value of f (x) in the closed
interval [- 4, 4] is
(A) 18 (B) 10
(C) - 225 (D) indeterminate

Q. 57 An examination consists of two papers, Paper 1 and Paper 2. The probability


of failing in Paper 1 is 0.3 and that in Paper 2 is 0.2. Given that a student has
failed in Paper 2, the probability of failing in Paper 1 is 0.6. The probability of a
student failing in both the papers is
(A) 0.5 (B) 0.18
(C) 0.12 (D) 0.06

Downloaded From : www.EasyEngineering.net


Downloaded From : www.EasyEngineering.net
GATE SOLVED PAPER - EC ENGINEERING MATHEMATICS

2006 ONE MARK


R V
S1 1 1 W
Q. 58 The rank of the matrix S1 - 1 0 W is
SS1 1 1 WW
(A) 0 T X (B) 1
(C) 2 (D) 3

Q. 59 4#4# P , where P is a vector, is equal to


(A) P # 4# P - 4 2 P (B) 4 2 P + 4 (4 # P)
(C) 4 2 P + 4# P (D) 4 (4$ P) - 4 2 P

Q. 60 ## (4 # P) $ ds , where P is a vector, is equal to


(A) # P $ dl (B) # 4#4# P $ dl
(C) # 4# P $ dl (D) ### 4$ Pdv
ww
Q. 61 A probability density function is of the form p (x) = Ke- a x , x ! (- 3, 3). The

w.E value of K is
(A) 0.5
(C) 0.5a
(B) 1
(D) a

Q. 62

x (0-) = 0 is
asy
A solution for the differential equation xo (t) + 2x (t) = d (t) with initial condition

(A) e - 2t u (t)
En (B) e2t u (t)
(C) e - t u (t)
gin (D) et u (t)

Q. 63
2006
eer TWO MARKS

The eigenvalue and the corresponding eigenvector of 2 # 2 matrix are given by


Eigenvalue
l1 = 8
Eigenvector
1
v1 = = G ing
l2 = 4
1

v2 = = G
1 .ne
The matrix is
6 2
(A) = G
2 6
-1

(B) =
4 6
6 4G
t
2 4 4 8
(C) =
4 2G
(D) =
8 4G

Q. 64 For the function of a complex variable W = ln Z (where, W = u + jv and Z = x + jy


, the u = constant lines get mapped in Z -plane as
(A) set of radial straight lines (B) set of concentric circles
(C) set of confocal hyperbolas (D) set of confocal ellipses
1 dz is positive sense is
Q. 65 The value of the constant integral # z2 + 4
z-j =2
jp
(A) (B) - p
2 2
jp
(C) - (D) p
2 2

Downloaded From : www.EasyEngineering.net


Downloaded From : www.EasyEngineering.net
GATE SOLVED PAPER - EC ENGINEERING MATHEMATICS

p
Q. 66 The integral #0 sin3 qdq is given by
(A) 1 (B) 2
2 3
(C) 4 (D) 8
3 3

Q. 67 Three companies X, Y and Z supply computers to a university. The percentage


of computers supplied by them and the probability of those being defective are
tabulated below

Company % of Computer Supplied Probability of being supplied defective


X 60% 0.01
Y 30% 0.02
Z 10% 0.03

ww Given that a computer is defective, the probability that was supplied by Y is


(A) 0.1 (B) 0.2

w.E (C) 0.3

For the matrix =


4 2
G
(D) 0.4
101
the eigenvalue corresponding to the eigenvector = G is
Q. 68

(A) 2 asy 2 4 101

(B) 4
(C) 6 En
(D) 8

d2 y
gin
Q. 69 For the differential equation
dx 2
eer
+ k2 y = 0 the boundary conditions are

(i) y = 0 for x = 0 and (ii) y = 0 for x = a


ing
The form of non-zero solutions of y (where m varies over all integers) are
(A) y = / Am sin mpx (B) y = / Am cos mpx
m

(C) y = / Am x
a
mp
m
.ne
a
(D) y = / Am e -
mpx

t
a a
m m

Q. 70 As x increased from - 3 to 3, the function f (x) = ex


1 + ex
(A) monotonically increases
(B) monotonically decreases
(C) increases to a maximum value and then decreases
(D) decreases to a minimum value and then increases

2005 ONE MARK

Q. 71 The following differential equation has


d2 y dy 3
3 c 2 m + 4 c m + y2 + 2 = x
dt dt
(A) degree = 2 , order = 1 (B) degree = 1, order = 2
(C) degree = 4 , order = 3 (D) degree = 2 , order = 3

Downloaded From : www.EasyEngineering.net


Downloaded From : www.EasyEngineering.net
GATE SOLVED PAPER - EC ENGINEERING MATHEMATICS

Q. 72 A fair dice is rolled twice. The probability that an odd number will follow an even
number is
(A) 1/2 (B) 1/6
(C) 1/3 (D) 1/4

d2 y dy
Q. 73 A solution of the following differential equation is given by -5 + 6y = 0
dx2 dx
(A) y = e2x + e-3x (B) y = e2x + e3x
(C) y = e-2x + 33x (D) y = e-2x + e-3x

2005 TWO MARKS

Q. 74 In what range should Re (s) remain so that the Laplace transform of the function
e(a + 2) t + 5 exits.

ww (A) Re (s) > a + 2


(C) Re (s) < 2
(B) Re (s) > a + 7
(D) Re (s) > a + 5

w.E
Q. 75 The derivative of the symmetric function drawn in given figure will look like

asy
En
gin
eer
ing
Match the following and choose the correct combination:

.ne
Q. 76
Group I Group 2
E. Newton-Raphson method 1. Solving nonlinear equations
F. Runge-kutta method

G. Simpson’s Rule
equations t
2. Solving linear simultaneous

3. Solving ordinary differential


equations
H. Gauss elimination 4. Numerical integration
5. Interpolation
6. Calculation of Eigenvalues
(A) E - 6, F - 1, G - 5, H - 3 (B) E - 1, F - 6, G - 4, H - 3
(C) E - 1, F - 3, G - 4, H - 2 (D) E - 5, F - 3, G - 4, H - 1
-4 2
Given the matrix =
4 3G
Q. 77 , the eigenvector is
3 4
(A) = G (B) = G
2 3
2 -1
(C) = G (D) = G
-1 2

Downloaded From : www.EasyEngineering.net


Downloaded From : www.EasyEngineering.net
GATE SOLVED PAPER - EC ENGINEERING MATHEMATICS

2 - 0.1 1
a
Q. 78 Let, A = = G and A - 1 = = 2 G. Then (a + b) =
0 3 0 b
(A) 7/20 (B) 3/20
(C) 19/60 (D) 11/20
2
1
#0 exp c- x m dx is
3
Q. 79 The value of the integral I =
2p 8
(A) 1 (B) p
(C) 2 (D) 2p

Q. 80 Given an orthogonal matrix


R1 1 1 1 V
S W
S1 1 - 1 - 1W
A =S
1 - 1 0 0W
S W

ww 6AA @- 1 is
T

R1 V
S0 0 1 1 W
T X
R1 V

w.E S4 0 0 0 W
S0 1 0 0 W
(A) S 4 1 W
S0 0 2 0 W
S2
S0
(B) S
S0
0 0 0W
1
2 0 0
1
0 2 0W
W
W

T asy
S0 0 0 12 W
R1 0 0 0 V
X
S0
TR
1
0 0 12 W

0 0 0W
XV
S W
S0 1 0 0 W
(C) S
0 0 1 0W En (D) S
S4
S0
S0
1
4 0 0
1
0 4 0W
W
W
S
T
W
S0 0 0 1 W
X gin S0
T
0 0 14 W
X

*********** eer
ing
.ne
t

Downloaded From : www.EasyEngineering.net


Downloaded From : www.EasyEngineering.net
GATE SOLVED PAPER - EC ENGINEERING MATHEMATICS

SOLUTIONS

Sol. 1 Option (B) is correct.


Here, as we know
Lim sin q . 0
q"0

but for 10% error, we can check option (B) first,


q = 18c = 18c # p = 0.314
180c
sin q = sin 18c = 0.309
% error = 0.314 - 0.309 # 100% = 0.49%

ww Now, we check it for q = 50c


0.309

q = 50c = 50c # p = 0.873


w.E sin q = sin 50c = 0.77
180c

% error = 0.77 - 0.873 =- 12.25%

asy 0.873
so, the error is more than 10% . Hence, for error less than 10%, q = 18c can have
the approximation
En
sin q . q
Sol. 2 Option (A) is correct.
gin
For, a given matrix 6A@ the eigen value is calculated as
A - lI = 0
eer
where l gives the eigen values of matrix. Here, the minimum eigen value among
the given options is
l =0
ing
We check the characteristic equation of matrix for this eigen value
A - lI = A
.ne
(for l = 0 )
3 5 2
= 5 12 7
2 7 5
t
= 3 ^60 - 49h - 5 ^25 - 14h + 2 ^35 - 24h
= 33 - 55 + 22
=0
Hence, it satisfied the characteristic equation and so, the minimum eigen value
is
l =0
Sol. 3 Option (D) is correct.
Given, the polynomial
f ^x h = a 4 x 4 + a 3 x3 + a2 x2 + a1 x - a 0
Since, all the coefficients are positive so, the roots of equation is given by
f ^x h = 0
It will have at least one pole in right hand plane as there will be least one sign
change from ^a1h to ^a 0h in the Routh matrix 1 st column. Also, there will be a

Downloaded From : www.EasyEngineering.net


Downloaded From : www.EasyEngineering.net
GATE SOLVED PAPER - EC ENGINEERING MATHEMATICS

corresponding pole in left hand plane


i.e.; at least one positive root (in R.H.P)
and at least one negative root (in L.H.P)
Rest of the roots will be either on imaginary axis or in L.H.P
Sol. 4 Option (B) is correct.
Consider the given matrix be
R V
S2 1 1 1W
S1 2 1 1W
Im + AB = S W
S1 1 2 1W
S1 1 1 2W
T
where m = 4 so, we obtain X
R V R V
S2 1 1 1W S1 0 0 0W
S1 2 1 1W S0 1 0 0W
AB = S W-S W

ww S1 1 2 1W S0 0 1 0W
S1 1 1 2W S0 0 0 1W
RT VX T X

w.E =S
S1 1 1 1W
S1 1 1 1W
S1 1 1 1W
W

asy S1 1 1 1W
TR V
S1W
X

En S1W
= S W 61 1 1 1@
S1W

Hence, we get
S1W
T X gin
R V
S1W
S1W
A = S W, B = 81 1 1 1B
eer
S1W
S1W ing
Therefore,
T X
BA = 81 1 1 1B S1W
R V
S1W .ne
=4
S1W
S W
S1W
T X
t
From the given property
Det ^Im + AB h = Det ^Im + BAh
R V ZR V _
S2 1 1 1W ]S1 0 0 0W b
S1 2 1 1W ]S0 1 0 0W b
& Det S W = Det [S W + 4`
S1 1 2 1W ]S
0 0 1 0W
b
S1 1 1 2W ]S0 0 0 1W b
T X \T X a
= 1+4
=5
Note : Determinant of identity matrix is always 1.
Sol. 5 Option (D) is correct.
t dx + x = t
dt

Downloaded From : www.EasyEngineering.net


Downloaded From : www.EasyEngineering.net
GATE SOLVED PAPER - EC ENGINEERING MATHEMATICS

dx + x = 1
dt t
dx + Px = Q (General form)
dt
IF = e # = e = e lnt = t
1

Integrating factor, Pdt # dt


t

Solution has the form,


x # IF = # ^Q # IF hdt + C
x # t = # (1) (t) dt + C
2
xt = t + C
2
Taking the initial condition,
x (1) = 0.5

ww 0.5 = 1 + C & C = 0
2

w.E
Sol. 6
So,

Option (C) is correct.


2
2
xt = t & x = t
2

asy
f (z) = 1 - 2
z+1 z+3
1
2p j C
En
# f (z) dz = sum of the residues of the poles which lie
inside the given closed region.
C & z+1 = 1
gin
Only pole z =- 1 inside the circle, so residue at z =- 1 is.
f (z) = -z + 1
(z + 1) (z + 3)
= lim
(z + 1) (- z + 1) 2
z "- 1 (z + 1) (z + 3) eer
= =1
2
So 1
2pj #C
f (z) dz = 1
ing
Sol. 7 Option (A) is correct.
x= - 1 = i = cos p + i sin p .ne
So,
p
x = ei 2
2 2

xx = ^ei 2 h & ^ei 2 h = e- 2


p x p i p
t
Sol. 8 Option (D) is correct.
d 2 y (t) 2dy (t)
+ + y (t) = d (t)
dt 2 dt
By taking Laplace transform with initial conditions
dy
;s Y (s) - sy (0) - dt E + 2 [sy (s) - y (0)] + Y (s) = 1
2

t=0

6s Y (s) + 2s - 0@ + 2 6sY (s) + 2@ + Y (s) = 1


2
&
Y (s) [s2 + 2s + 1] = 1 - 2s - 4
Y (s) = 2- 2s - 3
s + 2s + 1
L
We know that, If, y (t) Y (s)

Downloaded From : www.EasyEngineering.net


Downloaded From : www.EasyEngineering.net
GATE SOLVED PAPER - EC ENGINEERING MATHEMATICS

dy (t)
sY (s) - y (0)
L
then,
dt
(- 2s - 3) s
So, sY (s) - y (0) = +2
(s2 + 2s + 1)
2 2
= - 2s - 32 s + 2s + 4s + 2
(s + 2s + 1)
sY (s) - y (0) = s + 2 2 = s + 1 2 + 1
(s + 1) (s + 1) (s + 1) 2
= 1 + 1
s + 1 (s + 1) 2
Taking inverse Laplace transform
dy (t)
= e-t u (t) + te-t u (t)
dt
dy
At t = 0+ , = e0 + 0 = 1

ww dt t = 0 +

w.E
Sol. 9 Option (A) is correct.
Divergence of A in spherical coordinates is given as
d:A = 12 2 (r 2 Ar ) = 12 2 (krn + 2)

asy r 2r
= k2 (n + 2) rn + 1
r
r 2r

En = k (n + 2) rn - 1 = 0 (given)

Sol. 10 Option (C) is correct.


n+2 = 0
gin& n =- 2

eer
Probability of appearing a head is 1/2. If the number of required tosses is odd,
we have following sequence of events.
H, ,
TTH TTTTH
3
ing
, ...........
= +b l +b l +
5

Probability
P 1 1 1
P = 2 2 1 2= 2 2
1- 4 3 .ne
.....

Sol. 11 Option (B) is correct.


f (x) = x3 - 9x2 + 24x + 5
1
t
df (x)
= 3x2 - 18x + 24 = 0
dx
df (x)
& = x 2 - 6x + 8 = 0 x = 4, x = 2
dx
d 2 f (x)
= 6x - 18
dx 2
d 2 f (x)
For x = 2, = 12 - 18 =- 6 < 0
dx2
So at x = 2, f (x) will be maximum

f (x) max
= (2) 3 - 9 (2) 2 + 24 (2) + 5
= 8 - 36 + 48 + 5 = 25

Downloaded From : www.EasyEngineering.net


Downloaded From : www.EasyEngineering.net
GATE SOLVED PAPER - EC ENGINEERING MATHEMATICS

Sol. 12 Option (B) is correct.


Characteristic equation.
A - lI = 0
-5 - l -3
=0
2 -l
5l + l2 + 6 = 0
l2 + 5l + 6 = 0
Since characteristic equation satisfies its own matrix, so
A2 + 5A + 6 = 0 & A2 =- 5A - 6I
Multiplying with A
A 3 + 5A 2 + 6 A = 0
A3 + 5 (- 5A - 6I) + 6A = 0

ww
Sol. 13 Option (D) is correct.
A3 = 19A + 30I

w.E From Divergence theorem, we have


### 4$
The position vector
v = #A
v Adv v $ nt ds
s

asy
v = 5rv, thus
Here, A
rv = ^utx x + uty y + utz z h

En 4$ A c 2x 2z m ^
v = utx 2 + uty 2 + utz 2 : utx x + uty y + utz z
2y h

= c dx +
dy dz
dx dy dz m
+
gin5 = 3 # 5 = 15

So, ## 5rv $ nt ds = ### 15 dv = 15V


s
eer
Sol. 14 Option (C) is correct.
We have
dy
dx
= ky ing
Integrating # dy
y
= # k dx + A .ne
or
Since y (0) = c thus
So, we get,
ln y = kx + A
ln c = A
ln y = kx + ln c
t
or ln y = ln ekx + ln c
or y = cekx
Sol. 15 Option (A) is correct.
C R Integrals is # - 2
3z + 4 dz where C is circle z = 1
C
z + 4z + 5
# f (z) dz = 0 if poles are outside C.
C

Now z 2 + 4z + 5 = 0
(z + 2) 2 + 1 = 0
Thus z1, 2 =- 2 ! j & z1, 2 > 1
So poles are outside the unit circle.

Downloaded From : www.EasyEngineering.net


Downloaded From : www.EasyEngineering.net
GATE SOLVED PAPER - EC ENGINEERING MATHEMATICS

Sol. 16 Option (C) is correct.


We have f (x) = x + x - 3 = 0
f l (x) = 1 + 1
2 x
Substituting x 0 = 2 we get
f l (x 0) = 1.35355 and f (x 0) = 2 + 2 - 3 = 0.414
Newton Raphson Method
f (x 0)
x1 = x 0 -
f l (x 0)
Substituting all values we have
x 1 = 2 - 0.414 = 1.694
1.3535
Sol. 17 Option (B) is correct.

ww Writing A: B we have
R V
S1 1 1 : 6 W

w.E S1 4 6 : 20W
S
T
W
S1 4 l : m W
X

asy
Apply R 3 " R 3 - R2
R
S1 1 1 : 6 W
V

En S1 4 6 : 20 W
S W
S0 0 l - 6 : m - 20W
T X
gin
For equation to have solution, rank of A and A: B must be same. Thus for no
solution; l = 6, m ! 20
Sol. 18 Option (C) is correct.
eer
Total outcome are 36 out of which favorable outcomes are :

ing
(1, 2), (1, 3), (1, 4), (1, 5), (1, 6), (2, 3), (2, 4), (2, 5), (2, 6);
(3, 4), (3, 5), (3, 6), (4, 5), (4, 6), (5, 6) which are 15.
Thus P (E) = No. of favourable outcomes = 15 = 5
No. of total outcomes .ne
36 12
Sol. 19 Option (C) is correct.
t
Eigen value of a Skew-symmetric matrix are either zero or pure imaginary in
conjugate pairs.
Sol. 20 Option (C) is correct.
For a function x (t) trigonometric fourier series is
3
x (t) = Ao + / [An cos nwt + Bn sin nwt]
n=1

Where, Ao = 1 # x (t) dt T0 " fundamental period


T0 T
0

An = 2 # x (t) cos nwt dt


T0 T
0

Bn = 2 # x (t) sin nwt dt


T0 T
0

For an even function x (t), Bn = 0


Since given function is even function so coefficient Bn = 0 , only cosine and constant

Downloaded From : www.EasyEngineering.net


Downloaded From : www.EasyEngineering.net
GATE SOLVED PAPER - EC ENGINEERING MATHEMATICS

terms are present in its fourier series representation.


Constant term :
A0 = 1 #
3T/4
x (t) dt
T -T/4
= 1 : # Adt + # - 2AdtD
T/4 3T/4

T -T/4 T/4

= 1 :TA - 2AT D =- A
T 2 2 2
Constant term is negative.
Sol. 21 Option (D) is correct.
Given differential equation
d 2 n (x) n (x)
- 2 =0
dx 2 L

ww Let n (x) = Ae
So,
lx

L
lx
Al2 elx - Ae2 = 0

w.E l - 12 = 0 & l = ! 1
2
L L
Boundary condition, n (3) = 0 so take l =- 1
asy n (x) = Ae- L
x
L

En n (0) = Ae0 = K & A = K

Sol. 22
So,
Option (A) is correct.
n (x) = Ke- (x/L)
gin
Given that
or
y
e =x
1
x

ln ey = ln x x
1
eer
or y = 1 ln x
x ing
Now
For maxima and minima :
dy
dx
= 1 1 + ln x ^- x- x h = 12 - ln2
xx
1
2

x x.ne
dy
dx
= 12 (1 - ln x) = 0
x
ln x = 1 " x = e 1
t
d 2y
Now =- 23 - ln x b- 23 l - 12 b 1 l
dx 2 x x x x
=- 22 + 2 ln3 x - 13
x x x
2
d x = -22 + 23 - 13 < 0
dy 2 at x = e1 e e e
1
So, y has a maximum at x = e
Sol. 23 Option (D) is correct.
According to given condition head should comes 3 times or 4 times
P (Heads comes 3 times or 4 times) = 4C 4 b 1 l + 4C 3 b 1 l b 1 l
4 3

2 2 2
= 1: 1 +4:1 :1 = 5
16 8 2 16

Downloaded From : www.EasyEngineering.net


Downloaded From : www.EasyEngineering.net
GATE SOLVED PAPER - EC ENGINEERING MATHEMATICS

Sol. 24 Option (C) is correct.


v = xyatx + x 2 aty
A
v = dxatx + dyaty
dl
# Av : dl
v = # (xyatx + x 2 aty) : (dxatx + dyaty)
C C
= # (xydx + x 2 dy)
C

= #1/
2/ 3
xdx +
1/ 3
#2/ 3xdx + #1
3 4 dy + #3
1 1 dy
3 3 3 3
= 1 : 4 - 1 D + 3 :1 - 4 D + 4 [3 - 1] + 1 [1 - 3]
2 3 3 2 3 3 3 3
=1
Sol. 25 Option (C) is correct.
Given function

ww X (z ) =1 - 2z
z (z - 1) (z - 2)

w.E Poles are located at z = 0, z = 1, and z = 2


At Z = 0 residues is
R 0 = z : X (z) Z = 0 = 1 - 2 # 0 = 1

at z = 1, asy (0 - 1) (0 - 2)
R1 = (Z - 1) : X (Z ) Z = 1
2

En = 1-2#1 = 1
1 (1 - 2)
At z = 2 , R2 = (z - 2) : X (z) z = 2
gin
= 1 - 2 # 2 =- 3
2 (2 - 1) 2
eer
Sol. 26 Option (B) is correct.
Taking step size h = 0.1, y (0) = 0
ing
x y dy
dx
= x+y
.ne
yi + 1 = yi + h
dy
dx
0
0.1
0.2 0.01
0
0
0.21
0
0.1 t
y1 = 0 + 0.1 (0) = 0
y2 = 0 + 0.1 (0.1) = 0.01
y 3 = 0.01 + 0.21 # 0.1 = 0.031
0.3 0.031
From table, at x = 0.3, y (x = 0.3) = 0.031
Sol. 27 Option (D) is correct.
Given that
f (t) = L - 1 ; 3 3s + 1
s + 4s 2 + (K - 3) s E
lim f (t) = 1
t"3

By final value theorem


lim f (t) = lim sF (s) = 1
t"3 s"0

s : (3s + 1)
or lim =1
s"0 s3 + 4s2 + (K - 3) s

Downloaded From : www.EasyEngineering.net


Downloaded From : www.EasyEngineering.net
GATE SOLVED PAPER - EC ENGINEERING MATHEMATICS

s (3s + 1)
or lim =1
s"0 s [s2 + 4s + (K - 3)]
1 =1
K-3
or K =4
Sol. 28 Option (B) is correct.
The highest derivative terms present in DE is of 2nd order.
Sol. 29 Option (C) is correct.
Number of elements in sample space is
Only 210 . one element
1
"H, H, T, T, T, T, T, T, T, T , is event. Thus probability is 10
2
Sol. 30 Option (C) is correct.
We have

ww f (z) = c0 + c1 z - 1

f1 (z) =
1 + f (z) 1 + c0 + c1 z - 1
= =
z (1 + c0) + c1

w.E z z
Since f1 (z) has double pole at z = 0 , the residue at z = 0 is
Res f1 (z) z = 0 = lim z2 .f1 (z) = lim z2 . c
z2

z (1 + c0) + c1
m = c1
Hence asy z"0 z"0 z2

unit circle En
# f1(z) dz = # [1 +zf (z)] dz = 2pj [Residue at z = 0]
unit circle

Option (D) is correct.


= 2pjc1
gin
eer
Sol. 31

We have f (x) = sin x


x-p
Substituting x - p = y ,we get
sin (y + p) sin y
= - 1 (sin y)
ing
f (y + p) =
y
y 3
=-
y 5
= - 1 cy - + - ...m
y y
.ne
or
y

f (y + p) =- 1 +
Substituting x - p = y we get
3! 5!
y2 y 4
- + ...
3! 5!
t
(x - p) 2 (x - p) 4
f (x) =- 1 + - + ...
3! 5!
Sol. 32 Option (A) is correct.
dy y
(A) =
dx x
dy
or # = # dx
y x
or log y = log x + log c
or y = cx Straight Line
Thus option (A) and (C) may be correct.
dy y
(B) =-
dx x

Downloaded From : www.EasyEngineering.net


Downloaded From : www.EasyEngineering.net
GATE SOLVED PAPER - EC ENGINEERING MATHEMATICS

or # dyy =- # dx
x
or log y =- log x + log c
or log y = log 1 + log c
x
or y = c Hyperbola
x
Sol. 33 Option (D) is correct.
Sum of the principal diagonal element of matrix is equal to the sum of Eigen
values. Sum of the diagonal element is - 1 - 1 + 3 = 1.In only option (D), the
sum of Eigen values is 1.
Sol. 34 Option (C) is correct.
The product of Eigen value is equal to the determinant of the matrix. Since one

ww of the Eigen value is zero, the product of Eigen value is zero, thus determinant
of the matrix is zero.
Thus p11 p22 - p12 p21 = 0

w.E
Sol. 35 Option (B) is correct.
The given system is

asy
4 2 x 7
=2 1G=y G = =6 G

We have
En A == G
4 2
2 1

and A =
4 2
2 1
=0
gin Rank of matrix r (A) < 2

Now C ==
4 2 7
2 1 6
G
eer Rank of matrix r (C) = 2

Sol. 36
Since r (A) ! r (C) there is no solution.
Option (A) is correct. ing
Sol. 37
sin z can have value between - 1 to + 1. Thus no solution.
Option (A) is correct. .ne
We have
For x > 0 ,
For x < 0 ,
f (x) = ex + e-x
ex > 1 and 0 < e-x < 1
0 < ex < 1 and e-x > 1
t
Thus f (x) have minimum values at x = 0 and that is e0 + e-0 = 2 .
Sol. 38 Option (A) is correct.
3 5
sin x = x + x + x + ...
3! 5!
2 4
cos x = 1 + x + x + ...
2! 4!
Thus only sin (x3) will have odd power of x .
Sol. 39 Option (B) is correct.
dx (t)
We have + 3x (t) = 0
dt
or (D + 3) x (t) = 0
Since m =- 3 , x (t) = Ce - 3t Thus only (B) may be solution.

Downloaded From : www.EasyEngineering.net


Downloaded From : www.EasyEngineering.net
GATE SOLVED PAPER - EC ENGINEERING MATHEMATICS

Sol. 40 Option (C) is correct.


We have x = e-x
or f (x) = x - e - x
f'( x) = 1 + e - x
The Newton-Raphson iterative formula is
f (xn)
xn + 1 = xn -
f'( xn)
Now f (xn) = xn - e - x n

f'( xn) = 1 + e - x n

- xn
(1 + xn) e - x
xn + 1 = xn - xn - e- x =
n

Thus
1+e n
1 + e-x n

Sol. 41 Option (A) is correct.

ww Res f (z) z = a = 1 dn - 1 6(z - a) n f (z)@


(n - 1)! dzn - 1 z=a

w.E Here we have n = 2 and a = 2

Thus Res f (z) z = 2 = 1 d (z - 2) 2


(2 - 1)! dz ;
1
(z - 2) 2 (z + 2) 2 Ez = a

asy = d ; 1 2E -2
dz (z + 2) z = a ; (z + 2) 3 Ez = a
=

En =- 2 =- 1
64 32
Sol. 42 Option (D) is correct.
eP = L- 1 6(sI - A) - 1@
gin
s 0
= L- 1 e= G - =
0 1 -1
- 2 - 3Go
eer
= L- 1 e=
0 s
s - 1 -1
2 s + 3G
o ing
= L- 1 f>
s+3
(s + 1)( s + 2)
1
(s + 1)( s + 2)
Hp .ne
==
2e - 1 - e - 2
-2
(s + 1)( s + 2)

e-1 - e-2
- 2e - 1 + 2e - 2 - e - 1 + 2e - 2
s
(s + 1)( s + 2)

G
t
Sol. 43 Option (B) is correct.
Taylor series is given as
(x - a) 2
f (x) = f (a) + x - a f'( a) + f"( a) + ...
1! 2!
For x = p we have
(x - p) 2
Thus f (x) = f (p) + x - p f'( p) + f"( x)...
1! 2!
Now f (x) = ex + sin x
f'( x) = ex + cos x
f"( x) = ex - sin x
f"( p) = e p - sin p = e p

Downloaded From : www.EasyEngineering.net


Downloaded From : www.EasyEngineering.net
GATE SOLVED PAPER - EC ENGINEERING MATHEMATICS

f"( p)
Thus the coefficient of (x - p) 2 is
2!
Sol. 44 Option (A) is correct.
The equation of straight line from (0, 0) to (1, 2) is y = 2x .
Now g (x, y) = 4x3 + 10y 4
or, g (x, 2x) = 4x3 + 160x 4
1 1
Now #0 g (x, 2x) = #0 (4x3 + 160x4) dx
= [x 4 + 32x5] 10 = 33
Sol. 45 Option (B) is correct.
Q
I =2 #P (xdx + ydy)
Q Q

ww =2

=2
#P
#1
0
xdx + 2

xdx + 2
#P ydy
1
#0 ydy = 0
w.E
Sol. 46 Option (B) is correct.
The given plot is straight line whose equation is

asy
x +y =1
-1 1
or
En y = x+1
I =
2
#1 ydx =
2
#1 (x + 1) dx
Now

=;
(x + 1) 2 2 9 4 gin
E = - = 2. 5

Sol. 47 Option (C) is correct.


2 2 2
eer
coth x = cosh x
sinh x
ing
as x << 1, cosh x . 1 and sinh x . x
Thus coth x . 1
x .ne
Sol. 48 Option (A) is correct.

lim
q"0
sin ^ q2 h
q
= lim
sin ^ q2 h
q " 0 2^ q h
= 1 lim sin ^ 2 h = 1 = 0.5
2 q " 0 ^ q2 h
q

2
t
2

Sol. 49 Option (D) is correct.


We have, lim 12 = 3
x"0 x

lim x2 = 3
x"3

lim e - x = 3
x"3

lim e - x = 0
2

x"3

lim e - x = 1 Thus e - x is strictly bounded.


2 2

x"0
Sol. 50 Option (A) is correct.
We have f (x) = e - x = e - (x - 2) - 2 = e - (x - 2) e - 2
(x - 2) 2
= ;1 - (x - 2) + ...Ee - 2
2!

Downloaded From : www.EasyEngineering.net


Downloaded From : www.EasyEngineering.net
GATE SOLVED PAPER - EC ENGINEERING MATHEMATICS

= 61 - (x - 2)@e - 2 Neglecting higher powers


-2
= (3 - x) e
Sol. 51 Option (D) is correct.
d2 y
We have k2 = y - y2
dx2
d2 y y y
or 2
- 2 =- 22
dx k k

A.E. D2 - 12 = 0
k
or D =! 1
k
C.F. = C1 e - + C2 e
x x

ww
k k

1 - y22
P.I. = c m = y2
D2 - 1
k2

w.E
k2
Thus solution is
y = C1 e - + C2 e + y2
x x
k k

asy
From y (0) = y1 we get
C1 + C2 = y1 - y2

Thus En
From y (3) = y2 we get that C1 must be zero.
C2 = y1 - y2

Sol. 52 Option (B) is correct.


y = (y1 - y2) e - + y2
gin x
k

We have
f (x) = x3 - x2 + 4x - 4 eer
f'( x) = 3x2 - 2x + 4
Taking x0 = 2 in Newton-Raphosn method ing
x1 = x0 -
f (x0)
f'( x0)
= 2-
23 - 22 + 4 (2) - 4
3 (2) 2 - 2 (2) + 4
=4
3 .ne
Sol. 53 Option (C) is correct.
For two orthogonal signal f (x) and g (x)
+3
t
#- 3 f (x) g (x) dx = 0

i.e. common area between f (x) and g (x) is zero.


Sol. 54 Option (A) is correct.
We know that
# s2 -1 1 ds = 2pj [sum of residues]
D
Singular points are at s = ! 1 but only s =+ 1 lies inside the given contour,
Thus Residue at s =+ 1 is
lim (s - 1) f (s) = lim (s - 1) 2 1 = 1
s"1 s"1 s -1 2
1 ds = 2pj 1 = pj
# s2 - 1 `2j
D

Downloaded From : www.EasyEngineering.net


Downloaded From : www.EasyEngineering.net
GATE SOLVED PAPER - EC ENGINEERING MATHEMATICS

Sol. 55 Option (C) is correct.


For two orthogonal vectors, we require two dimensions to define them and
similarly for three orthogonal vector we require three dimensions to define them.
2M vectors are basically M orthogonal vector and we require M dimensions to
define them.
Sol. 56 Option (A) is correct.
We have
f (x) = x2 - x + 2
f'( x) = 2x - 1 = 0 " x = 1
2
f"( x) = 2
Since f"( x) = 2 > 0 , thus x = 1 is minimum point. The maximum value in
2
closed interval 6- 4, 4@ will be at x =- 4 or x = 4

ww Now maximum value


= max [f (- 4), f (4)]

w.E = max (18, 10)


= 18
Sol. 57

asy
Option (C) is correct.
Probability of failing in paper 1 is P (A) = 0.3

En
Possibility of failing in Paper 2 is
Probability of failing in paper 1, when
P (B) = 0.2

student has failed in paper 2 is


We know that ginP ^ BA h = 0.6

Pb A l =
B
(P + B)
P (B)
eer
Sol. 58
or

Option (C) is correct.


B
ing
P (A + B) = P (B) P b A l = 0.6 # 0.2 = 0.12

We have
R V R
S1 1 1 W S1 1 1 W
V .ne
A = S1 - 1 0 W + S1 - 1 0 W
SS1 1 1 WW SS0 0 0 WW
T X T X
t R3 - R1

Since one full row is zero, r (A) < 3


1 1
Now =- 2 ! 0 , thus r (A) = 2
1 -1
Sol. 59 Option (D) is correct.
The vector Triple Product is
A # (B # C) = B (A $ C) - C (A $ B)
Thus 4#4# P = 4 (4$ P) - P (4$4) = 4 (4$ P) - 4 2 P

Sol. 60 Option (A) is correct.


The Stokes theorem is
## (4 # F) $ ds = # A $ dl
Sol. 61 Option (C) is correct.

Downloaded From : www.EasyEngineering.net


Downloaded From : www.EasyEngineering.net
GATE SOLVED PAPER - EC ENGINEERING MATHEMATICS

# p (x) dx
3
We know =1
-3

#
3
Thus Ke- a x dx = 1
-3
0
# # Ke
3
or Keax dx + - ax
dx = 1
-3 0

a 6 @- 3 (- a) 6 @0
or K eax 0 + k e- ax 3 = 1

or K +K =1
a a
or K =a
2
Sol. 62 Option (A) is correct.
We have xo (t) + 2x (t) = s (t)

ww Taking Laplace transform both sides


sX (s) - x (0) + 2X (s) = 1
Since x (0 -) = 0

w.E or sX (s) + 2X (s) = 1


X (s) = 1
s+2

asy
Now taking inverse Laplace transform we have
x (t) = e - 2t u (t)
Sol. 63

En
Option (A) is correct.
Sum of the Eigen values must be equal to the sum of element of principal diagonal
of matrix.
6 2
Only matrix = G satisfy this condition. gin
Sol. 64
2 6
Option (B) is correct. eer
We have W = ln z
u + jv = ln (x + jy) ing
or
or
eu + jv = x + jy
eu e jv = x + jy .ne
Thus
eu (cos v + j sin v) = x + jy
Now x = eu cos v and y = eu sin v
x 2 + y 2 = e 2u
t Equation of circle
Sol. 65 Option (D) is correct.
We have
# z2 +1 4 dz = # 1
(z + 2i) (z - 2i)
dz
z-j = 2 z-j = 2

P (0, 2) lies inside the circle z - j = 2 and P (0, - 2) does not lie.
Thus By cauchy’s integral formula
I = 2pi lim (z - 2i) 1 = # 2p i = p
z " 2i (z + 2i)( z - 2i) 2i + 2i 2
C
Sol. 66 Option (C) is correct.
p
I = #0 sin3 qdq

Downloaded From : www.EasyEngineering.net


Downloaded From : www.EasyEngineering.net
GATE SOLVED PAPER - EC ENGINEERING MATHEMATICS

3 sin q - sin 3q dq
p
= #0 ` 4 j sin 3q = 3 sin q - 4 sin3 q

= :- 3 cos qD = : ws3q D = 8 3 + 3 B - 8 1 + 1 B = 4
p p

4 0
12 0 4 4 12 12 3
Sol. 67 Option (D) is correct.
Let d " defective and y " supply by Y
y P (y + d)
pa k =
d P (d)
P (y + d) = 0.3 # 0.02 = 0.006
P (d) = 0.6 # 0.1 + 0.3 # 0.02 + 0.1 # 0.03 = 0.015
y
P a k = 0.006 = 0.4
d 0.015
Sol. 68 Option (C) is correct.

ww We have A ==
4 2
2 4G

w.E Now

or
4 - l 2 101
6A - lI @[X] = 0
0
= 2 4 - l G=101G = =0 G

or
asy
(101)( 4 - l) + 2 (101) = 0

Sol. 69
or

En
Option (A) is correct.
l =6

We have
d2 y
dx 2
+ k2 y = 0
gin
or
The AE is
D2 y + k2 y = 0
m2 + k2 = 0 eer
The solution of AE is m = ! ik
Thus y = A sin kx + B cos kx ing
From x = 0 , y = 0 we get B = 0 and x = a, y = 0 we get
A sin ka = 0 .ne
or sin ka = 0
k = mpx
a
t
Thus y = / Am sin ` mpx j
m
a
Sol. 70 Option (A) is correct.
We have f (x) = ex
1 + ex
For x " 3, the value of f (x) monotonically increases.
Sol. 71 Option (B) is correct.
Order is the highest derivative term present in the equation and degree is the
power of highest derivative term.
Order = 2 , degree = 1
Sol. 72 Option (D) is correct.
Probability of coming odd number is 12 and the probability of coming even number
is 12 . Both the events are independent to each other, thus probability of coming

Downloaded From : www.EasyEngineering.net


Downloaded From : www.EasyEngineering.net
GATE SOLVED PAPER - EC ENGINEERING MATHEMATICS

odd number after an even number is 1


2 # 12 = 14 .
Sol. 73 Option (B) is correct.
d2 y dy
We have -5 + 6y = 0
dx2 dx
The A.E. is m2 - 5m + 6 = 0
m = 3, 2
The CF is yc = C1 e3x + C2 e2x
Since Q = 0 , thus y = C1 e3x + C2 e2x
Thus only (B) may be correct.
Sol. 74 Option (A) is correct.
We have f (t) = e(a + 2) t + 5 = e5 .e(a + 2) t
Taking Laplace transform we get

ww F (s) = e5 ; 1
s - (a + 2) E
Thus Re (s) > (a + 2)

w.E
Sol. 75

Sol. 76
Option (C) is correct.
For x > 0 the slope of given curve is negative. Only (C) satisfy this condition.
Option (C) is correct.
Newton - Raphson
asy
Runge - kutta Method
" Method-Solving nonlinear eq.
" Solving ordinary differential eq.
Simpson’s Rule
En " Numerical Integration

Sol. 77
Gauss elimination
Option (C) is correct. gin
" Solving linear simultaneous eq.

We have A ==
-4 2
4 3G
eer
Characteristic equation is
A - lI = 0 ing
or
4-l 2
4 3-l
=0 .ne
or
or
or
(- 4 - l)(3 - l) - 8 = 0
- 12 + l + l2 - 8 = 0
l2 + l - 20 = 0
t
or l =- 5, 4 Eigen values
Eigen vector for l =- 5
(A - lI) Xi = 0
1 - (- 5) 2 x1 0
= 4 G=
8 - 4 x2 G == G
0

1 2 x1 0
=0 0G=x G = =0 G R2 - 4R1
2

x1 + 2x2 = 0
Let - x1 = 2 & x2 =- 1,
2
X ==
-1G
Thus Eigen vector

Downloaded From : www.EasyEngineering.net


Downloaded From : www.EasyEngineering.net
GATE SOLVED PAPER - EC ENGINEERING MATHEMATICS

Sol. 78 Option (A) is correct.


We have
2 - 0. 1 1
a
A == G and A - 1 = = 2 G
0 3 0 b
Now AA - 1 = I
2 - 0. 1 1 a 1 0
or =0 3 G= 2 G = =0 1G
0 b
1 2a - 0.1b 1 0
or =0 3b G == G
0 1
or 2a - 0.1 = 0 and 3b = 1
Thus solving above we have b = 1 and a = 1
3 60

ww Therefore a+b = + 1
3 60
1 = 7
20

w.E
Sol. 79 Option (A) is correct.
Gaussian PDF is
f (x) = 1
#
3 - (x - m)2
e 2s2 dx for - 3 # x # 3

and asy # f (x) dx


3
2p s

=1
-3

En -3

Substituting m = 0 and s = 2 in above we get


1
#
3 -

2p 2 - 3
e dx = 1
x2
8

gin
or 1 2
# e
2p 2 0
3 - x2
8
dx = 1
eer
or 1
2p
#
0
3 - x2
e 8
dx = 1
ing
Sol. 80 Option (C) is correct.
From orthogonal matrix .ne
[AAT ] = I
Since the inverse of I is I , thus
[AAT ] -1 = I-1 = I
t
*********

Downloaded From : www.EasyEngineering.net


Downloaded From : www.EasyEngineering.net

No part of this publication may be reproduced or distributed in any form or any means, electronic, mechanical,
photocopying, or otherwise without the prior permission of the author.

ww
w.E
GATE SOLVED PAPER
Electronics & Communication
General Apitude
asy
Copyright © By NODIA & COMPANY
En
gin
eer
ing
Information contained in this book has been obtained by authors, from sources believes to be reliable. However,
neither Nodia nor its authors guarantee the accuracy or completeness of any information herein, and Nodia nor its

.ne
authors shall be responsible for any error, omissions, or damages arising out of use of this information. This book
is published with the understanding that Nodia and its authors are supplying information but are not attempting
to render engineering or other professional services.

t
NODIA AND COMPANY
B-8, Dhanshree Tower Ist, Central Spine, Vidyadhar Nagar, Jaipur 302039
Ph : +91 - 141 - 2101150
www.nodia.co.in
email : enquiry@nodia.co.in

Downloaded From : www.EasyEngineering.net


Downloaded From : www.EasyEngineering.net

GATE SOLVED PAPER - EC


GENERAL APITUDE

YEAR 2013 ONE MARK

Q. 1 Choose the grammatically CORRECT sentence:


(A) Two and two add four
(B) Two and two become four
(C) Two and two are four
(D) Two and two make four

ww
Q. 2 Statement: You can always give me a ring whenever you need.
Which one of the following is the best inference from the above statement?

w.E (A) Because I have a nice caller tune.


(B) Because I have a better telephone facility

asy
(C) Because a friend in need is a friend indeed
(D) Because you need not pay towards the telephone bills when you give me a

Q. 3
ring

En
In the summer of 2012, in New Delhi, the mean temperature of Monday to

gin
Wednesday was 41°C and of Tuesday to Thursday was 43cC . If the temperature
on Thursday was 15% higher than that of Monday, then the temperature in cC
on Thursday was
(A) 40 (B) 43 eer
Q. 4
(C) 46 (D) 49
ing
Complete the sentence: Dare .................. mistakes.
(A) commit
(C) committed .ne
(B) to commit
(D) committing

Q. 5 They were requested not to quarrel with others. t


Which one of the following options is the closest in meaning to the word quarrel?
(A) make out (B) call out
(C) dig out (D) fall out
Sol. 1 Option (D) is correct.
They were requested not to quarrel with others.
Quarrel has a similar meaning to ‘fall out’

YEAR 2013 TWO MARKS

Q. 6 A car travels 8 km in the first quarter of an hour, 6 km in the second quarter and
16 km in the third quarter. The average speed of the car in km per hour over the
entire journey is
(A) 30 (B) 36
(C) 40 (D) 24

Downloaded From : www.EasyEngineering.net


Downloaded From : www.EasyEngineering.net
GATE SOLVED PAPER - EC GENERAL APITUDE

Q. 7 Find the sum to n terms of the series 10 + 84 + 734 + ...


9 ^9n + 1h 9 ^9n - 1h
(A) +1 (B) +1
10 8

9 ^9n - 1h 9 ^9n - 1h
(C) +n (D) + n2
8 8

Q. 8 Statement: There were different streams of freedom movements in colonial India


carried out by the moderates, liberals, radicals, socialists, and so on.
Which one of the following is the best inference from the above statement?
(A) The emergence of nationalism in colonial India led to our Independence
(B) Nationalism in India emerged in the context of colonialism
(C) Nationalism in India is homogeneous
(D) Nationalism in India is heterogeneous

ww
Q. 9 The set of values of p for which the roots of the equation 3x2 + 2x + p ^p - 1h = 0
are of opposite sign is

w.E (A) ^- 3, 0h
(C) ^1, 3h
(B) ^0, 1h
(D) ^0, 3h

Q. 10
(A) 2/7 asy
What is the chance that a leap year, selected at random, will contain 53 Sundays?
(B) 3/7
(C) 1/7
En (D) 5/7

2012
gin ONE MARK

Q. 11
(A) 2.23 (B) 4.33 eer
If (1.001) 1259 = 3.52 and (1.001) 2062 = 7.85, then (1.001) 3321

Q. 12
(C) 11.37 (D) 27.64
ing
Choose the most appropriate alternate from the options given below to complete
the following sentence :
.ne
If the tired soldier wanted to lie down, he. ................the mattress out on the
balcony.
(A) should take
(C) should have taken
(B) shall take
(D) will have taken
t
Q. 13 Choose the most appropriate word from the options given below to complete the
following sentence :
Give the seriousness of the situation that he had to face, his. ......was impressive.
(A) beggary (B) nomenclature
(C) jealousy (D) nonchalance

Q. 14 Which one of the following options is the closest in meaning to the word given
below ?
Latitude
(A) Eligibility (B) Freedom
(C) Coercion (D) Meticulousness

Downloaded From : www.EasyEngineering.net


Downloaded From : www.EasyEngineering.net
GATE SOLVED PAPER - EC GENERAL APITUDE

Q. 15 One of the parts (A, B, C, D) in the sentence given below contains an ERROR.
Which one of the following is INCORRECT ?
I requested that he should be given the driving test today instead of tomorrow.
(A) requested that (B) should be given
(C) the driving test (D) instead of tomorrow

2012 TWO MARKS

Q. 16 One of the legacies of the Roman legions was discipline. In the legious, military
law prevailed and discipline was brutal. Discipline on the battlefield kept units
obedient, intact and fighting, even when the odds and conditions were against
them.
Which one of the following statements best sums up the meaning of the above
passage ?

ww (A) Through regimentation was the main reason for the efficiency of the Roman
legions even in adverse circumstances.

w.E (B) The legions were treated inhumanly as if the men were animals
(C) Disciplines was the armies inheritance from their seniors
(D) The harsh discipline to which the legions were subjected to led to the odds

asy
and conditions being against them.

Q. 17

En
Raju has 14 currency notes in his pocket consisting of only Rs. 20 notes and Rs.
10 notes. The total money values of the notes is Rs. 230. The number of Rs. 10
notes that Raju has is
(A) 5
gin(B) 6

Q. 18
(C) 9

eer
(D) 10

There are eight bags of rice looking alike, seven of which have equal weight and

ing
one is slightly heavier. The weighing balance is of unlimited capacity. Using this
balance, the minimum number of weighings required to identify the heavier bag is
(A) 2
(C) 4
(B) 3
(D) 8 .ne
Q. 19
average household.
t
The data given in the following table summarizes the monthly budget of an

Category Amount (Rs.)


Food 4000
Clothing 1200
Rent 2000
Savings 1500
Other Expenses 1800

The approximate percentages of the monthly budget NOT spent on savings is


(A) 10% (B) 14%
(C) 81% (D) 86%

Downloaded From : www.EasyEngineering.net


Downloaded From : www.EasyEngineering.net
GATE SOLVED PAPER - EC GENERAL APITUDE

Q. 20 A and B are friends. They decide to meet between 1 PM and 2 PM on a given


day. There is a conditions that whoever arrives first will not wait for the other for
more than 15 minutes. The probability that they will meet on that days is
(A) 1/4 (B) 1/16
(C) 7/16 (D) 9/16

2011 ONE MARK

Q. 21 There are two candidates P and Q in an election. During the campaign, 40% of
voter promised to vote for P , and rest for Q . However, on the day of election 15%
of the voters went back on their promise to vote for P and instead voted for Q .
25% of the voter went back on their promise to vote for Q and instead voted for
P . Suppose, P lost by 2 votes, then what was the total number of voters ?
(A) 100 (B) 110

ww (C) 90 (D) 95

w.E
Q. 22 The question below consists of a pair of related words followed by four pairs of
words. Select the pair that best expresses the relations in the original pair :
Gladiator : Arena

asy
(A) dancer : stage
(C) teacher : classroom
(B) commuter : train
(D) lawyer : courtroom

Q. 23
following sentence :En
Choose the most appropriate word from the options given below to complete the

gin
Under ethical guidelines recently adopted by the Indian Medical Association,
human genes are to be manipulated only to correct diseases for which. .................
treatments are unsatisfactory.
(A) similar (B) most eer
Q. 24
(C) uncommon (D) available
ing
Choose the word from the from the options given below that is most opposite in
meaning to the given word :
Frequency .ne
Q. 25
(A) periodicity
(C) gradualness
(B) rarity
(D) persistency t
Choose the most appropriate word from the options given below to complete the
following sentence :
It was her view that the country’s had been . ........... by foreign techno-crafts, so
that to invite them to come back would be counter-productive.
(A) identified (B) ascertained
(C) exacerbated (D) analysed

2011 TWO MARKS

Q. 26 The fuel consumed by a motor cycle during a journey while travelling at various
speed is indicated in the graph below.

Downloaded From : www.EasyEngineering.net


Downloaded From : www.EasyEngineering.net
GATE SOLVED PAPER - EC GENERAL APITUDE

The distance covered during four laps of the journey are listed in the table below
Lap Distance (km) Average speed (km/hour)
P 15 15
Q 75 45

ww R
S 10
40
10
75

w.E From the given data, we can conclude that the fuel consumed per kilometre was
least during the lap
(A) P (B) Q

Q. 27
(C) R
asy (D) S

The horse has played a little known but very important role in the field of

En
medicine. Horses were injected with toxins of disease until their blood build up
immunities. Then a serum was made from their blood. Serums to fight with

gin
diphteria and tetanus were developed this way.
It can be inferred from the passage, that horses were
(A) given immunity to diseases
(B) generally quite immune to diseases eer
(C) given medicines to fight toxins
(D) given diphtheria and tetanus serums ing
Q. 28 The sum of n terms of the series 4 44 + 444 + ........
.ne
Q. 29
(A) (4/81) [10n + 1 - 9n - 1]
(C) (4/81) [10n + 1 - 9n - 10]
+

t
(B) (4/81) [10n - 1 - 9n - 1]
(D) (4/81) [10n - 9n - 10]

Given that f (y) = y /y, and q is any non-zero real number, the value of
f (q) - f (- q) is
(A) 0 (B) - 1
(C) 1 (D) 2

Q. 30 Three friends R, S and T shared toffee from a bowl. R took 1/3 rd of the toffees,
but returned four to the bowl. S took 1/4 th of what was left but returned three
toffees to the bowl. T took half of the remainder but returned two back into the
bowl. If the bowl had 17 toffees left, how many toffees were originally there in
the bowl ?
(A) 38 (B) 31
(C) 48 (D) 41

Downloaded From : www.EasyEngineering.net


Downloaded From : www.EasyEngineering.net
GATE SOLVED PAPER - EC GENERAL APITUDE

2010 ONE MARK

Q. 31 Which of the following options is the closest in meaning to the word below ?
Circuitous
(A) Cyclic (B) Indirect
(C) Confusing (D) Crooked

Q. 32 The question below consist of a pair of related words followed by four pairs of
words. Select the pair that best expresses the relation in the original pair.
Unemployed : Worker
(A) Fallow : Land (B) Unaware : Sleeper
(C) Wit : Jester (D) Renovated : House

Q. 33 Choose the most appropriate word from the options given below to complete the
following sentence :

ww If we manage to . ...... our natural resources, we would leave a better planet for
our children.

w.E (A) unhold


(C) cherish
(B) restrain
(D) conserve

Q. 34

asy
Choose the most appropriate word from the options given below to complete the
following sentence :
His rather casual remarks on politics. ................his lack of seriousness about the
subject.
(A) masked En (B) belied
(C) betrayed
gin(D) suppressed

Q. 35

eer
25 persons are in a room 15 of them play hockey, 17 of them play football and
10 of them play hockey and football. Then the number of persons playing neither
hockey nor football is
(A) 2
(C) 13
(B) 17
(D) 3
ing
.ne
Q. 36
2010

t TWO MARKS

Modern warfare has changed from large scale clashes of armies to suppression of
civilian populations. Chemical agents that do their work silently appear to be
suited to such warfare ; and regretfully, their exist people in military establishments
who think that chemical agents are useful fools for their cause.
Which of the following statements best sums up the meaning of the above passage
?
(A) Modern warfare has resulted in civil strife.
(B) Chemical agents are useful in modern warfare.
(C) Use of chemical agents in ware fare would be undesirable.
(D) People in military establishments like to use chemical agents in war.

Q. 37 If 137 + 276 = 435 how much is 731 + 672 ?


(A) 534 (B) 1403
(C) 1623 (D) 1531

Downloaded From : www.EasyEngineering.net


Downloaded From : www.EasyEngineering.net
GATE SOLVED PAPER - EC GENERAL APITUDE

Q. 38 5 skilled workers can build a wall in 20 days; 8 semi-skilled workers can build a
wall in 25 days; 10 unskilled workers can build a wall in 30 days. If a team has 2
skilled, 6 semi-skilled and 5 unskilled workers, how long will it take to build the
wall ?
(A) 20 days (B) 18 days
(C) 16 days (D) 15 days

Q. 39 Given digits 2, 2, 3, 3, 3, 4, 4, 4, 4 how much distinct 4 digit numbers greater than


3000 can be formed ?
(A) 50 (B) 51
(C) 52 (D) 54

Q. 40 Hari (H), Gita (G), Irfan (I) and Saira (S) are siblings (i.e. brothers and sisters.)
All were born on 1st January. The age difference between any two successive

ww siblings (that is born one after another) is less than 3 years. Given the following
facts :
1. Hari’s age + Gita’s age > Irfan’s age + Saira’s age.

w.E 2. The age difference between Gita and Saira is 1 year. However, Gita is not
the oldest and Saira is not the youngest.

asy
3. There are no twins.
In what order were they born (oldest first) ?
(A) HSIG
(C) IGSH
En (B) SGHI
(D) IHSG

gin
***********

eer
ing
.ne
t

Downloaded From : www.EasyEngineering.net


Downloaded From : www.EasyEngineering.net
GATE SOLVED PAPER - EC GENERAL APITUDE

SOLUTIONS

Sol. 1 Option (D) is correct


Two and two make four
Sol. 2 Option (C) is correct.
You can always given me a ring whenever you need. Because a friend is need is a
friend indeed
Sol. 3 Option (C) is correct.
Let the temperature on Monday, Tuesday, Wednesday and Thursday be respectively

ww as TM , TTU , TW , TTH
So, from the given data we have

w.E and
TH + TTU + TW = 41
3
TTU + TW + TTH = 43
....(1)

....(2)

asy 3
also, as the temperature on Thursday was 15% higher than that of Monday
i.e.

En TTH = 1.15 TM
solving eq (1), (2) and (3), we obtain
....(3)

Sol. 4 Option (B) is correct. gin


TTH = 46cC

Sol. 5
Dare to commit mistakes
Option (D) is correct. eer
They were requested not to quarrel with others.
Quarrel has a similar meaning to ‘fall out’ ing
Sol. 6 Option (C) is correct.
.ne
Distance
8 km
Time Duration
1
4 hr
t
Given, the distance travelled by the car in each quarter intervals as

6 km 1
4 hr
16 km 1
4 hr
Therefore, the total time taken = 1 + 1 + 1 + 3 hr
4 4 4 4
Total distance travelled = 8 + 6 + 16 = 30 km
Hence, average speed = Total distance travelled
Total time taken
= 30 = 40 km/hr
3/4
Sol. 7 Option (D) is correct.
It will be easy to check the options for given series. From the given series.
10 + 84 + 734 + . ....

Downloaded From : www.EasyEngineering.net


Downloaded From : www.EasyEngineering.net
GATE SOLVED PAPER - EC GENERAL APITUDE

We get
Sum of 1 term = S1 = 10
Sum of 2 terms = S2 = 10 + 84 = 94
and sum of 3 terms = S 3 = 10 + 84 + 734 = 828
Checking all the options one by one, we observe that only (D) option satisfies as
9 ^9n - 1h
Sn = + n2
8
9 ^92 - 1h
so, S1 + 22 = 10
8
9 ^9 - 1h
S2 = + 22 = 94
8
9 ^93 - 1h
S3 = + 32 = 828
8
Sol. 8 Option (D) is correct.

ww
Sol. 9
Nationalism in India is heterogeneous
Option (B) is correct.

w.E Given, the quadratic equation


3x2 + 2x + P ^P - 1h = 0
It will have the roots with opposite sign if

asy P ^P - 1h < 0
So it can be possible only when

En P < 0 and P - 1 > 0


or
st
gin
P > 0 and P - 1 < 0
The 1 condition tends to no solution for P .
Hence, from the second condition, we obtain
0 <P<1 eer
Sol. 10
i.e., P is in the range ^0, 1h
Option (A) is correct. ing
.ne
In a leap year, there are 366 days So, 52 weeks will have 52 saturdays and for
remaining two days ^366 - 52 # 7 = 2h. We can have the following combinations
Saturday, Sunday
Sunday, Monday
Monday, Tuesday
t
Tuesday, Wednesday
Wednesday, Thursday
Thursday, Friday
Friday, Saturday
Out of these seven possibilities, only two consist a saturday. Therefore, the
probability of saturday is given as
P =2
7
Sol. 11 Option (D) is correct.
Let 1.001 = x
So in given data :
x1259 = 3.52
x2062 = 7.85

Downloaded From : www.EasyEngineering.net


Downloaded From : www.EasyEngineering.net
GATE SOLVED PAPER - EC GENERAL APITUDE

Again x3321 = x1259 + 2062


= x1259 x2062
= 3.52 # 7.85
= 27.64
Sol. 12 Option (C) is correct.
Sol. 13 Option (D) is correct.
Sol. 14 Option (B) is correct.
Sol. 15 Option (B) is correct.
Sol. 16 Option (A) is correct.
Sol. 17 Option (A) is correct.
Let no. of notes of Rs.20 be x and no. of notes of Rs. 10 be y .

ww Then from the given data.


x + y = 14

w.E 20x + 10y = 230


Solving the above two equations we get
x = 9, y = 5

Sol. 18 asy
So, the no. of notes of Rs. 10 is 5.
Option (A) is correct.

En
We will categorize the 8 bags in three groups as :
(i) A1 A2 A 3 , (ii) B1 B2 B 3 , (iii) C1 C2
Weighting will be done as bellow :
gin
1st weighting " A1 A2 A 3 will be on one side of balance and B1 B2 B 3 on the other.

Case 1 : A1 A 2 A 3 = B1 B 2 B 3 eer
It may have three results as described in the following cases.

ing
This results out that either C1 or C2 will heavier for which we will have to perform
weighting again.

if C1 > C 2 then C1 is heavier.


.ne
2 nd weighting " C1 is kept on the one side and C2 on the other.

Case 2 :
C1 < C 2 then C2 is heavier.
A1 A 2 A 3 > B1 B 2 B 3 t
it means one of the A1 A2 A 3 will be heavier So we will perform next weighting as:
2 nd weighting " A1 is kept on one side of the balance and A2 on the other.
if A1 = A 2 it means A 3 will be heavier
A1 > A 2 then A1 will be heavier
A1 < A 2 then A2 will be heavier
Case 3 : A1 A 2 A 3 < B1 B 2 B 3
This time one of the B1 B2 B 3 will be heavier, So again as the above case weighting
will be done.
2 nd weighting " B1 is kept one side and B2 on the other
if B1 = B 2 B 3 will be heavier
B1 > B 2 B1 will be heavier
B1 < B 2 B2 will be heavier
So, as described above, in all the three cases weighting is done only two times to

Downloaded From : www.EasyEngineering.net


Downloaded From : www.EasyEngineering.net
GATE SOLVED PAPER - EC GENERAL APITUDE

give out the result so minimum no. of weighting required = 2.


Sol. 19 Option (D) is correct.
Total budget = 4000 + 1200 + 2000 + 1500 + 1800 = 10, 500
The amount spent on saving = 1500
So, the amount not spent on saving
= 10, 500 - 1500 = 9000
So, percentage of the amount
= 9000 # 100% = 86%
10500
Sol. 20 Option (S) is correct.
The graphical representation of their arriving time so that they met is given as
below in the figure by shaded region.

ww
w.E
asy
En
gin
So, the area of shaded region is given by
eer
Area of 4PQRS
ing
- (Area of TEFQ + Area of
TGSH )

.ne
= 60 # 60 - 2 b 1 # 45 # 45 l
2
= 1575
So, the required probability = 1575 = 7
3600 16
t
Sol. 21 Option (A) is correct.
Let us assume total voters are 100. Thus 40 voter (i.e. 40 %) promised to vote for
P and 60 (rest 60 % ) promised to vote fore Q.
Now, 15% changed from P to Q (15 % out of 40)
Changed voter from P to Q 15 40 = 6
100 #
Now Voter for P 40 - 6 = 34
Also, 25% changed form Q to P (out of 60%)
Changed voter from Q to P 25 60 = 15
100 #
Now Voter for P 34 + 15 = 49
Thus P P got 49 votes and Q got 51 votes, and P lost by 2 votes, which is
given. Therefore 100 voter is true value.

Downloaded From : www.EasyEngineering.net


Downloaded From : www.EasyEngineering.net
GATE SOLVED PAPER - EC GENERAL APITUDE

Sol. 22 Option (A) is correct.


A gladiator performs in an arena. Commutators use trains. Lawyers performs,
but do not entertain like a gladiator. Similarly, teachers educate. Only dancers
performs on a stage.
Sol. 23 Option (D) is correct.
Available is appropriate because manipulation of genes will be done when other
treatments are not useful.
Sol. 24 Option (B) is correct.
Periodicity is almost similar to frequency. Gradualness means something happening
with time. Persistency is endurance. Rarity is opposite to frequency.
Sol. 25 Option (C) is correct.
The sentence implies that technocrats are counterproductive (negative). Only (C)
can bring the same meaning.

ww
Sol. 26 Option (B) is correct.
Since fuel consumption/litre is asked and not total fuel consumed, only average

w.E speed is relevant. Maximum efficiency comes at 45 km/hr, So least fuel consumer
per litre in lap Q
Sol. 27

asy
Option (B) is correct.
Option B fits the sentence, as they built up immunities which helped humans
create serums from their blood.
Sol. 28
En
Option (C) is correct.

gin
4 + 44 + 444 + .............. 4 (1 + 11 + 111 + .......)
= 4 (9 + 99 + 999 + ............)
9

eer
= [(9 - 1) + (100 - 1) + ........]
4 10

ing
= 4 [10 (1 + 10 + 102 + 103) - n]
9
n
= 4 :10 # 10 - 1 - nD
9 10 - 1 .ne
Sol. 29 Option (D) is correct.
= 4 610n + 1 - 10 - 9n@
81
t
y
f (y) =
y
-y
Now f (- y) = =- f (y)
y
or f (q) - f (- q) = 2f (q) = 2
Sol. 30 Option (C) is correct.
Let total no of toffees be x . The following table shows the all calculations.
Friend Bowl Status
R = x -4 = 2x + 4
3 3

Downloaded From : www.EasyEngineering.net


Downloaded From : www.EasyEngineering.net
GATE SOLVED PAPER - EC GENERAL APITUDE

S = 1 :2x + 4D - 3 = 2x + 4 - x + 2
4 3 3 6
= x +1-3 = x -2 = x +6
6 6 2

T = 1 a x + 6k - 2 = x +6-x -1
2 2 2 4
= x +1 = x +5
4 4

Now, x + 5 = 17
4
or x = 17 - 5 = 12
4
x = 12 # 4 = 48

ww
Sol. 31 Option (B) is correct.
Circuitous means round about or not direct. Indirect is closest in meaning to this

w.E circuitous
(A) Cyclic
(B) Indirect
: Recurring in nature
: Not direct

(D) Crooked asy


(C) Confusing : lacking clarity of meaning
: set at an angle; not straight
Sol. 32
En
Option (B) is correct.

Sol. 33 Option (D) is correct.


gin
A worker may by unemployed. Like in same relation a sleeper may be unaware.

Sol. 34
Here conserve is most appropriate word.
Option (C) is correct. eer
Sol. 35 Option (D) is correct. ing
Betrayed means reveal unintentionally that is most appropriate.

Number of people who play hockey n (A) = 15


Number of people who play football n (B) = 17 .ne
Persons who play both hockey and football n (A + B) = 10
Persons who play either hockey or football or both :
n (A , B) = n (A) + n (B) - n (A + B)
t
= 15 + 17 - 10 = 22
Thus people who play neither hockey nor football = 25 - 22 = 3
Sol. 36 Option (D) is correct.
Sol. 37 Option (C) is correct.
Since 7 + 6 = 13 but unit digit is 5 so base may be 8 as 5 is the remainder when
13 is divided by 8. Let us check.
137 8 731 8
276 8 672 8
435 Thus here base is 8. Now 1623
Sol. 38 Option (D) is correct.
Let W be the total work.
Per day work of 5 skilled workers =W
20

Downloaded From : www.EasyEngineering.net


Downloaded From : www.EasyEngineering.net
GATE SOLVED PAPER - EC GENERAL APITUDE

Per day work of one skill worker =W =W


5 # 20 100
Similarly per day work of 1 semi-skilled workers = W = W
8 # 25 200
Similarly per day work of one semi-skill worker = W = W
10 # 30 300
Thus total per day work of 2 skilled, 6 semi-skilled and 5 unskilled workers is
= 2W + 6W + 5W = 12W + 18W + 10W = W
100 200 300 600 15
Therefore time to complete the work is 15 days.
Sol. 39 Option (B) is correct.
As the number must be greater than 3000, it must be start with 3 or 4. Thus we
have two case:
Case (1) If left most digit is 3 an other three digits are any of 2, 2, 3, 3, 4, 4, 4, 4.

ww (1) Using 2, 2, 3 we have 3223, 3232, 3322 i.e. 3! = 3 no.


2!

w.E (2) Using 2, 2, 4 we have 3224, 3242, 3422 i.e. 3


2!
! = 3 no.

(3) Using 2, 3, 3 we have 3233, 3323, 3332 i.e. 3! = 3 no.

asy
(4) Using 2, 3, 4 we have 3! = 6 no.
2!

En
(5) Using 2, 4, 4 we have 3244, 3424, 3442 i.e. 3! = 3 no.
2!
(6) Using 3, 3, 4 we have 3334, 3343, 3433 i.e. 3! = 3 no.

gin 2!
(7) Using 3, 4, 4 we have 3344, 3434, 3443 i.e. 3! = 3 no.

(8) Using 4, 4, 4 we have 3444 i.e. 3!


3!
= 1 no.
2!
eer
Total 4 digit numbers in this case is
1 + 3 + 3 + 3 + 6 + 3 + 3 + 3 + 1 = 25
ing
.ne
Case 2 : If left most is 4 and other three digits are any of 2, 2, 3, 3, 3, 4, 4, 4.
(1) Using 2, 2, 3 we have 4223, 4232, 4322 i.e. . 3! = 3 no
2!
(2) Using 2, 2, 4 we have 4224, 4242, 4422 i.e. . 3! = 3 no
2!
t
(3) Using 2, 3, 3 we have 4233, 4323, 4332 i.e. . 3! = 3 no
2!
(4) Using 2, 3, 4 we have i.e. . 3! = 6 no
(5) Using 2, 4, 4 we have 4244, 4424, 4442 i.e. . 3! = 3 no
2!
(6) Using 3, 3, 3 we have 4333 i.e 3! = 1. no.
3!
(7) Using 3, 3, 4 we have 4334, 4343, 4433 i.e. . 3! = 3 no
2!
(8) Using 3, 4, 4 we have 4344, 4434, 4443 i.e. . 3! = 3 no
2!
(9) Using 4, 4, 4 we have 4444 i.e. 3 ! = 1. no
3!
Total 4 digit numbers in 2nd case = 3 + 3 + 3 + 6 + 3 + 3 + 1 + 3 + 1 = 26

Downloaded From : www.EasyEngineering.net


Downloaded From : www.EasyEngineering.net
GATE SOLVED PAPER - EC GENERAL APITUDE

Thus total 4 digit numbers using case (1) and case (2) is = 25 + 26 = 51
Sol. 40 Option (B) is correct.
Let H , G , S and I be ages of Hari, Gita, Saira and Irfan respectively.
Now from statement (1) we have H + G > I + S
Form statement (2) we get that G - S = 1 or S - G = 1
As G can’t be oldest and S can’t be youngest thus either GS or SG possible.
From statement (3) we get that there are no twins
(A) HSIG : There is I between S and G which is not possible
(B) SGHI : SG order is also here and S > G > H > I and G + H > S + I which
is possible.
(C) IGSH : This gives I > G and S > H and adding these both inequalities we
have I + S > H + G which is not possible.

ww (D) IHSG : This gives I > H and S > G and adding these both inequalities we
have I + S > H + G which is not possible.

w.E **********

asy
En
gin
eer
ing
.ne
t

Downloaded From : www.EasyEngineering.net


Downloaded From : www.EasyEngineering.net

No part of this publication may be reproduced or distributed in any form or any means, electronic, mechanical,
photocopying, or otherwise without the prior permission of the author.

ww
w.E
GATE SOLVED PAPER
Electronics & Communication
Network Analysis
asy
Copyright © By NODIA & COMPANY
En
gin
eer
ing
Information contained in this book has been obtained by authors, from sources believes to be reliable. However,
neither Nodia nor its authors guarantee the accuracy or completeness of any information herein, and Nodia nor its

.ne
authors shall be responsible for any error, omissions, or damages arising out of use of this information. This book
is published with the understanding that Nodia and its authors are supplying information but are not attempting
to render engineering or other professional services.

t
NODIA AND COMPANY
B-8, Dhanshree Tower Ist, Central Spine, Vidyadhar Nagar, Jaipur 302039
Ph : +91 - 141 - 2101150
www.nodia.co.in
email : enquiry@nodia.co.in

Downloaded From : www.EasyEngineering.net


Downloaded From : www.EasyEngineering.net

GATE SOLVED PAPER - EC


NETWORK ANALYSIS

2013 ONE MARK

Q. 1 Consider a delta connection of resistors and its equivalent star connection as


shown below. If all elements of the delta connection are scaled by a factor k , k > 0
, the elements of the corresponding star equivalent will be scaled by a factor of

ww
w.E (A) k2
(B) k
(C) 1/k
(D) k
asy V2 ^s h
Q. 2

En
The transfer function
V1 ^s h
of the circuit shown below is

gin
eer
(A) 0.5s + 1
ing
(B) 3s + 6
s+1
(C) s + 2
s+2
(D) s + 1
.ne
Q. 3
s+1 s+2
t
A source vs ^ t h = V cos 100pt has an internal impedance of ^4 + j3h W . If a purely
resistive load connected to this source has to extract the maximum power out of
the source, its value in W should be
(A) 3
(B) 4
(C) 5
(D) 7

2013 TWO MARKS

Q. 4 In the circuit shown below, if the source voltage VS = 100+53.13c V then the
Thevenin’s equivalent voltage in Volts as seen by the load resistance RL is

Downloaded From : www.EasyEngineering.net


Downloaded From : www.EasyEngineering.net
GATE SOLVED PAPER - EC NETWORK ANALYSIS

(A) 100+90c (B) 800+0c


(C) 800+90c (D) 100+60c

Q. 5 The following arrangement consists of an ideal transformer and an attenuator


which attenuates by a factor of 0.8. An ac voltage VWX1 = 100 V is applied
across WX to get an open circuit voltage VYZ1 across YZ. Next, an ac voltage
VYZ2 = 100 V is applied across YZ to get an open circuit voltage VWX2 across WX.
Then, VYZ1 /VWX1 , VWX2 /VYZ2 are respectively,

ww
w.E
asy
(A) 125/100 and 80/100 (B) 100/100 and 80/100

En
(C) 100/100 and 100/100 (D) 80/100 and 80/100

Q. 6

gin
Two magnetically uncoupled inductive coils have Q factors q1 and q2 at the chosen
operating frequency. Their respective resistances are R1 and R2 . When connected

(A) q1 + q2
(B) ^1/q1h + ^1/q2h
eer
in series, their effective Q factor at the same operating frequency is

(C) ^q1 R1 + q2 R2h / ^R1 + R2h


(D) ^q1 R2 + q2 R1h / ^R1 + R2h
ing
Q. 7 .ne
Three capacitors C1 , C2 and C 3 whose values are 10 mF , 5 mF , and 2 mF respectively,

t
have breakdown voltages of 10 V, 5 V and 2 V respectively. For the interconnection
shown below, the maximum safe voltage in Volts that can be applied across the
combination, and the corresponding total charge in mC stored in the effective
capacitance across the terminals are respectively,

(A) 2.8 and 36 (B) 7 and 119


(C) 2.8 and 32 (D) 7 and 80

Downloaded From : www.EasyEngineering.net


Downloaded From : www.EasyEngineering.net
GATE SOLVED PAPER - EC NETWORK ANALYSIS

Common Data For Q. 8 and 9:


Consider the following figure

Q. 8 The current IS in Amps in the voltage source, and voltage VS in Volts across the
current source respectively, are
(A) 13, - 20
(B) 8, - 10

ww (C) - 8, 20
(D) - 13, 20

w.E
Q. 9 The current in the 1W resistor in Amps is
(A) 2
(B) 3.33
(C) 10 asy
(D) 12
En
2012 gin ONE MARK

Q. 10

eer
In the following figure, C1 and C2 are ideal capacitors. C1 has been charged to
12 V before the ideal switch S is closed at t = 0. The current i (t) for all t is

ing
.ne
(A) zero
(B) a step function
t
(C) an exponentially decaying function
(D) an impulse function

Q. 11 The average power delivered to an impedance (4 - j3) W by a current


5 cos (100pt + 100) A is
(A) 44.2 W
(B) 50 W
(C) 62.5 W
(D) 125 W

Downloaded From : www.EasyEngineering.net


Downloaded From : www.EasyEngineering.net
GATE SOLVED PAPER - EC NETWORK ANALYSIS

Q. 12 In the circuit shown below, the current through the inductor is

ww (A) 2 A
1+j
(C) 1 A
(B) - 1 A
1+j
(D) 0 A

w.E 1+j

2012 TWO MARKS

Q. 13
asy
Assuming both the voltage sources are in phase, the value of R for which maximum
power is transferred from circuit A to circuit B is

En
gin
eer
(A) 0.8 W (B) 1.4 W ing
(C) 2 W (D) 2.8 W
.ne
Q. 14 If VA - VB = 6 V then VC - VD is
t
(A) - 5 V (B) 2 V
(C) 3 V (D) 6 V
Common Data For Q. 15 and 16 :
With 10 V dc connected at port A in the linear nonreciprocal two-port network
shown below, the following were observed :
(i) 1 W connected at port B draws a current of 3 A
(ii) 2.5 W connected at port B draws a current of 2 A

Downloaded From : www.EasyEngineering.net


Downloaded From : www.EasyEngineering.net
GATE SOLVED PAPER - EC NETWORK ANALYSIS

Q. 15 With 10 V dc connected at port A, the current drawn by 7 W connected at port


B is
(A) 3/7 A (B) 5/7 A
(C) 1 A (D) 9/7 A

Q. 16 For the same network, with 6 V dc connected at port A, 1 W connected at port


B draws 7/3 A. If 8 V dc is connected to port A, the open circuit voltage at port
B is
(A) 6 V (B) 7 V

ww (C) 8 V (D) 9 V

w.E
Q. 17
2011 ONE MARK

In the circuit shown below, the Norton equivalent current in amperes with respect

asy
to the terminals P and Q is

En
gin
(A) 6.4 - j 4.8 eer
(B) 6.56 - j 7.87
(C) 10 + j 0
ing
(D) 16 + j 0

Q. 18

RL is maximum is
.ne
In the circuit shown below, the value of RL such that the power transferred to

t
(A) 5 W (B) 10 W
(C) 15 W (D) 20 W

Q. 19 The circuit shown below is driven by a sinusoidal input vi = Vp cos (t/RC ). The
steady state output vo is

Downloaded From : www.EasyEngineering.net


Downloaded From : www.EasyEngineering.net
GATE SOLVED PAPER - EC NETWORK ANALYSIS

(A) (Vp /3) cos (t/RC ) (B) (Vp /3) sin (t/RC )
(C) (Vp /2) cos (t/RC ) (D) (Vp /2) sin (t/RC )

2011 TWO MARKS

Q. 20 In the circuit shown below, the current I is equal to

ww (A) 1.4+0c A
(C) 2.8+0c A
(B) 2.0+0c A
(D) 3.2+0c A

w.E
Q. 21 In the circuit shown below, the network N is described by the following Y matrix:
Y=>
0.1 S - 0.01 S
0.1 SH
. the voltage gain V2 is
0.01 S
asy V1

En
gin
(A) 1/90
eer
(B) –1/90
(C) –1/99 (D) –1/11
ing
Q. 22

.ne
In the circuit shown below, the initial charge on the capacitor is 2.5 mC, with the
voltage polarity as indicated. The switch is closed at time t = 0 . The current i (t)
at a time t after the switch is closed is
t

(A) i (t) = 15 exp (- 2 # 103 t) A


(B) i (t) = 5 exp (- 2 # 103 t) A
(C) i (t) = 10 exp (- 2 # 103 t) A
(D) i (t) =- 5 exp (- 2 # 103 t) A

Downloaded From : www.EasyEngineering.net


Downloaded From : www.EasyEngineering.net
GATE SOLVED PAPER - EC NETWORK ANALYSIS

2010 ONE MARK

Q. 23 For the two-port network shown below, the short-circuit admittance parameter
matrix is

4 -2 1 - 0.5
(A) >
4H
(B) >
1H
S S
-2 - 0. 5
1 0. 5 4 2
(C) >
1H
(D) >
4H
S S
0.5 2

ww
Q. 24 For parallel RLC circuit, which one of the following statements is NOT correct ?
(A) The bandwidth of the circuit decreases if R is increased

w.E (B) The bandwidth of the circuit remains same if L is increased


(C) At resonance, input impedance is a real quantity
(D) At resonance, the magnitude of input impedance attains its minimum
value.
asy
2010 En TWO MARKS

Q. 25
gin
In the circuit shown, the switch S is open for a long time and is closed at t = 0 .
The current i (t) for t $ 0+ is
eer
ing
.ne
(A) i (t) = 0.5 - 0.125e-1000t A
(B) i (t) = 1.5 - 0.125e-1000t A
t
(C) i (t) = 0.5 - 0.5e-1000t A
(D) i (t) = 0.375e-1000t A

Q. 26 The current I in the circuit shown is

(A) - j1 A (B) j1 A
(C) 0 A (D) 20 A

Downloaded From : www.EasyEngineering.net


Downloaded From : www.EasyEngineering.net
GATE SOLVED PAPER - EC NETWORK ANALYSIS

Q. 27 In the circuit shown, the power supplied by the voltage source is

(A) 0 W (B) 5 W
(C) 10 W (D) 100 W

ww GATE 2009 ONE MARK

w.E
Q. 28 In the interconnection of ideal sources shown in the figure, it is known that the
60 V source is absorbing power.

asy
En
gin
(A) 10 A (B) 13 A eer
Which of the following can be the value of the current source I ?

Q. 29
(C) 15 A (D) 18 A

If the transfer function of the following network is ing


Vo (s)
Vi (s)
= 1
2 + sCR .ne
t
The value of the load resistance RL is
(A) R (B) R
4 2
(C) R (D) 2R

Q. 30 A fully charged mobile phone with a 12 V battery is good for a 10 minute talk-
time. Assume that, during the talk-time the battery delivers a constant current of
2 A and its voltage drops linearly from 12 V to 10 V as shown in the figure. How
much energy does the battery deliver during this talk-time?

Downloaded From : www.EasyEngineering.net


Downloaded From : www.EasyEngineering.net
GATE SOLVED PAPER - EC NETWORK ANALYSIS

(A) 220 J (B) 12 kJ


(C) 13.2 kJ (D) 14.4 J

GATE 2009 TWO MARK

Q. 31 An AC source of RMS voltage 20 V with internal impedance Zs = (1 + 2j) W


feeds a load of impedance ZL = (7 + 4j) W in the figure below. The reactive power
consumed by the load is

ww
w.E
asy
(A) 8 VAR
(C) 28 VAR En (B) 16 VAR
(D) 32 VAR

Q. 32
gin
The switch in the circuit shown was on position a for a long time, and is move to
position b at time t = 0 . The current i (t) for t > 0 is given by

eer
ing
.ne
(A) 0.2e-125t u (t) mA
(C) 0.2e-1250t u (t) mA
t
(B) 20e-1250t u (t) mA
(D) 20e-1000t u (t) mA

Q. 33 In the circuit shown, what value of RL maximizes the power delivered to RL ?

(A) 2.4 W (B) 8 W


3
(C) 4 W (D) 6 W

Downloaded From : www.EasyEngineering.net


Downloaded From : www.EasyEngineering.net
GATE SOLVED PAPER - EC NETWORK ANALYSIS

Q. 34 The time domain behavior of an RL circuit is represented by


L di + Ri = V0 (1 + Be-Rt/L sin t) u (t).
dt
For an initial current of i (0) = V0 , the steady state value of the current is given
R
by
(A) i (t) " V0 (B) i (t) " 2V0
R R
(C) i (t) " V0 (1 + B) (D) i (t) " 2V0 (1 + B)
R R

GATE 2008 ONE MARK

Q. 35 In the following graph, the number of trees (P) and the number of cut-set (Q) are

ww
w.E (A) P = 2, Q = 2
(C) P = 4, Q = 6 asy (B) P = 2, Q = 6
(D) P = 4, Q = 10

Q. 36
En
In the following circuit, the switch S is closed at t = 0 . The rate of change of
current di (0+) is given by
dt
gin
eer
ing
(A) 0
(C)
(R + Rs) Is
(B) Rs Is
(D) 3
L
.ne
L

GATE 2008
t TWO MARKS

Q. 37 The Thevenin equivalent impedance Zth between the nodes P and Q in the
following circuit is

(A) 1 (B) 1 + s + 1
s
2
(C) 2 + s + 1 (D) s2 + s + 1
s s + 2s + 1

Downloaded From : www.EasyEngineering.net


Downloaded From : www.EasyEngineering.net
GATE SOLVED PAPER - EC NETWORK ANALYSIS

Q. 38 The driving point impedance of the following network is given by


Z (s) = 2 0.2s
s + 0.1s + 2

The component values are


(A) L = 5 H, R = 0.5 W, C = 0.1 F
(B) L = 0.1 H, R = 0.5 W, C = 5 F
(C) L = 5 H, R = 2 W, C = 0.1 F
(D) L = 0.1 H, R = 2 W, C = 5 F

ww
Q. 39 The circuit shown in the figure is used to charge the capacitor C alternately
from two current sources as indicated. The switches S1 and S2 are mechanically

w.E coupled and connected as follows:


For 2nT # t # (2n + 1) T , (n = 0, 1, 2,..) S1 to P1 and S2 to P2
For (2n + 1) T # t # (2n + 2) T, (n = 0, 1, 2,...) S1 to Q1 and S2 to Q2

asy
En
gin
eer
Assume that the capacitor has zero initial charge. Given that u (t) is a unit step

(A)
3
/ (- 1) n tu (t - nT) ing
function , the voltage vc (t) across the capacitor is given by

n=1
3
(B) u (t) + 2 / (- 1) n u (t - nT) .ne
n=1
3
(C) tu (t) + 2 / (- 1) n u (t - nT) (t - nT)
n=1
t
(D) / 60.5 - e- (t - 2nT) + 0.5e- (t - 2nT) - T @
3

n=1

Common Data For Q.40 and 41 :


The following series RLC circuit with zero conditions is excited by a unit impulse
functions d (t).

Downloaded From : www.EasyEngineering.net


Downloaded From : www.EasyEngineering.net
GATE SOLVED PAPER - EC NETWORK ANALYSIS

Q. 40 For t > 0 , the output voltage vC ^ t h is

(A) 2 ^e t - e t h (B) 2 te 2 t
-1 3 -1
2 2

3 3
(C) 2 e 2 t cos c 3 t m (D) 2 e 2 t sin c 3 t m
-1 -1

3 2 3 2
Q. 41 For t > 0 , the voltage across the resistor is

(A) 1 _e - e- 2 t i
3t 1
2
3
3 1 sin 3 t
=cos c 2 t m - c 2 mG
-1 t
(B) e 2
3
2 e -21 t sin 3 t
(C) c 2 m
3
(D) 2 e 2 t cos c 3 t m
-1

ww 3 2

w.E Statement for linked Answers Questions 42 and 43:


A two-port network shown below is excited by external DC source. The voltage

asy
and the current are measured with voltmeters V1, V2 and ammeters. A1, A2 (all
assumed to be ideal), as indicated

En
gin
Under following conditions, the readings obtained are:
eer
(1) S1 -open, S2 - closed A1 = 0,V1 = 4.5 V,V2 = 1.5 V, A2 = 1 A
(2) S1 -open, S2 - closed A1 = 4 A,V1 = 6 V,V2 = 6 V, A2 = 0
ing
Q. 42 The z -parameter matrix for this network is

(A) =
1. 5 1. 5
1. 5 G
(B) =
1.5 4. 5.ne
4. 5 G

(C) =
4. 5
1.5
1.5
4. 5
1. 5 G
(D) =
1.5
4. 5
1. 5
1.5
4.5G
t
Q. 43 The h -parameter matrix for this network is
-3 3 -3 -1
(A) =
- 1 0.67 G
(B) =
3 0.67 G
3 3 3 1
(C) =
1 0.67 G
(D) =
- 3 - 0.67 G

GATE 2007 ONE MARK

Q. 44 An independent voltage source in series with an impedance Zs = Rs + jXs delivers


a maximum average power to a load impedance ZL when
(A) ZL = Rs + jXs (B) ZL = Rs
(C) ZL = jXs (D) ZL = Rs - jXs

Downloaded From : www.EasyEngineering.net


Downloaded From : www.EasyEngineering.net
GATE SOLVED PAPER - EC NETWORK ANALYSIS

Q. 45 The RC circuit shown in the figure is

(A) a low-pass filter (B) a high-pass filter


(C) a band-pass filter (D) a band-reject filter

GATE 2007 TWO MARKS

Q. 46 Two series resonant filters are as shown in the figure. Let the 3-dB bandwidth of

ww Filter 1 be B1 and that of Filter 2 be B2 . the value B1 is


B2

w.E
asy
(A) 4
(C) 1/2 En (B) 1
(D) 1/4

Q. 47
gin
For the circuit shown in the figure, the Thevenin voltage and resistance looking
into X - Y are
eer
ing
.ne
(A)
(C)
4
3
4
3
V, 2 W
V, 23 W
(B) 4 V, 23 W
(D) 4 V, 2 W t
Q. 48 In the circuit shown, vC is 0 volts at t = 0 sec. For t > 0 , the capacitor current
iC (t), where t is in seconds is given by

(A) 0.50 exp (- 25t) mA


(B) 0.25 exp (- 25t) mA
(C) 0.50 exp (- 12.5t) mA
(D) 0.25 exp (- 6.25t) mA

Downloaded From : www.EasyEngineering.net


Downloaded From : www.EasyEngineering.net
GATE SOLVED PAPER - EC NETWORK ANALYSIS

Q. 49 In the ac network shown in the figure, the phasor voltage VAB (in Volts) is

(A) 0 (B) 5+30c


(C) 12.5+30c (D) 17+30c

GATE 2006 TWO MARKS

Q. 50 A two-port network is represented by ABCD parameters given by

ww V1 A B V2
=I G = =C D G=- I G
1 2

w.E If port-2 is terminated by RL , the input impedance seen at port-1 is given by


(A) A + BRL
C + DRL
(B) ARL + C
BRL + D

(C) DRL + A
BRL + C asy (D) B + ARL
D + CRL

Q. 51
En
In the two port network shown in the figure below, Z12 and Z21 and respectively

gin
eer
(A) re and br0 ing
(B) 0 and - br0
(C) 0 and bro (D) re and - br0
.ne
Q. 52

t
The first and the last critical frequencies (singularities) of a driving point
impedance function of a passive network having two kinds of elements, are a pole
and a zero respectively. The above property will be satisfied by
(A) RL network only (B) RC network only
(C) LC network only (D) RC as well as RL networks

Q. 53 A 2 mH inductor with some initial current can be represented as shown below,


where s is the Laplace Transform variable. The value of initial current is

(A) 0.5 A (B) 2.0 A


(C) 1.0 A (D) 0.0 A

Downloaded From : www.EasyEngineering.net


Downloaded From : www.EasyEngineering.net
GATE SOLVED PAPER - EC NETWORK ANALYSIS

Q. 54 In the figure shown below, assume that all the capacitors are initially uncharged.
If vi (t) = 10u (t) Volts, vo (t) is given by

(A) 8e -t/0.004 Volts (B) 8 (1 - e -t/0.004) Volts


(C) 8u (t) Volts (D) 8 Volts

Q. 55 A negative resistance Rneg is connected to a passive network N having driving


point impedance as shown below. For Z2 (s) to be positive real,

ww
w.E
asy
En
(A) Rneg # Re Z1 (jw), 6w
(C) Rneg # Im Z1 (jw), 6w
(B) Rneg # Z1 (jw) , 6w
(D) Rneg # +Z1 (jw), 6w

gin
Q. 56
GATE 2005

eer ONE MARK

The condition on R, L and C such that the step response y (t) in the figure has
no oscillations, is
ing
.ne
(A) R $ 1 L (B) R $L
t
2 C C
(C) R $ 2 L (D) R = 1
C LC
Q. 57 The ABCD parameters of an ideal n: 1 transformer shown in the figure are
n 0
>0 x H

The value of x will be

Downloaded From : www.EasyEngineering.net


Downloaded From : www.EasyEngineering.net
GATE SOLVED PAPER - EC NETWORK ANALYSIS

(A) n (B) 1
n
(C) n2 (D) 12
n
Q. 58 In a series RLC circuit, R = 2 kW , L = 1 H, and C = 1 mF The resonant
400
frequency is
(A) 2 # 10 4 Hz (B) 1 # 10 4 Hz
p
(C) 10 4 Hz (D) 2p # 10 4 Hz

Q. 59 The maximum power that can be transferred to the load resistor RL from the
voltage source in the figure is

ww
w.E (A) 1 W
(C) 0.25 W
asy (B) 10 W
(D) 0.5 W

Q. 60

En
The first and the last critical frequency of an RC -driving point impedance
function must respectively be
(A) a zero and a pole
(C) a pole and a pole gin
(B) a zero and a zero
(D) a pole and a zero

GATE 2005 eer TWO MARKS

Q. 61
ing
For the circuit shown in the figure, the instantaneous current i1 (t) is

.ne
t
(A) 10 3 90c A (B) 10 3 - 90c A
2 2
(C) 5 60c A (D) 5 - 60c A

Q. 62 Impedance Z as shown in the given figure is

(A) j29 W (B) j9 W


(C) j19 W (D) j39 W

Downloaded From : www.EasyEngineering.net


Downloaded From : www.EasyEngineering.net
GATE SOLVED PAPER - EC NETWORK ANALYSIS

Q. 63 For the circuit shown in the figure, Thevenin’s voltage and Thevenin’s equivalent
resistance at terminals a - b is

(A) 5 V and 2 W (B) 7.5 V and 2.5 W


(C) 4 V and 2 W (D) 3 V and 2.5 W

Q. 64 If R1 = R2 = R4 = R and R3 = 1.1R in the bridge circuit shown in the figure, then


the reading in the ideal voltmeter connected between a and b is

ww
w.E
(A) 0.238 V
asy (B) 0.138 V
(C) - 0.238 V
En (D) 1 V

Q. 65

gin
The h parameters of the circuit shown in the figure are

eer
ing
(A) =
0 . 1 0. 1
- 0. 1 0. 3 G
(B) =
10 - 1
.ne
1 0.05G

Q. 66
(C) =
30 20
20 20G
(D) =
10 1
- 1 0.05G t
A square pulse of 3 volts amplitude is applied to C - R circuit shown in the figure.
The capacitor is initially uncharged. The output voltage V2 at time t = 2 sec is

(A) 3 V (B) - 3 V
(C) 4 V (D) - 4 V

Downloaded From : www.EasyEngineering.net


Downloaded From : www.EasyEngineering.net
GATE SOLVED PAPER - EC NETWORK ANALYSIS

GATE 2004 ONE MARK

Q. 67 Consider the network graph shown in the figure. Which one of the following is
NOT a ‘tree’ of this graph ?

ww
w.E (A) a
(B) b
(C) c
(D) d asy
Q. 68
En
The equivalent inductance measured between the terminals 1 and 2 for the circuit
shown in the figure is
gin
eer
ing
(A) L1 + L2 + M
.ne
(B) L1 + L2 - M
(C) L1 + L2 + 2M
(D)L1 + L2 - 2M
t
Q. 69 The circuit shown in the figure, with R = 1 W, L = 1 H and C = 3 F
3 4
has input voltage v (t) = sin 2t . The resulting current i (t) is

(A) 5 sin (2t + 53.1c)


(B) 5 sin (2t - 53.1c)
(C) 25 sin (2t + 53.1c)
(D) 25 sin (2t - 53.1c)

Downloaded From : www.EasyEngineering.net


Downloaded From : www.EasyEngineering.net
GATE SOLVED PAPER - EC NETWORK ANALYSIS

Q. 70 For the circuit shown in the figure, the time constant RC = 1 ms. The input
voltage is vi (t) = 2 sin 103 t . The output voltage vo (t) is equal to

(A) sin (103 t - 45c) (B) sin (103 t + 45c)


(C) sin (103 t - 53c) (D) sin (103 t + 53c)

ww
Q. 71 For the R - L circuit shown in the figure, the input voltage vi (t) = u (t). The
current i (t) is

w.E
asy
En
gin
eer
ing
.ne
Q. 72
GATE 2004 t TWO MARKS

For the lattice shown in the figure, Za = j2 W and Zb = 2 W . The values of the open
z11 z12
circuit impedance parameters 6z @ = =
z21 z22 G
are

1-j 1+j 1-j 1+j


(A) =
1 + jG
(B) =
1+j -1 + j 1 - j G
1+j 1+j 1 + j -1 + j
(C) =
1 - jG
(D) =
1-j -1 + j 1 + j G

Downloaded From : www.EasyEngineering.net


Downloaded From : www.EasyEngineering.net
GATE SOLVED PAPER - EC NETWORK ANALYSIS

Q. 73 The circuit shown in the figure has initial current iL (0-) = 1 A through the
inductor and an initial voltage vC (0-) =- 1 V across the capacitor. For input
v (t) = u (t), the Laplace transform of the current i (t) for t $ 0 is

(A) s (B) s+2


s2 + s + 1 s2 + s + 1
(C) 2 s - 2 (D) 2 1
s +s+1 s +s+1

ww
Q. 74 The transfer function H (s) =
Vo (s)
Vi (s)
of an RLC circuit is given by

w.E H (s) = 106


s + 20s + 106
2

The Quality factor (Q-factor) of this circuit is


(A) 25
(C) 100 asy (B) 50
(D) 5000

Q. 75
En
For the circuit shown in the figure, the initial conditions are zero. Its transfer
V (s)
function H (s) = c
Vi (s)
is
gin
eer
ing
(A) 1
s2 + 106 s + 106
(B) 106
.ne
s2 + 103 s + 106

(C) 103
s + 103 s + 106
2
(D) 106
s + 106 s + 106
2 t
Q. 76 Consider the following statements S1 and S2
S1 : At the resonant frequency the impedance of a series RLC circuit is zero.
S2 : In a parallel GLC circuit, increasing the conductance G results in increase
in its Q factor.
Which one of the following is correct?
(A) S1 is FALSE and S2 is TRUE
(B) Both S1 and S2 are TRUE
(C) S1 is TRUE and S2 is FALSE
(D) Both S1 and S2 are FALSE

Downloaded From : www.EasyEngineering.net


Downloaded From : www.EasyEngineering.net
GATE SOLVED PAPER - EC NETWORK ANALYSIS

GATE 2003 ONE MARK

Q. 77 The minimum number of equations required to analyze the circuit shown in the
figure is

(A) 3 (B) 4
(C) 6 (D) 7

ww
Q. 78 A source of angular frequency 1 rad/sec has a source impedance consisting of 1 W
resistance in series with 1 H inductance. The load that will obtain the maximum
power transfer is

w.E (A) 1 W resistance


(B) 1 W resistance in parallel with 1 H inductance

asy
(C) 1 W resistance in series with 1 F capacitor
(D) 1 W resistance in parallel with 1 F capacitor

Q. 79

En
A series RLC circuit has a resonance frequency of 1 kHz and a quality factor
Q = 100 . If each of R, L and C is doubled from its original value, the new Q of
the circuit is
(A) 25 gin
(B) 50
(C) 100 eer
(D) 200
ing
Q. 80

.ne
The differential equation for the current i (t) in the circuit of the figure is

t
2 2
(A) 2 d 2i + 2 di + i (t) = sin t (B) d 2i + 2 di + 2i (t) = cos t
dt dt dt dt
2 2
(C) 2 d 2i + 2 di + i (t) = cos t (D) d 2i + 2 di + 2i (t) = sin t
dt dt dt dt

GATE 2003 TWO MARKS

Q. 81 Twelve 1 W resistance are used as edges to form a cube. The resistance between
two diagonally opposite corners of the cube is
(A) 5 W (B) 1 W
6
(C) 6 W (D) 3 W
5 2

Downloaded From : www.EasyEngineering.net


Downloaded From : www.EasyEngineering.net
GATE SOLVED PAPER - EC NETWORK ANALYSIS

Q. 82 The current flowing through the resistance R in the circuit in the figure has the
form P cos 4t where P is

(A) (0.18 + j0.72) (B) (0.46 + j1.90)


(C) - (0.18 + j1.90) (D) - (0.192 + j0.144)

Common Data For Q. 83 and 84 :


Assume that the switch S is in position 1 for a long time and thrown to position

ww 2 at t = 0 .

w.E
asy
Q. 83 En
At t = 0+ , the current i1 is
(A) - V
2R gin
(B) - V
R
(C) - V
4R
(D) zero
eer
I1 (s) and I2 (s) are the Laplace transforms of i1 (t) and i2 (t) respectively. The

ing
Q. 84
equations for the loop currents I1 (s) and I2 (s) for the circuit shown in the figure,
after the switch is brought from position 1 to position 2 at t = 0 , are

(A) >
R + Ls + Cs1 - Ls I1 (s)
- Ls 1 H=
R + Cs I2 (s)G
V
== G
s
0 .ne
(B) >
R + Ls + Cs1 - Ls I1 (s)
- Ls H=
R + Cs1 I2 (s)G = =
- Vs
0
G t
R + Ls + Cs1 - Ls I1 (s) - Vs
(C) > 1 H= G == G
- Ls R + Ls + Cs I2 (s) 0
R + Ls + Cs1 - Cs I1 (s) V
(D) >
R + Ls + Cs1 H I2 (s)
= G = G
s
=
- Ls 0

Q. 85 The driving point impedance Z (s) of a network has the pole-zero locations as
shown in the figure. If Z (0) = 3 , then Z (s) is

Downloaded From : www.EasyEngineering.net


Downloaded From : www.EasyEngineering.net
GATE SOLVED PAPER - EC NETWORK ANALYSIS

3 (s + 3) 2 (s + 3)
(A) 2
(B) 2
s + 2s + 3 s + 2s + 2
3 (s + 3) 2 (s - 3)
(C) 2 (D) 2
s + 2s + 2 s - 2s - 3

Q. 86 An input voltage v (t) = 10 2 cos (t + 10c) + 10 5 cos (2t + 10c) V is applied


to a series combination of resistance R = 1 W and an inductance L = 1 H. The
resulting steady-state current i (t) in ampere is
(A) 10 cos (t + 55c) + 10 cos (2t + 10c + tan-1 2)
(B) 10 cos (t + 55c) + 10 23 cos (2t + 55c)
(C) 10 cos (t - 35c) + 10 cos (2t + 10c - tan-1 2)
(D) 10 cos (t - 35c) + 3
2 cos (2t - 35c)

ww
Q. 87 The impedance parameters z11 and z12 of the two-port network in the figure are

w.E
asy
(A) z11 = 2.75 W and z12 = 0.25 W
(C) z11 = 3 W and z12 = 0.25 W
(B) z11 = 3 W and z12 = 0.5 W
(D) z11 = 2.25 W and z12 = 0.5 W

En
Q. 88
GATE 2002
gin
The dependent current source shown in the figure
ONE MARK

eer
ing
.ne
Q. 89
(A) delivers 80 W
(C) delivers 40 W
(B) absorbs 80 W
(D) absorbs 40 W t
In the figure, the switch was closed for a long time before opening at t = 0 . The
voltage vx at t = 0+ is

(A) 25 V (B) 50 V
(C) - 50 V (D) 0 V

Downloaded From : www.EasyEngineering.net


Downloaded From : www.EasyEngineering.net
GATE SOLVED PAPER - EC NETWORK ANALYSIS

GATE 2002 TWO MARKS

Q. 90 In the network of the fig, the maximum power is delivered to RL if its value is

(A) 16 W (B) 40 W
3
(C) 60 W (D) 20 W

Q. 91 If the 3-phase balanced source in the figure delivers 1500 W at a leading power
factor 0.844 then the value of ZL (in ohm) is approximately

ww
w.E (A) 90+32.44c
(C) 80+ - 32.44c asy (B) 80+32.44c
(D) 90+ - 32.44c

GATE 2001 En ONE MARK

Q. 92 The Voltage e0 in the figure is


gin
eer
ing
(A) 2 V (B) 4/3 V .ne
Q. 93
(C) 4 V (D) 8 V
t
If each branch of Delta circuit has impedance 3 Z , then each branch of the
equivalent Wye circuit has impedance
(A) Z (B) 3Z
3
(C) 3 3 Z (D) Z
3
Q. 94 The admittance parameter Y12 in the 2-port network in Figure is

(A) - 0.02 mho (B) 0.1 mho


(C) - 0.05 mho (D) 0.05 mho

Downloaded From : www.EasyEngineering.net


Downloaded From : www.EasyEngineering.net
GATE SOLVED PAPER - EC NETWORK ANALYSIS

GATE 2001 TWO MARKS

Q. 95 The voltage e0 in the figure is

(A) 48 V (B) 24 V
(C) 36 V (D) 28 V

Q. 96 When the angular frequency w in the figure is varied 0 to 3, the locus of the
current phasor I2 is given by

ww
w.E
asy
En
gin
eer
ing
.ne
t
Q. 97 In the figure, the value of the load resistor RL which maximizes the power delivered
to it is

(A) 14.14 W (B) 10 W


(C) 200 W (D) 28.28 W

Downloaded From : www.EasyEngineering.net


Downloaded From : www.EasyEngineering.net
GATE SOLVED PAPER - EC NETWORK ANALYSIS

Q. 98 The z parameters z11 and z21 for the 2-port network in the figure are

(A) z11 = 6 W; z21 = 16 W (B) z11 = 6 W; z21 = 4 W


11 11 11 11

(C) z11 = 6 W; z21 =- 16 W (D) z11 = 4 W; z21 = 4 W


11 11 11 11

GATE 2000 ONE MARK

ww
Q. 99 The circuit of the figure represents a

w.E
asy
(A) Low pass filter En (B) High pass filter
(C) band pass filter
gin (D) band reject filter

Q. 100 In the circuit of the figure, the voltage v (t) is


eer
ing
.ne
(A) eat - ebt
(C) aeat - bebt
(B) eat + ebt
(D) aeat + bebt t
Q. 101 In the circuit of the figure, the value of the voltage source E is

(A) - 16 V (B) 4 V
(C) - 6 V (D) 16 V

Downloaded From : www.EasyEngineering.net


Downloaded From : www.EasyEngineering.net
GATE SOLVED PAPER - EC NETWORK ANALYSIS

GATE 2000 TWO MARKS

Q. 102 Use the data of the figure (a). The current i in the circuit of the figure (b)

(A) - 2 A (B) 2 A
(C) - 4 A (D) 4 A

ww GATE 1999 ONE MARK

w.E
Q. 103 Identify which of the following is NOT a tree of the graph shown in the given
figure is

asy
En
(A) begh
gin (B) defg
(C) abfg
eer
(D) aegh

Q. 104
can be given by ing
A 2-port network is shown in the given figure. The parameter h21 for this network

.ne
t
(A) - 1/2 (B) + 1/2
(C) - 3/2 (D) + 3/2

GATE 1999 TWO MARK

Q. 105 The Thevenin equivalent voltage VTH appearing between the terminals A and B
of the network shown in the given figure is given by

Downloaded From : www.EasyEngineering.net


Downloaded From : www.EasyEngineering.net
GATE SOLVED PAPER - EC NETWORK ANALYSIS

(A) j16 (3 - j4) (B) j16 (3 + j4)


(C) 16 (3 + j4) (D) 16 (3 - j4)

Q. 106 The value of R (in ohms) required for maximum power transfer in the network
shown in the given figure is

(A) 2 (B) 4
(C) 8 (D) 16

ww
Q. 107 A Delta-connected network with its Wye-equivalent is shown in the given figure.
The resistance R1, R2 and R3 (in ohms) are respectively

w.E
asy
(A) 1.5, 3 and 9
(C) 9, 3 and 1.5 En (B) 3, 9 and 1.5
(D) 3, 1.5 and 9

gin
Q. 108
GATE 1998

eer ONE MARK

A network has 7 nodes and 5 independent loops. The number of branches in the
network is
(A) 13 (B) 12 ing
Q. 109
(C) 11 (D) 10

The nodal method of circuit analysis is based on .ne


(A) KVL and Ohm’s law
(C) KCL and KVL
(B) KCL and Ohm’s law
t
(D) KCL, KVL and Ohm’s law

Q. 110 Superposition theorem is NOT applicable to networks containing


(A) nonlinear elements (B) dependent voltage sources
(C) dependent current sources (D) transformers

Q. 111 The parallel RLC circuit shown in the figure is in resonance. In this circuit

(A) IR < 1 mA (B) IR + IL > 1 mA


(C) IR + IC < 1 mA (D) IR + IC > 1 mA

Downloaded From : www.EasyEngineering.net


Downloaded From : www.EasyEngineering.net
GATE SOLVED PAPER - EC NETWORK ANALYSIS

0 - 1/2
The short-circuit admittance matrix a two-port network is >
1/2 0 H
Q. 112
The two-port network is
(A) non-reciprocal and passive (B) non-reciprocal and active
(C) reciprocal and passive (D) reciprocal and active

Q. 113 The voltage across the terminals a and b in the figure is

(A) 0.5 V (B) 3.0 V

ww (C) 3.5 V (D) 4.0 V

A high-Q quartz crystal exhibits series resonance at the frequency ws and parallel

w.E
Q. 114
resonance at the frequency wp . Then
(A) ws is very close to, but less than wp
(B) ws << wp

asy
(C) ws is very close to, but greater than wp
(D) ws >> wp

En
Q. 115
GATE 1997

gin
The current i4 in the circuit of the figure is equal to
ONE MARK

eer
ing
.ne
(A) 12 A
(C) 4 A
(B) - 12 A t
(D) None or these

Q. 116 The voltage V in the figure equal to

(A) 3 V (B) - 3 V
(C) 5 V (D) None of these

Downloaded From : www.EasyEngineering.net


Downloaded From : www.EasyEngineering.net
GATE SOLVED PAPER - EC NETWORK ANALYSIS

Q. 117 The voltage V in the figure is always equal to

(A) 9 V (B) 5 V
(C) 1 V (D) None of the above

Q. 118 The voltage V in the figure is

ww
w.E (A) 10 V (B) 15 V

Q. 119
(C) 5 V
asy (D) None of the above

In the circuit of the figure is the energy absorbed by the 4 W resistor in the time
interval (0, 3) is
En
gin
eer
(A) 36 Joules (B) 16 Joules ing
Q. 120
(C) 256 Joules
.ne
(D) None of the above

In the circuit of the figure the equivalent impedance seen across terminals a, b, is

(A) b 16 l W (B) b 8 l W
3 3
(C) b 8 + 12j l W (D) None of the above
3

Downloaded From : www.EasyEngineering.net


Downloaded From : www.EasyEngineering.net
GATE SOLVED PAPER - EC NETWORK ANALYSIS

GATE 1996 ONE MARK

Q. 121 In the given figure, A1, A2 and A3 are ideal ammeters. If A2 and A3 read 3 A and
4 A respectively, then A1 should read

(A) 1 A (B) 5 A
(C) 7 A (D) None of these

ww
Q. 122 The number of independent loops for a network with n nodes and b branches is
(A) n - 1
(B) b - n

w.E (C) b - n + 1
(D) independent of the number of nodes

GATE 1996
asy TWO MARKS

Q. 123
En
The voltages VC1, VC2, and VC3 across the capacitors in the circuit in the given

gin
figure, under steady state, are respectively.

eer
ing
(A) 80 V, 32 V, 48 V
(C) 20 V, 8 V, 12 V .ne
(B) 80 V, 48 V, 32 V
(D) 20 V, 12 V, 8 V

t
***********

Downloaded From : www.EasyEngineering.net


Downloaded From : www.EasyEngineering.net
GATE SOLVED PAPER - EC NETWORK ANALYSIS

SOLUTIONS

Sol. 1 Option (B) is correct.


In the equivalent star connection, the resistance can be given as
RC = Rb Ra
Ra + Rb + Rc
RB = Ra Rc
Ra + Rb + Rc
RA = Rb Rc
Ra + Rb + Rc

ww So, if the delta connection components Ra , Rb and Rc are scaled by a factor k


then
^k Rb h^k Rc h
w.E RAl =
kRa + kRb + kRc
=k
2
Rb Rc
k Ra + Rb + Rc
Hence, it is also scaled by a factor k
= k RA

Sol. 2
asy
Option (D) is correct.
For the given capacitance, C = 100mF in the circuit, we have the reactance.

En
XC = 1 =
sc
1
s # 100 # 10-6
= 10 4
s
So,
V2 ^s h
= 4 s
10 4 + 10 4 gin
= s+1
V1 ^s h 10 + 10 4 + 10 4
s s
s+2
eer
Sol. 3 Option (C) is correct.
ing
For the purely resistive load, maximum average power is transferred when

.ne
2 2
RL = RTh + XTh
where RTh + jXTh is the equivalent thevenin (input) impedance of the circuit.

Sol. 4
Hence, we obtain
RL = 42 + 32 5 W
Option (C) is correct.
t
For evaluating the equivalent thevenin voltage seen by the load RL , we open the
circuit across it (also if it consist dependent source).
The equivalent circuit is shown below

As the circuit open across RL so


I2 = 0
or, j40I2 = 0

Downloaded From : www.EasyEngineering.net


Downloaded From : www.EasyEngineering.net
GATE SOLVED PAPER - EC NETWORK ANALYSIS

i.e., the dependent source in loop 1 is short circuited. Therefore,


^ j4h Vs
VL1 =
j4 + 3
j40 40 90c
VTh = 10 VL1 = 100 53.13c = 100 53.13c
j4 + 3 5 53.13c
= 800 90c
Sol. 5 Option (C) is correct.
For the given transformer, we have
V = 1.25
VWX 1

ww
w.E Since,

So,
VYZ = 0.8 (attenuation factor)
V
^ h^ h
VYZ = 0.8 1.25 = 1

or,
asy VWX
VYZ = VWX
V
VWX = 100 V ; YZ = 100
at

En
1
1
VWX 1
100
VWX
at VWZ = 100 V ; = 100

gin
2
2
VYZ 2
100
Sol. 6 Option (C) is correct.

eer
The quality factor of the inductances are given by
q 1 = wL 1

and
R1
w
q2 = L2
R2 ing
.ne
So, in series circuit, the effective quality factor is given by
XLeq
Q =

= R
Req

1 R2
= wL 1 + wL 2

R
R1 + R 2
wL 1 + wL 2
1 R2
= R
q1
2
q
+ 2
R 2
= 1 1
t
q R + q2 R2
1 + 1 1 + 1 R1 + R 2
R 2 R1 R 2 R1
Sol. 7 Option (C) is correct.

Consider that the voltage across the three capacitors C1 , C2 and C 3 are V1 , V2 and
V3 respectively. So, we can write
V2 = C 3 ....(1)
V3 C2
Since, Voltage is inversely proportional to capacitance

Downloaded From : www.EasyEngineering.net


Downloaded From : www.EasyEngineering.net
GATE SOLVED PAPER - EC NETWORK ANALYSIS

Now, given that C1 = 10 mF ; ^V1hmax = 10V


C2 = 5 mF ; ^V2hmax = 5 V
C 3 = 2 mF ; ^V3hmax = 2V
So, from Eq (1) we have
V2 = 2
V3 5
for ^V3hmax = 2
We obtain, V2 = 2 # 2 = 0.8 volt < 5
5
i.e., V2 < ^V2hmax
Hence, this is the voltage at C2 . Therefore,
V3 = 2 volt
V2 = 0.8 volt
and V1 = V2 + V3 = 2.8 volt

ww Now, equivalent capacitance across the terminal is


Ceq = C2 C 3 + C1 = 5 # 2 + 10 = 80 mF
C2 + C3 5+2 7

w.E Equivalent voltage is (max. value)


Vmax = V1 = 2.8
So, charge stored in the effective capacitance is

asy Q = Ceq Vmax = b 80 l # ^2.8h = 32 mC


7
Sol. 8

En
Option (D) is correct.

gin
eer
ing
At the node 1, voltage is given as .ne
V1 = 10 volt
Applying KCL at node 1
IS + V1 + V1 - 2 = 0
2 1
t
10 10
IS + + - 2 = 0
2 1
IS =- 13 A
Also, from the circuit,
VS - 5 # 2 = V1 & VS = 10 + V1 = 20 volt
Sol. 9 Option (C) is correct.
Again from the shown circuit, the current in 1 W resistor is
I = V1 = 10 = 10 A
1 1
Sol. 10 Option (D) is correct.
The s -domain equivalent circuit is shown as below.

Downloaded From : www.EasyEngineering.net


Downloaded From : www.EasyEngineering.net
GATE SOLVED PAPER - EC NETWORK ANALYSIS

vc (0) /s v (0)
I (s) = = c
1 + 1 1 + 1
C1 s C 2 s C1 C 2

I (s) = b C1 C2 l (12 V) = 12Ceq vC (0) = 12 V


C1 + C 2
Taking inverse Laplace transform for the current in time domain,

ww
Sol. 11
i (t) = 12Ceq d (t)

Option (B) is correct.


(Impulse)

w.E In phasor form, Z = 4 - j 3 = 5 - 36.86cW


I = 5 100c A

asy
Average power delivered.
Pavg. = 1 I 2 Z cos q = 1 # 25 # 5 cos 36.86c = 50 W
2 2
Alternate Method:
En
Pavg
2
Z = (4 - j3) W ,
2
2
gin
I = 5 cos (100pt + 100) A
= 1 Re $ I Z . = 1 # Re "(5) 2 # (4 - j3), = 1 # 100 = 50 W
2
Sol. 12 Option (C) is correct
eer
ing
.ne
t

Applying nodal analysis at top node.


V1 + 1 0c V1 + 1 0c
+ = 1 0c
1 j1
V1 (j 1 + 1) + j 1 + 1 0c = j1
V1 = - 1
1 + j1
1
V1 + 1 0c - 1 + j + 1 j
Current I1 = = = = 1 A
j1 j1 (1 + j) j 1 + j

Downloaded From : www.EasyEngineering.net


Downloaded From : www.EasyEngineering.net
GATE SOLVED PAPER - EC NETWORK ANALYSIS

Sol. 13 Option (A) is correct.


We obtain Thevenin equivalent of circuit B .

Thevenin Impedance :

ZTh = R

ww Thevenin Voltage :

w.E Now, circuit becomes as


VTh = 3 0c V

asy
En
gin
Current in the circuit, I1 = 10 - 3
2+R eer
Power transfer from circuit A to B
P = (I 12) 2 R + 3I1 ing
= :10 - 3D R + 3 :10 - 3D
2+R
2

2+R .ne
or P = 42 + 70R2
(2 + R)
2
dP = 2 + R) 70 - (42 + 70R) 2 (2 + R) = 0
(
t
dR (2 + R) 4
(2 + R) [(2 + R) 70 - (42 + 70R) 2] = 0 & R = 0.8 W

Sol. 14 Option (A) is correct.


In the given circuit
VA - VB = 6 V
So current in the branch will be
IAB = 6 = 3 A
2
We can see, that the circuit is a one port circuit looking from terminal BD as
shown below

Downloaded From : www.EasyEngineering.net


Downloaded From : www.EasyEngineering.net
GATE SOLVED PAPER - EC NETWORK ANALYSIS

For a one port network current entering one terminal, equals the current leaving
the second terminal. Thus the outgoing current from A to B will be equal to the
incoming current from D to C as shown
i.e. IDC = IAB = 3 A

ww
w.E
asy
The total current in the resistor 1 W will be

En I1 = 2 + IDC
= 2+3 = 5A
(By writing KCL at node D )

Sol. 15
So, VCD
Option (C) is correct. gin
= 1 # (- I1) =- 5 V

When 10 V is connected at port A the network is


eer
ing
.ne
t
Now, we obtain Thevenin equivalent for the circuit seen at load terminal, let
Thevenin voltage is VTh, 10 V with 10 V applied at port A and Thevenin resistance
is RTh .

VTh,10 V
IL =
RTh + RL
For RL = 1 W , IL = 3 A
VTh,10 V
3= ...(i)
RTh + 1

Downloaded From : www.EasyEngineering.net


Downloaded From : www.EasyEngineering.net
GATE SOLVED PAPER - EC NETWORK ANALYSIS

For RL = 2.5 W , IL = 2 A
VTh,10 V
2= ...(ii)
RTh + 2.5
Dividing above two
3 = RTh + 2.5
2 RTh + 1
3RTh + 3 = 2RTh + 5
RTh = 2 W
Substituting RTh into equation (i)
VTh,10 V = 3 (2 + 1) = 9 V
Note that it is a non reciprocal two port network. Thevenin voltage seen at port
B depends on the voltage connected at port A. Therefore we took subscript
VTh,10 V . This is Thevenin voltage only when 10 V source is connected at input port

ww A. If the voltage connected to port A is different, then Thevenin voltage will be


different. However, Thevenin’s resistance remains same.
Now, the circuit is as shown below :

w.E
asy
En VTh,10 V
= 9 = 1A

Sol. 16
For RL = 7 W ,

Option (B) is correct.


IL =
gin
2 + RL 2 + 7

eer
Now, when 6 V connected at port A let Thevenin voltage seen at port B is VTh,6 V
. Here RL = 1 W and IL = 7 A
3
ing
.ne
VTh, 6 V = RTh # 7 + 1 # 7 = 2 # 7 + 7 = 7 V
t
3 3 3 3
This is a linear network, so VTh at port B can be written as
VTh = V1 a + b
where V1 is the input applied at port A.
We have V1 = 10 V , VTh,10 V = 9 V
` 9 = 10a + b ...(i)
When V1 = 6 V , VTh, 6 V = 9 V
` 7 = 6a + b ...(ii)
Solving (i) and (ii)
a = 0.5 , b = 4
Thus, with any voltage V1 applied at port A, Thevenin voltage or open circuit
voltage at port B will be

Downloaded From : www.EasyEngineering.net


Downloaded From : www.EasyEngineering.net
GATE SOLVED PAPER - EC NETWORK ANALYSIS

So, VTh, V = 0.5V1 + 4


1

For V1 = 8 V
VTh,8 V = 0.5 # 8 + 4 = 8 = Voc (open circuit voltage)
Sol. 17 Option (A) is correct.
Replacing P - Q by short circuit as shown below we have

Using current divider rule the current Isc is

ww ISC = 25
25 + 15 + j30
(16 0 ) = (6.4 - j4.8) A

w.E
Sol. 18 Option (C) is correct.
Power transferred to RL will be maximum when RL is equal to the Thevenin
resistance. We determine Thevenin resistance by killing all source as follows :

asy
En
gin
RTH = 10 # 10 + 10 = 15 W
10 + 10 eer
Sol. 19 Option (A) is correct.
The given circuit is shown below ing
.ne
t
For parallel combination of R and C equivalent impedance is
R$ 1
jw C R
Zp = =
R+ 1 1 + j wRC
jw C
Transfer function can be written as
R
Vout = Z p 1 + jwRC
=
Vin Zs + Zp R+ 1 + R
jwC 1 + jwRC
jwRC
=
jwRC + (1 + jwRC) 2
j
= Here w = 1
j + (1 + j) 2 RC

Downloaded From : www.EasyEngineering.net


Downloaded From : www.EasyEngineering.net
GATE SOLVED PAPER - EC NETWORK ANALYSIS

Vout = j
=1
Vin (1 + j) 2 + j 3
Vp
Thus v out = b l cos (t/RC)
3
Sol. 20 Option (B) is correct.
From star delta conversion we have

ww Thus R1 = Ra Rb = 6.6
Ra + Rb + Rc 6 + 6 + 6
= 2W

w.E Here R1 = R 2 = R 3 = 2 W
Replacing in circuit we have the circuit shown below :

asy
En
gin
Now the total impedance of circuit is eer
Z =
(2 + j4) (2 - j4)
(2 + j4) (2 - j4)
+2 = 7W
ing
Current I = 14+0c = 2+0c
7 .ne
Sol. 21 Option (D) is correct.
From given admittance matrix we get
I1 = 0.1V1 - 0.01V2 and
t ...(1)
I2 = 0.01V1 + 0.1V2 ...(2)
Now, applying KVL in outer loop;
V2 =- 100I2
or I2 =- 0.01V2 ...(3)
From eq (2) and eq (3) we have
- 0.01V2 = 0.01V1 + 0.1V2
- 0.11V2 = 0.01V1
V2 = - 1
V1 11
Sol. 22 Option (A) is correct.
Here we take the current flow direction as positive.
At t = 0- voltage across capacitor is

Downloaded From : www.EasyEngineering.net


Downloaded From : www.EasyEngineering.net
GATE SOLVED PAPER - EC NETWORK ANALYSIS

-3
Q
VC (0-) =- =- 2.5 # 10-6 =- 50 V
C 50 # 10
+
Thus VC (0 ) =- 50 V
In steady state capacitor behave as open circuit thus
V (3) = 100 V
Now, VC (t) = VC (3) + (VC (0+) - VC (3)) e-t/RC
-t
= 100 + (- 50 - 100) e 10 # 50 # 10
-6

3
= 100 - 150e- (2 # 10 t)
Now ic (t) = C dV
dt
3
= 50 # 10-6 # 150 # 2 # 103 e-2 # 10 t A
3
= 15e-2 # 10 t

ww
Sol. 23 Option (A) is correct.
ic (t) = 15 exp (- 2 # 103 t) A

w.E Given circuit is as shown below

asy
En
Writing node equation at input port
gin
I1 = V1 + V1 - V2 = 4V1 - 2V2 ...(1)
0.5 0.5
Writing node equation at output port
eer
I2 = V2 + V2 - V1 =- 2V1 + 4V2
0.5 0.5
From (1) and (2), we have admittance matrix ing ...(2)

Y =>
4 -2
- 2 4H .ne
Sol. 24 Option (D) is correct.
A parallel RLC circuit is shown below :
t

Input impedance Z in = 1
1 + 1 + jw C
R jwL
At resonance 1 = wC
wL
So, Z in = 1 = R (maximum at resonance)
1/R
Thus (D) is not true.

Downloaded From : www.EasyEngineering.net


Downloaded From : www.EasyEngineering.net
GATE SOLVED PAPER - EC NETWORK ANALYSIS

Furthermore bandwidth is wB i.e wB \ 1 and is independent of L,


R
Hence statements A, B, C, are true.
Sol. 25 Option (A) is correct.
Let the current i (t) = A + Be-t/t t " Time constant
When the switch S is open for a long time before t < 0 , the circuit is

At t = 0 , inductor current does not change simultaneously, So the circuit is

ww
w.E
asy
Current is resistor (AB)
i (0) = 0.75 = 0.375 A
2

En
Similarly for steady state the circuit is as shown below

gin
eer
i (3) = 15 = 0.5 A
3 ing
t = L = 15 # 10
Req
-3

10 + (10 || 10)
= 10-3 sec
.ne
Now
and
t
i (t) = A + Be- 1 # 10 = A + Be-100t
i (0) = A + B = 0.375
i (3) = A = 0.5
-3

t
So, B = 0.375 - 0.5 =- 0.125
Hence i (t) = 0.5 - 0.125e-1000 t A
Sol. 26 Option (A) is correct.
Circuit is redrawn as shown below

Where, Z1 = jwL = j # 103 # 20 # 10-3 = 20j


Z2 = R || XC
XC = 1 = 1 =- 20j
jwC j # 103 # 50 # 10-6

Downloaded From : www.EasyEngineering.net


Downloaded From : www.EasyEngineering.net
GATE SOLVED PAPER - EC NETWORK ANALYSIS

1 (- 20j)
Z2 = R = 1W
1 - 20j
Voltage across Z2
- 20j
c 1 - 20j m
VZ = Z2 : 20 0 = : 20
Z1 + Z 2 20j
c 20j - 1 - 20j m
2

(- 20j)
=c
20j + 400 - 20j m
: 20 =- j

Current in resistor R is
V j
I = Z =- =- j A
2

R 1

ww
Sol. 27 Option (A) is correct.
The circuit can be redrawn as

w.E
asy
En
gin
Applying nodal analysis
VA - 10 + 1 + VA - 0 = 0 eer
2 2
2VA - 10 + 2 = 0 = V4 = 4 V ing
Current, I1 = 10 - 4 = 3 A
2
.ne
Current from voltage source is
I 2 = I1 - 3 = 0 t
Since current through voltage source is zero, therefore power delivered is zero.
Sol. 28 Option (A) is correct.
Circuit is as shown below

Since 60 V source is absorbing power. So, in 60 V source current flows from +


to - ve direction
So, I + I1 = 12
I = 12 - I1

Downloaded From : www.EasyEngineering.net


Downloaded From : www.EasyEngineering.net
GATE SOLVED PAPER - EC NETWORK ANALYSIS

I is always less then 12 A So, only option (A) satisfies this conditions.
Sol. 29 Option (C) is correct.
For given network we have
(RL XC ) Vi
V0 =
R + (RL XC )
RL
V0 (s) 1 + sRL C RL
= =
Vi (s) R+ R L R + RRL sC + RL
1 + sRL C
= RL = 1
R + RRL sC + RL 1+ R + RsC
RL
But we have been given
V (s) 1
T . F. = 0

ww Comparing, we get
Vi (s)

1 + R = 2 & RL = R
=
2 + sCR

w.E
Sol. 30
RL
Option (C) is correct.

asy
The energy delivered in 10 minutes is
E =
t t
#0 VIdt = I #0Vdt = I # Area

En= 2 # 1 (10 + 12) # 600 = 13.2 kJ


2
Sol. 31 Option (B) is correct.
From given circuit the load current is gin
IL = V
Zs + ZL
= 20+0c = 20+0c
(1 + 2j) + (7 + 4j) 8 + 6j
= 1 (8 - 6j) = 20+0c = 2+ - f
eer where f = tan - 1 3
5
The voltage across load is
10+f
ing 4

VL = IL ZL
The reactive power consumed by load is .ne
Pr = VL IL* = IL ZL # IL* = ZL IL 2
2
= (7 # 4j) 20+0c = (7 + 4j) = 28 + 16j
8 + 6j
t
Thus average power is 28 and reactive power is 16.
Sol. 32 Option (B) is correct.
At t = 0- , the circuit is as shown in fig below :

V (0-) = 100 V
Thus V (0+) = 100 V
At t = 0+ , the circuit is as shown below

Downloaded From : www.EasyEngineering.net


Downloaded From : www.EasyEngineering.net
GATE SOLVED PAPER - EC NETWORK ANALYSIS

I (0+) = 100 = 20 mA
5k
At steady state i.e. at t = 3 is I (3)= 0
i (t) = I (0+) e-
t
Now RCeq
u (t)
(0.5m + 0.3m) 0.2m
Ceq = = 0.16 m F
0.5m + 0.3m + 0.2m

ww 1 =
RCeq
1
5 # 103 # 0.16 # 10-6
i (t) = 20e-1250t u (t) mA
= 1250

w.E
Sol. 33 Option (C) is correct.
For Pmax the load resistance RL must be equal to thevenin resistance Req i.e. RL

asy
= Req . The open circuit and short circuit is as shown below

En
gin
eer
The open circuit voltage is
Voc = 100 V ing
From fig I1 = 100 = 12.5 A
8 .ne
Vx =- 4 # 12.5 =- 50 V
I2 = 100 + Vx = 100 - 50 = 12.5 A
4 4
t
Isc = I1 + I2 = 25 A
Rth = Voc = 100 = 4 W
Isc 25
Thus for maximum power transfer RL = Req = 4 W
Sol. 34 Option (A) is correct.
Steady state all transient effect die out and inductor act as short circuits and
forced response acts only. It doesn’t depend on initial current state. From the
given time domain behavior we get that circuit has only R and L in series with
V0 . Thus at steady state
i (t) " i (3) = V0
R
Sol. 35 Option (C) is correct.
The given graph is

Downloaded From : www.EasyEngineering.net


Downloaded From : www.EasyEngineering.net
GATE SOLVED PAPER - EC NETWORK ANALYSIS

There can be four possible tree of this graph which are as follows:

There can be 6 different possible cut-set.

ww
Sol. 36 Option (B) is correct.

w.E Initially i (0-) = 0 therefore due to inductor i (0+) = 0 . Thus all current Is will
flow in resistor R and voltage across resistor will be Is Rs . The voltage across

asy
inductor will be equal to voltage across Rs as no current flow through R.

En
gin
Thus vL (0+) = Is Rs eer
but vL (0+) = L
di (0+)
dt ing
Thus
di (0+)
dt
v (0+) Is Rs
= L
L
=
L
.ne
Sol. 37 Option (A) is correct.
Killing all current source and voltage sources we have, t

Zth = (1 + s) ( s1 + 1)
(1 + s)( s1 + 1) [ s1 + 1 + 1 + s]
= =
(1 + s) + ( s1 + 1) s + s1 + 1 + 1
or Zth = 1
Alternative :
Here at DC source capacitor act as open circuit and inductor act as short
circuit. Thus we can directly calculate thevenin Impedance as 1 W

Downloaded From : www.EasyEngineering.net


Downloaded From : www.EasyEngineering.net
GATE SOLVED PAPER - EC NETWORK ANALYSIS

Sol. 38 Option (D) is correct.


s
Z (s) = R 1 sL = 2 C
sC s + s
RC + 1
LC
We have been given
Z (s) = 2 0.2s
s + 0.1s + 2
Comparing with given we get
1 = 0.2 or C = 5 F
C
1 = 0.1 or R = 2 W
RC
1 = 2 or L = 0.1 H
LC

ww
Sol. 39 Option (C) is correct.
Voltage across capacitor is
t
Vc = 1 # idt

w.E C 0
Here C = 1 F and i = 1 A. Therefore
t
#0 dt
asy Vc =

For 0 < t < T , capacitor will be charged from 0 V

En Vc =
t
#0 dt = t
At t = T, Vc = T Volts
gin
For T < t < 2T , capacitor will be discharged from T volts as
Vc = T -
t
#T dt = 2T - t eer
At t = 2T, Vc = 0 volts
For 2T < t < 3T , capacitor will be charged from 0 V ing
Vc =
t
#2Tdt = t - 2T
.ne
At t = 3T, Vc = T Volts
For 3T < t < 4T , capacitor will be discharged from T Volts
Vc = T -
t
#3Tdt = 4T - t
t
At t = 4T, Vc = 0 Volts
For 4T < t < 5T , capacitor will be charged from 0 V
t
Vc = #4Tdt = t - 4T
At t = 5T, Vc = T Volts
Thus the output waveform is

Downloaded From : www.EasyEngineering.net


Downloaded From : www.EasyEngineering.net
GATE SOLVED PAPER - EC NETWORK ANALYSIS

Only option C satisfy this waveform.


Sol. 40 Option (D) is correct.
Writing in transform domain we have
Vc (s) 1
= 1 s = 2 1
Vs (s) ^ s + s + 1h (s + s + 1)
Since Vs (t) = d (t) " Vs (s) = 1 and
Vc (s) = 2 1
(s + s + 1)
3
or Vc (s) = 2 = 2
G
3 (s + 12 ) 2 + 43
Taking inverse Laplace transform we have
Vt = 2 e- sin c 3 t m
t
2

ww
Sol. 41 Option (B) is correct.
3

Let voltage across resistor be vR

w.E VR (s)
VS (s)
= 1 1 = s
( s + s + 1) (s2 + s + 1)
Since vs = d (t) " Vs (s) = 1 we get

asy
VR (s) = 2 s = s
(s + s + 1) (s + 12 ) 2 + 43

En =
(s + 12 )
-
1
2

or
(s + 12 ) 2 +

vR (t) = e- cos
1
3
4

3 t-1gin (s + 1 2
2) + 4

2 e- sin 3 t 1
3

eer
2 2

2 2# 3 2

3 t - 1 sin 3 t

ing
t
= e- 2 =cos
2 3 2 G
Sol. 42 Option (C) is correct.
From the problem statement we have
z11 = v1 = 6 = 1.5W
.ne
i1 i = 0 4
z12 = v1
i2 i = 0
v
2

= 4.5 = 4.5W
1

6
1
t
z21 = 2 = = 1.5W
i1 i = 0 4
2

z22 = v2 = 1.5 = 1.5W


i2 i = 0 2
1
Thus z -parameter matrix is
z11 z12 1.5 4.5
=z z G = =1.5 1.5 G
21 22

Sol. 43 Option (A) is correct.


From the problem statement we have
h12 = v1 = 4.5 = 3
v2 i = 0 1
1.5
h22 = i2 = 1 = 0.67
v2 i = 0 1
1.5
From z matrix, we have

Downloaded From : www.EasyEngineering.net


Downloaded From : www.EasyEngineering.net
GATE SOLVED PAPER - EC NETWORK ANALYSIS

v1 = z11 i1 + z12 i2
v2 = z21 i1 + z22 i2
If v2 = 0 then i2 = - z21 = - 1.5 =- 1 = h
21
i1 z22 1.5
or i2 =- i1
Putting in equation for v1, we get
v1 = (z11 - z12) i1
v1 = h11 = z11 - z12 = 1.5 - 4.5 =- 3
i1 v2 = 0

Hence h -parameter will be


h11 h12 -3 3
=h h G = =- 1 0.67 G
21 22

Sol. 44 Option (D) is correct.

ww According to maximum Power Transform Theorem


ZL = Zs* = (Rs - jXs)

w.E
Sol. 45 Option (C) is correct.
At w " 3 , capacitor acts as short circuited and circuit acts as shown in fig below

asy
En Here we get V0 = 0
gin Vi

At w " 0 , capacitor acts as open circuited and circuit look like as shown in fig
below
eer
ing
Here we get also V0 = 0
.ne
Vi
t
So frequency response of the circuit is as shown in fig and circuit is a Band pass
filter.

Sol. 46 Option (D) is correct.


We know that bandwidth of series RLC circuit is R . Therefore
L
Bandwidth of filter 1 is B1 = R
L1
Bandwidth of filter 2 is B2 = R = R = 4R
L2 L1 /4 L1
B
Dividing above equation 1 = 1
B2 4

Downloaded From : www.EasyEngineering.net


Downloaded From : www.EasyEngineering.net
GATE SOLVED PAPER - EC NETWORK ANALYSIS

Sol. 47 Option (D) is correct.


Here Vth is voltage across node also. Applying nodal analysis we get

Vth + Vth + Vth - 2i = 2


2 1 1
But from circuit i = Vth = Vth
1
Therefore
Vth + Vth + Vth - 2Vth = 2

ww or
2 1 1
Vth = 4 volt

w.E From the figure shown below it may be easily seen that the short circuit current
at terminal XY is isc = 2 A because i = 0 due to short circuit of 1 W resistor

asy
and all current will pass through short circuit.

En
gin
Therefore Rth = Vth = 4 = 2 W
isc 2 eer
Sol. 48 Option (A) is correct.
The voltage across capacitor is ing
At t = 0+ ,
At t = 3 ,
Vc (0+) = 0
VC (3) = 5 V .ne
The equivalent resistance seen by capacitor as shown in fig is
Req = 20 20 = 10kW
t

Time constant of the circuit is


t = Req C = 10k # 4m = 0.04 s
Using direct formula
Vc (t) = VC (3) - [Vc (3) - Vc (0)] e-t/t
= VC (3) (1 - e-t/t) + VC (0) e-t/t = 5 (1 - e-t/0.04)
or Vc (t) = 5 (1 - e-25t)

Downloaded From : www.EasyEngineering.net


Downloaded From : www.EasyEngineering.net
GATE SOLVED PAPER - EC NETWORK ANALYSIS

dVC (t)
Now IC (t) = C
dt
= 4 # 10-6 # (- 5 # 25e-25t) = 0.5e-25t mA

Sol. 49 Option (D) is correct.


(5 - 3j) # (5 + 3j)
Impedance = (5 - 3j) (5 + 3j) =
5 - 3j + 5 + 3j
(5) 2 - (3j) 2
= = 25 + 9 = 3.4
10 10
VAB = Current # Impedance = 5+30c # 34 = 17+30c

Sol. 50 Option (D) is correct.


The network is shown in figure below.

ww
w.E Now
and asy V1 = AV2 - BI2
I1 = CV2 - DI2
...(1)
...(2)
also
EnV2 =- I2 RL
From (1) and (2) we get
...(3)

Thus V1 = AV2 - BI2


I1 CV2 - DI2
gin
Substituting value of V2 from (3) we get
Input Impedance Zin = - A # I2 RL - BI2
- C # I2 RL - DI2
eer
or Zin = ARL + B
CRL + D ing
Sol. 51 Option (B) is correct.
The circuit is as shown below. .ne
t
At input port V1 = re I1
At output port V2 = r0 (I2 - bI1) =- r0 bI1 + r0 I2
Comparing standard equation
V1 = z11 I1 + z12 I2
V2 = z21 I1 + z22 I2
z12 = 0 and z21 =- r0 b
Sol. 52 Option (B) is correct.
For series RC network input impedance is
Zins = 1 + R = 1 + sRC
sC sC

Downloaded From : www.EasyEngineering.net


Downloaded From : www.EasyEngineering.net
GATE SOLVED PAPER - EC NETWORK ANALYSIS

Thus pole is at origin and zero is at - 1


RC
For parallel RC network input impedance is
1 R
Zin = sC = sC
1 +R 1 + sRC
sC
Thus pole is at - 1 and zero is at infinity.
RC
Sol. 53 Option (A) is correct.
We know v = Ldi
dt
Taking Laplace transform we get

ww V (s) = sLI (s) - Li (0+)


As per given in question

w.E Thus
- Li (0+) =- 1 mV
i (0+) = 1 mV = 0.5 A
2 mH
Sol. 54

asy
Option (B) is correct.
At initial all voltage are zero. So output is also zero.
Thus
En
v0 (0+) = 0
At steady state capacitor act as open circuit.

gin
eer
ing
Thus, v0 (3) = 4 # vi = 4 # 10 = 8 .ne
5 5
t
The equivalent resistance and capacitance can be calculate after killing all
source

Req = 1 4 = 0.8 kW
Ceq = 4 1 = 5 mF
t = Req Ceq = 0.8kW # 5mF = 4 ms
v0 (t) = v 0 (3) - [v 0 (3) - v 0 (0+)] e-t/t
= 8 - (8 - 0) e-t/0.004

Downloaded From : www.EasyEngineering.net


Downloaded From : www.EasyEngineering.net
GATE SOLVED PAPER - EC NETWORK ANALYSIS

v0 (t) = 8 (1 - e-t/0.004) Volts


Sol. 55 Option (A) is correct.
Here Z2 (s) = Rneg + Z1 (s)
or Z2 (s) = Rneg + Re Z1 (s) + j Im Z1 (s)
For Z2 (s) to be positive real, Re Z2 (s) $ 0
Thus Rneg + Re Z1 (s) $ 0
or Re Z1 (s) $- Rneg
But Rneg is negative quantity and - Rneg is positive quantity. Therefore
Re Z1 (s) $ Rneg
or Rneg # Re Z1 (jw) For all w.
Sol. 56 Option (C) is correct.
Transfer function is

ww Y (s)
U (s)
=
1
sC
R + sL + 1
= 2 1 =
1
LC
s2 + R s + 1
w.E sC
s LC + scR + 1

Comparing with s2 + 2xwn s + wn2 = 0 we have


L LC

Here
asy
2xwn = R ,

wn = 1
L
and
En R
LC
LC = R C
Thus x =
2L
For no oscillations, x $ 1
2
gin
L

Thus R C $1
2 L eer
or R $2 L
C
ing
Sol. 57 Option (B) is correct.
For given transformer
.ne
or
I2 = V1 = n
I1 V2

n
1
I1 = I2 and V1 = nV2
t
Comparing with standard equation
V1 = AV2 + BI2
I1 = CV2 + DI2
A B n 0
=C D G = =0 1 G
n

Thus x = 1
n
Sol. 58 Option (B) is correct.
We have L = 1H and C = 1 # 10-6
400
Resonant frequency
f0 = 1 == 1
2p LC 2p 1 # 1 # 10 - 6
400

Downloaded From : www.EasyEngineering.net


Downloaded From : www.EasyEngineering.net
GATE SOLVED PAPER - EC NETWORK ANALYSIS

3 4
= 10 # 20 = 10 Hz
2p p
Sol. 59 Option (C) is correct.
Maximum power will be transferred when RL = Rs = 100W
In this case voltage across RL is 5 V, therefore
2
Pmax = V = 5 # 5 = 0.25 W
R 100
Sol. 60 Option (C) is correct.
For stability poles and zero interlace on real axis. In RC series network the
driving point impedance is
Zins = R + 1 = 1 + sRC
Cs sC
Here pole is at origin and zero is at s =- 1/RC , therefore first critical

ww frequency is a pole and last critical frequency is a zero.


For RC parallel network the driving point impedance is

w.E Zinp =
R 1

R+
Cs =
1
Cs
1 +
R
sRC

asy
Here pole is s =- 1/RC and zero is at 3, therefore first critical frequency is a

Sol. 61
En
pole and last critical frequency is a zero.
Option (A) is correct.
Applying KCL we get
i1 (t) + 5+0c = 10+60c gin
or
or i1 (t) = 5 3 +90c = 10 3 +90c eer
i1 (t) = 10+60c - 5+0c = 5 + 5 3j - 5

Sol. 62 Option (B) is correct.


2
ing
.ne
If L1 = j5W and L3 = j2W the mutual induction is subtractive because current
enters from dotted terminal of j2W coil and exit from dotted terminal of j5W. If

from dotted terminal of both coil.


Thus Z = L1 - M13 + L2 + M23 + L3 - M31 + M32
t
L2 = j2W and L3 = j2W the mutual induction is additive because current enters

= j5 + j10 + j2 + j10 + j2 - j10 + j10 = j9


Sol. 63 Option (B) is correct.
Open circuit at terminal ab is shown below

Applying KCL at node we get


Vab + Vab - 10 = 1
5 5
or Vab = 7.5 = Vth

Downloaded From : www.EasyEngineering.net


Downloaded From : www.EasyEngineering.net
GATE SOLVED PAPER - EC NETWORK ANALYSIS

Short circuit at terminal ab is shown below

Short circuit current from terminal ab is


Isc = 1 + 10 = 3 A
5
Thus Rth = Vth = 7.5 = 2.5 W
Isc 3
Here current source being in series with dependent voltage source make it

ww
Sol. 64
ineffective.
Option (C) is correct.

w.E Here Va = 5 V because R1 = R2 and total voltage drop is 10 V.


Now Vb = R3 # 10 = 1.1 # 10 = 5.238 V
R3 + R4 2.1

Sol. 65 asy
Option (D) is correct.
V = Va - Vb = 5 - 5.238 =- 0.238 V

En
For h parameters we have to write V1 and I2 in terms of I1 and V2 .
V1 = h11 I1 + h12 V2

gin
I2 = h21 I1 + h22 V2
Applying KVL at input port
V1 = 10I1 + V2
Applying KCL at output port eer
V2 = I + I
20 1 2
ing
or I2 =- I1 + V2
20 .ne
Thus from above equation we get
h11 h12 10 1
=h h G = =- 1 0.05G
12 22
t
Sol. 66 Option (B) is correct.
Time constant RC = 0.1 # 10 - 6 # 103 = 10 - 4 sec
Since time constant RC is very small, so steady state will be reached in 2 sec.
At t = 2 sec the circuit is as shown in fig.

Vc = 3 V
V2 =- Vc =- 3 V

Downloaded From : www.EasyEngineering.net


Downloaded From : www.EasyEngineering.net
GATE SOLVED PAPER - EC NETWORK ANALYSIS

Sol. 67 Option (B) is correct.


For a tree there must not be any loop. So a, c, and d don’t have any loop. Only
b has loop.
Sol. 68 Option (D) is correct.
The sign of M is as per sign of L If current enters or exit the dotted terminals of
both coil. The sign of M is opposite of L If current enters in dotted terminal of
a coil and exit from the dotted terminal of other coil.
Thus Leq = L1 + L2 - 2M
Sol. 69 Option (A) is correct.
Here w = 2 and V = 1+0c
Y = 1 + jw C + 1
R jw L
= 3 + j2 # 3 + 1 1 = 3 + j4

ww j2 # 4
= 5+ tan - 1 4 = 5+53.11c
3

w.E Thus
I = V * Y = (1+0c)( 5+53.1c) = 5+53.1c
i (t) = 5 sin (2t + 53.1c)
Sol. 70

asy
Option (A) is correct.
vi (t) = 2 sin 103 t

En
Here w = 103 rad and Vi =
1
2 +0c

Now V0 =
jw C
R+ 1
jw C
.Vt =
1 +
1
j ginV
wCR i

= 1
1 + j # 103 # 10 - 3
2 +0c eer
= 1 - 45c
v0 (t) = sin (103 t - 45c)
ing
Sol. 71 Option (C) is correct.
.ne
Input voltage
Taking Laplace transform

Impedance
vi (t) = u (t)

Z (s) = s + 2
Vi (s) = 1
s
t
V (s) 1
I (s) = i =
s + 2 s (s + 2)
or I (s) = 1 ; 1 - 1 E
2 s s+2
Taking inverse Laplace transform
i (t) = 1 (1 - e-2t) u (t)
2
At t = 0 , i (t) = 0
At t = 2 ,
1
i (t) = 0.31
At t = 3 , i (t) = 0.5
Graph (C) satisfies all these conditions.

Downloaded From : www.EasyEngineering.net


Downloaded From : www.EasyEngineering.net
GATE SOLVED PAPER - EC NETWORK ANALYSIS

Sol. 72 Option (D) is correct.


We know that
V1 = z11 I1 + z12 I2
V2 = z11 I1 + z22 I2
where z11 = V1
I1 I = 0
2

z21 = V2
I1 I = 0
1

Consider the given lattice network, when I2 = 0 . There is two similar path in
the circuit for the current I1. So I = 1 I1
2

ww
w.E For z11 applying KVL at input port we get

Thus asy V1 = I (Za + Zb)


V1 = 1 I1 (Za + Zb)

En 2
z11 = 1 (Za + Zb)
2

gin
For Z21 applying KVL at output port we get
V2 = Za I1 - Zb I1

Thus
2 2
V2 = 1 I1 (Za - Zb)
eer
2
1
z21 = (Za - Zb) ing
2
For this circuit z11 = z22 and z12 = z21. Thus
R V .ne
z11 z12
SZa + Zb Za - Zb W
2 2
=z z G = SSZa - Zb Za + Zb WW
21 22
S 2
T
2 W
X
t
Here Za = 2j and Zb = 2W
z11 z12 1+j j-1
Thus =z z G = =j - 1 1 + j G
21 22

Sol. 73 Option (B) is correct.


Applying KVL,
Ldi (t) 1
#0
3
v (t) = Ri (t) + + i (t) dt
dt C
Taking L.T. on both sides,
I (s) vc (0+)
V (s) = RI (s) + LsI (s) - Li (0+) + +
sC sC
v (t) = u (t) thus V (s) = 1
s

Downloaded From : www.EasyEngineering.net


Downloaded From : www.EasyEngineering.net
GATE SOLVED PAPER - EC NETWORK ANALYSIS

Hence 1 = I (s) + sI (s) - 1 + I (s) - 1


s s s
2 + 1 = I (s) 6s2 + s + 1@
s s
or I (s) = 2 s + 2
s +s+1
Sol. 74 Option (B) is correct.
Characteristics equation is
s2 + 20s + 106 = 0
Comparing with s2 + 2xwn s + wn2 = 0 we have
wn = 106 = 103
2xw = 20
2x = 203 = 0.02

ww Thus

Now
10
Q = 1 = 1 = 50
2x 0.02

w.E
Sol. 75 Option (D) is correct.
1

asy
H (s) =
V0 (s)
Vi (s)
= sC
R + sL + 1
sC
= 2
s LC +
1
sCR + 1

En
= 2 -2 1
s (10 # 10-4) + s (10-4 # 10 4) + 1
= -6 2 1 = 2
gin
106
10 s + s + 1 s + 106 s + 106
Sol. 76 Option (D) is correct.

eer
Impedance of series RLC circuit at resonant frequency is minimum, not zero.
Actually imaginary part is zero.
Z = R + j ` wL - 1 j
wC ing
wC
Q =R C
.ne
At resonance wL - 1 = 0 and Z = R that is purely resistive. Thus S1 is false
Now quality factor

Since G = 1 ,
R
Q = 1 C
G
L

L
t
If G - then Q . provided C and L are constant. Thus S2 is also false.
Sol. 77 Option (B) is correct.
Number of loops = b - n + 1
= minimum number of equation
Number of branches = b = 8
Number of nodes = n = 5
Minimum number of equation = 8 - 5 + 1 = 4
Sol. 78 Option (C) is correct.
For maximum power transfer
ZL = ZS* = Rs - jXs
Thus ZL = 1 - 1j

Downloaded From : www.EasyEngineering.net


Downloaded From : www.EasyEngineering.net
GATE SOLVED PAPER - EC NETWORK ANALYSIS

Sol. 79 Option (B) is correct.


Q = 1 L
R C
When R, L and C are doubled,
Q' = 1 2L = 1 L =Q
2R 2C 2R C 2
Thus Q' = 100 = 50
2
Sol. 80 Option (C) is correct.
Applying KVL we get,
di (t) 1
sin t = Ri (t) + L + # i (t) dt
dt C
di (t)
or sin t = 2i (t) + 2 + # i (t) dt

ww dt
Differentiating with respect to t , we get
2di (t) 2d2 i (t)

w.E
Sol. 81
cos t =

Option (A) is correct.


dt
+
dt2
+ i (t)

asy
For current i there is 3 similar path. So current will be divide in three path

En
gin
eer
ing
so, we get
Vab - b i # 1l - b i # 1l - b 1 # 1l = 0 .ne
Sol. 82
3

i
Option ( ) is correct.
6

eq
3 6 3
3
Vab = R = 1 + 1 + 1 = 5 W
6 t
Data are missing in question as L1 &L2 are not given.
Sol. 83 Option (A) is correct.
At t = 0 - circuit is in steady state. So inductor act as short circuit and capacitor
act as open circuit.

Downloaded From : www.EasyEngineering.net


Downloaded From : www.EasyEngineering.net
GATE SOLVED PAPER - EC NETWORK ANALYSIS

At t = 0 - , i1 (0 -) = i2 (0 -) = 0
vc (0 -) = V
At t = 0+ the circuit is as shown in fig. The voltage across capacitor and
current in inductor can’t be changed instantaneously. Thus

At t = 0+ , i1 = i2 =- V

ww
Sol. 84 Option (C) is correct.
2R

w.E When switch is in position 2, as shown in fig in question, applying KVL in loop
(1),
RI1 (s) + V + 1 I1 (s) + sL [I1 (s) - I2 (s)] = 0

or asy
s sC
I1 (s) 8R + 1 + sL B - I2 (s) sL = - V
sc s

En z11 I1 + z12 I2 = V1
Applying KVL in loop 2,

gin
sL [I2 (s) - I1 (s)] + RI2 (s) + 1 I2 (s) = 0
sC

or
Z12 I1 + Z22 I2 = V2
- sLI1 (s) + 8R + sL + 1 BI2 (s) = 0 eer
Now comparing with
sc
ing
Z11 Z12 I1
21 22 2
V1
=Z Z G=I G = =V G
2
.ne
we get
R
SR + sL + 1
S
SS - sL
sC
- sL
R + sL + 1
V
W I1 (s)
W=
WW I2 (s)
G
-V
=> s H
0
t
sC
T X
Sol. 85 Option (B) is correct.
Zeros =- 3
Pole1 =- 1 + j
Pole 2 =- 1 - j
K (s + 3) K (s + 3) K (s + 3)
Z (s) = = =
(s + 1 + j)( s + 1 - j) 2
(s + 1) - j 2
(s + 1) 2 + 1
From problem statement Z (0) w = 0 = 3
Thus 3K = 3 and we get K = 2
2
2 (s + 3)
Z (s) = 2
s + 2s + 2

Downloaded From : www.EasyEngineering.net


Downloaded From : www.EasyEngineering.net
GATE SOLVED PAPER - EC NETWORK ANALYSIS

Sol. 86 Option (C) is correct.


v (t) = 10 2 cos (t + 10c) + 10 5 cos (2t + 10c)
1 4444 2 4444 3 1 4444 4 2 4444 43
v1 v2
Thus we get w1 = 1 and w2 = 2
Now Z1 = R + jw1 L = 1 + j1
Z2 = R + jw2 L = 1 + j2
v1 (t) v2 (t) 10 2 cos (t + 10c) 10 5 cos (2t + 10c)
i (t) = + = +
Z1 Z2 1+j 1 + j2
10 2 cos (t + 10c) 10 5 cos (2t + 10c)
= +
12 + 22 + tan-1 1 12 + 22 tan-1 2
10 2 cos (t + 10c) 10 5 cos (2t + 10c)
= +
2 + tan-1 45c 5 tan-1 2

ww
Sol. 87
i (t) = 10 cos (t - 35c) + 10 cos (2t + 10c - tan-1 2)
Option (A) is correct.

w.E Using 3- Y conversion

asy
En R1 = 2 # 1 = 2 = 0. 5
2+1+1 4
R2 = 1 # 1 = 1 = 0.25
gin
2+1+1 4
R3 = 2 # 1 = 0.5 eer
2+1+1
Now the circuit is as shown in figure below. ing
.ne
t
Now z11 = V1 = 2 + 0.5 + 0.25 = 2.75
I1 I2 = 0

z12 = R3 = 0.25
Sol. 88 Option (A) is correct.
Applying KCL at for node 2,

V2 + V2 - V1 = V1
5 5 5

Downloaded From : www.EasyEngineering.net


Downloaded From : www.EasyEngineering.net
GATE SOLVED PAPER - EC NETWORK ANALYSIS

or V2 = V1 = 20 V
Voltage across dependent current source is 20 thus power delivered by it is
PV2 # V1 = 20 # 20 = 80 W
5 5
It deliver power because current flows from its +ive terminals.
Sol. 89 Option (C) is correct.
When switch was closed, in steady state, iL (0 -) = 2.5 A

ww
w.E At t = 0+ , iL (0+) = iL (0 -) = 2.5 A and all this current of will pass through 2 W
resistor. Thus
Vx =- 2.5 # 20 =- 50 V
Sol. 90
asy
Option (A) is correct.
For maximum power delivered, RL must be equal to Rth across same terminal.

En
gin
eer
Applying KCL at Node, we get
0.5I1 = Vth + I1
ing
or
20
Vth + 10I1 = 0 .ne
but

Thus
I1 = Vth - 50
V
Vth + th -
4
50
40
=0
t
or Vth = 10 V
For Isc the circuit is shown in figure below.

Isc = 0.5I1 - I1 =- 0.5I1


but I1 =- 50 =- 1.25 A
40
Isc =- 0.5 # - 12.5 = 0.625 A

Downloaded From : www.EasyEngineering.net


Downloaded From : www.EasyEngineering.net
GATE SOLVED PAPER - EC NETWORK ANALYSIS

Rth = Vth = 10 = 16 W
Isc 0.625
Sol. 91 Option (D) is correct.
IP , VP " Phase current and Phase voltage
IL, VL " Line current and line voltage
Now VP = c VL m and IP = IL
3
So, Power = 3VP IL cos q
1500 = 3 c VL m (IL) cos q
3
IL = c V
3 ZL m
also L

1500 = 3 c VL mc VL m cos q

ww ZL =
3 3 ZL
(400) 2 (.844)
= 90 W

w.E As power factor is leading


So,
1500

cos q = 0.844 " q = 32.44

asy
As phase current leads phase voltage
ZL = 90+ - q = 90+ - 32.44c
Sol. 92
En
Option (C) is correct.
Applying KCL, we get
e0 - 12 + e0 + e0 = 0
4 2+2
gin
or
4
e0 = 4 V
eer
Sol. 93 Option (A) is correct.
The star delta circuit is shown as below ing
.ne
t
Here ZAB = ZBC = ZCA = 3 Z
and ZA = ZAB ZCA
ZAB + ZBC + ZCA
ZB = ZAB ZBC
ZAB + ZBC + ZCA
ZC = ZBC ZCA
ZAB + ZBC + ZCA
Now ZA = ZB = ZC = 3Z 3Z = Z
3Z+ 3Z+ 3Z 3
Sol. 94 Option (C) is correct.

Downloaded From : www.EasyEngineering.net


Downloaded From : www.EasyEngineering.net
GATE SOLVED PAPER - EC NETWORK ANALYSIS

y11 y12 y1 + y3 - y3
=y y G = = - y y + y G
21 22 3 2 3

y12 =- y3
y12 =- 1 =- 0.05 mho
20
Sol. 95 Option (D) is correct.
We apply source conversion the circuit as shown in fig below.

ww Now applying nodal analysis we have


e0 - 80 + e0 + e0 - 16 = 0
10 + 2 12 6

w.E or 4e0 = 112


e0 = 112 = 28 V
4
Sol. 96
asy
Option (A) is correct.
I2 = Em +01c = Em +0c
jw C

En R2 + jwC 1 + jwCR2

+I2 = +90c
gin
+ tan-1 wCR2
E m wC

At w = 0
I2 =

I2 = 0
1 + w2 C2 R 22
eer
+ (90c - tan-1 wCR2)

and at w = 3, I2 = Em
R2 ing
Sol. 97
Only figure given in option (A) satisfies both conditions.
Option (A) is correct. .ne
Xs = wL = 10 W
For maximum power transfer
RL = Rs2 + Xs2 = 102 + 102 = 14.14 W
t
Sol. 98 Option (C) is correct.
Applying KVL in LHS loop
E1 = 2I1 + 4 (I1 + I2) - 10E1
or E1 = 6I1 + 4I2
11 11
Thus z11 = 6
11
Applying KVL in RHS loop
E2 = 4 (I1 + I2) - 10E1
= 4 (I1 + I2) - 10 c 6I1 + 4I2 m =- 16I1 + 4I2
11 11 11 11
Thus z21 =- 16
11

Downloaded From : www.EasyEngineering.net


Downloaded From : www.EasyEngineering.net
GATE SOLVED PAPER - EC NETWORK ANALYSIS

Sol. 99 Option (D) is correct.


At w = 0 , circuit act as shown in figure below.

V0 = RL (finite value)
Vs RL + Rs
At w = 3 , circuit act as shown in figure below:

ww
w.E
asy V0 = RL
Vs RL + Rs
(finite value)

En
At resonant frequency w = 1 circuit acts as shown in fig and V = 0 .
LC 0

gin
eer
Thus it is a band reject filter.
ing
Sol. 100 Option (D) is correct.
.ne
Applying KCL we get

Now
iL = eat + ebt
t
V (t) = vL = L diL = L d [eat + ebt] = aeat + bebt
dt dt
Sol. 101 Option (A) is correct.
Going from 10 V to 0 V

10 + 5 + E + 1 = 0
or E =- 16 V

Downloaded From : www.EasyEngineering.net


Downloaded From : www.EasyEngineering.net
GATE SOLVED PAPER - EC NETWORK ANALYSIS

Sol. 102 Option (C) is correct.


This is a reciprocal and linear network. So we can apply reciprocity theorem
which states “Two loops A & B of a network N and if an ideal voltage source
E in loop A produces a current I in loop B , then interchanging positions an
identical source in loop B produces the same current in loop A. Since network is
linear, principle of homogeneity may be applied and when volt source is doubled,
current also doubles.
Now applying reciprocity theorem
i = 2 A for 10 V
V = 10 V, i = 2 A
V =- 20 V, i =- 4 A
Sol. 103 Option (C) is correct.
Tree is the set of those branch which does not make any loop and connects all

ww the nodes.
abfg is not a tree because it contains a loop l node (4) is not connected

w.E
asy
Sol. 104
En
Option (A) is correct.
For a 2-port network the parameter h21 is defined as
h21 = I2 gin
I1 V = 0 (short circuit)

eer
2

ing
.ne
Applying node equation at node a we get
Va - V1 + Va - 0 + Va - 0 = 0
R R R
t
3Va = V1 & Va = V1
3
V1 - V1
Now I1 = V1 - Va = 3 = 2V1
R R 3R
0 - V1 - V
and I2 = 0 - Va = 3 = 1
R R 3R
I2 - V1 /3R - 1
Thus = h21 = =
I1 V = 0
2
2V1 /3R 2
Sol. 105 Option (A) is correct.
Applying node equation at node A
Vth - 100 (1 + j0) Vth - 0
+ =0
3 4j

Downloaded From : www.EasyEngineering.net


Downloaded From : www.EasyEngineering.net
GATE SOLVED PAPER - EC NETWORK ANALYSIS

or 4jVth - 4j100 + 3Vth = 0


or Vth (3 + 4j) = 4j100
4j100
Vth =
3 + 4j
By simplifying
4j100 3 - 4j
Vth =
3 + 4j # 3 - 4j
Vth = 16j (3 - j4)
Sol. 106 Option (C) is correct.
For maximum power transfer RL should be equal to RTh at same terminal.
so, equivalent Resistor of the circuit is

ww
w.E Req = 5W 20W + 4W

asy Req = 5.20 + 4 = 4 + 4 = 8 W


5 + 20
Sol. 107

En
Option (D) is correct.
Delta to star conversion
R1 = Rab Rac
gin
= 5 # 30 = 150 = 3 W
Rab + Rac + Rbc 5 + 30 + 15
Rab Rbc
50
= 5 # 15 = 1.5 W
R2 =
Rab + Rac + Rbc 5 + 30 + 15
Rac Rbc = 15 # 30 = 9 W
eer
Sol. 108
R3 =
Rab + Rac + Rbc 5 + 30 + 15
Option (C) is correct. ing
No. of branches = n + l - 1 = 7 + 5 - 1 = 11
Option (B) is correct. .ne
t
Sol. 109

In nodal method we sum up all the currents coming & going at the node So it is
based on KCL. Furthermore we use ohms law to determine current in individual
branch. Thus it is also based on ohms law.
Sol. 110 Option (A) is correct.
Superposition theorem is applicable to only linear circuits.
Sol. 111 Option (B) is correct.
Sol. 112 Option (B) is correct.
For reciprocal network y12 = y21 but here y12 =- 12 ! y21 = 12 . Thus circuit is non
reciprocal. Furthermore only reciprocal circuit are passive circuit.
Sol. 113 Option (C) is correct.
Taking b as reference node and applying KCL at a we get
Vab - 1 + Vab = 3
2 2
or Vab - 1 + Vab = 6

Downloaded From : www.EasyEngineering.net


Downloaded From : www.EasyEngineering.net
GATE SOLVED PAPER - EC NETWORK ANALYSIS

or Vab = 6 + 1 = 3.5 V
2
Sol. 114 Option (A) is correct.
Sol. 115 Option (B) is correct.
The given figure is shown below.

Applying KCL at node a we have

ww I = i 0 + i1 = 7 + 5 = 12 A
Applying KCL at node f

w.E so
I =- i 4
i 4 =- 12 amp
Sol. 116

asy
Option (A) is correct.

En
gin
eer
Sol. 117
so V = 3 - 0 = 3 volt
Option (D) is correct. ing
Sol. 118
Can not determined V without knowing the elements in box.
Option (A) is correct. .ne
Sol. 119
The voltage V is the voltage across voltage source and that is 10 V.
Option (B) is correct.
Voltage across capacitor
t
-t
VC (t) = VC (3) + (VC (0) - VC (3)) e RC
Here VC (3) = 10 V and (VC (0) = 6 V. Thus
-t -t -t
VC (t) = 10 + (6 - 10) e RC = 10 - 4e RC = 10 - 4e 8
Now VR (t) = 10 - VC (t)
-t -t
= 10 - 10 + 4e RC = 4e RC
Energy absorbed by resistor
2 -t

#0 3V RR(t) = #0 3 164e 4
-t
E = #0 3 4e 4 = 16 J

Sol. 120 Option (B) is correct.


It is a balanced whetstone bridge
R1 R 3
b R2 = R 4 l

Downloaded From : www.EasyEngineering.net


Downloaded From : www.EasyEngineering.net
GATE SOLVED PAPER - EC NETWORK ANALYSIS

so equivalent circuit is

Zeq = (4W 8W) = 4 # 8 = 8


4+8 3
Sol. 121 Option (B) is correct.
Current in A2 , I2 = 3 amp

ww Inductor current can be defined as I2 =- 3j


Current in A 3 , I3 = 4

w.E Total current I 1 = I 2 + I 3 = 4 - 3j


I = (4) 2 + (3) 2 = 5 amp

Sol. 122
asy
Option (C) is correct.
For a tree we have (n - 1) branches. Links are the branches which from a loop,

En
when connect two nodes of tree.
so if total no. of branches = b

gin
No. of links = b - (n - 1) = b - n + 1
Total no. of links in equal to total no. of independent loops.
Sol. 123 Option (B) is correct.
eer
In the steady state condition all capacitors behaves as open circuit & Inductors
behaves as short circuits as shown below :
ing
.ne
t
Thus voltage across capacitor C1 is
VC = 100 # 40 = 80 V
1
10 + 40
Now the circuit faced by capacitor C2 and C 3 can be drawn as below :

Voltage across capacitor C2 and C 3 are


VC = 80 C 3 = 80 # 3 = 48 volt
2
C2 + C3 5

Downloaded From : www.EasyEngineering.net


Downloaded From : www.EasyEngineering.net
GATE SOLVED PAPER - EC NETWORK ANALYSIS

VC = 80 C2 = 80 2 = 32 volt
3
C2 + C3 #5

***********

ww
w.E
asy
En
gin
eer
ing
.ne
t

Downloaded From : www.EasyEngineering.net


Downloaded From : www.EasyEngineering.net

No part of this publication may be reproduced or distributed in any form or any means, electronic, mechanical,
photocopying, or otherwise without the prior permission of the author.

ww
w.E
GATE SOLVED PAPER
Electronics & Communication
Signals & Systems
asy
Copyright © By NODIA & COMPANY
En
gin
eer
ing
Information contained in this book has been obtained by authors, from sources believes to be reliable. However,
neither Nodia nor its authors guarantee the accuracy or completeness of any information herein, and Nodia nor its

.ne
authors shall be responsible for any error, omissions, or damages arising out of use of this information. This book
is published with the understanding that Nodia and its authors are supplying information but are not attempting
to render engineering or other professional services.

t
NODIA AND COMPANY
B-8, Dhanshree Tower Ist, Central Spine, Vidyadhar Nagar, Jaipur 302039
Ph : +91 - 141 - 2101150
www.nodia.co.in
email : enquiry@nodia.co.in

Downloaded From : www.EasyEngineering.net


Downloaded From : www.EasyEngineering.net

GATE SOLVED PAPER - EC


SIGNALS & SYSTEMS

2013 ONE MARK

Q. 1 Two systems with impulse responses h1 ^ t h and h2 ^ t h are connected in cascade.


Then the overall impulse response of the cascaded system is given by
(A) product of h1 ^ t h and h2 ^ t h
(B) sum of h1 ^ t h and h2 ^ t h
(C) convolution of h1 ^ t h and h2 ^ t h

ww
Q. 2
(D) subtraction of h2 ^ t h from h1 ^ t h

The impulse response of a system is h ^ t h = tu ^ t h. For an input u ^t - 1h, the

w.E output is
2
(A) t u ^ t h
2
(B)
t ^t - 1h
2
u ^t - 1h

(C)
^t - 1h2
2 asy
u ^t - 1h
2
(D) t - 1 u ^t - 1h
2
Q. 3

En
For a periodic signal v ^ t h = 30 sin 100t + 10 cos 300t + 6 sin ^500t + p/4h, the
fundamental frequency in rad/s
(A) 100
(C) 500 gin(B) 300
(D) 1500

Q. 4
eer
A band-limited signal with a maximum frequency of 5 kHz is to be sampled.

(C) 15 kHz
(B) 12 kHz
(D) 20 kHz
ing
According to the sampling theorem, the sampling frequency which is not valid is
(A) 5 kHz

Q. 5 .ne
Which one of the following statements is NOT TRUE for a continuous time
causal and stable LTI system?
t
(A) All the poles of the system must lie on the left side of the jw axis
(B) Zeros of the system can lie anywhere in the s-plane
(C) All the poles must lie within s = 1
(D) All the roots of the characteristic equation must be located on the left side
of the jw axis.

Q. 6 Assuming zero initial condition, the response y ^ t h of the system given below to a
unit step input u ^ t h is

(A) u ^ t h (B) tu ^ t h
2
(C) t u ^ t h (D) e-t u ^ t h
2

Downloaded From : www.EasyEngineering.net


Downloaded From : www.EasyEngineering.net
GATE SOLVED PAPER - EC SIGNALS & SYSTEMS

Let g ^ t h = e- pt , and h ^ t h is a filter matched to g ^ t h. If g ^ t h is applied as input


2
Q. 7
to h ^ t h, then the Fourier transform of the output is
2 2
(A) e- pf (B) e- pf /2
2
(C) e- p f (D) e-2pf

2013 TWO MARKS

Q. 8 The impulse response of a continuous time system is given by h ^ t h = d ^t - 1h + d ^t - 3h


. The value of the step response at t = 2 is
(A) 0 (B) 1
(C) 2 (D) 3

d2 y dy
Q. 9 A system described by the differential equation + 5 + 6y ^ t h = x ^ t h. Let
dt2 dt
x ^ t h be a rectangular pulse given by
ww x^t h = *
1
0
0<t<2
otherwise

w.E Assuming that y ^0 h = 0 and


(A) e -2s

s ^s + 2h^s + 3h
dy
dt
= 0 at t = 0 , the Laplace transform of y ^ t h is
(B) 1 - e-2s
s ^s + 2h^s + 3h
(C) e -2s

^s + 2h^s + 3hasy (D) 1 - e-2s


^s + 2h^s + 3h
Q. 10
En
A system described by a linear, constant coefficient, ordinary, first order differential
equation has an exact solution given by y ^ t h for t > 0 , when the forcing function
gin
is x ^ t h and the initial condition is y ^0 h. If one wishes to modify the system so that
the solution becomes - 2y ^ t h for t > 0 , we need to

eer
(A) change the initial condition to - y ^0 h and the forcing function to 2x ^ t h
(B) change the initial condition to 2y ^0 h and the forcing function to - x ^ t h

ing
(C) change the initial condition to j 2 y ^0 h and the forcing function to
j 2 x^t h

.ne
(D) change the initial condition to - 2y ^0 h and the forcing function to - 2x ^ t h

Q. 11

R V
Sa b c d W
Sd a b c W
t
The DFT of a vector 8a b c dB is the vector 8a b g dB . Consider the product

8p q r sB = 8a b c dBSc d a b W
S W
Sb c d aW
T X
The DFT of the vector 8p q r sB is a scaled version of
(A) 9a2 b2 g2 d2C (B) 9 a b g dC
(C) 8a + b b + d d + g g + aB (D) 8a b g dB

2012 ONE MARK

Q. 12 The unilateral Laplace transform of f (t) is 2 1 . The unilateral Laplace


s +s+1
transform of tf (t) is
(A) - 2 s 2 (B) - 2 2s + 1 2
(s + s + 1) (s + s + 1)

Downloaded From : www.EasyEngineering.net


Downloaded From : www.EasyEngineering.net
GATE SOLVED PAPER - EC SIGNALS & SYSTEMS

(C) s (D) 2s + 1
(s2 + s + 1) 2 (s2 + s + 1) 2
Q. 13 If x [n] = (1/3) n - (1/2) n u [n], then the region of convergence (ROC) of its z
-transform in the z -plane will be
(A) 1 < z < 3 (B) 1 < z < 1
3 3 2
(C) 1 < z < 3 (D) 1 < z
2 3

2012 TWO MARKS


t
Q. 14 The input x (t) and output y (t) of a system are related as y (t) = # x (t) cos (3t) dt .
-3
The system is

ww (A) time-invariant and stable


(C) time-invariant and not stable
(B) stable and not time-invariant
(D) not time-invariant and not stable

w.E
Q. 15 The Fourier transform of a signal h (t) is H (jw) = (2 cos w) (sin 2w) /w . The value
of h (0) is
(A) 1/4 (B) 1/2
(C) 1
asy (D) 2

Let y [n] denote the convolution of h [n] and g [n], where h [n] = (1/2) n u [n] and g [n]

En
Q. 16
is a causal sequence. If y [0] = 1 and y [1] = 1/2, then g [1] equals
(A) 0
(C) 1 gin
(B) 1/2
(D) 3/2

2011 eer ONE MARK

Q. 17
2

The differential equation 100 ddty - 20 dy


2
ing
dt + y = x (t) describes a system with an

.ne
input x (t) and an output y (t). The system, which is initially relaxed, is excited
by a unit step input. The output y ^ t h can be represented by the waveform

Downloaded From : www.EasyEngineering.net


Downloaded From : www.EasyEngineering.net
GATE SOLVED PAPER - EC SIGNALS & SYSTEMS

Q. 18 The trigonometric Fourier series of an even function does not have the
(A) dc term (B) cosine terms
(C) sine terms (D) odd harmonic terms

Q. 19 A system is defined by its impulse response h (n) = 2n u (n - 2). The system is


(A) stable and causal (B) causal but not stable
(C) stable but not causal (D) unstable and non-causal

Q. 20 If the unit step response of a network is (1 - e- at), then its unit impulse response
is
(A) ae- at (B) a-1 e- at
(C) (1 - a-1) e- at (D) (1 - a) e- at

ww
Q. 21
2011 TWO MARKS

An input x (t) = exp (- 2t) u (t) + d (t - 6) is applied to an LTI system with impulse
response h (t) = u (t) . The output is

w.E (A) [1 - exp (- 2t)] u (t) + u (t + 6)


(B) [1 - exp (- 2t)] u (t) + u (t - 6)

asy
(C) 0.5 [1 - exp (- 2t)] u (t) + u (t + 6)
(D) 0.5 [1 - exp (- 2t)] u (t) + u (t - 6)

Q. 22
En
Two systems H1 (Z ) and H2 (Z ) are connected in cascade as shown below. The

gin
overall output y (n) is the same as the input x (n) with a one unit delay. The
transfer function of the second system H2 (Z ) is

eer
ing
(A) 1 - 0.6z-1
z (1 - 0.4z-1)
-1 (B)
(1 - 0.4z-1)
.ne
z-1 (1 - 0.6z-1)

Q. 23
(C)
z-1 (1 - 0.4z-1)
(1 - 0.6z-1)
(D) 1 - 0.4 z-1
z (1 - 0.6z-1)
-1
t
The first six points of the 8-point DFT of a real valued sequence are
5, 1 - j 3, 0, 3 - j 4, 0 and 3 + j 4 . The last two points of the DFT are respectively
(A) 0, 1 - j 3 (B) 0, 1 + j 3
(C) 1 + j3, 5 (D) 1 - j 3, 5

2010 ONE MARK

Q. 24 Consider the z -transform x (z) = 5z2 + 4z-1 + 3; 0 < z < 3. The inverse z -
transform x [n] is
(A) 5d [n + 2] + 3d [n] + 4d [n - 1]
(B) 5d [n - 2] + 3d [n] + 4d [n + 1]
(C) 5u [n + 2] + 3u [n] + 4u [n - 1]
(D) 5u [n - 2] + 3u [n] + 4u [n + 1]

Downloaded From : www.EasyEngineering.net


Downloaded From : www.EasyEngineering.net
GATE SOLVED PAPER - EC SIGNALS & SYSTEMS

Q. 25 The trigonometric Fourier series for the waveform f (t) shown below contains

(A) only cosine terms and zero values for the dc components
(B) only cosine terms and a positive value for the dc components
(C) only cosine terms and a negative value for the dc components

ww (D) only sine terms and a negative value for the dc components

w.E
Q. 26 Two discrete time system with impulse response h1 [n] = d [n - 1] and h2 [n] = d [n - 2]
are connected in cascade. The overall impulse response of the cascaded system is
(A) d [n - 1] + d [n - 2] (B) d [n - 4]

Q. 27
(C) d [n - 3]
asy (D) d [n - 1] d [n - 2]

For a N -point FET algorithm N = 2m which one of the following statements is


TRUE ?
En
(A) It is not possible to construct a signal flow graph with both input and
output in normal order
gin
(B) The number of butterflies in the m th stage in N/m

eer
(C) In-place computation requires storage of only 2N data
(D) Computation of a butterfly requires only one complex multiplication.

ing
Q. 28
2010

Given f (t) = L-1 ; 3 3s + 1


s + 4s2 + (k - 3) s E
.ne TWO MARKS

. If lim f (t) = 1, then the value of k is

Q. 29
(A) 1
(C) 3

A continuous time LTI system is described by


t"3
(B) 2
(D) 4 t
d 2 y (t) dy (t) dx (t)
+4 + 3y (t) = 2 + 4x (t)
dt 2 dt dt
Assuming zero initial conditions, the response y (t) of the above system for the
input x (t) = e-2t u (t) is given by
(A) (et - e3t) u (t) (B) (e-t - e-3t) u (t)
(C) (e-t + e-3t) u (t) (D) (et + e3t) u (t)

Q. 30 The transfer function of a discrete time LTI system is given by


2 - 34 z-1
H (z) =
1 - 34 z-1 + 18 z-2
Consider the following statements:
S1: The system is stable and causal for ROC: z > 1/2
S2: The system is stable but not causal for ROC: z < 1/4

Downloaded From : www.EasyEngineering.net


Downloaded From : www.EasyEngineering.net
GATE SOLVED PAPER - EC SIGNALS & SYSTEMS

S3: The system is neither stable nor causal for ROC: 1/4 < z < 1/2
Which one of the following statements is valid ?
(A) Both S1 and S2 are true (B) Both S2 and S3 are true
(C) Both S1 and S3 are true (D) S1, S2 and S3 are all true

2009 ONE MARK

Q. 31 The Fourier series of a real periodic function has only


(P) cosine terms if it is even
(Q) sine terms if it is even
(R) cosine terms if it is odd
(S) sine terms if it is odd
Which of the above statements are correct ?
(A) P and S (B) P and R

ww (C) Q and S (D) Q and R

w.E
Q. 32 A function is given by f (t) = sin2 t + cos 2t . Which of the following is true ?
(A) f has frequency components at 0 and 21p Hz
(B) f has frequency components at 0 and p1 Hz

asy
(C) f has frequency components at 21p and p1 Hz
(D) f has frequency components at 0. 1
2p and 1
p Hz

Q. 33
En
The ROC of z -transform of the discrete time sequence

gin
x (n) = b 1 l u (n) - b 1 l u (- n - 1) is
n n

3 2
(A) 1 < z < 1 (B) z > 1
3
(C) z < 1
2 2
(D) 2 < z < 3 eer
2009
3

ing TWO MARKS

Q. 34
.ne
Given that F (s) is the one-side Laplace transform of f (t), the Laplace transform

t
t
of #0 f (t) dt is
(A) sF (s) - f (0) (B) 1 F (s)
s
(D) 1 [F (s) - f (0)]
s
(C) #0 F (t) dt
s

Q. 35 A system with transfer function H (z) has impulse response h (.) defined as
h (2) = 1, h (3) =- 1 and h (k) = 0 otherwise. Consider the following statements.
S1 : H (z) is a low-pass filter.
S2 : H (z) is an FIR filter.
Which of the following is correct?
(A) Only S2 is true
(B) Both S1 and S2 are false
(C) Both S1 and S2 are true, and S2 is a reason for S1
(D) Both S1 and S2 are true, but S2 is not a reason for S1

Downloaded From : www.EasyEngineering.net


Downloaded From : www.EasyEngineering.net
GATE SOLVED PAPER - EC SIGNALS & SYSTEMS

Q. 36 Consider a system whose input x and output y are related by the equation
# x (t - t) g (2t) dt where h (t) is shown in the graph.
3
y (t) =
-3

Which of the following four properties are possessed by the system ?


BIBO : Bounded input gives a bounded output.
Causal : The system is causal,
LP : The system is low pass.
LTI : The system is linear and time-invariant.
(A) Causal, LP (B) BIBO, LTI

ww
Q. 37
(C) BIBO, Causal, LTI (D) LP, LTI

The 4-point Discrete Fourier Transform (DFT) of a discrete time sequence

w.E {1,0,2,3} is
(A) [0, - 2 + 2j , 2, - 2 - 2j ]
(C) [6, 1 - 3j , 2, 1 + 3j ]
(B) [2, 2 + 2j , 6, 2 - 2j ]
(D) [6, - 1 + 3j , 0, - 1 - 3j ]

Q. 38 asy
An LTI system having transfer function s +s 2+s 1+ 1 and input x (t) = sin (t + 1) is in
2
2

En
steady state. The output is sampled at a rate ws rad/s to obtain the final output
{x (k)}. Which of the following is true ?

gin
(A) y (.) is zero for all sampling frequencies ws
(B) y (.) is nonzero for all sampling frequencies ws
(C) y (.) is nonzero for ws > 2 , but zero for ws < 2
(D) y (.) is zero for ws > 2 , but nonzero for w2 < 2 eer
2008 ing ONE MARK

Q. 39
.ne
The input and output of a continuous time system are respectively denoted by
x (t) and y (t). Which of the following descriptions corresponds to a causal system
?
(A) y (t) = x (t - 2) + x (t + 4)
(C) y (t) = (t + 4) x (t - 1)
t
(B) y (t) = (t - 4) x (t + 1)
(D) y (t) = (t + 5) x (t + 5)

Q. 40 The impulse response h (t) of a linear time invariant continuous time system is
described by h (t) = exp (at) u (t) + exp (bt) u (- t) where u (- t) denotes the unit
step function, and a and b are real constants. This system is stable if
(A) a is positive and b is positive
(B) a is negative and b is negative
(C) a is negative and b is negative
(D) a is negative and b is positive

2008 TWO MARKS

Q. 41 A linear, time - invariant, causal continuous time system has a rational transfer
function with simple poles at s =- 2 and s =- 4 and one simple zero at s =- 1.

Downloaded From : www.EasyEngineering.net


Downloaded From : www.EasyEngineering.net
GATE SOLVED PAPER - EC SIGNALS & SYSTEMS

A unit step u (t) is applied at the input of the system. At steady state, the output
has constant value of 1. The impulse response of this system is
(A) [exp (- 2t) + exp (- 4t)] u (t)
(B) [- 4 exp (- 2t) - 12 exp (- 4t) - exp (- t)] u (t)
(C) [- 4 exp (- 2t) + 12 exp (- 4t)] u (t)
(D) [- 0.5 exp (- 2t) + 1.5 exp (- 4t)] u (t)

Q. 42 The signal x (t) is described by


1 for - 1 # t # + 1
x (t) = )
0 otherwise
Two of the angular frequencies at which its Fourier transform becomes zero are
(A) p, 2p (B) 0.5p, 1.5p
(C) 0, p (D) 2p, 2.5p

ww
Q. 43 A discrete time linear shift - invariant system has an impulse response h [n] with
h [0] = 1, h [1] =- 1, h [2] = 2, and zero otherwise The system is given an input

w.E sequence x [n] with x [0] = x [2] = 1, and zero otherwise. The number of nonzero
samples in the output sequence y [n], and the value of y [2] are respectively
(A) 5, 2 (B) 6, 2
(C) 6, 1
asy (D) 5, 3

Let x (t) be the input and y (t) be the output of a continuous time system. Match

En
Q. 44
the system properties P1, P2 and P3 with system relations R1, R2, R3, R4
Properties Relations

P2 : Time - invariant but NOT linear gin


P1 : Linear but NOT time - invariant R1 : y (t) = t2 x (t)
R2 : y (t) = t x (t)
P3 : Linear and time - invariant
eer
R3 : y (t) = x (t)
R4 : y (t) = x (t - 5)
(A) (P1, R1), (P2, R3), (P3, R4)
(C) (P1, R3), (P2, R1), (P3, R2) ing
(B) (P1, R2), (P2, R3), (P3, R4)
(D) (P1, R1), (P2, R2), (P3, R3)

Q. 45
.ne
{x (n)} is a real - valued periodic sequence with a period N . x (n) and X (k) form
N-point Discrete Fourier Transform (DFT) pairs. The DFT Y (k) of the sequence

(A) X (k) 2
N-1
y (n) = 1 / x (r) x (n + r) is
N r=0
N-1
t
(B) 1 / X (r) X (k + r)
N r=0
N-1
(C) 1 / X (r) X (k + r) (D) 0
N r=0

Statement for Linked Answer Question 46 and 47:


In the following network, the switch is closed at t = 0- and the sampling starts
from t = 0 . The sampling frequency is 10 Hz.

Downloaded From : www.EasyEngineering.net


Downloaded From : www.EasyEngineering.net
GATE SOLVED PAPER - EC SIGNALS & SYSTEMS

Q. 46 The samples x (n), n = (0, 1, 2, ...) are given by


(A) 5 (1 - e-0.05n) (B) 5e-0.05n
(C) 5 (1 - e-5n) (D) 5e-5n

Q. 47 The expression and the region of convergence of the z -transform of the sampled
signal are
(A) 5z 5 , z < e-5 (B) 5z , z < e-0.05
z-e z - e-0.05
(C) 5z , z > e-0.05 (D) 5z -5 , z > e-5
z - e-0.05 z-e

Statement for Linked Answer Question 48 & 49:


The impulse response h (t) of linear time - invariant continuous time system is
given by h (t) = exp (- 2t) u (t), where u (t) denotes the unit step function.

ww
Q. 48 The frequency response H (w) of this system in terms of angular frequency w, is
given by H (w)

w.E (A)

(C) 1
1
1 + j 2w
(B) sin w

(D)
w
jw

Q. 49
2 + jw
asy 2 + jw

The output of this system, to the sinusoidal input x (t) = 2 cos 2t for all time t , is
(A) 0
En
(C) 2-0.5 cos (2t - 0.125p)
(B) 2-0.25 cos (2t - 0.125p)
(D) 2-0.5 cos (2t - 0.25p)

2007
gin ONE MARK

Q. 50 If the Laplace transform of a signal Y (s) = eer


1
s (s - 1)
, then its final value is
(A) - 1
(C) 1
(B) 0
ing
(D) Unbounded

2007 .ne TWO MARKS

Q. 51
function, is given by
(A) 1 Hz (B) 1 2 - 1 Hz
t
The 3-dB bandwidth of the low-pass signal e-t u (t), where u (t) is the unit step

2p 2p
(C) 3 (D) 1 Hz

Q. 52 A 5-point sequence x [n] is given as x [- 3] = 1, x [- 2] = 1, x [- 1] = 0, x [0] = 5


and x [1] = 1. Let X (eiw) denoted the discrete-time Fourier transform of x [n]. The
p
value of #
-p
X (e jw) dw is
(A) 5 (B) 10p
(C) 16p (D) 5 + j10p

Q. 53 The z -transform X (z) of a sequence x [n] is given by X [z] = 1 -0.25z . It is given that
-1

the region of convergence of X (z) includes the unit circle. The value of x [0] is
(A) - 0.5 (B) 0
(C) 0.25 (D) 05

Downloaded From : www.EasyEngineering.net


Downloaded From : www.EasyEngineering.net
GATE SOLVED PAPER - EC SIGNALS & SYSTEMS

Q. 54 A Hilbert transformer is a
(A) non-linear system (B) non-causal system
(C) time-varying system (D) low-pass system

Q. 55 The frequency response of a linear, time-invariant system is given by


H (f) = 1 + j510pf . The step response of the system is
(B) 5 61 - e- 5@ u (t)
t
(A) 5 (1 - e-5t) u (t)
(C) 1 (1 - e-5t) u (t) (D) 1 ^1 - e- 5 h u (t)
t

2 5

2006 ONE MARK

Q. 56 Let x (t) * X (jw) be Fourier Transform pair. The Fourier Transform of the signal
x (5t - 3) in terms of X (jw) is given as

ww jw jw
j3w j3w
(A) 1 e- 5 X b l (B) 1 e 5 X b l
5 5 5 5
jw jw
(C) 1 e-j3w X b l (D) 1 e j3w X b l

w.E
Q. 57
5 5

The Dirac delta function d (t) is defined as


5 5

(A) d (t) = )
1
0
asyt=0
otherwise
3 t=0
(B) d (t) = )
0
En
otherwise
1 t=0

gin
and # d (t) dt = 1
3
(C) d (t) = )
0 otherwise -3

3 t=0
and # d (t) dt = 1
3
(D) d (t) = )

Q. 58
0 otherwise -3

If the region of convergence of x1 [n] + x2 [n] eer


is 1 < z < 2 then the region of
convergence of x1 [n] - x2 [n] includes
(A) 1 < z < 3 (B)
3
ing
2 < z <3
3

3
(C) 3 < z < 3
2
(D)
3
1< z <2
3 3 .ne
Q. 59

will be
t
In the system shown below, x (t) = (sin t) u (t) In steady-state, the response y (t)

1 sin t - p 1 sin t + p
(A) ` 4j
(B) ` 4j
2 2
(C) 1 e-t sin t (D) sin t - cos t
2

2006 TWO MARKS

Q. 60 Consider the function f (t) having Laplace transform


F (s) = 2 w0 2 Re [s] > 0
s + w0

Downloaded From : www.EasyEngineering.net


Downloaded From : www.EasyEngineering.net
GATE SOLVED PAPER - EC SIGNALS & SYSTEMS

The final value of f (t) would be


(A) 0 (B) 1
(C) - 1 # f (3) # 1 (D) 3

Q. 61 A system with input x [n] and output y [n] is given as y [n] = (sin 56 pn) x [n]. The
system is
(A) linear, stable and invertible
(B) non-linear, stable and non-invertible
(C) linear, stable and non-invertible
(D) linear, unstable and invertible

Q. 62 The unit step response of a system starting from rest is given by c (t) = 1 - e-2t
for t $ 0 . The transfer function of the system is
(A) 1 (B) 2

ww (C) 1
1 + 2s

2+s
2+s
(D) 2s
1 + 2s

w.E
Q. 63 The unit impulse response of a system is f (t) = e-t, t $ 0 . For this system the
steady-state value of the output for unit step input is equal to
(A) - 1
(C) 1 asy (B) 0
(D) 3

2005 En ONE MARK

Q. 64
gin
Choose the function f (t); - 3 < t < 3 for which a Fourier series cannot be defined.
(A) 3 sin (25t)
(C) exp (- t ) sin (25t)
eer
(B) 4 cos (20t + 3) + 2 sin (710t)
(D) 1

Q. 65
function u (t) are respectively, ing
The function x (t) is shown in the figure. Even and odd parts of a unit step

.ne
t
(A) 1 , 1 x (t) (B) - 1 , 1 x (t)
2 2 2 2
1 1
(C) , - x (t) (D) - 1 , - 1 x (t)
2 2 2 2
Q. 66 The region of convergence of z - transform of the sequence
5 n 6 n
b 6 l u (n) - b 5 l u (- n - 1) must be
(A) z < 5 (B) z > 5
6 6
5
(C) < z < 6 6
(D) < z < 3
6 5 5

Q. 67 Which of the following can be impulse response of a causal system ?

Downloaded From : www.EasyEngineering.net


Downloaded From : www.EasyEngineering.net
GATE SOLVED PAPER - EC SIGNALS & SYSTEMS

Q. 68 Let x (n) = ( 12 ) n u (n), y (n) = x2 (n) and Y (e jw) be the Fourier transform of y (n)
then Y (e j0)
(A) 1 (B) 2
4

ww (C) 4 (D) 4
3

w.E
Q. 69 The power in the signal s (t) = 8 cos (20p - p2 ) + 4 sin (15pt) is
(A) 40
(B) 41
(C) 42
(D) 82 asy
En
Q. 70
2005

gin TWO MARKS

The output y (t) of a linear time invariant system is related to its input x (t) by
the following equations
eer
y (t)= 0.5x (t - td + T) + x (t - td ) + 0.5x (t - td + T)

d ing
The filter transfer function H (w) of such a system is given by
(A) (1 + cos wT) e-jwt (B) (1 + 0.5 cos wT) e-jwt d

(C) (1 - cos wT) e-jwtd

.ne
(D) (1 - 0.5 cos wT) e-jwt d

Q. 71 Match the following and choose the correct combination.


Group 1
E. Continuous and aperiodic signal
F. Continuous and periodic signal
t
G. Discrete and aperiodic signal
H. Discrete and periodic signal
Group 2
1. Fourier representation is continuous and aperiodic
2. Fourier representation is discrete and aperiodic
3. Fourier representation is continuous and periodic
4. Fourier representation is discrete and periodic
(A) E - 3, F - 2, G - 4, H - 1
(B) E - 1, F - 3, G - 2, H - 4
(C) E - 1, F - 2, G - 3, H - 4
(D) E - 2, F - 1, G - 4, H - 3

Downloaded From : www.EasyEngineering.net


Downloaded From : www.EasyEngineering.net
GATE SOLVED PAPER - EC SIGNALS & SYSTEMS

Q. 72 A signal x (n) = sin (w0 n + f) is the input to a linear time- invariant system having
a frequency response H (e jw). If the output of the system Ax (n - n0) then the most
general form of +H (e jw) will be
(A) - n0 w0 + b for any arbitrary real
(B) - n0 w0 + 2pk for any arbitrary integer k
(C) n0 w0 + 2pk for any arbitrary integer k
(D) - n0 w0 f

Statement of linked answer question 73 and 74 :


A sequence x (n) has non-zero values as shown in the figure.

ww
w.E
Q. 73 asy
The sequence y (n) = *
x ( n2 - 1), For n even
will be

En 0, For n odd

gin
eer
ing
.ne
t

Downloaded From : www.EasyEngineering.net


Downloaded From : www.EasyEngineering.net
GATE SOLVED PAPER - EC SIGNALS & SYSTEMS

Q. 74 The Fourier transform of y (2n) will be


(A) e-j2w [cos 4w + 2 cos 2w + 2] (B) cos 2w + 2 cos w + 2
-jw
(C) e [cos 2w + 2 cos w + 2] (D) e-j2w [cos 2w + 2 cos + 2]

Q. 75 For a signal x (t) the Fourier transform is X (f). Then the inverse Fourier transform
of X (3f + 2) is given by
j4pt
(A) 1 x` t j e j3pt (B) 1 x` t j e - 3
2 2 3 3
(C) 3x (3t) e-j4pt (D) x (3t + 2)

2004 ONE MARK

Q. 76 The impulse response h [n] of a linear time-invariant system is given by


h [n] = u [n + 3] + u [n - 2) - 2n [n - 7] where u [n] is the unit step sequence. The

ww above system is
(A) stable but not causal (B) stable and causal

w.E
Q. 77
(C) causal but unstable (D) unstable and not causal

The z -transform of a system is H (z) = z -z0.2 . If the ROC is z < 0.2 , then the
impulse response of the system is
(A) (0.2) n u [n]
(C) - (0.2) n u [n] asy (B) (0.2) n u [- n - 1]
(D) - (0.2) n u [- n - 1]

Q. 78
En
The Fourier transform of a conjugate symmetric function is always
(A) imaginary
(C) real gin
(B) conjugate anti-symmetric
(D) conjugate symmetric

2004 eer TWO MARKS

Q. 79
part of the sequence is - ing
Consider the sequence x [n] = [- 4 - j51 + j25]. The conjugate anti-symmetric

(A) [- 4 - j2.5, j2, 4 - j2.5]


(C) [- j2.5, j2, 0]
(B) [- j2.5, 1, j2.5]
(D) [- 4, 1, 4] .ne
Q. 80 A causal LTI system is described by the difference equation
2y [n] = ay [n - 2] - 2x [n] + bx [n - 1]
t
The system is stable only if
(A) a = 2 , b < 2 (B) a > 2, b > 2
(C) a < 2 , any value of b (D) b < 2 , any value of a

Q. 81 The impulse response h [n] of a linear time invariant system is given as


-2 2 n = 1, - 1
h [ n] = * 4 2 n = 2, - 2
0 otherwise
If the input to the above system is the sequence e jpn/4 , then the output is
(A) 4 2 e jpn/4 (B) 4 2 e-jpn/4
(C) 4e jpn/4 (D) - 4e jpn/4

Downloaded From : www.EasyEngineering.net


Downloaded From : www.EasyEngineering.net
GATE SOLVED PAPER - EC SIGNALS & SYSTEMS

Q. 82 Let x (t) and y (t) with Fourier transforms F (f) and Y (f) respectively be related as
shown in Fig. Then Y (f) is

(A) - 1 X (f/2) e-jpf (B) - 1 X (f/2) e j2pf


2 2
(C) - X (f/2) e j2pf (D) - X (f/2) e-j2pf

ww
Q. 83
2003

The Laplace transform of i (t) is given by I (s) = 2


s (1 + s)
ONE MARK

At t " 3, The value of

w.E i (t) tends to


(A) 0
(B) 1
(C) 2
(D) 3 asy
Q. 84
En
The Fourier series expansion of a real periodic signal with fundamental frequency

(A) 5 + j3 n =- 3
gin
f0 is given by gp (t) = / cn e j2pf t . It is given that c3 = 3 + j5 . Then c-3 is
0

(B) - 3 - j5

Q. 85
(C) - 5 + j3
eer
(D) 3 - j5

Let x (t) be the input to a linear, time-invariant system. The required output is

(A) 4e j4pf (B) 2e-j8pfing


4p (t - 2). The transfer function of the system should be

(C) 4e-j4pf (D) 2e j8pf


.ne
Q. 86

t
A sequence x (n) with the z -transform X (z) = z 4 + z2 - 2z + 2 - 3z-4 is applied
as an input to a linear, time-invariant system with the impulse response
h (n) = 2d (n - 3) where
1, n = 0
d (n) = )
0, otherwise
The output at n = 4 is
(A) - 6 (B) zero
(C) 2 (D) - 4

2003 TWO MARKS

Q. 87 Let P be linearity, Q be time-invariance, R be causality and S be stability. A


discrete time system has the input-output relationship,
x (n) n$1
y (n) = *0, n= 0
x (n + 1) n # - 1

Downloaded From : www.EasyEngineering.net


Downloaded From : www.EasyEngineering.net
GATE SOLVED PAPER - EC SIGNALS & SYSTEMS

where x (n) is the input and y (n) is the output. The above system has the
properties
(A) P, S but not Q, R (B) P, Q, S but not R
(C) P, Q, R, S (D) Q, R, S but not P

Common Data For Q. 88 & 89 :


The system under consideration is an RC low-pass filter (RC-LPF) with R = 1 k
W and C = 1.0 mF.
Q. 88 Let H (f) denote the frequency response of the RC-LPF. Let f1 be the highest
H (f1)
frequency such that 0 # f # f1 $ 0.95 . Then f1 (in Hz) is
H (0)
(A) 324.8 (B) 163.9

ww
Q. 89
(C) 52.2 (D) 104.4

Let tg (f) be the group delay function of the given RC-LPF and f2 = 100 Hz. Then

w.E tg (f2) in ms, is


(A) 0.717
(C) 71.7
(B) 7.17
(D) 4.505

2002
asy ONE MARK

Q. 90
En
Convolution of x (t + 5) with impulse function d (t - 7) is equal to
(A) x (t - 12)
(C) x (t - 2) gin
(B) x (t + 12)
(D) x (t + 2)

Q. 91
eer
Which of the following cannot be the Fourier series expansion of a periodic signal?
(A) x (t) = 2 cos t + 3 cos 3t
(C) x (t) = cos t + 0.5 ing
(B) x (t) = 2 cos pt + 7 cos t
(D) x (t) = 2 cos 1.5pt + sin 3.5pt

Q. 92 The Fourier transform F {e-1 u (t)} is equal to 1


1 + j 2p f .ne
. Therefore, F ' 1
1 + j2pt 1
is
(A) e f u (f)
(C) e f u (- f)
(B) e-f u (f)
(D) e-f u (- f) t
Q. 93 A linear phase channel with phase delay Tp and group delay Tg must have
(A) Tp = Tg = constant
(B) Tp \ f and Tg \ f
(C) Tp = constant and Tg \ f ( f denote frequency)
(D) Tp \ f and Tp = constant

2002 TWO MARKS

Q. 94 The Laplace transform of continuous - time signal x (t) is X (s) = 5-s


s2 - s - 2
. If the
Fourier transform of this signal exists, the x (t) is
(A) e2t u (t) - 2e-t u (t) (B) - e2t u (- t) + 2e-t u (t)
(C) - e2t u (- t) - 2e-t u (t) (D) e2t u (- t) - 2e-t u (t)

Downloaded From : www.EasyEngineering.net


Downloaded From : www.EasyEngineering.net
GATE SOLVED PAPER - EC SIGNALS & SYSTEMS

Q. 95 If the impulse response of discrete - time system is h [n] =- 5n u [- n - 1], then the
system function H (z) is equal to
(A) - z and the system is stable (B) z and the system is stable
z-5 z-5
(C) - z and the system is unstable (D) z and the system is unstable
z-5 z-5

2001 ONE MARK

Q. 96 The transfer function of a system is given by H (s) = 2 1 . The impulse


response of the system is s (s - 2)
(A) (t2 * e-2t) u (t) (B) (t * e2t) u (t)
(C) (te-2 t) u (t) (D) (te-2t) u (t)

Q. 97 The region of convergence of the z - transform of a unit step function is

ww (A) z > 1
(C) (Real part of z ) > 0
(B) z < 1
(D) (Real part of z ) < 0

w.E
Q. 98 Let d (t) denote the delta function. The value of the integral
(A) 1 (B) - 1
#
-3
3
d (t) cos b 3t l dt is
2

Q. 99
(C) 0
asy (D) p2

If a signal f (t) has energy E , the energy of the signal f (2t) is equal to
(A) 1
En (B) E/2
(C) 2E

gin (D) 4E

Q. 100
2001
eer
The impulse response functions of four linear systems S1, S2, S3, S4 are given
TWO MARKS

respectively by
h1 (t) = 1, h2 (t) = u (t), h3 (t) =
u (t) ing
and h 4 (t) = e-3t u (t)
t+1
.ne
where u (t) is the unit step function. Which of these systems is time invariant,
causal, and stable?
(A) S1
(C) S3
(B) S2
(D) S4
t
2000 ONE MARK

Given that L [f (t)] = s2+ 2 , L [g (t)] = s2 + 1 t


Q. 101
s +1 (s + 3) (s + 2)
and h (t) = #0 f (t) g (t - t) dt .
L [h (t)] is
2
(A) s + 1 (B) 1
s+3 s+3
(C) s2 + 1 + s+2 (D) None of the above
(s + 3)( s + 2) s2 + 1
2
Q. 102 The Fourier Transform of the signal x (t) = e-3t is of the following form, where A
and B are constants :
2
(A) Ae-B f (B) Ae-Bf
(C) A + B f 2 (D) Ae-Bf

Downloaded From : www.EasyEngineering.net


Downloaded From : www.EasyEngineering.net
GATE SOLVED PAPER - EC SIGNALS & SYSTEMS

Q. 103 A system with an input x (t) and output y (t) is described by the relations :
y (t) = tx (t). This system is
(A) linear and time - invariant (B) linear and time varying
(C) non - linear and time - invariant (D) non - linear and time - varying

Q. 104 A linear time invariant system has an impulse response e2t, t > 0 . If the initial
conditions are zero and the input is e3t , the output for t > 0 is
(A) e3t - e2t (B) e5t
(C) e3t + e2t (D) None of these

2000 TWO MARKS

Q. 105 One period (0, T) each of two periodic waveforms W1 and W2 are shown in the
figure. The magnitudes of the nth Fourier series coefficients of W1 and W2 , for

ww n $ 1, n odd, are respectively proportional to

w.E
asy
(A) n and n-2
-3

(C) n-1 and n-2 En (B) n-2 and n-3


(D) n-4 and n-2

Q. 106
gin
Let u (t) be the step function. Which of the waveforms in the figure corresponds
to the convolution of u (t) - u (t - 1) with u (t) - u (t - 2) ?

eer
ing
.ne
t

Q. 107 A system has a phase response given by f (w), where w is the angular frequency.
The phase delay and group delay at w = w0 are respectively given by
f (w0) df (w) d2 f (w0)
(A) - , - (B) f (wo), -
w0 dw w = w 0 dw2 w = w o

df (w)
(C) wo , -
w
(D) wo f (wo), # f (l)
o

f (wo) d (w) w = w
o
-3

Downloaded From : www.EasyEngineering.net


Downloaded From : www.EasyEngineering.net
GATE SOLVED PAPER - EC SIGNALS & SYSTEMS

1999 ONE MARK

Q. 108 The z -transform F (z) of the function f (nT) = anT is


(A) z T (B) z T
z-a z+a
(C) z (D) z
z - a-T z + a-T
Q. 109 If [f (t)] = F (s), then [f (t - T)] is equal to
(A) esT F (s) (B) e-sT F (s)
F (s) F (s)
(C) (D)
1 - esT 1 - e-sT
Q. 110 A signal x (t) has a Fourier transform X (w). If x (t) is a real and odd function of
t , then X (w) is
(A) a real and even function of w

ww (B) a imaginary and odd function of w


(C) an imaginary and even function of w

w.E (D) a real and odd function of w

Q. 111
1999

asy
The Fourier series representation of an impulse train denoted by
TWO MARKS

En
s (t) =
3
/ d (t - nT0) is given by
n =- 3

(A) 1 / exp -
j2pnt

gin (B) 1 / exp -


jpnt
3 3

T0 n =- 3 T0 T0 n =- 3 T0
jpnt j2pnt
(C) 1 / exp (D) 1 / exp
3 3

T0 n =- 3 T0
eer
T0 n =- 3 T0
1z-1 (1 - z-4)
Q. 112 The z -transform of a signal is given by C (z) =
(A) 1/4 ing
4 (1 - z-1) 2
(B) zero
. Its final value is

(C) 1.0 (D) infinity


.ne
Q. 113
1998

If F (s) = w , then the value of Limf (t)


s2 + w2 t"3
t ONE MARK

(A) cannot be determined (B) is zero


(C) is unity (D) is infinite

Q. 114 The trigonometric Fourier series of a even time function can have only
(A) cosine terms (B) sine terms
(C) cosine and sine terms (D) d.c and cosine terms
1, t < T1
Q. 115 A periodic signal x (t) of period T0 is given by x (t) = *
0, T1 < t < T0
The dc component of x (t) is 2
(A) T1
(B) T1
T0 2T0
(C) 2T1 T
(D) 0
T0 T1

Downloaded From : www.EasyEngineering.net


Downloaded From : www.EasyEngineering.net
GATE SOLVED PAPER - EC SIGNALS & SYSTEMS

Q. 116 The unit impulse response of a linear time invariant system is the unit step
function u (t). For t > 0 , the response of the system to an excitation e-at u (t), a > 0
will be
(A) ae-at (B) (1/a) (1 - e-at)
(C) a (1 - e-at) (D) 1 - e-at
3
Q. 117 The z-transform of the time function / d (n - k) is
k=0

(A) z - 1 (B) z
z z-1
z (z - 1) 2
(C) (D)
(z - 1) 2 z
Q. 118 A distorted sinusoid has the amplitudes A1, A2, A 3, .... of the fundamental, second
harmonic, third harmonic,..... respectively. The total harmonic distortion is

ww (A) 2 + A 3 + ....
A1
A A 2 + A 2 + ..... 2
2

A1
2
2
(B) A 2 + A 3 + .....

(D) c A 2 + A 3 + ..... m
w.E (C) 2 3
2 2 2
A 1 + A 2 + A 3 + .... A1
dX (t)
Q. 119 The Fourier transform of a function x (t) is X (f). The Fourier transform of
df
will be

(A)
dX (f) asy (B) j2pfX (f)
df
(C) jfX (f) En (D)
X (f)
jf

1997 gin ONE MARK

Q. 120
eer
The function f (t) has the Fourier Transform g (w). The Fourier Transform

ing
3
ff (t) g (t) e = # g (t) e-jwt dt o is
-3
(A) 1 f (w) (B) 1 f (- w)
2p
(C) 2pf (- w)
2p
.ne
(D) None of the above

Q. 121 The Laplace Transform of eat cos (at) is equal to


(A)
(s - a)
(s - a) 2 + a2
(B)
(s + a)
(s - a) 2 + a2
t
(C) 1 (D) None of the above
(s - a) 2

1996 ONE MARK

Q. 122 The trigonometric Fourier series of an even function of time does not have the
(A) dc term (B) cosine terms
(C) sine terms (D) odd harmonic terms

Q. 123 The Fourier transform of a real valued time signal has


(A) odd symmetry (B) even symmetry
(C) conjugate symmetry (D) no symmetry
***********

Downloaded From : www.EasyEngineering.net


Downloaded From : www.EasyEngineering.net
GATE SOLVED PAPER - EC SIGNALS & SYSTEMS

SOLUTIONS

Sol. 1 Option (C) is correct.


If the two systems with impulse response h1 ^ t h and h2 ^ t h are connected in
cascaded configuration as shown in figure, then the overall response of the system
is the convolution of the individual impulse responses.

ww
w.E
Sol. 2 Option (C) is correct.
Given, the input x ^ t h = u ^t - 1h
It’s Laplace transform is

asyX ^s h = e
-s

En
The impulse response of system is given
h^t h = t u^t h
Its Laplace transform is
H ^s h = 12
gin
s
Hence, the overall response at the output is eer
-s
Y ^s h = X ^s h H ^s h = e 3
s ing
Its inverse Laplace transform is

y^t h =
^t - 1h2
u ^t - 1h
.ne
Sol. 3 Option (A) is correct.
Given, the signal
2
t
v ^ t h = 30 sin 100t + 10 cos 300t + 6 sin ^500t + p4 h
So we have
w1 = 100 rad/s , w2 = 300 rad/s and w3 = 500 rad/s
Therefore, the respective time periods are
T1 = 2p = 2p sec , T2 = 2p = 2p sec and T3 = 2p sec
w1 100 w2 300 500
So, the fundamental time period of the signal is
LCM ^2p, 2p, 2ph
L.C.M. ^T1, T2 T3h =
HCF ^100, 300, 500h
or, T0 = 2p
100
Hence, the fundamental frequency in rad/sec is w0 = 2p = 100 rad/s
10

Downloaded From : www.EasyEngineering.net


Downloaded From : www.EasyEngineering.net
GATE SOLVED PAPER - EC SIGNALS & SYSTEMS

Sol. 4 Option (A) is correct.


Given, the maximum frequency of the band-limited signal
fm = 5 kHz
According to the Nyquist sampling theorem, the sampling frequency must be
greater than the Nyquist frequency which is given as
fN = 2fm = 2 # 5 = 10 kHz
So, the sampling frequency fs must satisfy
fs $ fN
fs $ 10 kHz
only the option (A) doesn’t satisfy the condition therefore, 5 kHz is not a valid
sampling frequency.
Sol. 5 Option (C) is correct.

ww For a system to be casual, the R.O.C of system transfer function H ^s h which is


rational should be in the right half plane and to the right of the right most pole.

w.E For the stability of LTI system. All poles of the system should lie in the
left half of S -plane and no repeated pole should be on imaginary axis. Hence,
options (A), (B), (D) satisfies an LTI system stability and causality both.

asy
But, Option (C) is not true for the stable system as, S = 1 have one pole
in right hand plane also.
Sol. 6

En
Option (B) is correct.
The Laplace transform of unit step fun n is
U ^s h = 1
s
So, the O/P of the system is given as
gin
Y ^s h = b 1 lb 1 l = 12
s s s eer
For zero initial condition, we check
u^t h =
dy ^ t h
dt
ing
& U ^s h = SY ^s h - y ^0 h
U ^s h = s c 12 m - y ^0 h
.ne
&

or, U ^s h = 1
s
s

Hence, the O/P is correct which is


t
^y ^0 h = 0h

Y ^s h = 12
s
its inverse Laplace transform is given by
y ^ t h = tu ^ t h
Sol. 7 No Option is correct.
The matched filter is characterized by a frequency response that is given as
H ^ f h = G * ^ f h exp ^- j2pfT h
g^t h G^f h
f
where
Now, consider a filter matched to a known signal g ^ t h. The fourier transform of
the resulting matched filter output g 0 ^ t h will be
G 0 ^ f h = H ^ f h G ^ f h = G * ^ f h G ^ f h exp ^- j2pfT h
= G ^ f h 2 exp ^- j2pfT h
T is duration of g ^ t h

Downloaded From : www.EasyEngineering.net


Downloaded From : www.EasyEngineering.net
GATE SOLVED PAPER - EC SIGNALS & SYSTEMS

Assume exp ^- j2pfT h = 1


So, G0 ^ f h = G_ f i 2
Since, the given Gaussian function is
g ^ t h = e- pt
2

Fourier transform of this signal will be


g ^ t h = e- pt e- pf = G ^ f h
2 f 2

Therefore, output of the matched filter is


G 0 ^ f h = e- pf
2 2

Sol. 8 Option (B) is correct.


Given, the impulse response of continuous time system
h ^ t h = d ^t - 1h + d ^t - 3h
From the convolution property, we know
x ^ t h * d ^t - t 0h = x ^t - t 0h

ww So, for the input


x ^ t h = u ^ t h (Unit step fun n )
The output of the system is obtained as

w.E y ^ t h = u ^ t h * h ^ t h = u ^ t h * 6d ^t - 1h + d ^t - 3h@

= u ^t - 1h + u ^t - 3h

Sol. 9
At t = 2
asy y ^2 h = u ^2 - 1h + u ^2 - 3h = 1
Option (B) is correct.

En
Given, the differential equation
d2y dy
+ 5 + 6y ^ t h = x ^ t h
dt2 dt
gin
Taking its Laplace transform with zero initial conditions, we have
s2 Y ^s h + 5sY ^s h + 6Y ^s h = X ^s h
Now, the input signal is eer ....(1)

x^t h = *
1 0<t<2
0 otherwise ing
i.e., x ^ t h = u ^ t h - u ^t - 2h
Taking its Laplace transform, we obtain .ne
X ^s h = 1 - e
s
-2s

s
Substituting it in equation (1), we get
X ^s h
= 1-e
s
-2s

2s
t
1 - e-2s
Y ^s h = 2 = 21 - e =
s + 5s + 6 s ^s + 5s + 6h s ^s + 2h^s + 3h

Sol. 10 Option (D) is correct.


The solution of a system described by a linear, constant coefficient, ordinary, first
order differential equation with forcing function x ^ t h is y ^ t h so, we can define a
function relating x ^ t h and y ^ t h as below
dy
P + Qy + K = x ^ t h
dt
where P , Q , K are constant. Taking the Laplace transform both the sides, we
get
P sY ^s h - Py ^0 h + Q Y ^s h = X ^s h ....(1)
Now, the solutions becomes
y1 ^ t h =- 2y ^ t h

Downloaded From : www.EasyEngineering.net


Downloaded From : www.EasyEngineering.net
GATE SOLVED PAPER - EC SIGNALS & SYSTEMS

or, Y1 ^s h =- 2Y ^s h
So, Eq. (1) changes to
P sY1 ^s h - P y1 ^0 h + Q Y1 ^s h = X1 ^s h
or, - 2PSY ^s h - P y1 ^0 h - 2QY1 ^s h = X1 ^s h ....(2)
Comparing Eq. (1) and (2), we conclude that
X1 ^s h =- 2X ^s h
y1 ^0 h =- 2y ^0 h
Which makes the two equations to be same. Hence, we require to change the
initial condition to - 2y ^0 h and the forcing equation to - 2x ^ t h
Sol. 11 Option (A) is correct.
Given, the DFT of vector 8a b c dB as
D.F.T. %8a b c dB/ = 8a b g dB

ww Also, we have
R V
Sa b c d W
Sd a b c W

w.E 8p q r sB = 8a b c dBSc d a b W
S
Sb c d aW
T
W
X
...(1)

asy
For matrix circular convolution, we know
Rh h h VRx V
S 0 2 1WS 0W
x 6n@ * h 6n@ = Sh1 h 0 h2WSx1W

En Sh h h WSx W
T
2 1 0 1
XT X
where "x 0, x1, x2, are three point signals for x 6n@ and similarly for h 6n@, h 0 , h1
gin
and h2 are three point signals. Comparing this transformation to Eq(1), we get
R
Sa d c W
Sb a d W
VT

6p q r s@ = Sc b aW 8a b c dB
eer
S
Sd c bW
T
W
X
ing
Sa W
R V
Sa W .ne
= 6a b c d @T * 6a b c d @T
R V
Sb W
=S W * S W
Sc W
Sd W
T X
Sb W
Sc W
Sd W
T X
t
Now, we know that
x1 6n@ * x2 6n@ = X1DFT 6k @ X2, DFT 6k @
R V R V R V R V
Sa W Sa W SaW SaW
Sb W Sb W SbW SbW
So, Sc W * Sc W = SgW * SgW
S W S W S W S W
Sd W Sd W Sd W Sd W
T X T X T X2 2 2T X2
= 9a b g d C
Sol. 12 Option (D) is correct.
Using s -domain differentiation property of Laplace transform.
L
If f (t) F (s)
dF (s)
-
L
tf (t)
ds

Downloaded From : www.EasyEngineering.net


Downloaded From : www.EasyEngineering.net
GATE SOLVED PAPER - EC SIGNALS & SYSTEMS

L [tf (t)] = - d ; 2 1 2s + 1
ds s + s + 1E (s2 + s + 1) 2
So, =

Sol. 13 Option (C) is correct.


x [ n] = b 1 l - b 1 l u [ n]
n n

3 2
x [n] = b 1 l u [n] + b 1 l u [- n - 1] - b 1 l u (n)
n -n n

3 3 2
Taking z -transform

X 6z @ =
3 3 3
1 n -n 1 -n -n 1 n -n
/ b 3 l z u [ n] + / b 3 l z u [ - n - 1] - / b 2 l z u [ n]
n =- 3 n =- 3 n =- 3
3 -1 3
/ b 13 l z 1
/ b 12 l z
n -n n
= -n
+ / b3l z -
-n -n

n=0 n =- 3 n=0
3 3 3
/ b 31z l + / b 13 z l - /b 1 l
n m n
= Taking m =- n
2z

ww Series I converges if
n=0
14 2
I
43
1 < 1 or z > 1
m=1
1 44 2
II
4
4 3
n=0
1 4 2
III
43

w.E 3z
Series II converges if 1 z < 1 or z < 3
3
3

asy
Series III converges if 1 < 1 or z > 1
2z 2
Region of convergence of X (z) will be intersection of above three
So,
En
ROC : 1 < z < 3
2
Sol. 14 Option (D) is correct.
y (t) =
t
gin
# x (t) cos (3t) dt
Time Invariance :
-3

eer
Let, x (t) = d (t)
y (t) =
t
# d (t) cos (3t) dt ing
= u (t) cos (0) = u (t)

.ne
-3
For a delayed input (t - t 0) output is
t
# d (t - t ) cos (3t) dt
y (t, t 0) =
Delayed output,
-3

y (t - t 0) = u (t - t 0)
0

t
= u (t) cos (3t 0)

y (t, t 0) ! y (t - t 0) System is not time invariant.


Stability :
Consider a bounded input x (t) = cos 3t
y (t) = # cos2 3t = # 1 - cos 6t = 1 # 1dt - 12 # cos 6t dt
t t t t

-3 -3 2 2 -3 -3

As t " 3, y (t) " 3 (unbounded) System is not stable.


Sol. 15 Option (C) is correct.
(2 cos w) (sin 2w)
H (jw) = = sin 3w + sin w
w w w
We know that inverse Fourier transform of sin c function is a rectangular function.

Downloaded From : www.EasyEngineering.net


Downloaded From : www.EasyEngineering.net
GATE SOLVED PAPER - EC SIGNALS & SYSTEMS

ww So, inverse Fourier transform of H (jw)


h (t) = h1 (t) + h2 (t)

w.E
Sol. 16
h (0) = h1 (0) + h2 (0) = 1 + 1 = 1

Option (A) is correct.


2 2

asy
Convolution sum is defined as

y [n] = h [n] * g [n] =


3
/ h [n] g [n - k]
En
For causal sequence, y [ n] =
3
k =- 3

/ h [n] g [n - k]
k=0

gin
y [n] = h [n] g [n] + h [n] g [n - 1] + h [n] g [n - 2] + .....
For n = 0 ,
eer
y [0] = h [0] g [0] + h [1] g [- 1] + ...........
y [0] = h [0] g [0] g [- 1] = g [- 2] = ....0

For n = 1, y [1]
y [0] = h [0] g [0]
ing
= h [1] g [1] + h [1] g [0] + h [1] g [- 1] + ....
...(i)

y [1]
1
= h [ 1] g [ 1 ] + h [ 1 ] g [ 0 ]
= 1 g [1] + 1 g [0] h [1] = b 1 l = 1
1 .ne
2 2 2
1 = g [1] + g [0]
g [1] = 1 - g [0]
2 2
t
y [0] 1
From equation (i), g [0] = = =1
h [ 0] 1
So, g [1] = 1 - 1 = 0
Sol. 17 Option (A) is correct.
d2 y dy
We have 100 2 - 20 + y = x (t)
dt dt
Applying Laplace transform we get
100s2 Y (s) - 20sY (s) + Y (s) = X (s)
Y (s) 1
or H (s) = =
X (s) 100s2 - 20s + 1
1/100 A
= 2 =
s - (1/5) s + 1/100 s2 + 2xwn s + w2

Downloaded From : www.EasyEngineering.net


Downloaded From : www.EasyEngineering.net
GATE SOLVED PAPER - EC SIGNALS & SYSTEMS

Here wn = 1/10 and 2xwn =- 1/5 giving x =- 1


Roots are s = 1/10, 1/10 which lie on Right side of s plane thus unstable.
Sol. 18 Option (C) is correct.
For an even function Fourier series contains dc term and cosine term (even and
odd harmonics).
Sol. 19 Option (B) is correct.
Function h (n) = an u (n) stable if a < 1 and Unstable if a H 1We We have
h (n) = 2n u (n - 2);
Here a = 2 therefore h (n) is unstable and since h (n) = 0 for n < 0
Therefore h (n) will be causal. So h (n) is causal and not stable.
Sol. 20 Option (A) is correct.
Impulse response = d (step response)

ww dt
= d (1 - e- at) = 0 + ae- at = ae- at
dt

w.E
Sol. 21 Option (D) is correct.
We have x (t) = exp (- 2t) m (t) + s (t - 6) and h (t) = u (t)
Taking Laplace Transform we get

asyX (s) = b 1 + e-6s l and H (s) = 1


s+2 s
Now
En
Y (s) = H (s) X (s)
= 1 : 1 + e-6sD = 1 +e
-6s

or
s s+2

Y (s) = 1 - 1 gin
+e
s (s + 2)
-6s
s

Thus
2s 2 (s + 2) s
eer
y (t) = 0.5 [1 - exp (- 2t)] u (t) + u (t - 6)
Sol. 22 Option (B) is correct.
y (n) = x (n - 1) ing
or
Y (z)
Y (z) = z-1 X (z)
.ne
or

Now
X (z)
= H (z) = z-1

H1 (z) H2 (z) = z-1


1 - 0.4z-1
c 1 - 0.6z-1 m H2 (z) = z
-1
t
z-1 (1 - 0.6z-1)
H2 (z) =
(1 - 0.4z-1)
Sol. 23 Option (B) is correct.
For 8 point DFT, x* [1] = x [7]; x* [2] = x [6]; x* [3] = x [5] and it is conjugate
symmetric about x [4], x [6] = 0 ; x [7] = 1 + j3
Sol. 24 Option (A) is correct.
Inverse Z - transform
We know that aZ ! a ad [n ! a]
2 -1
Given that X (z) = 5z + 4z + 3
Inverse z-transform x [n] = 5d [n + 2] + 4d [n - 1] + 3d [n]
Sol. 25 Option (C) is correct.
For a function x (t) trigonometric fourier series is

Downloaded From : www.EasyEngineering.net


Downloaded From : www.EasyEngineering.net
GATE SOLVED PAPER - EC SIGNALS & SYSTEMS

3
x (t) = Ao + / [An cos nwt + Bn sin nwt]
n=1

Where, Ao 1 # x (t) dt T0 "fundamental period


T0 T
0

and An = 2 # x (t) cos nwt dt


T0 T
0

Bn = 2 # x (t) sin nwt dt


T0 T
0

For an even function x (t), Bn = 0


Since given function is even function so coefficient Bn = 0 , only cosine and constant
terms are present in its fourier series representation
A0 = 1 # x (t) dt = 1 : # Adt + # - 2AdtD
3T/4 T/4 3T/4
Constant term
T -T/4 T -T/4 T/4

= 1 :TA - 2AT D =- A
ww Constant term is negative.
T 2 2 2

w.E
Sol. 26 Option (C) is correct.
We have h1 [n] = d [n - 1] or H1 [Z ] = Z - 1
h 2 [n] = d [n - 2] or H2 (Z ) = Z - 2
and

asy
Response of cascaded system
H (z ) = H1 (z ) : H2 (z ) = z-1 : z-2 = z-3
or,
En h [n] = d [n - 3]
Sol. 27 Option (D) is correct.
gin
For an N-point FET algorithm butterfly operates on one pair of samples and

Sol. 28
involves two complex addition and one complex multiplication.
Option (D) is correct. eer
We have f (t) = L -1 3s + 1
ing
;s3 + 4s 2 + (k - 3) s E
and

By final value theorem


lim f (t) = 1
t"3

.ne
or lim
lim f (t) = lim sF (s) = 1
t"3

s. (3s + 1)
=1
s"0
t
s"0 s + 4s2 + (k - 3) s
3

s (3s + 1)
or lim 2 =1
s " 0 s [s + 4s + (k - 3)]

1 =1
k-3
or k =4
Sol. 29 Option (B) is correct.
System is described as
d 2 y (t) dt (t) dx (t)
+4 + 3y (t) = 2 + 4x (t)
dt 2 dt dt
Taking Laplace transform on both side of given equation
s 2 Y (s) + 4sY (s) + 3Y (s) = 2sX (s) + 4X (s)
(s 2 + 4s + 3) Y (s) = 2 (s + 2) X (s) s

Downloaded From : www.EasyEngineering.net


Downloaded From : www.EasyEngineering.net
GATE SOLVED PAPER - EC SIGNALS & SYSTEMS

Transfer function of the system


Y (s) 2 (s + 2) 2 (s + 2)
H (s) = = =
X (s) s 2 + 4s + 3 (s + 3) (s + 1)
Input x (t) = e-2t u (t)
or, X (s) = 1
(s + 2)
2 (s + 2)
Output Y (s) = H (s) : X (s) = : 1
(s + 3) (s + 1) (s + 2)
By Partial fraction
1 - 1
Y (s) =
s+1 s+3
Taking inverse Laplace transform
y (t) = (e-t - e-3t) u (t)

ww
Sol. 30 Option (C) is correct.

We have H (z) =
2 - 34 z - 1
1 - 34 z - 1 + 18 z - 2

w.E By partial fraction H (z ) can be written as


H (z ) =
^ -
1
1 -1 +
h ^ -
1
1 -1
h
asy
For ROC : z > 1/2
1 2 z 1 4z

2
En
h [n] = b 1 l u [n] + b 1 l u [n], n > 0
n

4
n
1
1 - z -1
= an u [n], z > a

gin
Thus system is causal. Since ROC of H (z ) includes unit circle, so it is stable
also. Hence S1 is True
For ROC : z < 1
4
eer
h [n] =-b 1 l u [- n - 1] + b 1 l u (n), z > 1 , z < 1
ing
n n

2 4 4 2
System is not causal. ROC of H (z ) does not include unity circle, so it is not
stable and S 3 is True
.ne
Sol. 31

Sol. 32
Option (A) is correct.

t
The Fourier series of a real periodic function has only cosine terms if it is even
and sine terms if it is odd.
Option (B) is correct.
Given function is
f (t) = sin2 t + cos 2t = 1 - cos 2t + cos 2t = 1 + 1 cos 2t
2 2 2
The function has a DC term and a cosine function. The frequency of cosine
terms is
w = 2 = 2pf " f = 1 Hz
p
The given function has frequency component at 0 and 1 Hz.
p
Sol. 33 Option (A) is correct.
x [n] = b 1 l u (n) - b 1 l u (- n - 1)
n n

3 2
Taking z transform we have

Downloaded From : www.EasyEngineering.net


Downloaded From : www.EasyEngineering.net
GATE SOLVED PAPER - EC SIGNALS & SYSTEMS

n=3 n n =- 1
X (z) = / b 13 l z-n - / 1 n -n
b2l z
n=0 n =- 3
n=3 n =- 1
= / b 1 z-1 l - / b 1 z-1 l
n n

n=0
3 n =- 3
2
First term gives 1 z-1 < 1 " 1< z
3 3
Second term gives 1 z-1 > 1 " 1> z
2 2
Thus its ROC is the common ROC of both terms. that is
1< z <1
3 2
Sol. 34 Option (B) is correct.
By property of unilateral Laplace transform

#- 3 f (t) dt L Fs(s) + s1 #- 3 f (t) dt


ww
t 0 -

Here function is defined for 0 < t < t , Thus

w.E #0 f (t) L Fs(s)


t

Sol. 35

asy
Option (A) is correct.
We have h (2) = 1, h (3) =- 1 otherwise h (k) = 0 . The diagram of response is as
follows :

En
gin
eer
ing
It has the finite magnitude values. So it is a finite impulse response filter. Thus

Sol. 36
S2 is true but it is not a low pass filter. So S1 is false.
Option (B) is correct. .ne
gives bounded output y (t). Thus it is BIBO stable.
Here we can conclude that option (B) is correct.
t
Here h (t) ! 0 for t < 0 . Thus system is non causal. Again any bounded input x (t)

Sol. 37 Option (D) is correct.


We have x [n] = {1, 0, 2, 3) and N = 4
N-1
X [k ] = / x [ n] e -j2pnk/N
k = 0, 1...N - 1
n=0
3
For N = 4 , X [k ] = / x [ n] e -j2pnk/4
k = 0, 1,... 3
n=0
3
Now X [ 0] = / x [n] = x [0] + x [1] + x [2] + x [3] = 1 + 0 + 2 + 3 = 6
n=0
3
x [1] = / x [ n] e -jpn/2
= x [0] + x [1] e-jp/2 + x [2] e-jp + x [3] e-jp3/2
n=0

= 1 + 0 - 2 + j3 =- 1 + j3
3
X [ 2] = / x [ n] e -jpn
= x [0] + x [1] e-jp + x [2] e-j2p + x [3] e-jp3
n=0

Downloaded From : www.EasyEngineering.net


Downloaded From : www.EasyEngineering.net
GATE SOLVED PAPER - EC SIGNALS & SYSTEMS

= 1+0+2-3 = 0
3
X [ 3] = / x [ n] e -j3pn/2
= x [0] + x [1] e-j3p/2 + x [2] e-j3p + x [3] e-j9p/2
n=0

= 1 + 0 - 2 - j3 =- 1 - j3
Thus [6, - 1 + j3, 0, - 1 - j3]
Sol. 38 Option (A) is correct.
Sol. 39 Option (C) is correct.
The output of causal system depends only on present and past states only.
In option (A) y (0) depends on x (- 2) and x (4).
In option (B) y (0) depends on x (1).
In option (C) y (0) depends on x (- 1).
In option (D) y (0) depends on x (5).
Thus only in option (C) the value of y (t) at t = 0 depends on x (- 1) past value.

ww
Sol. 40
In all other option present value depends on future value.
Option (D) is correct.

w.E We have h (t) = eat u (t) + e bt u (- t)


This system is stable only when bounded input has bounded output For

asy
stability at < 0 for t > 0 that implies a < 0 and bt > 0 for t > 0 that implies
b > 0 . Thus, a is negative and b is positive.
Sol. 41

En
Option (C) is correct.
K (s + 1)
, and R (s) = 1
G (s) =
(s + 2)( s + 4)
C (s) = G (s) R (s) =
gin
K (s + 1)
s

= K + K
s (s + 2)( s + 4)
- 3K eer
Thus
8s 4 (s + 2) 8 (s + 4)
c (t) = K :1 + 1 e-2t - 3 e-4tD u (t)
8 4 8 ing
At steady-state , c (3) = 1
K = 1 or K = 8 .ne
Thus

Then,
8
G (s) =
8 (s + 1)
(s + 2)( s + 4)
= 12 - 4
(s + 4) (s + 2)
t
h (t) = L-1 G (s) = (- 4e-2t + 12e-4t) u (t)
Sol. 42 Option (A) is correct.
1 for - 1 # t # + 1
We have x (t) = )
0 otherwise
Fourier transform is
= 1 [e-jwt]-11
1
#- 33e-jwt x (t) dt = #-1 e-jwt 1dt
- jw
= 1 (e-jw - e jw) = 1 (- 2j sin w) = 2 sin w
- jw - jw w
This is zero at w = p and w = 2p
Sol. 43 Option (D) is correct.
Given h (n) = [1, - 1, 2]

Downloaded From : www.EasyEngineering.net


Downloaded From : www.EasyEngineering.net
GATE SOLVED PAPER - EC SIGNALS & SYSTEMS

x (n) = [1, 0, 1]
y (n) = x (n)* h (n)
The length of y [n] is = L1 + L2 - 1 = 3 + 3 - 1 = 5
3
y (n) = x (n) * h (n) = / x (k) h (n - k)
k =- 3
3
y (2) = / x (k) h (2 - k)
k =- 3

= x (0) h (2 - 0) + x (1) h (2 - 1) + x (2) h (2 - 2)


= h (2) + 0 + h (0) = 1 + 2 = 3
There are 5 non zero sample in output sequence and the value of y [2] is 3.
Sol. 44 Option (B) is correct.
Mode function are not linear. Thus y (t) = x (t) is not linear but this functions is

ww time invariant. Option (A) and (B) may be correct.


The y (t) = t x (t) is not linear, thus option (B) is wrong and (a) is correct. We
can see that

w.E R1: y (t) = t2 x (t) Linear and time variant.


R2: y (t) = t x (t) Non linear and time variant.
R3: y (t) = x (t) Non linear and time invariant

Sol. 45
asy
R4: y (t) = x (t - 5) Linear and time invariant
Option (A) is correct.

Given : En
y (n) = 1
N-1
/ x (r) x (n + r)
It is Auto correlation.
N r=0

gin
eer
DFT
Hence y (n) = rxx (n) X (k) 2
Sol. 46 Option (B) is correct.
Current through resistor (i.e. capacitor) is
I = I (0+) e-t/RC ing
Here, I (0+) = V = 5 = 25mA
R 200k
.ne
RC = 200k # 10m = 2 sec
I = 25e- m A = VR # R = 5e- V
t
2
t
2
t
Here the voltages across the resistor is input to sampler at frequency of 10 Hz.
Thus
-n
x (n) = 5e 2 # 10
= 5e-0.05n For t > 0
Sol. 47 Option (C) is correct.
Since x (n) = 5e-0.05n u (n) is a causal signal
Its z transform is
X (z) = 5 : 1 5z
1 - e-0.05 z-1 D z - e-0.05
=

Its ROC is e-0.05 z-1 > 1 " z > e-0.05


Sol. 48 Option (C) is correct.
h (t) = e-2t u (t)
H (jw) = #- 33 h (t) e-jwt dt

Downloaded From : www.EasyEngineering.net


Downloaded From : www.EasyEngineering.net
GATE SOLVED PAPER - EC SIGNALS & SYSTEMS

= #0 3e-2t e-jwt dt = #0 3e-(2 + jw)t dt = 1


(2 + jw)
Sol. 49 Option (D) is correct.
H (jw) = 1
(2 + jw)
The phase response at w = 2 rad/sec is
+H (jw) =- tan-1 w =- tan-1 2 =- p =- 0.25p
2 2 4
Magnitude response at w = 2 rad/sec is
H (jw) = 1 = 1
22 + w2 2 2
Input is x (t) = 2 cos (2t)
Output is = 1 # 2 cos (2t - 0.25p)

ww 2 2
= 1 cos [2t - 0.25p]
2

w.E
Sol. 50 Option (D) is correct.
Y (s) = 1
s (s - 1)

asy
Final value theorem is applicable only when all poles of system lies in left half of
S -plane. Here s = 1 is right s -plane pole. Thus it is unbounded.
Sol. 51

En
Option (A) is correct.
x (t) = e-t u (t)
Taking Fourier transform
X (jw) = 1 gin
1 + jw
X (jw) = 1 2 eer
1+w
Magnitude at 3dB frequency is 1
2 ing
Thus 1 =
2
1
1 + w2 .ne
or
or
w = 1 rad
f = 1 Hz
2p
t
Sol. 52 Option (B) is correct.
For discrete time Fourier transform (DTFT) when N " 3
x [n] = 1 # X (e jw) e jwn dw
p

2p - p
Putting n = 0 we get
x [0] = 1 X (e jw) e jw0 dw = 1
p p

2p #
-p 2p #
-p
X (e jw) dw
p
or #
-p
X (e jw) dw = 2px [0] = 2p # 5 = 10p

Sol. 53 Option (B) is correct.


X (z) = 0.5
1 - 2z-1
Since ROC includes unit circle, it is left handed system

Downloaded From : www.EasyEngineering.net


Downloaded From : www.EasyEngineering.net
GATE SOLVED PAPER - EC SIGNALS & SYSTEMS

x (n) =- (0.5) (2) -n u (- n - 1)


x (0) = 0
If we apply initial value theorem
x (0) = lim X (z) = lim 0.5 -1 = 0.5
z"3 z " 31 - 2z

That is wrong because here initial value theorem is not applicable because
signal x (n) is defined for n < 0 .
Sol. 54 Option (A) is correct.
A Hilbert transformer is a non-linear system.
Sol. 55 Option (B) is correct.
H (f) = 5
1 + j10pf

ww Step response
H (s) = 5 =
1 + 5s
5

Y (s) = 1 a 1 = 1 1 1 = 5 - 5
s ^s + 5 h
= 1
5^s + 15 h s + 15

s ^s + 5 h

w.E s s+ 1
5
-t/5
or y (t) = 5 (1 - e ) u (t)
Sol. 56 Option (A) is correct.

asy
x (t)
Using scaling we have
F
X (jw)

x (5t)
En F 1 X jw
5 c 5 m
Using shifting property we get
x ;5 bt - 3 lE
5
F
5 b5l gin
1 X jw e- j35w

Sol. 57 Option (D) is correct.


eer
Dirac delta function d (t) is defined at t = 0 and it has infinite value a t = 0 . The
area of dirac delta function is unity.
ing
Sol. 58 Option (D) is correct.

.ne
The ROC of addition or subtraction of two functions x1 (n) and x2 (n) is R1 + R2 .

Sol. 59
subtraction of two function. It will be same.
Option (A) is correct.
t
We have been given ROC of addition of two function and has been asked ROC of

As we have x (t) = sin t , thus w = 1


Now H (s) = 1
s+1
or H (jw) = 1 = 1
jw + 1 j+1
or H (jw) = 1 + - 45c
2
Thus y (t) = 1 sin (t - p4 )
2
Sol. 60 Option (C) is correct.
F (s) = w0
s + w2
2

L-1 F (s) = sin wo t


f (t) = sin wo t

Downloaded From : www.EasyEngineering.net


Downloaded From : www.EasyEngineering.net
GATE SOLVED PAPER - EC SIGNALS & SYSTEMS

Thus the final value is - 1 # f (3) # 1


Sol. 61 Option (C) is correct.
y (n) = b sin 5 pn l x (n)
6
Let x (n) = d (n)
Now y (n) = sin 0 = 0 (bounded) BIBO stable
Sol. 62 Option (B) is correct.
c (t) = 1 - e-2t
Taking Laplace transform
C (s) 2
C (s) = = #s = 2
U (s) s (s + 2) s+2
Sol. 63 Option (C) is correct.

ww h (t) = e-t
L

L
1
H (s) =
s+1
X (s) = 1

w.E x (t) = u (t)


s
Y (s) = H (s) X (s) = 1 # 1 = 1 - 1
s+1 s s s+1

asy y (t) = u (t) - e-t


In steady state i.e. t " 3, y (3) = 1
Sol. 64
En
Option (C) is correct.
Fourier series is defined for periodic function and constant.
3 sin (25t) is a periodic function.
gin
4 cos (20t + 3) + 2 sin (710t) is sum of two periodic function and also a periodic
function.
eer
e- t sin (25t) is not a periodic function, so FS can’t be defined for it.

Sol. 65
1 is constant
Option (A) is correct. ing
Ev{g (t)} =
g (t) + g (- t)
2 .ne
Here
odd{g (t)} =
g (t) - g (- t)

g (t) = u (t)
2 t
u (t) + u (- t) 1
Thus ue (t) = =
2 2
u (t) - u (- t) x (t)
uo (t) = =
2 2
Sol. 66 Option (C) is correct.
Here x1 (n) = ` 5 jn u (n)
6
X1 (z) = 1 ROC : R1 " z > 5
1 - ^ 65 z-1h 6

x2 (n) =-` 6 jn u (- n - 1)
5
X1 (z) = 1 - 1 ROC : R2 " z < 6
1 - ^ 65 z-1h 5

Downloaded From : www.EasyEngineering.net


Downloaded From : www.EasyEngineering.net
GATE SOLVED PAPER - EC SIGNALS & SYSTEMS

Thus ROC of x1 (n) + x2 (n) is R1 + R2 which is 5 < z < 6


6 5
Sol. 67 Option (D) is correct.
For causal system h (t) = 0 for t # 0 . Only (D) satisfy this condition.
Sol. 68 Option (D) is correct.
x (n) = b 1 l u (n)
n

2
y (n) = x2 (n) = b 1 l u2 (n)
2n

2
2 n
y (n) = ;b 1 l E u (n) = b 1 l u (n)
n
or ...(1)
2 4
n=3 n=3
/ b 14 l e-jwn
n
Y (e jw) = / y (n) e-jwn =
n =- 3 n=0

ww n=3 n
= 1 +b1l +b1l+b1l +b1l = 1
1 3 4
or Y (e j0) = / ` 14 j 4 4 4 4 1- 1
=4
3
n=0 4
Alternative :

w.E Taking z transform of (1) we get


Y (z) = 1

asy 1 - 14 z-1
Substituting z = e jw we have
1

En
Y (e jw) =
1 - 14 e-jw
Y (e j0) = 1 1 = 4

Sol. 69 Option (A) is correct.


1- 4 3
gin
s (t) = 8 cos ` p - 20pt j + 4 sin 15pt
2 eer
= 8 sin 20pt + 4 sin 15pt
Here A1 = 8 and A2 = 4 . Thus power is ing
2 2 2 2
P = A1 + A2 = 8 + 4 = 40
2 2 2 2 .ne
Sol. 70 Option (A) is correct.
y (t) = 0.5x (t - td + T) + x (t - td ) + 0.5x (t - td - T)
Taking Fourier transform we have
t
Y (w) = 0.5e-jw (- t + T) X (w) + e-jwt X (w) + 0.5e-jw (- t - T) X (w)
d d d

Y (w)
or = e-jwt [0.5e jwT + 1 + 0.5e-jwT ]
d

X (w)
= e-jwt [0.5 (e jwT + e-jwT ) + 1] = e-jwt [cos wT + 1]
d d

Y (w)
or H (w) = = e-jwt (cos wT + 1) d

X (w)
Sol. 71 Option (C) is correct.
For continuous and aperiodic signal Fourier representation is continuous and
aperiodic.
For continuous and periodic signal Fourier representation is discrete and aperiodic.
For discrete and aperiodic signal Fourier representation is continuous and periodic.
For discrete and periodic signal Fourier representation is discrete and periodic.

Downloaded From : www.EasyEngineering.net


Downloaded From : www.EasyEngineering.net
GATE SOLVED PAPER - EC SIGNALS & SYSTEMS

Sol. 72 Option (B) is correct.


y (n) = Ax (n - no)
Taking Fourier transform
Y (e jw) = Ae-jw n X (e jw)
o o

Y (e jw)
or H (e jw) = = Ae-jw n o o

X (e jw)
Thus +H (e jw) =- wo no
For LTI discrete time system phase and frequency of H (e jw) are periodic with
period 2p. So in general form
q (w) =- no wo + 2pk
Sol. 73 Option (A) is correct.
From x (n) = [ 12 , 1, 2, 1, 1, 12 ]

ww y (n) = x ^ n2 - 1h, n even


= 0 , for n odd

w.E n =- 2 ,
n =- 1,
y (- 2) = x ( -22 - 1) = x (- 2) = 12
y (- 1) = 0
n = 0,
n = 1,
asy y (0) = x ( 20 - 1) = x (- 1) = 1
y (1) = 0
n=2
n = 3,
En y (2) = x ( 22 - 1) = x (0) = 2
y (3) = 0
n=4
n = 5, gin
y (4) = x ( 24 - 1) = x (1) = 1
y (5) = 0
n=6
Hence
y (6) = x ( 26 - 1) = x (2) = 12
eer
y (n) = 1 d (n + 2) + d (n) + 2d (n - 2) + d (n - 4)
+ 1 d (n - 6)
2
2
ing
Sol. 74 Option (C) is correct.
.ne
Here y (n) is scaled and shifted version of x (n) and again y (2n) is scaled version
of y (n) giving
z (n) = y (2n) = x (n - 1)
= 1 d (n + 1) + d (n) + 2d (n - 1) + d (n - 2) + 1 d (n - 3)
2 2
t
Taking Fourier transform.
Z (e jw) = 1 e jw + 1 + 2e-jw + e-2jw + 1 e-3jw
2 2
= e-jw b 1 e2jw + e jw + 2 + e-jw + 1 e-2jw l
2 2
2jw
= e-jw b e + e-2jw + e jw + 2 + e-jw
2 l

or Z (e jw) = e-jw [cos 2w + 2 cos w + 2]


Sol. 75 Option (B) is correct.
F
x (t) X (f)
Using scaling we have
1 X f
a ca m
F
x (at)

Downloaded From : www.EasyEngineering.net


Downloaded From : www.EasyEngineering.net
GATE SOLVED PAPER - EC SIGNALS & SYSTEMS

xb 1 f l
F
Thus 3X (3f)
3
Using shifting property we get
e-j2pf t x (t) = X (f + f0)
0

1 e-j 43 pt x 1 t
b3 l
F
Thus X (3f + 2)
3
e-j2p t x b 1 t l
2 F
3
3X (3 (f + 23 ))
3
1 e-jp t x 1 t
b3 l
4 F
3
X [3 (f + 23 )]
3
Sol. 76 Option (A) is correct.
3
A system is stable if / h (n) < 3. The plot of given h (n) is

ww
n =- 3

w.E
asy
En
3 6
Thus / h (n) = /
n =- 3
h (n)
n =- 3

= 15 < 3 gin
= 1+1+1+1+2+2+2+2+2

Sol. 77 Option (D) is correct. eer


Hence system is stable but h (n) ! 0 for n < 0 . Thus it is not causal.

We know that
H (z) = z
z - 0.2 ing z < 0.2

- an u [- n - 1] * 1
1 - az-1 .ne z <a

Sol. 78
Thus
Option (C) is correct.
h [n] =- (0.2) n u [- n - 1]

The Fourier transform of a conjugate symmetrical function is always real.


t
Sol. 79 Option (A) is correct.
We have x (n) = [- 4 - j5, 1 + 2j, 4]
-

x *( n) = [- 4 + j5, 1 - 2j, 4]
-

x *( - n) = [4, 1 - 2j, - 4 + j5]


-

x (n) - x* (- n)
xcas (n) = = [- 4 - j 25 , 2j 4 - j 25 ]
2 -

Sol. 80 Option (C) is correct.


We have 2y (n) = ay (n - 2) - 2x (n) + bx (n - 1)
Taking z transform we get
2Y (z) = aY (z) z-2 - 2X (z) + bX (z) z-1

Downloaded From : www.EasyEngineering.net


Downloaded From : www.EasyEngineering.net
GATE SOLVED PAPER - EC SIGNALS & SYSTEMS

Y (z) bz-1 - 2
or =c m ...(i)
X (z) 2 - az-2
z ( b - z)
or H (z) = 22 a
(z - 2 )
It has poles at ! a/2 and zero at 0 and b/2 . For a stable system poles must
lie inside the unit circle of z plane. Thus
a <1
2
or a <2
But zero can lie anywhere in plane. Thus, b can be of any value.
Sol. 81 Option (D) is correct.
We have x (n) = e jpn/4

ww and h (n) = 4 2 d (n + 2) - 2 2 d (n + 1) - 2 2 d (n - 1)
+ 4 2 d (n - 2)

w.E Now

=
3
y (n) = x (n)* h (n)

/ x (n - k) h (k) = / x (n - k) h (k)
k =- 3
2

k =- 2

or
asy
y (n) = x (n + 2) h (- 2) + x (n + 1) h (- 1) + x (n - 1) h (1) + x (n - 2) h (2)
= 4 2 ej
p
4
(n + 2)
- 2 2 ej
p
4
(n + 1)
- 2 2 ej
p
4
(n - 1)
+ 4 2 ej
p
4
(n - 2)

En = 4 2 6e j
p
p
4
@ - 2 2 6e j (n + 1) + e j
(n + 2)

p
+ ej
p
4
(n - 2)

= 4 2 e j n 6e j + e-j @ - 2 2 e j n 6e j + e-j @
p p
p
4

p p
p
4
@
(n - 1)

gin
4 2 2 2 4 4

p p
= 4 2 e j n [0] - 2 2 e j n [2 cos p4 ]
4 4

y (n) =- 4e j n
r

or

eer
4

Sol. 82 Option (B) is correct.


From given graph the relation in x (t) and y (t) is
y (t) =- x [2 (t + 1)] ing
Using scaling we have
x (t)
F
X (f)
.ne
Thus
x (at)

x (2t)
F

F
1 X f
a ca m
1X f
2 c2m
t
Using shifting property we get
x (t - t0) = e-j2pft X (f) 0

j2pf
f f
e-j2pf (- 1) 1 X b l = e X b l
F
Thus x [2 (t + 1)]
2 2 2 2
j 2p f f
-e Xc m
F
- x [2 (t + 1)]
2 2
Sol. 83 Option (C) is correct.
From the Final value theorem we have
lim i (t) = lim sI (s) = lim s 2 = lim 2 =2
t"3 s"0 s"0 s (1 + s) s " 0 (1 + s)
Sol. 84 Option (D) is correct.
Here C3 = 3 + j5

Downloaded From : www.EasyEngineering.net


Downloaded From : www.EasyEngineering.net
GATE SOLVED PAPER - EC SIGNALS & SYSTEMS

For real periodic signal


C-k = Ck*
Thus C-3 = Ck = 3 - j5
Sol. 85 Option (C) is correct.
y (t) = 4x (t - 2)
Taking Fourier transform we get
Y (e j2pf ) = 4e-j2pf2 X (e j2pf ) Time Shifting property
Y (e j2pf )
or = 4e-4jpf
X (e j2pf )
Thus H (e j2pf ) = 4e-4jpf
Sol. 86 Option (B) is correct.
We have h (n) = 3d (n - 3)

ww or H (z) = 2z-3
4 2
X (z) = z + z - 2z + 2 - 3z -4
Taking z transform

w.E Now Y (z) = H (z) X (z) = 2z-3 (z 4 + z2 - 2z + 2 - 3z-4)


= 2 (z + z-1 - 2z-2 + 2z-3 - 3z-7)

asy
Taking inverse z transform we have
y (n) = 2[ d (n + 1) + d (n - 1) - 2d (n - 2)

At n = 4 , En
+ 2d (n - 3) - 3d (n - 7)]
y (4) = 0
Sol. 87 Option (A) is correct.
gin
System is non causal because output depends on future value
For n # 1 y (- 1) = x (- 1 + 1) = x (0)
y (n - n0) = x (n - n0 + 1) eer Time varying

i.e.
y (n) = x (n + 1)
y (1) = x (2)
ing Depends on Future
None causal
For bounded input, system has bounded output. So it is stable.
.ne
y (n) = x (n) for n $ 1
= 0 for n = 0
= x (x + 1) for n #- 1
t
So system is linear.
Sol. 88 Option (C) is correct.
The frequency response of RC-LPF is
H (f) = 1
1 + j2pfRC
Now H (0) = 1
H (f1) 1
= $ 0.95
H (0) 1 + 4p2 f12 R2 C2
or 1 + 4p2 f12 R2 C2 # 1.108
or 4p2 f12 R2 C2 # 0.108
or 2pf1 RC # 0.329
or f1 # 0.329
2pRC

Downloaded From : www.EasyEngineering.net


Downloaded From : www.EasyEngineering.net
GATE SOLVED PAPER - EC SIGNALS & SYSTEMS

or f1 # 0.329
2pRC
or f1 # 0.329
2p1k # 1m
or f1 # 52.2 Hz
Thus f1 max = 52.2 Hz
Sol. 89 Option (A) is correct.
H (w) = 1
1 + jwRC
q (w) =- tan-1 wRC
dq (w) RC 10-3
tg =- = = = 0.717 ms
dw 2 2
1+w R C 2
1 + 4p # 10 4 # 10-6
2

ww
Sol. 90 Option (C) is correct.
If
Then
x (t)* h (t) = g (t)
x (t - t1)* h (t - t2) = y (t - t1 - t2)

w.E
Sol. 91
Thus x (t + 5)* d (t - 7) = x (t + 5 - 7) = x (t - 2)
Option (B) is correct.

asy
In option (B) the given function is not periodic and does not satisfy
Dirichlet condition. So it cant be expansion in Fourier series.

En
x (t) = 2 cos pt + 7 cos t
T1 = 2p = 2
w
T2 = 2p = 2p
1
gin
T1 = 1 = irrational
T2 p eer
Sol. 92 Option (C) is correct.
From the duality property of fourier transform we have ing
If x (t)
FT
X (f)
.ne
t
FT
Then X (t) x (- f)
Therefore if e-t u (t)
FT 1
1 + j 2p f
Then 1 FT
e f u (- f)
1 + j 2p t
Sol. 93 Option (A) is correct.
q (w) =- wt0
- q (w)
tp = = t0
w
dq (w)
and tg =- = t0
dw
Thus tp = tg = t0 = constant
Sol. 94 Option (*) is correct.
X (s) = 2 5 - s = 5-s = -2 + 1
s - s - 2 (s + 1)( s - 2) s+1 s-2
Here three ROC may be possible.

Downloaded From : www.EasyEngineering.net


Downloaded From : www.EasyEngineering.net
GATE SOLVED PAPER - EC SIGNALS & SYSTEMS

Re (s) < - 1
Re (s) > 2
- 1 < Re (s) < 2
Since its Fourier transform exits, only - 1 < Re (s) < 2 include imaginary axis.
so this ROC is possible. For this ROC the inverse Laplace transform is
x (t) = [- 2e-t u (t) - 2e2t u (- t)]
Sol. 95 Option (B) is correct.
For left sided sequence we have
- an u (- n - 1)
z 1 where z < a
1 - az-1
Thus - 5n u (- n - 1)
z 1 where z < 5
1 - 5z-1
z

ww
z
or - 5n u (- n - 1) where z < 5
z-5
Since ROC is z < 5 and it include unit circle, system is stable.

w.E Alternative :
h (n) =- 5n u (- n - 1)
-1 -1

asy
3
H (z) = / h (n) z -n
= /-5 z n -n
=- / (5z -1 n
)
n =- 3 n =- 3 n =- 3

Let n =- m, then

H (z) =-
En
-3
/ (5z -1 -m
) = 1- / (5
3
-1
z) -m

= 1-
n =- 1
1
1 - 5-1 z
,
gin
m=0

5-1 z < 1 or z < 5

= 1- 5 = z
5-z z-5
eer
Sol. 96 Option (B) is correct.
1 = 12 # 1 ing
s2 (s - 2)
1 # 1 L
s s-2

(t * e2t) u (t) .ne


s 2 s - 2
Here we have used property that convolution in time domain is multiplication in
s - domain
LT
t
X1 (s) X2 (s) x1 (t)* x2 (t)
Sol. 97 Option (A) is correct.
We have h (n) = u (n)
3 3 3
H (z) = / x (n) .z -n
= / 1.z -n
= / (z -1 n
)
n =- 3 n=0 n=0

H (z) is convergent if
3
/ (z-1) n < 3
n=0

and this is possible when z-1 < 1. Thus ROC is z-1 < 1 or z > 1
Sol. 98 Option (A) is correct.
# d (t) = 1
3
We know that d (t) x (t) = x (0) d (t) and
-3

Downloaded From : www.EasyEngineering.net


Downloaded From : www.EasyEngineering.net
GATE SOLVED PAPER - EC SIGNALS & SYSTEMS

Let x (t) = cos ( 23 t), then x (0) = 1

# d (t) x (t) = # x (0) d (t) dt # d (t) dt = 1


3 3 3
Now =
-3 -3 -3

Sol. 99 Option (B) is correct.


Let E be the energy of f (t) and E1 be the energy of f (2t), then
#
3
E = [f (t)] 2 dt
-3

E1 = #
3
and [f (2t)] 2 dt
-3

Substituting 2t = p we get
dp 1
# # [f (p)] 2 dp = E
3 3
E1 = [f (p)] 2 =
-3 2 2 -3 2
Sol. 100 Option (B) is correct.

ww Since h1 (t) ! 0 for t < 0 , thus h1 (t) is not causal


h2 (t) = u (t) which is always time invariant, causal and stable.

w.E h3 (t) =
u (t)
1+t
is time variant.
h 4 (t) = e-3t u (t) is time variant.
Sol. 101

asy
Option (B) is correct.
h (t) = f (t)* g (t)

En
We know that convolution in time domain is multiplication in s - domain.
f (t)* g (t) = h (t)
L
H (s) = F (s) # G (s)
Thus H (s) = s2+ 2 # s2 + 1
gin
= 1
s + 1 (s + 2)( s + 3) s + 3
Sol. 102 Option (B) is correct.
Since normalized Gaussion function have Gaussion FT eer
Thus e-at
2
p e-
a
FT p2 f2
a

ing
Sol. 103 Option (B) is correct.
Let x (t) = ax1 (t) + bx2 (t) .ne
ay1 (t) = atx1 (t)
by2 (t) = btx2 (t)
Adding above both equation we have
t
ay1 (t) + by2 (t) = atx1 (t) + btx2 (t) = t [ax1 (t) + bx2 (t)] = tx (t)
or ay1 (t) + by2 (t) = y (t) Thus system is linear
If input is delayed then we have
yd (d) = tx (t - t0)
If output is delayed then we have
y (t - t0) = (t - t0) x (t - t0)
which is not equal. Thus system is time varying.
Sol. 104 Option (A) is correct.
We have h (t) = e2t
LS
H (s) = 1
s-2
X (s) = 1
LS
and x (t) = e3t
s-3

Downloaded From : www.EasyEngineering.net


Downloaded From : www.EasyEngineering.net
GATE SOLVED PAPER - EC SIGNALS & SYSTEMS

Now output is Y (s) = H (s) X (s) = 1 # 1 = 1 - 1


s-2 s-3 s-3 s-2
Thus y (t) = e3t - e2t
Sol. 105 Option (C) is correct.
We know that for a square wave the Fourier series coefficient
sin nw t
Cnsq = At nw t2
0

...(i)
T 2
0

Thus Cnsq \ 1
n
If we integrate square wave, triangular wave will be obtained,
Hence Cntri \ 12
n
Sol. 106 Option (B) is correct.

ww u (t) - u (t - 1) = f (t)
L
F (s) = 1 [1 - e-s]
s
G (s) = 1 [1 - e-2s]
w.E
L
u (t) - u (t - 2) = g (t)
s
L
f (t)* g (t) F (s) G (s)

asy = 12 [1 - e-s] [1 - e-2s]


s
= 12 [1 - e-2s - e-s + e-3s]

En L
s
-2s -s
= 12 - e 2 - e 2 + e 2
-3s
or f (t)* g (t)
s s
Taking inverse Laplace transform we have
s s
gin
eer
f (t)* g (t) = t - (t - 2) u (t - 2) - (t - 1) u (t - 1) + (t - 3) u (t - 3)
The graph of option (B) satisfy this equation.
Sol. 107 Option (A) is correct.
ing
Sol. 108 Option (A) is correct.
We have f (nT) = anT
.ne
Taking z -transform we get
F (z) =
n =- 3
3
/ anT z-n =
3
/ (aT ) n z-n
n =- 3
= t
/ b az
3

n=0
T n
l =
z
z - aT
Sol. 109 Option (B) is correct.
If L [f (t)] = F (s)
Applying time shifting property we can write
L [f (t - T)] = e-sT F (s)
Sol. 110 Option (A) is correct.
Sol. 111 Option (A) is correct.
Sol. 112 Option (C) is correct.
Given z transform
z-1 (1 - z-4)
C (z) =
4 (1 - z-1) 2

Applying final value theorem

Downloaded From : www.EasyEngineering.net


Downloaded From : www.EasyEngineering.net
GATE SOLVED PAPER - EC SIGNALS & SYSTEMS

lim f (n) = lim (z - 1) f (z)


n"3 z"1

z-1 (1 - z-4) z-1 (1 - z-4) (z - 1)


lim (z - 1) F (z) = lim (z - 1) - = lim
z"1 z"1 4 (1 - z ) 1 2 z"1 4 (1 - z-1) 2
z-1 z-4 (z 4 - 1) (z - 1)
= lim
z"1 4z-2 (z - 1) 2
-3 (z - 1) (z + 1) (z2 + 1) (z - 1)
= lim z
z"1 4 (z - 1) 2
-3
= lim z (z + 1) (z2 + 1) = 1
z"1 4

Sol. 113 Option (A) is correct.


We have F (s) = w
s2 + w2
lim f (t) final value theorem states that:

ww t"3
lim f (t) = lim sF (s)
t"3 s"0

It must be noted that final value theorem can be applied only if poles lies in –ve

w.E half of s -plane.


Here poles are on imaginary axis (s1, s2 = ! jw) so can not apply final value

Sol. 114 asy


theorem. so lim f (t) cannot be determined.
t"3
Option (D) is correct.

En
Trigonometric Fourier series of a function x (t) is expressed as :
x (t) = A 0 +
3
/ [An cos nwt + Bn sin nwt]
n=1
For even function x (t), Bn = 0
gin
eer
3
So x (t) = A 0 + / An cos nwt
n=1
Series will contain only DC & cosine terms.
Sol. 115 Option (C) is correct.
Given periodic signal ing
x (t) = *
1, t < T1
0, T1 < t < T0
.ne
The figure is as shown below.
2
t

For x (t) fourier series expression can be written as


3
x (t) = A 0 + / [An cos nwt + Bn sin nwt]
n=1
where dc term
A0 = 1 = 1
T0 /2

T0 #T x (t) dt
0 T0 #-T /2 x (t) dt
0

Downloaded From : www.EasyEngineering.net


Downloaded From : www.EasyEngineering.net
GATE SOLVED PAPER - EC SIGNALS & SYSTEMS

= 1 : # x (t) dt + x (t) dtD = 1 60 + 2T1 + 0@


-T T1 T0 /2

T0 -T /20
1
#-T x (t) dt + #T
1 1 T0
A 0 = 2T1
T0
Sol. 116 Option (B) is correct.
The unit impulse response of a LTI system is u (t)
Let h (t) = u (t)
Taking LT we have H (s) = 1
s
If the system excited with an input x (t) = e-at u (t), a > 0 , the response
Y (s) = X (s) H (s)
X (s) = L [x (t)] = 1
(s + a)
Y (s) = 1 1 = 1 :1 - 1 D

ww so
(s + a) s
Taking inverse Laplace, the response will be
a s s+a

w.E
Sol. 117 Option (B) is correct.
y (t) = 1 61 - e-at@
a

We have
asy
x [ n] =
k=0
3
/ d (n - k)

En
X (z) =
3

k=0
/ x [n] z-n =
3
/ ; / d (n - k) z-nE
3

n =- 3 k = 0
Since d (n - k) defined only for n = k so
3
X (z) = / z-k = 1
(1 - 1/z)
= z
(z - 1)
gin
Sol. 118 Option (B) is correct.
k=0

eer
Sol. 119 Option (B) is correct.
x (t)
F
X (f) ing
by differentiation property;
F;
dx (t)
dt E
= jwX (w) .ne
or F;
dx (t)
dt E
= j2pfX (f) t
Sol. 120 Option (C) is correct.
g (w)
F
We have f (t)
by duality property of fourier transform we can write
2pf (- w)
F
g (t)
3
so F [g (t)] = # g (t) e-jwt dt = 2pf (- w)
-3

Sol. 121 Option (B) is correct.


Given function
x (t) = eat cos (at)
Now cos (at)
L s
s2 + a2
L
If x (t) X (s)

Downloaded From : www.EasyEngineering.net


Downloaded From : www.EasyEngineering.net
GATE SOLVED PAPER - EC SIGNALS & SYSTEMS

es t x (t) X (s - s 0)
L
then 0
shifting in s-domain
(s - a)
eat cos (at)
L
so
(s - a) 2 + a2
Sol. 122 Option (C) is correct.
For a function x (t), trigonometric fourier series is :
3
x (t) = A 0 + / [An cos nwt + Bn sin nwt]
n=1

where A 0 = 1 # x (t) dt T0 =Fundamental period


T0 T 0

An = 2 # x (t) cos nwtdt


T0 T 0

Bn = 2 # x (t) sin nwtdt


T0 T 0

For an even function x (t), coefficient Bn = 0

ww for an odd function x (t), A0 = 0


An = 0

w.E
Sol. 123
so if x (t) is even function its fourier series will not contain sine terms.
Option (C) is correct.

asy
The conjugation property allows us to show if x (t) is real, then X (jw) has conjugate
symmetry, that is

Proof :
En
X (- jw) = X)(jw) [ x (t) real]

X (jw) =
-3
3

gin
# x (t) e-jwt dt
replace w by - w then

X (- jw) =
3
# x (t) e jwt dt eer
X)(jw) = =
-3
3
# x (t) e-jwt dtG
)
=
3
ing
# x)(t) e jwt dt
if x (t) real x)(t) = x (t)
-3 -3

.ne
t
3
then X)(jw) = # x (t) e jwt dt = X (- jw)
-3

***********

Downloaded From : www.EasyEngineering.net

You might also like